1.2 Far1 Vol II Autumn 2022

You might also like

Download as pdf or txt
Download as pdf or txt
You are on page 1of 417

CAF-01

Financial Accounting
and Reporting I

Vol. II

Practice Manual Examination


Questions & Answers

Book Contains Translations of


QURANIC AYATS & AHADITHS,
Therefore Handle Carefully.
table of contents
Chapter Topics Page No.
No.
1 IAS 16 and Impairment IAS 36 01

2 IAS 36 Extra Questions 35

3 Depreciation 38

4 Exchange of Assets 46

5 IAS - 20 Govt Grants Flowchart 51

6 IAS 20 52

7 IAS 40 81

8 Capital Maintenance 110

9 Extra Practice 126

10 Autumn 2020 155

11 Spring 2021 169

12 Basic Earnings per Share 184

13 Diluted EPS 224

14 IAS 33 EPS Graded Questions 237

15 IAS 33 EPS Graded Answers 248

16 EPS Test 279

17 EPS Test Solution 281

18 Rectification of Error 287

19 Old Income and Expenditure Account 318

20 Accounting for NPO Final Updated 358

21 Autumn 2021 398


“If you learn from failure then it becomes like success rather than failure.”

Property Plant and Equipment (IAS-16) and Impairment of assets (IAS 36)

Definition of Property, Plant & Equipment [PPE]:


These are those tangible items that:
• Are held for use in business for production or supply of goods or for administrative purposes; and
• Are expected to be used during more than on accounting period.
 The asset are capitalized and then depreciated over their useful life.
 Examples of these assets may be;
• Land & Building
• Plant & Machinery
• Furniture & Fixtures
• Motor Vehicles
• Computer Equipment etc

Depreciation is the systematic allocation of the depreciable amount of an asset over its useful life.

Depreciable amount is the cost of an asset, or other amount substituted for cost, less its residual value.

Residual value of an asset is the current estimate of the amount that an entity would obtain from disposal of the
asset at the end of its useful life.

Useful life is:


(a) The period over which an asset is expected to be available for use by an entity; or
(b) The number of production or similar units expected to be obtained from the asset by an entity.

Carrying amount is the amount at which an asset is recognized after deducting any accumulated depreciation and
accumulated impairment losses. (Net book value (NBV) is a term that is often used instead of carrying amount)

Recognition
The cost of an item of PPE shall be recognised as an asset if, and only if:
(a) it is probable that future economic benefits associated with the item will flow to the entity; and
(b) the cost of the item can be measured reliably.

Spare parts and servicing equipment are usually carried as inventory and recognised in profit or loss as consumed.
However, major spare parts and stand-by equipment qualify as property, plant and equipment when an entity
expects to use them during more than one period.

As a practical expedient, immaterial items are not recognised as PPE even if they meet the definition criteria, for
example, staplers and calculators etc.

1 Page 1 of 34
Initial Measurement of an item of PPE
Initially an item of PPE is measured at cost. Cost components of may be different depending upon whether the
asset is
→ Purchased locally from within Pakistan.
→ Imported from an outside country
→ Self-manufactured.

As a general rule;
“All amounts incurred to bring the asset into working condition as intended by management are added to the cost
of an item of PPE.”

Note: Points to remember for Depreciation


➢ Depreciation shall begin when the item of PPE is available for use (Irrespective of whether or not it is actually
being used)
➢ Depreciation shall cease when the asset is derecognized i.e. disposed of /written off.
➢ Depreciation shall not cease when the asset is temporarily idle unless usage methods of depreciation (e.g.
machine hour and production unit methods are used).
 Depreciation is charged as an expense unless asset is used to bring another asset into working condition. In
such a case depreciation of asset is capitalized to the cost of that other asset e.g. an equipment is used to
install a plant and machinery for a period of three months during the year. Three months depreciation should
be capitalized to cost of plant and machinery and remaining 9 months depreciation of equipment should be
expensed out.
 An item of property plant and equipment shall be measured at its cost.

Element of Cost [Para 15 to 20]


Amounts that should be Included In the initial cost of an asset are those that are necessary to bring the asset to a
location and condition that enables it to be used as intended by management. It is calculated as follows;
a) Purchase price less trade discount; plus import duties and non-refundable. i.e. taxes (sales and Income)
b) Directly attributable cost
Examples of directly attributable cost
1) Cost of site preparation
2) Carriage inward
3) Installation and assembly cost
4) Professional fees and
5) Cost of testing that the asset Is functioning correctly(net off any proceeds made from the sale of items
produced during testing)
c) Initial estimates of future costs where the entity has the obligation to incur these costs [IAS-37].

Cost of dismantling, removing the asset and site restoration:


A company may be required to dismantle, remove the equipment and ‘clean up’ a location where it has been
working when production ceases.

This is often the case in industries where companies are only granted licenses to operate on condition that they
undertake to perform these activities. Such industries include, oil and gas, mining and nuclear power.

2 Page 2 of 34
Suppose a company has an obligation to perform these activities because it has agreed in a contract with the
relevant government of the country.

As the performance of these activities is agreed in a binding contract therefore a liability to dismantle and remove
the asset is recognized immediately after the installation of equipment (at the start of contract).
However as the liability is to be settled at the end of the contract, therefore liability is measured on the basis of
present value of expected future cash outflows.

Conclusion:
As the entity can’t obtain future economic benefits without agreeing to the clauses of the agreement therefore
second effect of this liability is capitalized to the cost of related assets.
Asset xxx
Liability to dismantle xxx
(At present value of expected future cash outflows)

Example of costs not part of an item of PPE


1) Abnormal wastages
2) Staff training e.g. training to operate newly acquired asset
3) Administrative expenses
4) Advertisement expenses
5) initial operating losses of business while the demand of output builds up
6) Relocation expenses(means cost of moving the asset to another location when it was working properly at a
place)

Q.1 A Limited bought a special bread-making plant on 1 January 20X1, details of which follow:
Rupees
Purchase price (including sales tax of 14%) 570,000
Import duties - non-refundable 100,000
Installation costs 30,000
Fuel (incurred when transporting the plant to the factory) 45,000
Administration costs 10,000
Staff party to celebrate the acquisition of the new plant 14,000
Staff training 12,000
Testing to ensure plant fully operational before start of production 10,980
Proceeds from sale of samples and by-products made during testing 13,000
Advertising of the ‘special bread’ to be made by the new plant 50,000
Initial operating loss 35,000
Estimated costs of dismantling/ removal costs at the end of its useful life (future amount 27,000
payable of Rs 70,031 present valued at a discount rate of 10%)

Useful life is 10 years.


The company is registered person in sales tax (Means Company can take refund of sales tax paid).

3 Page 3 of 34
Required:
Calculate the cost to be capitalized to the plant account.

Depreciating the whole asset or the parts thereof:


When an item of PPE has more than one significant parts having different useful life and a significant cost, it is
more appropriate to depreciate each of these parts individually, rather than depreciating the asset as a whole.

Parts of an asset
Each part of an asset that has a cost that is significant in relation to the total cost of the item must be depreciated
separately. This means that the cost of an asset might be split into several different assets and each depreciated
separately.

Illustration: Cost
A company has purchased a new airplane for Rs 5,500 million.

The company has identified the following cost components and useful lives in respect of this airplane.

Rs. Million Useful lives


Engines 2,000 3 years
Airframe 1,500 10 years
Avionics 1,500 20 years
Fittings 500 5 years
Total cost 5,500

Q.2 Ancient Waters Limited is a company involved in bottling spring water. The company purchased a bottling
plant on 2 January 20X2. The plant is made up of three significant components, the cost of which is as follows:
Depreciation of component: Cost price Residual value Expected useful life
Engine 1,500,000 500,000 5 years
Conveyer Belt and finings 2,000,000 - 8 years
Outer structure 800,000 50,000 3 years

Other costs incurred in relation to the bottling plant are as follows:


Description of cost: Rupees Transaction date
Delivery and installation 750,000 5 January 20X2
Staff training 60,000 16 January 20X2
Testing to ensure plant fully operational before start of 33,000 19 January 20X2
Production
Launch Party 210,000 21 January 20X2
initial Operating Loss 45,000 March 20X2

1) The plant was available for use in production on 1 February 20X2, although production only began on 1
March 20X2,
2) The plant was temporarily idle during December 20X2 when the factory closed down for its annual holiday
period.

4 Page 4 of 34
3) The company uses the straight-line method when depreciating its bottling plant (not apportioned for part
of a month).
4) All 'other costs' are considered to be incurred evenly between the three significant components of the
bottling plant (i.e. where appropriate, a third of the cost is allocated to each component).
5) The only other asset owned by Ancient Waters Ltd is land which was purchased on 5 December 20X0 for
Rs 4,000,000. The land is not depreciated.

Required:
a) Show all related journal entries relating to the bottling plant for the year ended 31 December 20X2 and 20X3.
Round to the nearest Rupee.
b) Disclose the “property, plant and equipment" note in the financial statements of Ancient Waters Limited for
the year ended 31 December 20X3.
 Day to day servicing costs (also known as repair and maintenance cost) are expensed when incurred.

Subsequent expenditure
Expenditure relating to non-current assets, after their initial acquisition, should be capitalised if it:
• improves the asset (for example, by enhancing its performance or extending its useful life); or
• is for a replacement part (provided that the part that it replaces is derecognized).

A basic rule is that improvements are capitalized but repairs are expensed.

Major inspections
A company might only be allowed to operate some assets if those assets are subjected to regular major
inspections for faults.

The cost of such major inspections is recognized in the carrying amount of the asset if such costs benefit the
business for more than one accounting period.

Example: Major overhaul


A shipping company is required to put its ships into dry port every three years for an overhaul, at a cost of Rs.
3,000,000. The ships have a useful life of 20 years. A ship was purchased from a shipbuilder at a cost of Rs. 200
million.

Initial recognition
Ship will be recorded as an asset at a cost of Rs. 200 million.

End of year 3: When an overhaul is required.


The cost of the new overhaul is capitalised and added to the asset’s cost.

At the end of year 6, the cost of the old overhauling and accumulated depreciation of overhaul which would have
been equal to cost should be removed from the books. The cost of the new overhaul will be capitalized and to be
depreciated over its useful life.

This treatment is just like part replacement.

5 Page 5 of 34
Impairment of Asset:
Asset is said to be impaired if its carrying amount exceeds recoverable amount.
• Asset can be tangible asset (IAS-16) or an intangible asset (IAS-38). In addition concept of impairment is
applicable to IAS 40 if cost model is used.
• Carrying Amount before impairment loss means:
For Assets at Cost Model For Assets at Revaluation Model
Cost - Revalued Amount -
Less Acc Depreciation (-) Less Acc Depreciation (-)
_ -__ __-__
Recoverable Amount: it is higher of:
• Fair value less cost to sell; and
• Value in use

Scenario 1
As on 30-6-2011, a machine measured at cost model has following carrying amount:
Cost 1,000,000
Less Acc Depreciation (350,000)
650,000

This machine is an asset from which we will obtain future economic benefits. There are two ways of obtaining
benefits from this machine; either:
a) Use the machine over remaining life and obtain production of goods; or
b) Immediately sell it and receive cash
• Suppose we have estimated as on 30-6-2011; that by using the machine over the remaining useful life we
can obtain benefit of Rs 700,000. In addition it is estimated that if the machine is sold on 30-6-2011
(today) we can obtain 800,000.
• What should we do; either use it or sell it. Obviously we should go towards higher figure; i.e 800,000. It
means our machine has a potential of Rs 800,000 benefit.
• The carrying amount is Rs 650,000 as on 30-6-2011; which means we are expecting a gain of Rs 150,000
(which should not be recorded immediately)
• Machine will remain at Rs 650,000 in statement of financial position as on 30-6-2011.

Scenario 2
As on 30-6-2014. A machine measured at cost model has the following carrying amount:
Cost 500,000
Less Acc Depreciation (300,000)
200,000

Suppose benefit from use over remaining life is expected to be Rs 150,000. It is further estimated that we can
obtain Rs 70,000 if sold immediately.
• It means we can obtain maximum benefit of Rs 150,000 by using the machine in this case (which is higher
figure).
• However, even if we use the machine, this benefit is less than the carrying amount; which means we are
expecting a loss of Rs 50,000.

6 Page 6 of 34
“Don’t let failure discourage you but take failure as guidance to achieve the success.”

• This expected loss is called as impairment loss which should be recognized immediately at the reporting
date i.e 30-6-2014.

Impairment Loss 50,000


Acc impairment Loss 50,000
After impairment loss; the revised carrying amount will be
Cost 500,000
Less Acc Depreciation (300,000)
Less Acc Impairment loss (50,000)
Carrying Amount 150,000
• After impairment loss revised carrying amount is depreciated over remaining useful life.
 Value in use
Present value of future cash flows expected to be derived from an asset over its remaining useful life.

Calculation of value in use:


1) Estimate future cash inflows from continuing use of asset.
2) Estimate future cash outflows from continuing use of asset.
3) Estimate *net disposal proceeds at the end of its useful life (if any).
*Net disposal proceeds means ultimate sale proceeds at the end of remaining useful life less estimated cost of
disposal.
4) Calculate the present value of these figures.
5) Add the calculated present values. Total is called as value in use.
 Fair value less cost to sell (FV less CTS)
It is the price that would be received from selling an asset less estimated cost of disposal. (It is immediate net
sale proceeds from asset at the reporting date).

Q.3 A machine has a carrying amount of Rs. 600,000 as on 31-12-2010. It is expected that the machine has a
remaining useful life of five years. The net cash flows generated from the use of that machine are
estimated as follows:
2011 2012 2013 2014 2015
Rs. Rs. Rs. Rs. Rs.
200,000 200,000 100,000 100,000 100,000

Net disposal proceeds at the end of 2015 are expected to be Rs 5,000.

Appropriate discount rate is 10% p.a.


Fair value of the machine is Rs 570,000 and costs to sell the machine is Rs. 22,000 as on 31-12-2010

Required: Calculate the impairment loss, if any, as on 31-12-2010.

7 Page 7 of 34
Procedure for calculation of impairment loss:
 Assess at the end of each reporting period whether there is any *indication of impairment of an asset.
*Examples of indication of impairments.
a) Physical damage of the asset; or
b) Obsolescence of asset; or
c) Economic conditions of the country are deteriorating in which the company operates.
 If there is any indication then calculate *recoverable amount of the asset.
*Recoverable amount is higher of:
→ Value in use; and
→ FV less CTS
 Compare recoverable amount with carrying amount.
 If carrying amount exceeds recoverable amount then recognize the impairment loss as follows:
For a Non-revalued Asset → recognize in statement of profit or loss immediately.
For a Revalued asset → First adjust against revaluation surplus if any as a result of a previous revaluation.

If however, impairment loss is more than revaluation surplus recognize the difference immediately in statement of
profit or loss.

Impairment loss in case of Revalued Asset:


Plant purchased on 1-1-2007
Cost = 1,000,000
Useful Life = 20 years

Plant is carried at Revaluation Model for subsequent measurement.


Year end 31-12-2007
Carrying Amount 1000,000 – 50,000 = 950,000
Revalued Amount (FV) 1,500,000
Revaluation Surplus 550,000
No indication of impairment loss on 31-12-2007
1,500,000 ÷ 19 550,000 ÷ 19
Depreciation (78,948) Surplus transferred to Retained (28,947)
Earnings
Carrying Amount 31-12-2008 1,421,052 Remaining Surplus 521,053

As on 31-12-2008
No indication of impairment loss
No revaluation as no material difference
Depreciation (78,948) (28,947)
Carrying Amount 31-12-2009 1,342,104 Remaining Surplus 492,106

As on 31-12-2009
No revaluation as no material difference
No indication of impairment loss
Depreciation (78,948) (28,947)
Carrying Amount 31-12-2010 1,263,156 Remaining Surplus 463,159

8 Page 8 of 34
As on 31-12-2010
Suppose there is an indication of Impairment loss i.e Plant is now obsolete; therefore calculate the
recoverable amount.
Value in use 700,000
FV less CTS 600,000
Higher 700,000 which is now called as recoverable amount

Therefore impairment loss = 1,263,156-700,000 = 563,156


Accounting Entry
Revaluation Surplus (OCI) 463,159
impairment Loss (bal) (P.L) 99,997
Acc Impairment Loss 563,156

Final carrying amount as on 31-12-2010


Plant 1,500,000
Less Acc depreciation (78,948 x 3) (236,844)
Less Acc Impairment Loss _(563,156)_
_700,000_

After charging impairment loss, revised carrying amount; i.e 700,000 is depreciated over remaining useful life i.e
16 years.

Relevant Ledgers
Plant
1-1-07 Bank 1,000,000 31-12-07 Acc Dep 50,000
31-12-07 Revaluation Surplus 550,000 31-12-07 c/d 1,500,000

1-1-08 b/d 1,500,000 31-12-08 c/d 1,500,000

1-1-09 b/d 1,500,000 31-12-09 c/d 1,500,000

1-1-10 b/d 1,500,000 31-12-10 c/d 1,500,000

Accumulated Depreciation
31-12-07 Plant 50,000 31-12-07 Depreciation 50,000
c/d -
1-1-08 b/d -
31-12-08 c/d 78,948 31-12-08 Depreciation 78,948

1-1-09 b/d 78,948


31-12-09 c/d 157,896 31-12-09 Depreciation 78,948

1-1-10 b/d 157,896


31-12-10 c/d 236,844 31-12-10 Depreciation 78,948

9 Page 9 of 34
Revaluation Surplus
31-12-07 Plant 550,000
31-12-07 c/d 550,000

31-12-08 Retained earnings 28,947 1-1-08 b/d 550,000


31-12-08 c/d 521,053

31-12-09 Retained earnings 28,947 1-1-09 b/d 521,053


31-12-09 c/d 492,106

31-12-10 Retained earnings 28,947 1-1-10 b/d 492,106


31-12-10 Acc impairment loss 463,159
31-12-10 c/d -

Acc Impairment Loss


31-12-2010 Revaluation surplus 463,159
c/d 563,157 31-12-2010 Impairment Loss 99,998

Q.4 Scientific Pharma Limited (SPL) is a manufacturer of pharmaceutical products. In January 2010, one of its
plants suffered a major break down. It was repaired at a cost of Rs. 1.5 million but the production capacity
was reduced significantly. The plant was ready for production on June 30, 2010.

At that time the company’s engineers advised that the plant could be used at a reduced level for 3 years only. The
plant is expected to have a recoverable amount of Rs 19,227,000 as on 30-6-2010.

Other related information is as under:


1) The plant was imported at price of US$ 800,000. The payment was made at the time of shipment on July 1,
2000 at Rs. 52 per US$. Other charges including installation cost amounted to Rs. 7 million. Installation of the
plant was completed on December 31, 2000 and commercial production commenced from April 1, 2001.
2) The company uses straight line method of deprecation. Depreciation is charged from the month the asset is
available for use up to the month prior to disposal. At the time of purchase, the estimated useful life of the
plant was estimated at 15 years whereas the salvage value was estimated at Rs. 2.0 million.
3) Based on the report of a professional independent valuer, the plant was revalued on July 1, 2005 at Rs. 45
million. There was however, no change in estimated useful life of the plant.
4) The factory remained closed from April 1, to June 30, 2007 due to law and order situation.
5) The salvage value has not changed since it was first estimated at the time of purchase.

Required:
Prepare accounting entries for the year ended June 30, 2010. Give all the necessary calculations. (Ignore taxation)

Note: Points to remember for Depreciation


➢ Depreciation shall begin when the item of PPE is available for use (Irrespective of whether or not it is actually
being used)
➢ Depreciation shall cease when the asset is derecognized i.e disposed off /written off or fully depreciated
whichever is earlier.

10 Page 10 of 34
“Always wake up with a goal.”

➢ Depreciation shall not cease when the asset is temporarily idle unless usage methods of depreciation (e.g.
machine hour and production unit methods are used).

 Depreciation is charged as an expense unless asset is used to bring another asset into working condition. In
such a case depreciation of asset is capitalized to the cost of that other asset e.g an equipment is used to
install a plant and machinery for a period of three months during the year. Three months depreciation should
be capitalized to cost of plant and machinery and remaining 9 months depreciation of equipment should be
expensed out.

Concept of Present value:

Present Value

Discounting Annuity

(1+i)-n 1-(1+i)-n

i=interest rate i

n= no of years

A company is required to pay 100,000 at the end of five years and suppose interest rate is 10%.Calculate PV.
=100,000 (1.1)-5
=Rs 62,092

A company is required to pay 100,000 at the end of each year for next five years.
− Interest rate = 10%
− Suppose today’s date is 1-1-2014

31-12-2014 100,000 x (1+0.1)-1 90,909


31-12-2015 100,000 x (1+0.1)-2 82,645
31-12-2016 100,000 x (1+0.1)-3 75,131
31-12-2017 100,000 x (1+0.1)-4 68,301
31-12-2018 100,000 x (1+0.1)-5 62,092
379,079

Important
Annuity 1-(1+i)-n
i

11 Page 11 of 34
In order to simplify the above calculation of PV, we can use formula of annuity if there are more than one
instalments. However this formula can only be used if amounts are same and interval between the amounts is
constant.
100,000 [1-(1+0.1)-5]
0.1
= Rs 379,079

• Four annual instalments of Rs 71,465 payable at the end of each year. Interest rate is 16%.
= 71,465 [1-(1+0.16)-4]
0.16
= Rs 199,972
• Four annual instalments of Rs 2,000 payable at the beginning of each year. Interest rate is 10%.

1-1-2014 2,000
1-1-2015 2,000

1-1-2016 2,000
1-1-2017 2,000

PV = 2,000 + 2,000 [1-(1.1)-3]


0.1
PV = Rs 6,974

Further discussion of impairment of assets:


IAS 36 applies to accounting for impairment of assets except for the followings:
• Inventories (IAS 2)
• Assets (means trade receivables) arising from contracts as per IFRS 15
• Investment properties that are measured at fair value (IAS 40)

Summary of the approach


Impairment of an asset should be identified and accounted for as follows:
(1) At the end of each reporting period, the entity should assess whether there are any indications that an
asset may be impaired.
(2) If there are such indications, the entity should estimate the asset’s recoverable amount.
(3) When the recoverable amount is less than the carrying value of the asset, the entity should reduce the
asset’s carrying value to its recoverable amount. The amount by which the value of the asset is written
down is an impairment loss.
(4) This impairment loss is recognised as a loss for the period.
(5) However, if the impairment loss relates to an asset that has previously been re-valued upwards, it is first
offset against any remaining revaluation surplus for that asset.
(6) Depreciation charges for the impaired asset in future periods should be adjusted to allocate the asset’s
revised carrying amount, minus any residual value, over its remaining useful life (revised if necessary).

12 Page 12 of 34
Every success is based on the deeds so do great deeds for great success.”

Identifying impairment or possible impairment


An entity must carry out an impairment review when there is evidence or an indication that impairment may
have occurred. At the end of each reporting period, an entity should assess whether there is any indication that
impairment might have occurred. If such an indication exists, the entity must estimate the recoverable amount
of the asset, in order to establish whether impairment has occurred and if so, the amount of the impairment.

Indicators of impairment
The following are given by IAS 36 as possible indicators of impairment.
When assessing whether there is an indication of impairment, IAS 36 requires that, at a minimum, the
following sources are considered:

External sources Internal sources


An unexpected decline in the asset’s Evidence that the asset is damaged or
market value. no longer of use to the entity.
Significant changes in technology, There are plans to discontinue or
markets, economic factors or laws restructure the operation for which the
and regulations that have an adverse asset is currently used.
effect on the company.
An increase in interest rates, affecting There is a reduction in the asset’s
the value in use of the asset. expected remaining useful life.
The company’s net assets have a There is evidence that the entity’s
higher carrying value than the expected performance is worse than
company’s market capitalization expected.
(which suggests that the assets are
over-valued in the statement of
financial position).

Internal indicators for impairment are generally refers to items under control of management while external
indicators are outside the control of management.

If there is an indication that an asset is impaired then it is tested for impairment. This involves the calculating the
recoverable amount of the item in question and comparing this to its carrying amount.

Measuring recoverable amount

It has been explained that recoverable amount is the higher of an asset’s:


• fair value less costs of disposal; and
• its value in use.

If either of these amounts is higher than the carrying value of the asset, there has been no impairment.

13 Page 13 of 34
Measuring fair value less costs of disposal
Fair value is normally current market value of asset. If no active market exists, it may be possible to estimate the
amount that entity could obtain from disposal.

Direct selling costs normally include legal costs, taxes and costs necessary to bring the asset into a condition to
be sold (e.g. transaction taxes like Octori). However, redundancy and similar costs (for example, where a
business is reorganized following the disposal of an asset (means cost to reorganize the business after disposal
of asset)) are not direct selling costs.

for example direct selling costs include:


• legal costs;
• costs of removal of the asset;
• Costs of delivery to customer
• costs incurred in bringing the asset to a saleable condition;
• Transaction taxes.
Example : recoverable amount – fair value less costs to sell
A company has an asset with the following details at 31 December 20X3:

Expected selling price 200 000


Costs of delivery to potential customer 20 000
Legal costs involved in sale agreement 10 000
Required:
Calculate the fair value less costs to sell of the asset at 31 December 20X3

Solution to example: recoverable amount – fair value less costs to sell

Expected selling price 200 000


Less the costs of disposal (20 000 + 10 000) 30 000
Fair value less costs to sell 170 000

Approach of calculating the recoverable amount:


• First calculate the fair value less cost to sell (it is generally easy)
• If fair value less cost to sell is higher than carrying amount then there is no need to calculate the value in use.
If it is not the case then calculate the value in use and then select the higher figure.
• If fair value less cost to sell is not available then calculate the value in use and take it as a recoverable amount.

Calculating the value in Use:


The following elements should be reflected in the calculation of an asset’s value in use:
• An estimate of the future cash flows the entity expects to derive from the asset
• Expectations about possible variations in the amount or timing of those future cash flows
• The time value of money (represented by the current market risk-free rate of interest)

14 Page 14 of 34
“You should work in a way that your work should encourage you to do more better.”

Estimates of future cash flows must include:



cash inflows from the continuing use of the asset;

cash outflows that will be necessarily incurred to generate the cash inflows from continuing use of the asset;
and


net disposal proceeds at the end of the asset’s useful life.

Estimates of Cash outflows should include only future expenditure that is necessary to maintain the asset at the
standard of performance assessed immediately before the expenditure is made (means repair and maintenance).

Cash flows to be excluded


Future cash flows are estimated based on the asset’s current condition. Care must therefore be taken not to
include the expected:
• cash inflows that relate to other assets, (since these will be taken into account when assessing the value in use
of these other assets);
• cash outflows that have already been recognized as liabilities (either as part of the asset or as an expense);
• cash inflows and outflows that relate to future expenditure that will ‘improve or enhance the asset.
• cash inflows and outflows from financing activities (means interest payments) (because the cash flows are
later discounted to present values using a discount rate that takes into account the time-value of money); and
• Cash flows in respect of tax receipts and tax payments (because the discount rate used to discount the cash
flows is a pre-tax discount rate).
• Non cash transactions.
• Cash flows that are expected to arise from a future restructuring to which entity is not yet committed(IAS 37)

Cash outflows that improve or enhance the performance


Until the entity incurs cash outflows that improve or enhance the asset’s performance, estimates of future cash
flows do not include the estimated future cash inflows that are expected to arise from increase in economic
benefits associated with cash outflows. Resultant outflows to improve the asset will be included in carrying
amount of asset when incurred.

The discount rate must be a pre-tax rate that reflects current market assessments of:

the time value of money; and


the risks specific to the asset for which the future cash flow estimates have not been adjusted.

Q.5 Property, plant and equipment as disclosed in the draft financial statements of Apricot Pakistan Limited (APL)
for the year ended 30 June 2018 include a plant having a carrying value of Rs. 610 million. The performance of the
plant has been deteriorating since last year which is affecting APL’s sales.

15 Page 15 of 34
“Patience plays major role in our life, never let run out your patience.”

Following information/estimates relate to the plant for the year ending 30 June 2019:

Rs. in million
Inflows from sale of product under existing condition of the plant 250
Operational cost other than depreciation 25
Depreciation 170
Expenses to be paid in respect of 30 June 2018 accruals 8

Cost of increasing the plant’s capacity 60


Additional inflows (net) expected from the upgrade 40
Interest on loan 30
Maintenance cost 15
Tax payment on profits 18

Cash flows from the plant are expected to decrease by 15% each year from 2020 and onward. The plant’s residual
value after its remaining useful life of 3 years is estimated at Rs. 100 million.

An offer has been received to buy the plant immediately for Rs. 570 million but APL has to incur the following
costs.
Rs. in million
Cost of delivery to the customer 45
Legal cost 10
Costs to re-organize the production process after 50
disposal of plant

Applicable discount rate is 9%.

Required:
Calculate the amount of impairment loss (if any) on plant for the year ended 30 June 2018

Answers:
A.1
Initial cost

Purchase price (excluding refundable sale tax: 570,000 x 100/114) (note 1) 500,000
Import duties - non-refundable 100,000
Installation costs 30,000
Fuel 45,000
Administration costs (note 2) 0
Staff party (note 2) 0
Staff training (note 2) 0

16 Page 16 of 34
Testing 10,980
Proceeds from sale of samples and by-products (13,000)
Advertising 0
Initial operating losses (note 3) 0
Estimated cost to dismantle and remove the plant at end of useful life (note 4) 27,000
Debit to the asset account 699,980

Note 1: since the company is registered under sale tax act therefore sale tax paid is refundable and should not be
capitalised.

Note 2: these costs are excluded since they are not costs directly and necessarily associated with bringing the asset
to a location and condition enabling it to be used as intended by management.

Note 3: these costs are excluded because they are incurred after the asset was brought to a location and condition
that enabled it to be used as intended by management.

Note 4: Since these costs will only be paid for at a future date, the credit entry will be to a liability account: a
certain degree of uncertainty exists regarding the Rs 70,031 of future costs, so the liability is classified as a
provision.

A.2
a) Journals
Debit Credit
2 January 20X2
Cost: plant – engine 1 500,000
Cost: plant - conveyor belt and fittings 2,000,000
Cost: plant - outer structure 800,000
Bank/ liability 4 300,000
Purchase of bottling plant
5 January 20X2
Cost: plant – engine 250,000
Cost: plant - conveyor belt and fittings 250,000
Cost: plant - outer structure 250,000
Bank/ liability 750,000
Delivery and installation: 1/3 allocated to each component
16 January 20X2
Staff training (expense) 60,000
Bank/ liability 60,000
Staff training expensed (not a ‘directly attributable cost’ in bringing the plant to a location and condition that
enabled it to be used as intended by management)

19 January 20X2
Cost: plant – engine 11,000
Cost: plant - conveyor belt and fittings 11,000
Cost: plant - outer structure 11,000
Bank/ liability 33,000

17 Page 17 of 34
Testing that plant. fully operational: 1/3 allocated to each component
21 January 20X2
Entertainment/ advertising (expense) 210,000
Bank/ liability 210,000
‘Bottling plant launch party ’ expensed
31 December 20X2
Depreciation 799 173
Accumulated depreciation: plant - engine 231 183
Accumulated depreciation: plant - conveyor belt and fittings 259,073
Accumulated depreciation: plant - outer structure 308 917

Depreciating each component of plant separately:


Engine = (1 500,000 + 250,000 + 11,000 - 500,000) / 5 years x 11/12 231,183
Conveyor belt etc = (2,000,000 + 250,000 + 11,000 - 0) / 8 years x 11/12 259,073
Outer structure = (800,000 + 250,000 + 11,000 - 50,000) / 3 years x 11/12 308,917
799173

Depreciation starts from the date that the asset was available for use i.e 1 February 20X2:

31 December 20X3
Depreciation 871 825
Accumulated depreciation: plant - engine 252 200
Accumulated depreciation: plant - conveyor belt and fittings 282 625
Accumulated depreciation: plant - outer structure 337,000
Depreciating each component of plant separately:
Engine = (1 500,000 + 250,000 + 11,000 - 500,000) / 5 years 252,200
Conveyor belt etc = (2,000,000 + 250,000 + 11,000 -0) / 8 years 282,625
Outer structure = (800,000 + 250,000 + 11,000 - 50,000) / 3 years 337,000
871,825

Comments:
There is no journal entry allocating the operating loss incurred during March 20X2 (the excess expenses incurred
over income earned) to the plant since this cost was incurred after the plant had been brought to a location and
put into a condition that enabled it to be used as intended by management.

Depreciation is calculated on each individual component separately.


Depreciation begins when the asset is available for use as intended by management.
Depreciation does not cease when the asset is temporarily idle.

18 Page 18 of 34
“Success is achieved by honest efforts but not just dream.”

b) Disclosure
ANCIENT WATERS LIMITED
NOTES TO THE FINANCIAL STATEMENTS
FOR THE YEAR ENDED 31 DECEMBER 20X3
Land Bottling Plant Total
Gross carrying amount
Opening Balance 4,000,000 5,083,000 9,083,000
Addition - - -
Deletion - - -
Closing Balance 4,000,000 5,083,000 9,083,000
Acc Depreciation
Opening Balance - 799,173 799,173
For the year - 871,825 871,825
Disposal Adjustment - - -
Closing Balance - 1,670,000 1,670,000
Carrying Amount 4,000,000 3,412,000 7,412,000

Accounting Policies:
1. PPE is stated at cost less accumulated depreciation.
2. Depreciation is provided on all assets, except land. The plant is made up of three significant components, the
cost of which is as follows:
Depreciation of component: Cost price Residual value Expected useful life
Engine 500,000 500,000 5 years
Conveyer Belt and finings 2,000,000 - 8 years
Outer structure 800,000 50,000 3 years

A.3
Carrying Amount = 600,000

Calculation of recoverable amount:


Value in use (W-1) = 555,736
FV less CTS (W-2) = 548,000
Higher of is Rs 555,736. After selection of higher figure, that figure is called as recoverable amount. As carrying
amount is Rs 600,000 therefore difference of Rs 44,264 is impairment loss which should be immediately
recognized as follows

Impairment loss 44,264


Accumulated Impairment loss 44,264

After impairment loss, revised carrying amount i.e 600,000 – 44,264 = 555,736 is depreciated over remaining
useful life.

19 Page 19 of 34
W-1 Calculation of value in use
2011 200,000 (1.1)-1 = 181,818
2012 200,000 (1.1)-2 = 165,289
2013 100,000 (1.1)-3 = 75,131
2014 100,000 (1.1)-4 = 68,301
2015 105,000 (1.1)-5 = 65,197
555,736
W-2 FV less CTS
Fair value 570,000
Less: Cost to sell (22,000)
548,000

A.5 Scientific Pharma Limited

Journal entries for the year ended June 30, 2010


Debit Credit
Rs.000 Rs.000
30.06.2010 Repair and maintenance expenses 1,500
Account payable / Bank 1,500
(Repair cost of major break down of the plant)

30.06.2010 Depreciation expense (45,000-2,000)/10.5 years 4,095


Accumulated depreciation 4,095
(Depreciation expense for the year)
30.06.2010 Revaluation surplus (10,380/10.5) 989
Retained earnings 989
(Incremental depreciation credited to retained earnings)

30.06.2010 Revaluation surplus (OCI) W 5,296


Accumulated impairment loss 5,296
(Impairment of plant due to break down)

Working
Purchase Price (800,000 x 52) 41,600,000
Installation Charges 7,000,000
Total cost 48,600,000
Depreciation till 30-6-2005 (13,980,000)
(48,600 – 2,000) x 4.5 /15
Carrying Amount 34,620,000
Revalued Amount 45,000,000
Revaluation Surplus as on 30-6-2005 10,380,000

Plant was ready for intended use as on 31 Dec, 2000 but production started on 1 April, 2001.

As per IAS-16 depreciation commenced when asset is available for use from 1 Jan, 2001.

20 Page 20 of 34
Depreciation till 30-6-2010 Transfer of Surplus till 30-6-2010
(45,000,000 – 2,000,000) ÷ 10.5 10,380,000 ÷ 10.5
4,095,238/annum 988,571/annum
4,095,238 x 5 = (20,476,190) 988,571 x 5 = (4,942,855)
(From 1-7-2005 to 30-6-2010) From 1-7-2005 to 30-6-2010

Carrying Amount as on 24,523,810 Remaining Surplus as on 5,437,145


30-6-2010 30-6-2010
(45.000.000 – 20,470,190) (10,380,000 – 4,942,855)
Recoverable Amount 19,227,000

Impairment Loss 5,296,810

As surplus is more than the impairment loss; therefore whole amount of impairment loss can be
adjusted against surplus.

Note: If an asset remains idle temporarily, it is still depreciated unless machine hour method or
production units method is used.

A.5 Computation of impairment of plant Rs. in million


Carrying value 610
Less : Recoverable amount
Value in use (W-1) 537
Fair value less cost of sell (W-2) 515

Higher of above 537


Impairment 73

W-1: Value in use 2019 2020 2021 Total


--------------- Rs. in million ---------------
Inflows from sale of product 250
Maintenance cost (15)
Operational cost other than dep. (25)

Cash flows – Undiscounted 210 178.50 151.73


PV of cash flows at 9% 193 150 117 460
PV of residual value (100 (1.09)-3) 77 77
Total value in use 537

21 Page 21 of 34
W-2: Fair value less cost to sell Rs. in million
Fair value of machine 570
Cost to sell

Cost of delivery to the customer (45)


Legal cost involved in sale agreement (10)
515

Self-Test Questions
Q.1 Roads International Limited constructed its own specially designed ‘tarring vehicle’ Details of related costs
incurred are as follows:

Description of cost: Rs Transaction date


Cost of raw materials purchased 500,000 1 February 20X2
Cost of raw materials used in construction of tarring 100,000
vehicle during June 20X2
Tests to ensure vehicle safe before brought into use 20,000 30 September 20X2
Depreciation on machinery to 31 December 20X2 200,000
Factory labor costs to 31 December 20X2 300,000

Additional information:
1) The company-owned machinery was used for a quarter of the year in the construction of the tarring vehicle.
2) 80% of the total labor costs for the year were incurred on building roads and 20% thereof were incurred in
construction of the tarring vehicle. Of the total labor cost incurred on the construction of the tarring vehicle,
an estimated 5% was as a direct result of a country-wide labor union strike during which laborers were paid
but yet did not turn up for work.
3) The vehicle was first brought into use on a contract that started on 1 November 20X2, although it was
available for use from 1 October 20X2.
4) The company uses the straight-line method to depreciate its vehicles. This vehicle is expected to be sold for Rs
7,000 at the end of its expected useful life of 5 years.

Required:
Journalize all related transactions for the year ended 31 December 20X2.

Q.2 Dominant Fertilizers has two plants. Following information is available for the purpose of impairment
testing:
(i) The remaining useful life of both plants is expected to be 3 years.
(ii) The fair values and written down values of the plants as on 31 December 2012 were as follows:

Incremental
WDV Fair value
Plants selling costs
Rupees in million
P-1 220 210 7
P-2 160 150 4

22 Page 22 of 34
“Spend your life by serving the people and doing good deeds.”

(iii) Expected cash flows from each plant in next three years are as follows:

P-1 P-2
Rs. in million
Annual inflows 105 55
Annual outflows 11 5
Sale proceeds at the end of year 3 8 3
Disposal costs at the end of year 3 2 1

(iv) Present value factor, based on a discount factor of 10%, for year 1, year 2 and year 3 are 0.909, 0.826 and
0.751 respectively.

Required: Compute impairment (if any) on each plant.

Q.3 On 1 January 2013 Delta acquired a specialized machine for its production department. The available
information is as follows:
Rupees
List price of machine 9,200,000
Freight charges 263,000
Electrical installation cost 245,000
Staff training for use of machine 351,000
Pre-production testing 193,000
Purchase of a three-year maintenance contract 528,000
Estimated residual value 175,000

Trade discount on list price 5%


Early settlement discount taken 3%
Estimated life (in machine hours) 12,000

Machine hours used during the years ended 31 December 2013, 2014 and 2015 were 2000, 3200 and 1400
respectively.

On 1 January 2015 Delta decided to upgrade the machine by adding new components at a cost of Rs. 1,753,000.
This upgrade led to a reduction in the production time per unit of goods being manufactured by the machine. The
upgrade also increased the estimated remaining life of the machine at 1 January 2015 to 8,000 machine hours and
its estimated residual value to Rs. 350,000.

Required:
For the years ended 31 December 2013, 2014 and 2015, compute the relevant amounts to be included (under each
head) in the statement of profit or loss and statement of financial position.
(15)

23 Page 23 of 34
Q.4 On 1 January 2013 Delta Limited acquired a specialized machine for its production department. The available
information is as follows:
Rupees
List price of machine 9,200,000
Freight charges 263,000
Electrical installation cost 245,000
Staff training for use of machine 351,000
Pre-production testing 193,000
Purchase of a three-year maintenance contract 528,000
Estimated residual value 175,000

Trade discount on list price 5%


Early settlement discount taken 3%
Estimated life 10 years

Delta Limited is using straight line method.

On 1 January 2015 Delta decided to upgrade the machine by adding new components at a cost of Rs. 1,753,000.
This upgrade led to a reduction in the production time per unit of goods being manufactured by the machine. The
upgrade increased its estimated residual value to Rs. 350,000. The company has now decided to use sum of years
digit method.

Required:
Prepare the following ledgers for the year ended 31.12.2013,2014 and 2015:
a) Machinery Account (7)
b) Accumulated depreciation account (7)

Q 5: Rooney has recently finished building a new item of plant for its own use. The item is a press for use in the
manufacture of industrial diamonds. Rooney commenced construction of the asset on 1st April 2013 and
completed it on 1st April 2015. The cost of manufacturing the asset were Rs. 30,800,000. The cost of the hydraulic
system is 30% of the total cost of manufacture.

The press comprises two significant parts, the hydraulic system and the ‘frame’. The hydraulic system has a three-
year life and the ‘frame’ has an eight-year life. Rooney depreciates plant on a straight line basis.

Rooney uses the IAS 16 revaluation model in accounting for diamond presses and revalue these assets on an
annual basis.

Revaluation surpluses or deficits are apportioned between the hydraulic system and the ‘frame’ on the basis of
their year-end book values before the revaluation.

Required: Explain the IAS 16 rules on accounting for significant parts of property, plant and equipment and show
the accounting treatment of the diamond press in the financial statements for the financial years ending:
(i) 31st March 2016 (assume that the press has a fair value of Rs. 21 million)
(ii) 31st March 2017 (assume that the press has a fair value of Rs. 19.6 million).

24 Page 24 of 34
“In difficult situations person needs help but not just advice.”

Q.6 ABA LIMITED


Aba Limited conducts its activities from two properties, a head office in the city centre and a property in the
countryside where staff training is conducted. Both properties were acquired on 1 April 2013 and had estimated
lives of 25 years with no residual value. The company has a policy of carrying its land and buildings at current
values. However, until recently property prices had not changed for some years. On 1 October 2015 the properties
were revalued by a firm of surveyors. Details of this and the original costs are:
Land Buildings

Rs. Rs.
Head office – cost 1 April 2013 500,000 1,200,000
– revalued 1 October 2015 700,000 1,350,000
Training premises – cost 1 April 2013 300,000 900,000
(property) – revalued 1 October 2015 350,000 600,000

The fall in the value of the training premises is due mainly to damage done by the use of heavy equipment during
training. The surveyors have also reported that the expected life of the training property in its current use will only
be a further 10 years from the date of valuation. The estimated life of the head office remained unaltered.

Note: Aba Limited treats its land and its buildings as separate assets. Depreciation is based on the straight-line
method from the date of purchase or subsequent revaluation.

Required: Prepare extracts of the financial statements of Aba Limited in respect of the above properties for the
year to 31 March 2016.

Q.7 Fam had the following tangible fixed assets at 31 December 2014.

Cost Depreciation NBV


Rs. 000 Rs. 000 Rs. 000
Land 500 — 500
Buildings 400 80 320
Plant and machinery 1,613 458 1,155
Fixtures and fittings 390 140 250
Assets under construction 91 - 91

2,994 678 2,316

In the year ended 31 December 2015 the following transactions occur.


(i) Further costs of Rs. 53,000 are incurred on buildings being constructed by the company. A building costing
Rs. 100,000 is completed during the year.
(ii) A deposit of Rs. 20,000 is paid for a new computer system which is undelivered at the year end.
(iii) Additions to plant are Rs. 154,000.
(iv) Additions to fixtures, excluding the deposit on the new computer system, are Rs. 40,000.
(v) The following assets are sold.
25 Page 25 of 34
Cost Depreciation brought forward Proceeds
Rs. 000 Rs. 000 Rs. 000
Plant 277 195 86
Fixtures 41 31 2
(vi) Land and buildings were revalued at 1 January 2015 to Rs. 1,500,000, of which land is worth Rs. 900,000.
The revaluation was performed by Messrs Jackson & Co, Chartered Surveyors, on the basis of existing use
value on the open market.
(vii) The useful economic life of the buildings is unchanged. The buildings were purchased ten years before the
revaluation.
(viii) Depreciation is provided on all assets in use at the year end at the following rates.
Buildings 2% per annum straight line (which means total life is 50 years)
Plant 20% per annum straight line (which means total life is 5 years)
Fixtures 25% per annum reducing balance

Required
Show the disclosure under IAS 16 in relation to fixed assets in the notes to the published financial statement for
the year ended 31 December 2015. [QB#7.5]

Solutions to self test questions:


A.1
a) Journals
Debit Credit
Year ended 31 December 20X2:
1 February 20X2
Raw materials (asset) 500,000
Bank/ liability 500,000
Purchase of raw materials

30 June 20X2
Tarring vehicle (asset) 100,000
Raw materials (asset) 100,000
Raw materials used on the construction of the tarring vehicle

30 September 20X2
Tarring vehicle (asset) 20,000
Bank/ liability 20,000
Safety test performed before vehicle could be brought into use

31 December 20X2
Depreciation - machinery (expense) 150,000
Tarring vehicle (asset) 50,000
Accumulated depreciation: machinery 200,000
Depreciation of machinery
Allocation of 25% of the depreciation on machinery to the construction of the
tarring vehicle: 200,000 x 25%=50,000

26 Page 26 of 34
The labour costs paid over the course of 20X2 would have totaled:
Labour costs (expense) 243,000
Tarring vehicle (asset) 57,000
Bank 300,000
Allocation of 20% of labour costs (excluding 5% unnecessary wastage due
to strike) to construction of tarring vehicle: 300,000 x 20% x 95% = 57,000
31 December 20X2
Depreciation - tarring vehicle 11,000
Accumulated depreciation: tarring vehicle 11,000
Tarring vehicle depreciated since first available for use:
(100,000 + 20,000 + 50,000 + 57,000 — 7,000) / 5 years x 3/12

A.2 Solution
P-1 P-2
Rs. in million
Fair Value as given 210.00 150.00

Less: Incremental Selling Cost (7.00) (4.00)


Fair value less incremental selling cost 203.00 146.00

Value in use (W-l) 238.19 125.80

Recoverable Value (Being higher of “value in use” and “fair


value less incremental selling cost”) 238.19 146.00

Written down value 220.00 (160.00)


Impairment loss - (14.00)

Working

W-l: Value in use


P-1 P-2
Year 1 Year 2 Year 3 Year 1 Year 2 Year 3
Rs. in million
Expected cash inflows each year 105 105 105 55 55 55
Expected outflows per year (ID (ID (11) (5) (5) (5)
Sale proceeds - - 8 - - 3
Disposal costs - - (2) - - (1)
Net cash flows 94 94 100 50 50 52
Present value factor 0.909 0.826 0.751 0.909 0.826 0.751
Present value of net cash flows 85.45 77.64 75.1 45.45 41.3 39.05
Total value in use 238.19 125.8

27 Page 27 of 34
A.3

Year ended 31-Dec-13 31-Dec-14 31-Dec-15


--------------- Amount in Rs. ---------------

Operating expenses (W-3) (1,720,333) (2,646,934) (1,371,028)


[2,470,933+176,0 [1,195,028+17
[1,544,333+176,000] 00] 6,000]
Other income (Discount received i.e.
(8,740,000×3%)) 262,200 - -
Administration expenses
(Staff training) (351,000) - -
31-Dec-13 31-Dec-14 31-Dec-15
Statement of financial position: --------------- Amount in Rs. ---------------
Non-Current Assets:
Property, plant and equipment (W-4) 7,896,667 5,425,733 5,983,705
Long term prepayments 176,000
Current Assets:
Short term prepayments 176,000 176,000

Not required for extra information:


Machine Account
1-1-2013 Cash 9,441,000
31-12-2013 c/d 9,441,000
1-1-2014 b/d 9,441,000
31-12-2014 c/d 9,441,000
1-1-2015 b/d 9,441,000
1-1-2015 Cash 1,753,000
31-12-2015 c/d 11,194,000

Accumulated Depreciation Account


b/d -
Depreciation 1,544,333
c/d 1,544,333
b/d 1,544,333
Depreciation 2,470,934
c/d 4,015,267
b/d 4,015,267
Depreciation 1,195,028
c/d 5,210,295

28 Page 28 of 34
“Don’t break heart of anyone but do your best to mend the broken hearts.”

A.4
Machine Account
1-1-2013 Cash 9,441,000
31-12-2013 c/d 9,441,000
1-1-2014 b/d 9,441,000
31-12-2014 c/d 9,441,000
1-1-2015 b/d 9,441,000
1-1-2015 Cash 1,753,000
31-12-2015 c/d 11,194,000

Accumulated Depreciation Account


b/d -
Depreciation 926,600
c/d 926,600 [9,441,000-175,000] / 10
b/d 926,600
Depreciation 926,600
c/d 1,853,200
b/d 1,853,200
Depreciation (working) 1,997,956
c/d 3,851,156

Workings:
WDV on 01.01.2015 = [9,441,000 – (926,600 x 2)] = 7,587,800
Year
Year Factor
2015 1 8/36 [7,587,800 + 1,753,000] – 350,000 x 8/36 = 1,997,956
2016 2 7/36 [7,587,800 + 1,753,000] – 350,000 x 7/36 = 1,748,211
2017 3 6/36 [7,587,800 + 1,753,000] – 350,000 x 6/36 = 1,498,467
2018 4 5/36 [7,587,800 + 1,753,000] – 350,000 x 5/36 = 1,248,722
2019 5 4/36 [7,587,800 + 1,753,000] – 350,000 x 4/36 = 998,978
2020 6 3/36 [7,587,800 + 1,753,000] – 350,000 x 3/36 = 771,733
2021 7 2/36 [7,587,800 + 1,753,000] – 350,000 x 2/36 = 499,488
2022 8 1/36 [7,587,800 + 1,753,000] – 350,000 x 1/36 = 249,744
36

Answer5
The IAS-16 rule on accounting for significant parts of Property, Plant and Equipment is as under:

As per IAS-16, each part of an item (that has a cost which is significant in relation to total cost) is depreciated
separately. Therefore, cost recognized at initial recognition must be allocated to each part accordingly.

29 Page 29 of 34
i) a) 31.3.2016 Rs. In ‘000’
Hydraulic System Frame Total

9,240 21,560 30,800


Cost (1.4.2015) (30%) (70%)
Depreciation (3,080) (2,695) (5,775)
[9,240 / 3] [21,560 / 8]
Carrying Amount on 6,160 18,865 25,025
31.3.2016
Fair Value 21,000
Revaluation Loss (P.L) 991 (w-1) 3,034 (w-1) 4,025
Revised Carrying 5,169 15,831 21,000
Amount on 31.3.2016

(W-1) Allocation of Revaluation Loss :


Hydraulic System : 6,160 / 25,025 x 4,025 = 991
Frame : 18,865 / 25,025 x 4,025 = 3,034

After revaluation , hydraulic system will be depreciated over remaining life of two years and the frame over seven
years from the next year onwards.

b) 31.3.2017
Carrying Amount 5,169 15,831 21,000
1.4.2016
Depreciation for 2017 (2,585) (2,262) (4,847)
[5,169 / 2] [15,831 / 7]
Carrying Amount on 2,584 13,569 16,153
31.3.2017
Fair value ` 19,600
Revaluation Surplus on 551 (w-2) 2,896 (w-2) 3,447
Revised Carrying 3,135 16,465 19,600
Amount on 31.3.2017

(W-2) Allocation of revaluation surplus:


Hydraulic System : 2,584 / 16,153 x 3,447 = 551
Frame: 13,569 / 16,153 x 3,447 = 2,896
However, this revaluation surplus is after the revaluation loss in previous year, therefore we have to calculate
reversal of loss to profit and loss and balancing figure (if any) of revaluation surplus.

30 Page 30 of 34
(w-2.1) Hydraulic System:
Revaluation Surplus 551

Reversal of Loss 991 Cr Revaluation Surplus 55


Depreciation to be charged [3,080-2,585] 495 Dr
Net Reversal of loss 496 Cr.

(w -2.2) Frame:
Revaluation Surplus 2,896

Loss to be Reversed 3,034 Cr Revaluation Surplus 295


Depreciation to be charged (2,695 – 2,262) 433 Dr
Net reversal of loss 2,601 Cr

A. 6 Aba Limited
Aba Limited statement of profit or loss (extracts) – year to 31 March 2016

Note: workings in brackets are in Rs.000 Rs. Rs.


Depreciation: head office – 6 months to 1 October
2015
(1,200/25 x 6/12) 24,000
– 6 months to 31 March
2016
(1,350/22.5 (W1) x 6/12) 30,000
–––––––
54,000
–––––––
Depreciation: training premises
– 6 months to 1 October
2016
(900/25 x 6/12) 18,000
– 6 months to 31 March
2016
(600/10 x 6/12) 30,000
––––––––
48,000
––––––––
Revaluation loss (W2) 210,000
––––––––
258,000

31 Page 31 of 34
Statement of financial position (extracts) as at
31 March 2016 Rs. Rs.
Non-current assets
Land and buildings – head office (700 + 1,350 – 30) 2,020,000
– training premises (350 + 600 –30) 920,000
––––––––
2,940,000
––––––––
Revaluation reserve
Head office land (700 – 500) 200,000
Building (1,350 – 1,080 – 6,000) (W1)) 264,000
Training premises land (350 – 300) 50,000
––––––––
514,000

Working:
Transfer to realised profit (270/22.5 (W1) x 6/12
(6,000)
––––––––
514,000
Workings
(W1) The date of the revaluation is two and a half years after acquisition. This means the remaining life of the
head office would be 22.5 years. The carrying value of the head office building at the date of revaluation is
Rs. 1,080,000 i.e. its cost less two and a half years at Rs. 48,000 per annum (Rs. 1,200,000 – Rs. 120,000).
(W2) Revaluation loss: the carrying value of training premises at date of revaluation is Rs. 810,000 i.e. its cost less
two and a half years at Rs. 36,000 per annum (Rs. 900,000 – Rs. 90,000). It is revalued down to Rs. 600,000
giving a loss of Rs. 210,000. As the land and the buildings are treated as separate assets the gain on the land
cannot be used to offset the loss on the buildings.

A.7 FAM
Accounting policies
(a) Property, plant and equipment is stated at historical cost less depreciation, or at valuation.
(b) Depreciation is provided on all assets, except land, and is calculated to write down the cost or valuation over
the estimated useful life of the asset.

The principal rates are as follows.


Buildings 2% pa straight line
Plant and machinery 20% pa straight line
Fixtures and fittings 25% pa reducing balance

32 Page 32 of 34
Fixed asset movements (disclosure of Property, Plant & Equipment)
For the year ended 31-12-2013
Land Building Plant and Fixtures, Assets in the Total
Gross Carrying Amount Machinery fittings, course of
tools and construction
equipment

Balance as on 1 January 2015 500 400 1,613 390 91 2,994


Revaluation adjustment 400 280 - - - 680
Acc Depreciation Adjustment - (80) - - - (80)
Additions - - 154 40 53 247
Reclassifications - 100 - - (100) -
Disposals - - (277) (41) - (318)

Balance as on 31 December 900 700 1,490 389 44 3,523


2015
Accumulated Depreciation
Balance as on 1 January 2015 - 80 458 140 - 678
Revaluation adjustment - (80) - - - (80)
Depreciation - 17 298 70 - 385
Disposals - - (195) (31) - (226)

Balance as on 31 December - 17 561 179 - 757


2015
Carrying Amount
At 31 December 2015 900 683 929 210 44 2,766

At 31 December 2014 500 320 1,155 250 91 2,316

Disclosure of Revaluation:
• Name of revaluer (if available) Messer Jackson & Co
• Basis of revaluation Existing use value in open market
• Carrying amount of the revalued assets at the reporting date if those assets were carried at cost model.

Land as on 31-12-2015
Carrying amount 500
Building as on 31-12-2015
Carrying amount
Cost 400
Less Acc depreciation (80+8) (88)
312

33 Page 33 of 34
Addition
Cost 100
Less Acc Depreciation (2)
98
Carrying amount 410
Land and Building as on 31-12-2015 910

WORKINGS
Rs.000
1. Depreciation on buildings 600 + (100 x 2%) 17
40
2% straight line depreciation is equivalent to a 50 year life. The buildings are ten
years old at valuation and therefore have 40 years remaining.
2. Depreciation on plant (1,613 + 154 - 277) x 20% 298
3. Depreciation on fixtures (250-10)x 25% + 40 x 25% 70

34 Page 34 of 34
“The journey of millions of kilo meters begins with one step.”

Extra Practice [Impairment of assets]

EXAMPLE 01: HUSSAIN ASSOCIATES LTD


The assistant financial controller of the Hussain Associates Ltd has identified the matters below which he believes may
indicate impairment of one or more assets:
Hussain Associates Ltd owns and operates an item of plant that cost Rs. 640,000 and had accumulated
depreciation of Rs. 400,000 at 1 October 2015. It is being depreciated at 12½% on cost.

On 30 Sep 2016 the plant was damaged when a factory vehicle collided into it. Due to the unavailability of replacement
parts, it is not possible to repair the plant, but it still operates, albeit at a reduced capacity. It is also expected that as a
result of the damage the remaining life of the plant from the date of the damage will be only two years.

Based on its reduced capacity, the estimated present value of the plant in use is Rs. 150,000. The plant has a current
disposal value of Rs. 20,000 (which will be nil in two years’ time), but Hussain Associates Ltd has been offered a trade-
in value of Rs. 180,000 against a replacement machine which has a cost of Rs. 1 million (there would be no disposal
costs for the replaced plant). Hussain Associates Ltd is reluctant to replace the plant as it is worried about the long-
term demand for the product produced by the plant. The trade-in value is only available if the plant is replaced.

Required; discuss the above situation with respect to extracts from the statement of financial position and
statement of profit or loss of Hussain Associates Ltd for the year ended 30 September 2016.

Answer:
The plant had a carrying amount of Rs. 240,000 (640,000-400,000) on 1 October 2015.

The depreciation on the plant from 1 October 2015 to 30 Sept 2016 would be Rs. 80,000 (640,000 x 12.5% giving a
carrying amount of Rs. 160,000 at the date of impairment. An impairment test requires the plant’s carrying amount
to be compared with its recoverable amount. The recoverable amount of the plant is the higher of its value in use of Rs.
150,000 or its fair value less costs to sell.

If Hussain Associates Ltd trades in the plant it would receive Rs. 180,000 by way of a part exchange, but this is conditional
on buying new plant which Hussain Associates Ltd. is reluctant to do. A more realistic amount of the fair value of the
plant is its current disposal value of only Rs. 20,000.

35
Page 1 of 3
Thus the recoverable amount would be its value in use of Rs. 150,000 giving an impairment loss of Rs. 10,000 (Rs.
160,000 – Rs. 150,000). Thus extracts from the financial statements for the year ended 30 September 2016 would
be:

Statement of financial position


Non-current assets Rs.
Plant 150,000
Statement of profit or loss
Plant depreciation 80,000
Plant impairment loss 10,000

EXAMPLE 3: PREMIER LIMITED (PL)


Question: Premier Limited (PL) owns a plant which has a carrying amount of Rs.248 million as at 1 April 2019. It is
being depreciated at 12½% per annum on a reducing balance basis.

The plant is used to manufacture a specific product which has been suffering a decline in salesdue to obsolescence.
PL has estimated that the plant will be retired from use on 31 March 2023.

The estimated net cash flows from the use of the plant and their present values are:

Net cash Present


flows values
Rs. In millions
Year to 31 March 2021 120 109.2
Year to 31 March 2022 80 66.4

Year to 31 March 2023 52 39

252 214.6

On 1 April 2020, PL had an alternative offer from the competitor to purchase the plant forRs.200 million.

Required: Calculate the impairment loss as on 31.03.2020.

Answer:
At 31 March 2020
Recoverable amount is the higher of value in use [PV of future net cash flows (Rs.214.6 million) and fair value less
costs of disposal (Rs.200 million)].
Carrying amount = Rs.217 million [248 m – (248 m x 12·5%)] Impairment loss = Carrying
amount – Recoverable amount
= Rs.217 million – Rs.214.6 million
= Rs.2.4 million

36
Page 2 of 3
EXAMPLE 04: NAVEED LIMITED
Question: Naveed Limited has an item of plant which has a carrying value of Rs.1, 800,000 as atthe end of the year
December 2020. It has undergone an impairment review and the following estimates were produced:

Fair value of plant = Rs.1, 400,000 Costs to sell 2%


of selling price

Revenue and associated costs per annum for remaining useful life:
(Assume all cash flows occur at the end of the year).

Revenue Costs
2021 Rs.960,000 Rs.240,000
2022 Rs.880,000 Rs.220,000
2023 Rs.700,000 Rs.290,000

The plant has an estimated residual value of Rs.50, 000.

A discount rate of 10% is applicable to investments equivalent in risk to this plant.

Required:
Calculate the impairment loss if the appropriate discount rate is 10%.

Answer:

Cash flows
2021 2022 2023
--------------- Amount in Rs. --------------
Revenue 960,000 880,000 700,000

Costs (240,000) (220,000) (290,000)


Net Cash Inflow 720,000 660,000 410,000
Discount factor 0.909 0.826 0.751
Present Value 654,545 545,457 308,037

Residual Value (50,000 x 0.751) = 37,566

Value in use = 654,545 + 545,457 + 308,037 + 37,566 = Rs.1, 545,605

Fair value less cost to sell = Rs.1, 400,000 – 2% of Rs.1.4 million = Rs.z1, 372,000Recoverable amount = Rs.
1,545,605
Carrying value = Rs.1, 800,000
Impairment loss = Rs.254, 395

37
Page 3 of 3
“Keep your thoughts and attitude positive, your life will be awesome.”

Property, Plant & Equipment [PPE] (IAS 16)

CHANGE IN ACCOUNTING ESTIMATES:


Whenever there is change in accounting estimates of useful life or residual value or there is a change in method of
depreciation then the WDV of the asset at the time of change is depreciated over the future periods. Depreciation
of previous periods is not adjusted. Review of useful life

Example:
Chiniot Engineering owns a machine which originally cost Rs. 60,000 on 1 January 2012.

The machine was being depreciated over its useful life of 10 years on a straight line basis and has no residual value.

On 1 January 2015 Chiniot Engineering revised the total useful life for the machine to eight years (down from the
previous 10 years).

Required
Calculate the depreciation charge for 2015 and subsequent years.

Solution:
The change in accounting estimate is to be applied to the financial statements from 2015 onwards.

Cost on 1 January 2012 60,000


Depreciation for 2012 to 2014 (60,000 × 3/10) (18,000)
–––––––
Carrying amount at end of 2014 42,000
–––––––
Remaining useful life at the end of 2014 = 8 – 3 years = 5 years.

Depreciation for 2015 and subsequent years = Rs. 42,000 ÷ 5 years = Rs. 8,400.

Review of Residual value

Example
A machine was purchased three years ago on 1 January, 2012. It cost Rs 150,000 and its expected life was 10 years
with an expected residual value of Rs 30,000.

Due to technological changes, the estimated life of the asset was reassessed during 2015. The total useful life of
the asset is now expected to be 7 years and the machine is now considered to have no residual value.

The financial year of the entity ends on 31 December.

38
Page 1 of 8
What is the depreciation charge for the year ending 31 December 2015?

Solution
Original depreciation = (150,000 – 30,000) /10 = Rs 12,000 per annum
Carrying amount at start of year 2015 = 150,000 – (12,000 x 3) = Rs 114,000
If the total useful life is anticipated to be 7 years then there are four years remaining.
Depreciation charge for year 2015 =Rs 114,000 /4 = Rs 28,500

Review of depreciation method

Example:
Marden Fabrics owns a machine which originally cost Rs. 30,000 on 1 January 2012. It has no residual value.

It was being depreciated over its useful life of 10 years on a straight-line basis. During 2015, Marden Fabrics
decided to change the method of depreciation, from straight-line to the reducing balance method, using a rate of
25%.

Required
Calculate the depreciation charge for 2015.

Answer
Cost on 1 January 2012 30,000
Depreciation for 2012 to 2014 (30,000 × 3/10) (9,000)
–––––––
Carrying amount at end of 2014 21,000
–––––––
Depreciation for 2015 will therefore be Rs. 21,000 × 25% = Rs. 5,250

Capitalization rule:
All amounts incurred to bring the asset into working condition as intended by management are added to the cost
of the asset.

Q. 1 ABC Limited imported technical equipment costing Rs. 3 million on July 1, 2003. It further incurred the
following expenses on the equipment:
• Import duty Rs. 1,000,000
• Income tax Rs. 276,000 adjustable against company’s income tax liability
• Other non-refundable taxes Rs. 60,000
• Transportation cost Rs. 10,000 to bring the equipment to factory premises
• Insurance in transit Rs. 4,000
• Fire insurance Rs. 10,000

Initially the useful life was estimated to be 5 years and depreciation was provided on straight line basis. The
estimated salvage value was Rs. 350,000.

39
Page 2 of 8
“Never do cheating with anyone in your life.”

During the year 2004-05 the company estimated the remaining life of the equipment to be five years instead of
four years. The salvage value was re-estimated at Rs. 400,000.

The equipment was sold on July 1, 2006 for Rs. 2,800,000.

Required:
• Calculate depreciation expense for the years ended June 30, 2004, 2005 and 2006.
• Pass journal entry to record the sale of equipment.

Q.2 The written down value of plant and machinery of Azfar and Company as at 30 June 2011 is Rs. 831,128.
Following additional information is also available:
1. On 1 July 2007, second-hand machinery was purchased for Rs. 300,000. An amount of Rs. 200,000 was spent
on its overhauling, before use.
2. On 1 January 2008 machinery costing Rs. 250,000 was purchased.
3. The machinery purchased on 1 July 2007 became obsolete and was sold for Rs. 100,000 on 1 January 2010. On
the same date, new machinery was purchased at a cost of Rs. 600,000.
4. Machinery purchased on 1 January 2008 was sold on 30 June 2011 at its book value plus Rs. 50,000.

Azfar and Company provides depreciation on machinery @ 15% on written down value. Depreciation on addition /
deletion is provided in proportion to the period of use.

Required:
a) Machinery Account from 1 July 2009 to 30 June 2011
b) Machinery Disposal Account for the years ended 30 June 2010 and 2011
(22 marks)

Q.3 Naveed Enterprises commenced business on 01 July 2009. Certain information about their vehicles, for the
years ended 30 June 2011 and 2012 can be ascertained from the following ledger accounts:

Accumulated depreciation on vehicles


28-02-11 Vehicle disposal account 435,467 01-07-10 Balance b/d 1,360,000
30-06-11 Balance c/d 2,160,800 30-06-11 Dep. for the year 1,236,267
2,596,267 2,596,267

30-04-12 Vehicle disposal account 560,000 01-07-11 Balance b/d 2,160,800


30-06-12 Balance c/d 3,025,040 30-06-12 Dep. for the year 1,424,240
3,585,040 3,585,040

40
Page 3 of 8
Vehicle disposal account
28-02-11 Cost at 01-07-2009 Profit 1,420,000 28-02-11 Accumulated Dep. Cash 435,467
28-02-11 on disposal 165,467 28-02-11 received 1,150,000
1,585,467 1,585,467
30-04-12 Cost at 01-07-2009
30-04-12 Accumulated Dep. Cash 560000
1,200,000 30-04-12 received 500000
30-04-12 Loss on disposal 140,000
1,200,000 1,200,000

Following further information is available in respect of the vehicles for the last three years (01-07-2009 to 30-06-
2012):
1) Depreciation is being provided at the rate of 20% per annum on diminishing balance method.
2) Accumulated depreciation brought down on 1 July 2010 represents depreciation for the whole year on
vehicles bought on 1 July 2009.
3) Two vehicles were purchased on 1 November 2010 and 1 September 2011.

Required: Prepare Vehicles (Asset) Account for the years ended 30 June 2011 and 2012.

Answers:
A.1
(a) Depreciation:
Cost = 4,074,000
Residual value = 350,000
Useful life = 5 years
4,074,000 − 350,000
Depreciation = = 744,800
5
Depreciation for the year ended 30-06-2005:
[4,074,000 − 744,800] − 400,000
=
5
= 585,840

Since straight line method is being used and there is no change in estimates during 2006; depreciation for the year
ended 30-06-2006 will also be same i.e. 585,840.

(b) WDV of Machine at the time of Disposal


Cost = 4,074,000
Accumulated depreciation (744,800 + 585,840 + 585,840) = 2,157,520
Written down value = 2,157,520
Entry of Disposal:
Accumulated depreciation 1,916,480
Cash / Bank 2,800,000
Equipment 4,074,000
Gain on disposal 642,480

41
Page 4 of 8
Ledgers of all relevant years (not required for extra information)
Equipment Account
Date Particular Rs. Date Particular Rs.
1-7-2003 Cash 4,074,000
30.06.2004 c/d 4,074,000
1-7-2004 b/d 4,074,000
30.06.2005 c/d 4,074,000
1-7-2005 b/d 4,074,000
30.06.2006 c/d 4,074,000
1-7-2006 b/d 4,074,000 1-7-2006 Disposal 4,074,000
c/d ----

Accumulated Depreciation
Date Particular Rs. Date Particular Rs.
30-6-2004 Depreciation 744,800
30.06.2004 c/d 744,800
1-7-2004 b/d 744,800
Depreciation 585,840
30.06.2005 c/d 1,330,640
1-7-2005 b/d 1,330,640
Depreciation 585,840
30.06.2006 c/d 1,916,480
1-7-2006 Disposal A/c 1,916,480 1-7-2006 b/d 1,916,480
30.06.2007 c/d ----

A.2
Machinery Account – At WDV
Date Particular Rs. Date Particular Rs.
1-7-2009 b/d (W-3) 1,055,222 1-1-2010 Disposal (W-1) 334,156
1-1-2010 Cash 600,000 Depreciation (bal) 176,190
30-6-2010 c/d 1,144,876

b/d
1-7-2010  831,128   1,144,876 30-6-2011 Depreciation 171,732
 85  100  + 167 ,078 
  
30-6-2011 Disposal (W-2) 142,016
30-6-2011 c/d 831,128

42
Page 5 of 8
(b)
Disposal Account
Date Particular Rs. Date Particular Rs.
1-1-2010 Machinery A/c 334,156 1-1-2010 Cash 100,000

Loss (bal) 234,156

Disposal Account
Date Particular Rs. Date Particular Rs.
1-1-2010 Machinery A/c 142,016 1-1-2010 Cash (142,016 + 50,000) 192,016

Gain 50,000

WORKINGS:
(W-1) WDV of Asset Disposed of on 1-1-10
Purchase date = 1-7-2007
Cost = 500,000
× 15% = (75,000)
As on 30-6-2008 = 425,000
× 15% = (63,750)
As on 30-6-2009 = 361,250
× 15% × 6/12 = (27,094)
As on 1-1-2010 = 334,156

(W-2) WDV of Asset Disposed of on 30-6-11


Purchase date = 1-1-2008
Cost = 250,000
× 15% × 6/12 = (18,750)
As on 30-6-2008 = 231,250
× 15% = (34,688)
As on 30-6-2009 = 196,562
× 15% = (29,484)
As on 1-1-2010 = 167,078
× 15% = (25,062)
As on 1-1-2010 = 142,016

(W-3) Opening WDV of First Year


Closing WDV = 1,144,876 however this includes additions
 1,144,876 − WDV of additons 
  100 + WDV of disposal as on 1-7-2009
 85 

1,144,876 − (600,000 − 600,000  15%  6 / 12)


[ × 100] + 361,250 = 1,055,222
85

43
Page 6 of 8
“Strive to bring goodness in your behavior and character.”

A.3 Vehicle Account – At Cost


Date Particular Rs. Date Particular Rs.
1-7-2010 b/d 6,800,000 28-2-2010 Disposal A/c 1,420,000
1-11-2010 Cash 1,680,000
30.06.2011 c/d 7,080,000
1-7-2011 b/d 7,060,000 30-4-2012 Disposal A/c 1,200,000
1-9-2011 Cash 2,820,000
30.06.2012 c/d 8,680,000

(W-1) Vehicles – Opening Balance


1-7-2010 (1,360,000 ÷ 20%) = 6,800,000

(W-2) Cost of Additions of Vehicles


2011 2012
Depreciation for the year 1,236,267 1,424,240
On deletions: (1,420,000 × 80% × 20% × 8/12) (151,467)
On deletions (working below) (128,000)

WDV
1,200,000 × 20% = 240,000 960,000
× 20% = 192,000 768,000
×20% = 128,000

On other b/f balances (860,800)*


On other b/f balances (826,240)**

Balance being depreciation on additions during the 224,000 470,000


year
Cost of vehicle purchased on 1-11-2010 (working 1,680,000
below)
Cost of vehicles purchased on 1-9-2011(working below) 2,820,000

44
Page 7 of 8
Workings:

*[(6,800,000 – 1,360,000) – 1,136,000*] × 20% = **[(7,060,000 – 2,160,800) – 768,000*] × 20% =


860,800 826,240
*2009 1,420,000 *1,200,000 x 20% = 240,000
% (284,000) 960,000 x 20% = 192,000
2010 1,136,000 Total = 432,000
8/12 151,467 1,200,000 – 432,000 = 768,000
Depreciation on additions: Depreciation on addition:
Cost × 20% × 8/12 = 224,000 Cost × 20% × 10/12 = 470,000
Cost = 1,680,000
Cost = 2,820,000

45
Page 8 of 8
“Don’t stress nor consider your life as burden, no matter what.”

Exchange of Non-Monetary Asset


1. If fair value (i.e., its cost) of only new asset received is available or is more reliably measureable then measure
the new asset at its fair value.
2. If fair value of new asset received as well as fair value of old asset given up both are reliable measureable or
only fair value of old asset given up is reliably measureable, then measure the new asset received at fair value
of asset given up plus cash paid minus cash received (if any)

Example 1:
Book value of old machine 8,000 (comprising of cost of 12,000 and accumulated depreciation of 4,000).
Fair value of machine given up = 6,000
List price of new machine = 16,000
Trade in allowance for old machine = 9,000

(It may not be the same as fair value of old machine because supplier may be charging an inflated price for his new
machine).

Cash paid (16,000 – 9,000) = 7,000

Required:
Prepare accounting entry to record gain / loss on exchange

Answer:
New machine (6,000 + 7,000) 13,000
Acc. Depreciation 4,000
Loss on exchange (P&L) 2,000
Old machine 12,000
Cash 7,000

3. If fair value of both the asset received or given up are not reliably measureable or are not available then
measure the new asset at carrying amount of asset given up plus cash paid minus cash received.

Comprehensive Example:
Following information pertains to three exchange transactions relating to fixed assets:
--------- Rs. in million ---------
1 2 3
Cash received/(paid) 1.1 (2.1) -
Assets given-up:
Original cost 10.3 12.4 14.5
Book value 6.4 7.3 3.4
Estimated fair value 8.5 6.6 4.6
Assets received:
Estimated fair value 7.1 9.0 4.1

46
Page 1 of 5
Additional information:
i. In case of transaction (i), fair values of both assets are reliably measurable.

ii. In case of transaction (ii), fair value of the asset received is clearly more evident.
iii. In case of transaction (iii), fair value of neither asset is reliably measurable.

Required:
Prepare accounting entries in respect of the above exchange transactions. (06)

Answer
(i)
New asset (8.5-1.1) 7.4
Cash 1.1
Old asset 6.4
Gain (P.L) 2.1

(ii)
New asset 9.0
Loss (P.L) 0.4
Cash 2.1
Old asset 7.3

(iii)
New asset (3.4+2) 5.4
Cash 3.4
Old asset 2.0

47
Page 2 of 5
Practice question
Q.1 Following information pertains to plant and machinery of Alpha Enterprises (AE):

(i) As at 1 January 2018, balances of cost and accumulated depreciation amounted to Rs. 12,700,000
and Rs. 6,240,000 respectively.

(ii) On 1 April 2018, an old machine having fair value of Rs. 340,000 was exchanged for a new machine.
The balance of the purchase price was paid through a cheque of Rs. 680,000. The list price of the
new machine was Rs.1,130,000. The old machine had been acquired for Rs. 870,000 on 1 September
2015.

(iii) On 1 February 2018, a plant having a list price of Rs. 10,000,000 was acquired. A trade discount of
5% was allowed on the list price. The plant was ready for use on 1 August 2018 after incurring the
following costs:
Rs. in '000
Freight charges 660
Consultant fees 540
Installation and testing 600
Administration and other general overheads 160
Staff training 120
Opening ceremony 100
2,180

(iv) On 31 October 2018, another machine was sold for Rs. 334,000. It was acquired on 1 January
2015 and had a net book value of Rs. 512,000 on 1 January 2018. A cost of Rs. 25,000 was
incurred on its disposal.

(v) AE depreciates plant and machinery at 20% per annum using the reducing balance method

Required:

Prepare following ledger accounts pertaining to the plant and machinery for the year ended 31 December
2018:
(a) Cost (06)
(b) Accumulated depreciation (06)
(c) Assets disposal (04)

48
Page 3 of 5
A.1 Alpha Enterprises

Plant and machinery - Cost


Date Description Rs. in '000 Date Description Rs. in '000
1-Jan-2018 Balance 12,700 1-Apr-2018 Assets disposal 870
1-Apr-2018 Bank 680 31-Oct-2018 Assets disposal (W-2) 1,000
Disposal 340
1,020
1-Aug-2018 Cash (W-4) 11,300 31-Dec-2018 Balance 23,150
25,020 25,020

Accumulated depreciation - Plant and machinery


1-Apr-2018 Assets disposal (W-1) 376 1-Jan-2018 Balance 6,240
31-Oct-2018 Assets disposal (W-2) 573 31-Dec- Depreciation exp. (W-3) 2,292
2018
31-Dec-2018 Balance 7,583
8,532 8,532

Assets disposal - Plant and machinery


1-Apr-2018 Old machine 870 1-Apr-2018 Acc. depreciation (W-1) 376
31-Oct-2018 Cost 1,000 1-Apr-2018 New machine 340
31-Oct-2018 Acc. depreciation (W-2) 573
31-Oct-2018 Cash (334-25) 309
31-Dec-2018 Loss on disposal (P.L) 272
1,870 1,870

W-1: Accumulated depreciation – Machine exchange Rs. in '000


Depreciation for 2015 870×0.2×4÷12 58
Depreciation for 2016 (870–58)×0.2 162
Depreciation for 2017 (870–58–162)×0.2 130
Accumulated depreciation upto 31-12-2017 (WDV = 870-350=520) 350
Depreciation for 2018 (870–350)×0.2×3÷12 26
376

W-2:Accumulated depreciation – Machine sold


1-1-2015 100
x 20% (20)
31-12-2015 80
x 20% (16)
31-12-2016 64
x 20% (12.8)
31-12-2017 51.2
512,000/51.2 x 100
Cost = 1,000,000

49
Page 4 of 5
Acc.dep on 31-12-2017=1,000,000-512,000=488,000
+ 512,000 x 20% x 10/12=85,333
Acc.dep on 10-10-2018=573,333

W-3 : Depreciation for the year


Opening WDV (12,700 – 6,240) 6,460
Less:Opening WDV of Disposal ( 512 + 520) (1,032)
5,428
x 20% 1086
+depreciation on Disposals (26+85) 111
+depreciation on Additions
1,020 x 20% x 9/12 153
11,300 x 20% x 5/12 942
Total depreciation 2,292

W-4 : Cost of the plant :


Rs.in ‘’000’’
Purchase price of the plant 10,000 × 0.95 9,500
Other directly related cost (660 + 540 + 1,800
600)
11,300

50
Page 5 of 5
“Don’t be aggressive but try to understand feelings of others.”

Government Grant
(IAS-20)

Grant related to Grant related to Assets


income as a
compensation
for Expenses
Monetary Asset Non- monetary
(cash) is received to asset (i.e. Actual
acquire asset asset e.g. land or
Grant for past expenses Grant for future machinery) is
or for immediate Expenses received
financial support
(unrelated to future Cash for depreciable Cash for non-
Recognize in profit or
costs) Asset is received depreciable asset is
loss over the period
the expenditure is (e.g. machinery) received (e.g. Land)
recognized
Recognize as
income in the
Grant is credited to Grant is credited to
period in which
Grant Income Grant is deferred grant asset account to
grant becomes
credited credited to income account which the grant
receivable
separately as related and is recognized as relates [ i.e Grant should be
(Example-1)
other income expense grant income over indirectly recognized as an
(Direct (indirect the useful life of recognized as income on the basis
approach) Approach) the asset (direct income over the that best reflects the
(Example-2) (Example-3) approach) period of grant by manner in which the
(example - 4) way of reduced conditions are met
depreciation] (Example-6)
(indirect approach)
[Example - 5]

Recognize the asset at either:

Fair value (Example-7A) Nominal amount of cash paid


(if any) (Example-7B) Repayment of Government Grants

Grant for Expenses Grant for Assets


Grants received as a package Debited first against
(means a combination of) the balance on the
deferred income
Account (if any) and Treated as a Treated as
balance (if any) to deferred income
Grant Grant for future Grant for past Grants for immediate reduction from
profit or loss separately
for asset expenses expenses financial support cost of Asset
(Example - 9)

Debited first against Debited to asset


the balance on the A/C & recognize
All these amounts will be
separately recognized as deferred income cumulative
an income as discussed account (if any) and additional
previously (Example - 8) balance (if any) to depreciation
profit or loss immediately in
(Example - 10A)
51 profit or loss
(Example - 10B)

Page 1 of 1
“Trust in the plans of Allah. Be patient, Keep smiling and never overthink.”

Government Grants and Government Assistance


(IAS 20)

Government grants are provided to encourage an entity to become involved in certain activities that it may
otherwise not have involved itself in (or may even be used to discourage certain activities). It is often provided to
assist businesses in starting up. This obviously benefits the business but also benefits the government through
creation of jobs and thus a larger base of taxpayers.

There are two types of grants:

1. Grants related to income (as a compensation of expenses)


2. Grants related to assets:

Grants related to income as a compensation of expenses


If the grant received does not relate to an asset it could be used as:
• Compensation for past expenses or as immediate financial support; or as
• Compensation for future expenses still to be incurred.

Grant for past expenses or immediate financial support


The grant may be receivable as either:
• Immediate financial support (unrelated to future costs); or
• For expenses or losses already incurred.

Accounting treatment:
Where the grant relates to immediate financial support or past expenses, it is recognised as income in the period in
which the grant becomes receivable.

Example 1: grant for past expenses


The government offers companies that incur certain labor expenditure cash equal to
30% of the expenditure. Ahmed limited incurred 30 000 of labor expenses during 2010 and presented the
government with an audited statement of expenses on 31 March 2011.

Required:
Show the related journal entries in the records of Ahmed Limited for the year ended 31.12.2010 and 31.12.2011
assuming that the grant becomes receivable:
A. In the year in which the company incurs the specified expenses;
B. In the year in which the company provides the government with an audited statement of expenses.

52
Page 1 of 29
Solution to example 1A: grant for past expenses

31 December 2010 Debit Credit

Salary expenses
30 000
Salaries and wages payable
30 000
Labour costs incurred during 20X0

9 000
Grant income receivable (asset) 30 000 x 30%
Grant income 9 000
Grant income recognised based on past expenses; recognised
when the expenses were incurred

Solution to example 1B: grant for past expenses

31 December 2010 Debit Credit

Salary expenses
30 000
Salaries and wages payable
30 000
Labour costs incurred during 20X0

31 March 2011 9 000


Grant income receivable (asset) 30 000 x 30%
Grant income 9 000
Grant income recognised based on past expenses; recognised when
the required audited expense statement was presented to
government

53
Page 2 of 29
“Sins take you away from Allah while prayers take you back to Allah.”

Grant as a compensation for future expenses

If the grant is to be used to subsidise certain future expenditure, then it should be recognised in the statement of
comprehensive income over the period that the expenditure is recognised.

Approaches to present government grant in income statement:


There are two approaches that the company may use in presenting the government grant:
a. direct income approach: the grant is credited to a grant income account in other income (i.e. the grant is
recognised directly as income over the period of the grant);
b. indirect income approach: the grant is credited to the expense account to which the grant relates (indirectly
recognised as income over the period of the grant by way of the reduced expenditure).

Example 2: grant for future expenses – direct approach

The government grants a company a cash of 10 000 to contribute 10% towards 100 000 of future wage expenditure.
The grant was received on 1 January 2011.

Required:
Show the journal entries for the year ended 31 December 2011 assuming that the company policy is to present such
a grant as grant income (i.e. direct income approach):
A. The company incurs 100 000 wage expenditure in 2011;
B. The company incurs 20 000 of the related wage expenditure in the year ended 31 December 2011 and 80 000
thereof in the year ended 31 December 2012.

Solution to example 2A: grant for future expenses – direct approach

1 January 2011 Debit Credit


Bank 10 000
Deferred grant
income (presented in
liabilities) 10 000
Recognising a government grant intended to reduce future expenses

31 December 2011
Wage expenditure 100 000
Wages payable / Cash 100 000
Wage expenditure incurred

Deferred grant income 10 000


Grant income Presented in other income 10 000
Recognising 100% of the government grant since all related expenses
that the grant was intended to compensate have been incurred

54
Page 3 of 29
Note: the statement of comprehensive income will reflect a wage expense of 100 000 and grant income
of 10 000 (the net effect on profit is a net expense of 90 000).

Solution to example 2B: grant for future expenses – direct approach

1 January 2011 Debit Credit


Bank 10 000
Deferred grant income 10 000
Recognising a government grant intended to reduce future expenses

31 December 2011
Wage expenditure 20 000
Wages payable/cash 20 000
Wage expenditure incurred

Deferred grant income 10 000 x 20% 2 000


Grant income 2 000
Recognising 20% of the government grant since 20% of the expenses
that the grant was intended to compensate have been incurred

31 December 2012
Wage expenditure 80 000
Wages payable/cash 80 000
Wage expenditure incurred

Deferred grant income 10 000 x 80% 8 000


Grant income Recognised directly as income 8 000
Recognising 80% of the government grant since 80% of the expenses
that the grant was intended to compensate have been incurred

Note: the statement of comprehensive income will reflect:


2011: a wage expense of 20 000 and grant income of 2 000 (the net decrease in profits: 18 000);
2012: a wage expense of 80 000 and grant income of 8 000 (net decrease in profits: 72 000).

Example 3: grant for future expenses – indirect approach


The government grants a company a cash of 10 000 to contribute 10% towards future specified wages. The grant
was received on 1 January 2011. The year-end is 31 December.

Required:
Show the journal entries assuming that the company policy is to recognise government grants as a credit to the
related expense (i.e. indirect income approach):
a. The company incurs all intended expenditure in the year ended 31 December 2011;
b. The company incurs 20% of the wages in 2011 and 80% in 2012.

55
Page 4 of 29
“Prayer is just like water, It keeps the root of Imaan alive so never forget to pray salat.”

Solution to example 3A: grant for future expenses – indirect approach

1 January 2011 Debit Credit


Bank 10 000
Deferred grant income 10 000
Recognising a government grant intended to reduce future expenses

31 December 2011
Wage expenditure 100 000
Wages payable/cash 100 000
Wage expenditure incurred

Deferred grant income 10 000


Wage expenditure Recognised indirectly as income 10 000
Recognising 100% of the government grant since all related expenses that
the grant was intended to compensate have been incurred

Note: the statement of comprehensive income will reflect a wage expense of 90 000 (the net effect on profit
is a decrease of 90 000).

Compare this to example 2A: the effect on profit is the same.

Solution to example 3B: grant for future expenses – indirect approach

1 January 2011 Debit Credit


Bank 10 000
Deferred grant income 10 000
Recognising a government grant intended to reduce future expenses

31 December 2011
Wage expenditure 20 000
Wages payable/cash 20 000
Wage expenditure incurred

Deferred grant income 10 000 x 20% 2 000


Wage expenditure Recognised indirectly as income 2 000
Recognising 20% of the government grant since 20% of the expenses
that the grant was intended to compensate have been incurred

56
Page 5 of 29
31 December 2012

Wage expenditure 80 000


Wages payable/cash 80 000
Wage expenditure incurred

Deferred grant income 10 000 x 80% 8 000


Wage expenditure Recognised indirectly as income 8 000

Recognising 80% of the government grant since 80% of the expenses that
the grant was intended to compensate have been incurred

Note: the statement of comprehensive income will reflect:


20X1: a wage expense of 18 000 (the net decrease in profits: 18 000);
20X2: a wage expense of 72 000 (net decrease in profits: 72 000); Compare this to example 2B: the effect on profit
is the same.

Another example
A company receives a cash grant from government of Rs. 30,000 on 1.1.2011.

The grant is towards the cost of training young trainees, and the training programme is expected to last for 18
months from 1 January 2011.Total training costs are expected to be 75,000.
Actual costs of the training were Rs. 50,000 in 2011 and Rs. 25,000 in 2012.

Reuired: Discuss the accounting treatment under both the methods.

Solution: The grant would be accounted for as follows:

Grant income for the year ended 31 December 2011 will be:
50,000/75,000 x 30,000 = 20,000

At 31.12.2011 there would be a current liability of Rs. 10,000 (30,000-20,000). This would be recognised in income
statement in 2012.

Extracts from the financial statements are as follows:

Statement of financial position (extracts)


31 31
December December
2011 2012
Current liabilities
Deferred income 10,000 -
Non-current liabilities
Deferred income - -
Statement of profit or loss (extracts)
Method 1 (Direct income)

57
Page 6 of 29
Training costs expense (50,000) (25,000)
Government grant (other income) 20,000 10,000
Method 2 (Indirect income)
Training costs exp (50,000 – 20,000) 30,000
Training costs exp (25,000 – 10,000) 15,000

Grants related to assets


Grants related to assets could be provided as:
• a monetary asset (i.e. cash) that must be used to acquire a non-monetary asset (e.g. land or machinery); or as
• a non-monetary asset (i.e. the actual asset is provided)

The non-monetary asset itself could be:


• a depreciable asset (e.g. a machinery); or
• a non-depreciable asset (e.g. land)
1. cash grant is received to acquire the asset:
a. Grant related to a depreciable asset

If the asset received or to be acquired is a depreciable asset, the grant is usually recognised as income over the same
period that the asset is depreciated.

Where the grant relates to an asset, the initial grant may be recorded using either of the following approaches:
• direct income approach: the grant is credited to a deferred grant income account and is recognised as grant
income over the useful life of the asset (i.e. the grant is recognised directly as income over the life of the asset);
• Indirect income approach: the grant is credited to the asset account to which the grant relates (i.e. indirectly
recognised as income over the period of the grant by way of a reduced depreciation charge).

Example 4: grant related to a depreciable asset – direct approach

The government grants a company a cash sum of 12 000 on 1 January 2011 to assist in the acquisition of a nuclear
plant. The nuclear plant was acquired on 1 January 2011 for 90 000, was available for use immediately and has a
useful life of 3 years (the plant has a nil residual value).

Required:
Show the journal entries in the years ended 31 December 2011, 2012 and 2013. The company has the policy of
recognizing government grants directly in income.

58
Page 7 of 29
59
Page 8 of 29
“Prayers take you near to Allah.”

Note: the statement of comprehensive income will reflect:


2011 – 2013: a depreciation expense of 30 000 and grant income of 4 000 (net decrease in profits: 26 000 per year).

Example 5: grant related to a depreciable asset – indirect approach

The government grants a company a cash of 12 000 on 1 January 2011 to assist in the acquisition of a nuclear plant.
The nuclear plant:
• was acquired on 1 January 2011 for 90 000;
• was available for use immediately; and
• has a useful life of 3 years (the plant has a nil residual value).

Required:
Show the journal entries in the years ended 31 December 2011, 2012 and 2013. The company has the policy of
recognizing government grants indirectly in income (i.e. as a reduction of the cost of the asset).

Solution to example 5: grant related to a depreciable asset – indirect approach

1 January 2011 Debit Credit


Bank 12 000
Deferred grant income 12 000
Recognising a government grant intended to assist in the acquisition
of a nuclear plant

2 January 2011
Nuclear plant: cost (asset) 90 000
Bank 90 000
Purchase of plant

Deferred grant income 12 000


Nuclear plant: cost (asset) 12 000
Recognising the government grant as a reduction of the plant’s cost

31 December 2011
Depreciation - plant (expense) (90 000 – 12 000 – 0) / 3 years 26 000
Nuclear plant: accumulated depreciation (asset) 26 000
Depreciation on plant

31 December 2012
Depreciation - plant (expense) (90 000 – 12 000 – 0) / 3 years 26 000
Nuclear plant: accumulated depreciation (asset) 26 000

60
Page 9 of 29
Depreciation on plant (net of
30,000 – 4,000 as was in
previous example)

31 December 2013
Depreciation - plant (expense) (90 000 – 12 000 – 0) / 3 years 26 000
Nuclear plant: accumulated depreciation (asset) 26 000
Depreciation on plant

Note: the statement of comprehensive income will reflect:

2011 – 2013: a depreciation expense of 26 000 (net decrease in profits: 26 000 per year). Compare this to example 4.

Another example
A company receives a government grant of Rs. 400,000 towards the cost of an asset with a cost of Rs. 1,000,000 on
1.1.2011

The asset has an estimated useful life of 10 years and no residual value.

Required
Show how the asset and the grant would be reflected in the financial statements at 31.12.2011 under both methods
of accounting for the grant allowed by IAS 20.

Solution:
The amounts could be reflected in the financial statements prepared at the end of 31.12.2011 in accordance with
IAS 20 in the following ways:

Method 1: (indirect income Approach)


Statement of financial position (extract) as on 31.12.2011
Property, plant and equipment Rs.
Cost (1,000,000 – 400,000) 600,000
Accumulated depreciation (600,000/10) (60,000)

Carrying amount 540,000

Statement of profit or loss (extract) Rs.


For the year ended 31.12.2011
Depreciation charge (Rs. 600,000/10 years) 60,000
Method 2: (direct income Approach)
Statement of financial position (extract) as on 31.12.2011
Property, plant and equipment Rs.
Cost 1,000,000
Accumulated depreciation (100,000)

61
Page 10 of 29
Carrying amount 900,000

Current liabilities
Deferred income 40,000
Non-current liabilities
Deferred income 320,000

At the end of year 1 there would be Rs. 360,000 of the grant left to recognise in income statement in the future at
Rs. 40,000 per annum. Rs. 40,000 would be recognised in the next year and is therefore current. The balance is
non-current.

Statement of profit or loss (extract) Rs.


For the year ended 31.12.2011
Expense: Depreciation charge (Rs. 1,000,000/10 years) (100,000)

Other Income: Government grant (Rs. 400,000/10 years) 40,000

Practice question
On January 2011 Entity purchased a non-current asset with a cost of Rs. 500,000 and received a grant of Rs. 100,000
in relation to that asset.

The asset is being depreciated on a straight-line basis over five years.

Required
Show how the asset and the grant would be reflected in the financial statements at 31.12.2011 under both methods
of accounting for the grant allowed by IAS 20.

Solution
The amounts could be reflected in the financial statements prepared at the end of 2011 in accordance with IAS 20
in the following ways:

Method 1: (indirect income approach)


Statement of financial position as on 31.12.2011
Property, plant and equipment Rs.
Cost (500,000 -100,000) 400,000
Accumulated depreciation (400,000/5) (80,000)
Carrying amount 320,000

Statement of profit or loss Rs.


Depreciation charge (Rs.400,000/5 years) 80,000

Method 2: (Direct income approach)


Statement of financial position as on 31.12.2011 Rs
Property, plant and equipment 500,000

62
Page 11 of 29
Cost (100,000)
Accumulated depreciation
400,000
Carrying amount
Current liabilities 20,000
Deferred income
Non-current liabilities
Deferred income 60,000

At the end of 2011 there would be Rs. 80,000 of the grant left to recognise in income statement in the future at Rs.
20,000 per annum. Rs. 20,000 would be recognised in the next year and is therefore current. The balance is non-
current.

Statement of profit or loss


Expense: Depreciation charge (Rs. 500,000/5 years) 100,000
Income: Government grant (Rs. 100,000/5 years) 20,000

b. Grant related to non-depreciable assets:


If the grant relates to the cost of a non-depreciable asset, the grant should be recognized as an income on a basis
that best reflects the manner in which the conditions are met.

If the asset to be acquired is a non-depreciable asset, the grant may require certain obligations to be met, in which
case the grant would be recognised as the obligations were met. By way of example, a grant could be provided by
way of cash to purchase land on condition that a building is erected on it. In this case, the grant could be recognised
as income once the building is erected or the grant could be recognised as income over the life of the building (being
a depreciable asset).

Example 6: grant related to a non-depreciable asset – direct approach


The government grants a company a cash of 12 000 on 1 January 2011 to assist in the acquisition of land. A condition
of the grant is that the company builds a factory on the land. The land was acquired on 1 January 2011 for 90 000.
Land is not depreciated. The factory was completed on 31 March 2011 (total building costs of 300 000 were paid in
cash on this date), was available for use immediately and has a useful life of 3 years (the factory has a nil residual
value).

Required:
Show the journal entries in the years ended 31 December 2011, 2012, 2013 and 2014. The company’s policy is to
recognise grants directly in income.

Solution to example 6: grant related to a non-depreciable asset – direct approach

1 January 2011 Debit Credit


Bank 12 000
Deferred grant income 12 000

63
Page 12 of 29
Government grant received to assist in the acquisition of land

Land: cost 90 000


Bank 90 000
Purchase of land

31 March 2011
Factory building: cost 300 000
Bank 300 000
Building costs related to factory, paid in cash

31 December 2011
Depreciation – factory building (300 000 – 0) / 3 years x 9 / 12 75 000
Factory building: accumulated depreciation 75 000
Depreciation of factory building

Deferred grant income 12 000 / 3 years x 9 / 12 3 000


Grant income 3 000
Grant income recognised on the same basis as depreciation on the
factory building

31 December 2012
Depreciation – factory building (300 000 – 0) / 3 years 100 000
Factory building: accumulated depreciation 100 000
Depreciation of factory building

Deferred grant income 12 000 / 3 years 4 000


Grant income 4 000
Grant income recognised on the same basis as depreciation on the
factory building

31 December 2013
Depreciation – factory building (300 000 – 0) / 3 years 100 000
Factory building: accumulated depreciation 100 000
Depreciation of factory building

Deferred grant income 12 000 / 3 years 4 000


Grant income 4 000
Grant income recognised on the same basis as depreciation on the
factory building

31 December 2014
Depreciation – factory building (300 000 – 0) / 3 years x 3 / 12 25 000
Factory building: accumulated depreciation 25 000
Depreciation of factory building

Deferred grant income 12 000 / 3 years x 3 / 12 1 000

64
Page 13 of 29
Grant income 1 000
Grant income recognised on the same basis as depreciation on the
factory building

2. Non monetary grants


Where the grant is in cash, the measurement thereof is discussed previously. If, however, the company is granted
an asset such as a licence to operate or land or machinery etc., the company may either measure the grant at its fair
value or at the nominal cost thereof, being the actual cash paid for the asset, if any. If nothing is paid then if the
asset is not measured at fair value the government grant is not measured at all.

Example 7: grant asset – fair value or nominal amount


Pakistani government grants the company a licence to fish off the coast of Gwadar. The fair value of the licence is
50 000 and the company is required to pay a nominal amount of 1 000 for the licence.

Required:
Show the journal entries assuming:
A. The company chooses to measure the licence at its fair value.
B. The company chooses to measure the licence at its nominal amount.

Solution to example 7A: grant asset – fair value

Debit Credit
Fishing licence (asset) Given 50 000
Deferred income 50 000 – 1 000 49 000
Bank Given 1 000
Recognising the licence granted by the government at fair value

Solution to example 7B: grant asset – nominal amount

Debit Credit
Fishing licence (asset) Given 1 000
Bank Given 1 000
Recognising the licence granted by the government at nominal value

Grants received as a package (IAS 20.19)


If the grant is received as a package to which a number of varying sets of conditions are attached, it may be
appropriate to recognise each part of the grant on a different basis. The first step is to identify each part of the
package to which there are different conditions affecting when the grant is earned.

The grant may, for instance, relate to a combination of:


• an asset
• future expenses
• past expenses
• immediate financial support.

65
Page 14 of 29
“A prayer with sincere heart is never wasted.”

Accounting treatment:
In such cases, the grant package may be viewed as multiple parts. The grant relating to:
A. the asset should be recognised as income in a way that reflects the pattern of depreciation;
B. future expenses should be recognised as income in a way that reflects the pattern of future expenses;
C. past expenses should be recognised as income in the period in which the grant becomes receivable;
D. general and immediate financial support should be recognised as income in the period in which the grant
becomes receivable

Example 8: grant is a package deal


The government grants a company a cash of 120 000 on 1 January 2011. The grant relates to two aspects:
• 30 000 is a cash sum as immediate financial support with no associated future costs;
• 90 000 is to assist in the future acquisition of vehicles.

The vehicles were acquired on 1 January 2011 for 210 000. The vehicles were available for use immediately and have
a useful life of 3 years (the vehicles all have nil residual values).

The company has decided to follow the direct Approach related to vehicle grant; i.e recognize separately as income
in statement of profit or loss.

Required:
Show the journal entries in the years ended 31 December 2011, 2012 and 2013.

Solution to example 8: grant is a package deal

1 January 2011 Debit Credit


Bank 120 000
Deferred grant income 90 000
Grant income 30 000
Recognising a government grant: package deal and
Portion of grant income recognised immediately – not attached to any
asset or future expenses and all criteria met in a prior year: 30000

1 January 2011
Vehicles: cost 210 000
Bank 210 000
Purchase of vehicles

31 December 2011
Depreciation – vehicles (210 000 – 0) / 3 years 70 000
Vehicles: accumulated depreciation 70 000
Depreciation of vehicles

66
Page 15 of 29
31 December 2011 Debit Credit
Deferred grant income (120 000 – 30 000) / 3 years 30 000
Grant income 30 000
Portion of grant income related to purchase of vehicles recognised on
the same basis as vehicle depreciation

31 December 2012
Depreciation – vehicles (210 000 – 0) / 3 years 70 000
Vehicles: accumulated depreciation 70 000
Depreciation of vehicles

Deferred grant income (120 000 – 30 000) / 3 years 30 000


Grant income 30 000
Portion of grant income related to purchase of vehicles recognised on
the same basis as vehicle depreciation

31 December 2013
Depreciation – vehicles (210 000 – 0) / 3 years 70 000
Vehicles: accumulated depreciation 70 000
Depreciation of vehicles

Deferred grant income (120 000 – 30 000) / 3 years 30 000


Grant income 30 000
Portion of grant income related to purchase of vehicles recognised on the
same basis as vehicle depreciation

Repayment of government grant:


A government grant might become repayable by the entity if the grant was provided on certain conditions and these
conditions were later breached by the entity.

If the grant becomes repayable it is accounted for as a change in accounting estimate (means prospectively)

Accounting treatment:
Where the grant has to be repaid the treatment depends on whether the grant was related to expenses or
assets.
1. If the original grant related to income for the compensation of expenses, the repayment of the grant is debited:
• first against the balance on the deferred income account, if any; and
• balance if any to profit or loss as an expense.

Example 9: grant related to expenses – repaid


The local government granted the company 10 000 on 1 January 2011 to assist in the financing of mining expenses.
The grant was conditional upon the company mining for a period of at least two years. Initially it was expected that
company can meet this condition and therefore it was decided to recognize the grant as income equally over two
years.

67
Page 16 of 29
The company ceased mining on 30 September 2012 due to unforeseen circumstances. The terms of the grant
required that the grant be repaid immediately and in full.

Mining expenses incurred to date were as follows:


2011: 80 000
2012: 60 000

Required:
Show the journal entries assuming:
A. The company recognises grants directly as ‘grant income’.
B. The company recognises grants indirectly as income by reducing the related expense.

Solution to example 9A: grant related to expenses – repaid

1 January 2011 Debit Credit


Bank 10 000
Deferred grant income 10 000
Recognising a government grant intended to reduce future expenses

31.12.2011

Mining expenses 80 000


Accounts payable / cash 80 000

Debit Credit
Deferred grant income 5 000
Grant income 5 000

30.09.2012 Debit Credit


Mining expenses 60 000
Accounts payable / cash 60 000

Debit Credit
Deferred grant income 3 750
Grant income 3 750
(10 000 x 50% x 9/12)

Debit Credit
1 250
Deferred grant income 8 750
Other expenses
Bank 10 000
(repayment)

68
Page 17 of 29
Solution to example 9B: grant related to expenses – repaid
1 January 2011 Debit Credit
Bank 10 000
Deferred grant income 10 000
Recognising a government grant intended to reduce future expenses

31.12.2011

Mining expenses 80 000


Accounts payable / cash 80 000

Debit Credit
Deferred grant income 5 000
Mining expenses 5 000

30.09.2012 Debit Credit


Mining expenses 60 000
Accounts payable / cash 60 000

Debit Credit
Deferred grant income 3 750
Mining expenses 3 750
(10 000 x 50% x 9/12)

Debit Credit
1 250
Deferred grant income 8 750
Other expenses
Bank 10 000
(repayment)

2. If the original grant related to an asset, and


A. it was accounted for as a deferred income, then:
• the repayment of the grant is debited to the balance on the deferred income account, if any; and
• then to the profit or loss as an expense.

B. if it was accounted for as a reduction from the cost of the asset; then
• Repayment is debited to the asset account; and
• Recognize the cumulative additional depreciation that would have been recognized to date had the grant
not been received immediately as an expense.

69
Page 18 of 29
Example 10: grant related to assets – repaid
The local government granted the company 10 000 on 1 January 2011 to assist in the purchase of a manufacturing
plant. The grant was conditional upon the company manufacturing for a period of at least two unbroken years.

The company purchased the plant on 1 January 2011 for 100 000. The plant is depreciated on the straight-line basis
over its useful life of 4 years to a nil residual value.

The company ceased manufacturing on 30 September 2012 due to unforeseen circumstances. The terms of the grant
required that the grant be repaid immediately and in full. The company intended to resume manufacturing in the
next year.

Required:
Show the journal entries for the year ended 31.12.2011 and 31.12.2012. assuming that the company:
A. recognises grants as grant income (direct income).
B. recognises grants as a reduction of the cost of the related asset (indirect income).

Solution to example 10A: grant related to assets – repaid

1 January 2011 Debit Credit


Bank 10 000
Deferred grant income 10 000
Recognising a government grant
1 January 2011
Plant: cost 100 000
Accounts payable/ bank 100 000
Purchase of plant

31 December 2011
Depreciation - plant (100 000 – 0) / 4 years x 12 / 12 25 000
Plant: accumulated depreciation 25 000
Depreciation of plant
Deferred grant income 10 000 / 4 years x 12 / 12 2 500
Grant income 2 500
Recognising 25% of the government grant since the grant relates to
the acquisition of an asset that is depreciated over 4 years
30 September 2012
Depreciation - plant (100 000 – 0) / 4 years x 9 / 12 18 750
Plant: accumulated depreciation 18 750
Depreciation of plant: (manufacture ceases on 30 September 20X2)

Deferred grant income 10 000 / 4 years x 9 / 12 1 875


Grant income 1 875

70
Page 19 of 29
Recognising 9 months of the government grant to the date of repayment of the grant

30 September 2012 Debit Credit

Deferred grant income 10 000 – 2 500 – 1 875 5 625


Other expense (balancing) 4 375
Bank Given 10 000
Repayment of the full grant, first reducing the balance on the deferred income account and then expensing the
rest

31 December 2012 Debit Credit


Depreciation – plant 6 250
Plant: accumulated depreciation 6 250
Depreciation of plant: (100 000/4 x 3/12)

Solution to example 10B: grant related to assets – repaid

1 January 2011 Debit Credit


Bank 10 000
Deferred grant income 10 000
Recognising a government grant

1 January 2011
Plant: cost 100 000
Accounts payable/ bank 100 000
Purchase of plant

Deferred grant income 10 000


Plant: cost 10 000
Recognising the grant income as a decrease in the asset’s cost

31 December 2011
Depreciation - plant (100 000 –10 000 – 0) / 4 years x 12 / 12 22 500
Plant: accumulated depreciation 22 500
Depreciation of plant:

30 September 2012
Depreciation - plant (100 000 – 10 000 – 0) / 4 years x 9 / 12 16 875
Plant: accumulated depreciation 16 875
Depreciation of plant: (manufacture ceases on 30 September 20X2)

71
Page 20 of 29
Plant: cost Original grant refunded 10 000
Bank 10 000
Depreciation – plant W1: 2 500 + 1 875 4 375
Plant: accumulated depreciation W1: 2 500 + 1 875 4 375

Repayment of the full grant: increase cost and increase accumulated depreciation with extra cumulative
depreciation that would otherwise have been expensed if no grant had been received on 1 January 20X1

31 December 2012 Debit Credit


Depreciation – plant 6 250
Plant: accumulated depreciation 6 250
Depreciation of plant: (100 000/4 x 3/12)
Or [100,000-10,000-22,500-16,875+10,000-4,375]/2.25x3/12=6,250

Working extra depreciation to be charged:


Date Calculations Was Is Difference
Cost 1.1.2011 100,000 - 90,0000 100,000 10,000
10,000
Depreciation 31.12.2011 22,500 25,000 2,500
(90,000/4) (100,000/4)
Depreciation 31.12.2012 9 months 16,875 18,750 1,875
Carrying amount 31.12.2012 50,625 56,250 5,625

Example: Repayment of grant related to income


On 1 January 2011 X Limited received a cash grant of Rs. 500,000 towards the cost of employing an environmental
impact analyst on a new project for a 5 year period.

It is decided to treat the grant income as a reduction of analyst salary, which is 300,000 per annum.

The grant is repayable in full if the project is not completed.

The analyst was employed and the project commenced from the 1 January 2011.

On 1 January 2013 the project was abandoned and the grant became repayable in full.

The grant and its subsequent repayment is accounted for as follows:

2011 Debit Credit


Cash 500,000
Deferred income 500,000
Salary 300,000

72
Page 21 of 29
Cash 300,000

Deferred income (500,000/5) 100,000

Salary 100,000
2012
Salary 300,000
Cash 300,000

Deferred income 100,000

Salary 100,000

The balance on the deferred income account at the end of 2012 is Rs. 300,000 (Rs. 500,000 – Rs. (2 x Rs. 100,000).

The double entry to reflect the repayment is as follows

Deferred income 300,000


Other expenses 200,000
Cash 500,000

Disclosures:
The following issues must be disclosed in the notes to the financial statements:
1. Accounting policy regarding both recognition and method of presentation, for example:
o Government grants are recognised as income over the period to which the grant applies; and
o The grant is presented as a decrease in the expenditure to which it relates (or: the grant is presented as a
separate line item: grant income).;
2. The nature and extent of government grants recognised in the financial statements;
3. An indication of other forms of government assistance not recognised as government grants but from which the
entity has benefited directly (e.g. provision of guarntee and free marketing and technical advice);
4. Unfulfilled conditions and other contingencies attached to recognised government grants.

Definitions:
Government: includes: government; government agencies; similar bodies; whether local, national or international.

Government assistance: means action by government designed to provide an economic benefit to an entity or range
of entities that qualify under certain criteria. Government assistance does not include benefits provided only
indirectly through action affecting general trading conditions, such as the provision of infrastructure in developing
areas or the imposition of trading constraints on competitors.

Government grants: are assistance by government in the form of transfers of resources to an entity in return for
past or future compliance with certain conditions relating to the operating activities of the entity.
They exclude
1. those forms of government assistances which cannot reasonably have a value placed upon them (free advice
and provision of guarantees) and

73
Page 22 of 29
“Prayer is a weapon for believers to overcome the hard situations.”

2. Transactions with government which can not be distinguished from normal trading transactions of the entity
(e.g. a government procurement policy that is responsible for a portion of the entity’s sales).

If these types of assistances are material then they should be disclosed in notes to the financial statements

There are two types of grants:


Grants related to assets:
a government grant that has a primary condition requiring:

The qualifying entity to purchase, construct or otherwise acquire long-term assets;


Secondary conditions may also be attached restricting:
• the type or location of the assets, and/ or
• The periods during which the assets are to be acquired or held.

Grants related to income (as a compensation of expenses):


a government grant that is not a grant related to an asset.

Forgivable loan (e.g. loans from government bank)


a loan that the lender has undertaken to waive repayment of under certain conditions. (Q.2 on page 82)

Accounting Treatment
It is treated as government grant when there is a reasonable assurance the entity will be able to meet the terms for
the forgiveness of loan.

Fair value:
Price that would be received from selling an asset or paid to settle a liability.

Recognition criteria:
Government grants are only recognized when there is reasonable assurance:
• that the entity will comply with the conditions; and
• The grants will be received.

Basic principle:
Government grants are recognized as:
• income over the relevant periods on a rational basis
• That matches the grant income with the costs that they were intended to compensate.

Presentation of grant income:


The grant income can be presented as:
• ‘other income’ in the statement of comprehensive income: direct income approach; or
• A reduction to the expense or asset to which it related: indirect income approach.

74
Page 23 of 29
Low interest Loans are loans which the government provides at lower interest rate as compared to the market
interest rate.

Accounting Treatment
The benefit of a government loan at a below market rate of interest is treated as a government grant under IAS 20.
The benefit of below market rate of interest shall be measured as the difference between the cash received under
the government loan and the fair value of the liability.

Question:
During the year ended 30 June 2012, Ahmed Limited (AL) received three grants, the details of which are set out
below.
(1) On 1 September 2011, a grant of Rs. 40,000 from local government. This grant was in respect of training costs
of Rs. 70,000 which AL had incurred during the year.
(2) On 1 November AL bought a machine for Rs. 350,000. A grant of Rs. 100,000 was received from central
government in respect of this purchase. The machine, which has a residual value of Rs. 50,000, is depreciated
on a straight-line basis over its useful life of five years.
(3) On 1 June 2012 a grant of Rs. 100,000 from local government. This grant was in respect of relocation costs
that AL had incurred moving part of its business from outside the local area. The grant is repayable in full
unless AL recruits ten employees locally by the end of 30.06.2012. AL is finding it difficult to recruit as the
local skill base does not match the needs of the business.

Required
Show the journal entries and how the above transactions should be reflected in the financial statements for the year
ended 30 June 2012, under both the options: i.e.:
1. Treat the relevant grants as separate income.
2. Treat the relevant grants as deduction.

Solution:

(a) Journal entries:

1. Grant 1:
Option-1:
[as separate income]

Training exp 70,000


Cash 70,000
1-9-2011
Cash 40,000
Grant Income 40,000
(treat it as a separate other
income)

75
Page 24 of 29
Option2{As a deduction from cost of Assets]

Training exp 70,000


Cash 70,000
1-9-2011
Cash 40,000
Training exp 40,000

2. Grant 2:

1-11-2011
Machine 350,000
Cash 350,000

Option 1: Option 2:
1-11-2011 Grant As a deferred income means a Grant As a deduction from cost of
liability asset:
Cash 100,000 Cash 100,000
Deferred Income 100,000 Machine 100,000
30-6-2012
Depreciation 40,000 Depreciation 26,667
Acc. dep 40,000 Acc. Dep 26,667

([350,000 – 50,000] / 5)x ([350,000 – 100,000]-50,000] / 5)x 8/12


8/12 = 40,000 = 26,667

Deferred Income 13,333


Grant Income 13,333
[100,000/5] x 8/12 = 13,333

3. Grant 3 : As the grant is conditional on future recruitment of 10 employees and AL is finding it difficult to recruit;
therefore amount received should be treated as a liability (as it is likely to be repaid).

Cash 100,000
Grant payable 100,000
Note
The Rs. 100,000 grant (3) has conditions attached to it. In such a situation, IAS 20 states that grants should not be
recognized until there is reasonable assurance that the entity will comply with any conditions attaching to the grant.
Since AL is struggling to recruit. Hence the grant should not be recognized as such, but should be held in current
liabilities, pending repayment.

76
Page 25 of 29
(b) Presentation in financial statements

Option 1 – Show grants separately from related expenditure


Statement of financial position as at 30 June 2012 (extracts)
Rs.
Non-current assets
Property, plant and equipment (Note 1) 310,000
Non-Current liabilities:
Deferred Income (Grant 2 ) (Note 2) 66,667
Current liabilities
Deferred Income (Grant 2 ) 20,000
Grant payable(Grant 3) 100,000

Notes to the financial statements for the year ended 30 June 2012 (extracts)

Rs.
1.Property, plant and equipment
Cost 350,000
Accumulated depreciation ((350,000 – 50,000) ÷ 5 x 8/12) (40,000)
––––––––
Carrying amount 310,000
––––––––

2.Deferred income relating to government grants- As on 30-6-2012 86,667


(100,000 - (100,000 ÷ 5 x 8/12))
Less: Grant income of 30-6-2013 [ Current liability]
(100,000/5) (20,000)
––––––––
Balance of Non-Current Liability 66,667
––––––––
Included in statement of profit or loss for the year ended 30 June 2012
Operating expenses: Rs.
Depreciation charge (40,000)
Training costs (70,000)
Other Income:
Government grant received (Grant 1) 40,000
Government grant income (Grant 2) 13,333

Option 2 – net off grant from related expenditures


Statement of financial position as at 30 June 2012 (extracts)

Rs.
Non-current assets
Property, plant and equipment (Note 1) 223,333
Current liabilities
Grant payable (Grant 3) 100,000

77
Page 26 of 29
“Prayer can change your situation so remember Allah and offer prayers.”

Notes to the financial statements for the year ended 30 June 2012 (extracts)

1. Property, plant and equipment Rs.


Cost (350,000 – 100,000) 250,000
Accumulated depreciation ((250,000 – 50,000) ÷ 5 x 8/12) (26,667)
––––––––
Carrying amount 223,333
––––––––
Included in statement of profit or loss for the year ended 30 June 2012
Rs.
Depreciation charge 26,667
Training costs (70,000 – 40,000) 30,000

78
Page 27 of 29
Extra practice question
Q.1 Discuss how the following should be dealt with in the financial statements of relevant entities according to
IAS 20 Accounting for Government Grants and Disclosure of Government Assistance:

(a) The government makes a grant to an entity which is planning to develop teaching software for
children with learning difficulties. The purpose of the grant is to help the entity to meet its general
financing requirement in the initial phase. There are no further conditions attached to the grant.
(01)

(b) A manufacturing entity sets up a plant in an area of high unemployment. A government grant of Rs.
4 million is received with a condition that the grant is repayable in full if the number of its employees
fell below 100 at any time during the next four years. It is highly probable that the entity will comply
with the condition attached to the grant.
(03)

(c) Free technical advice has been provided by the government’s export promotion department to
help an exporter to market his new technology in North America.
(01)

A. (i) The grant has been provided for the purpose of giving immediate financial support to the entity with no further
conditions, so this grant should be immediately recognised in profit or loss in full in the period in which the entity
qualifies to receive it (when it is receivable) with disclosure to ensure that its effect is clearly understood.

(ii) Since there is reasonable assurance that conditions attaching to the grant will be met, the grant is recognised in
statement of profit or loss over the four year period in which the entity incurs the costs of employing 100 people.
Amount taken to statement of profit or loss may be either be presented as other income or shown as deduction
from the related expense. The remaining amount of grant will be presented as deferred income under liabilities in
the balance sheet.

(iii) Free technical advice is government assistance that cannot reasonably have a value placed upon it and therefore
should not be recognised. However, an indication of such assistance should be disclosed in financial statements.

Q.2 ABC Pharmaceutical Company received cash from government for a research and development project of a children
vaccine. Explain whether the amount received can be treated as a forgiveable loan in each of the following
independent scenarios.

Scenario 1
As per the terms of the loan, the cash received from the government shall be waived off if the entity is able to
develop the vaccine within 3 years and sell it free of cost for 5 years.

If the entity takes more time than three years in the development or sells the vaccine for a pricebefore 5 years, it will
be liable to repay the loan and the loan will not be considered a forgivable loan.

79
Page 28 of 29
Scenario 2
As per the terms of the loan, the cash received from the government is repayable in cash only if the entity decides
to commercialize the results of the research phase of the project. If the entitydecides not to commercialize the results
of the research phase, the cash received is not repayable in cash, but instead the entity must transfer to the
government the rights to the research.

In this scenario, cash received from the government does not meet the definition of a forgivableloan in IAS 20. This
is because, in this arrangement, the government does not undertake to waive repayment of the loan, but rather to
require settlement in cash or by transfer of the rights to the research. The cash receipt described in the scenario
gives rise to a financial liability (means simply a loan).

80
Page 29 of 29
Allah wants to forgive us, We need repent often and ask Allah to forgive us.

Investment Property IAS 40

Investment property:
1. it is a property (land or buildings or part of a building or both)
2. held by an owner
3. to earn rentals or for capital appreciation or both, rather than for:
• use in the production or supply of goods and services or for administrative use (Owner-occupied property
IAS 16);or
• sale in ordinary course of business ( Inventories IAS 2)

Examples of investment property

The following are examples of investment property:


• Land held for long-term capital appreciation rather than for short-term sale in the ordinary course of
business.
• Land held for a currently undetermined future use. (if an entity has not yet determined that it will use the
land as owner-occupied property or for short-term sale in the ordinary course of business, the land is
regarded as asset held for capital appreciation (means treated as an investment property))
• Property that is being constructed or developed for future use as investment property.

Followings are not investment property


The following are examples of items that are not investment property:
• property intended for sale in the ordinary course of business (IAS 2; e.g. property acquired exclusively with a
view to subsequent disposal in the near future (e.g. property dealers) or for development and resale (housing
societies));
• owner-occupied property i.e.
a. property held for future use as owner-occupied property (IAS 16)
b. property held for future development and subsequent use as owner-occupied property,
c. property occupied by employees (whether or not the employees pay rent at market rates) and
d. owner-occupied property awaiting disposal;

Partly occupied buildings


An entity might use part of a property for the production or supply of goods or services or for administrative
purposes and hold another part of the same property to earn rentals or for capital appreciation. In other words, a
part of a property might be owner occupied and a part held as an investment property. The two parts are
accounted for separately if they could be sold separately.

If this is not the case, the entire property is investment property only if an insignificant portion is held for use in
the production or supply of goods or services or administrative purpose.

81
Page 1 of 29
Example: joint use properties
How should Stunning Ltd account for the following properties in its financial statements:
A. Stunning Ltd owns two buildings on two separate sites in Islamabad. The first building is used by Stunning Ltd
for administration purposes and the second building is leased out (means given on rent)
B. Stunning Ltd owns a 20 storey building in Karachi, which it uses for administration purposes. The top floor of
the building is leased to Unpleasant Ltd (assume portions cannot be sold separately)
C. Stunning Limited owns a 20-storey building in Lahore. It leases out 19 floors and uses the top floor for the
administration of the building (assume portions cannot be sold separately)

Solution to example: joint use properties


A. There are two distinct and separate buildings: owner-occupied and leased out. Since the two buildings are on
two separate sites, it is assumed that they can be sold separately. The owner-occupied building must be
accounted for as property, plant and equipment as it is used for administrative purposes. The leased out
buildings must be accounted for as investment property.
B. There are two components of same building: owner-occupied and leased out. Since these two components
are within one building and therefore cannot be sold separately. Nineteen of the twenty floors is owner-
occupied and is therefore the significant component. Stunning Ltd must therefore report the entire building
as owner-occupied (i.e. as property, plant and equipment under IAS 16).
C. There are two components of the same building: owner-occupied and leased out. Since these two
components are within one building and therefore cannot be sold separately. Nineteen of the twenty floors
are leased out therefore investment property portion is the significant component. Stunning Ltd must
therefore report the entire building as investment property under IAS 40.

Accounting treatment of investment property


The recognition criteria for investment property are the same as for property, plant and equipment under IAS 16.
An owned investment property should be recognised as an asset only when:
• It is probable that future economic benefits associated with the property will flow to the entity; and
• The cost of the property can be measured reliably.

Measurement at initial recognition


Investment property should be measured initially at cost plus any directly attributable expenditure (means
transaction taxes e.g. legal fees, property transfer taxes and other transaction costs) incurred to acquire the
property.

The followings are not added to cost of an investment property:


• Start-up costs (unless necessary to bring the property to the condition necessary for it to be capable of
operating in the manner intended by management);
• Operating losses incurred before the investment property achieves the planned level of occupancy (e.g
initially property is not fully on rent because of which there are losses); or
• Abnormal waste incurred in constructing or developing the property.
• Selling and administrative expenses.

82
Page 2 of 29
Fear Allah wherever you are. Allah is one who knows what is in every heart.

Measurement after initial recognition


After initial recognition an entity may choose as its accounting policy:
• The fair value model; or
• The cost model.

The chosen policy must be applied to all the investment property of the entity.

Once a policy has been chosen it cannot be changed unless the change will result in a more appropriate
presentation. IAS 40 states that a change from the fair value model to the cost model is unlikely to result in a
more appropriate presentation.

Even if the entity uses cost model, fair value of the property is to be disclosed in notes to the financial statements.

Fair value model for investment property


Definition: Fair value
Fair value is the price that would be received on selling an asset or paid to settle a liability in an orderly
transaction between market participants at the measurement date.

Under the fair value model the entity should:


• revalue all its investment property to ‘fair value’ at the end of each financial year; and
• recognise any resulting gain or loss in profit or loss for the period.

The investment property measured under fair value model would not be depreciated.

This is different to the revaluation model of IAS 16, where gains are accumulated as a revaluation surplus within
equity.

If an entity’s policy is to measure investment properties at fair value but fair value of a property cannot be
measured reliably such investment property shall be measured at cost model (for example an investment
property under construction)

However, if the entity expects the fair value of the investment property under construction to be reliably
measureable when construction is complete it shall measure that investment property under construction at cost
until either its fair value becomes reliably measurable or construction is completed (whichever is earlier).

If it is not possible to arrive at a reliable fair value figure (because it may be at a remote location) then the cost
model should be adopted for that investment property as in IAS 16. All other investment properties will remain at
fair value model.

This is an exception to normal rule that all investment properties must be valued under either cost model or fair
value model.

83
Page 3 of 29
Cost model for investment property
The cost model in IAS 40 follows the provisions of IAS 16. The property is measured at cost less accumulated
depreciation (related to the non-land element) and less accumulated impairment loss if any.

Example: Accounting for investment property


On 1 January 2011 Entity P purchased a building for its investment potential. The building cost Rs. 1 million with
transaction costs of Rs. 10,000.

The depreciable amount of the building component of the property at this date was Rs. 300,000.

The building element has a useful life of 50 years.

At the end of 2011 the property’s fair value had risen to Rs. 1.3 million.

Answer:
Cost model
The amounts which would be included in the financial statements of Entity P at 31 December 2011, under the cost
model are as follows:

The property will be included in the statement of financial position as follows:

Rs.
Cost (1,000,000 + 10,000) 1,010,000
Accumulated depreciation (300,000 ÷ 50 years) (6,000)

Carrying amount 1,004,000

The statement of profit or loss will include depreciation of Rs. 6,000. Fair value of Rs. 1.3 millions will be disclosed
in notes to financial statements.

Fair value model


The amounts which would be included in the financial statements of Entity P at 31 December 2011, under the fair
value model are as follows:

The property will be included in the statement of financial position at its fair value of Rs.

1,300,000.

The statement of profit or loss will include a gain of Rs. 290,000 (Rs. 1,300,000 – Rs.

1,010,000) in respect of the fair value adjustment.

Example: In the light of poor market interest rates, the financial controller of Abbott Inc was instructed by the
managing director, in June 2019 to invest some of the company’s surplus cash in a plot of land costing 1 million.

84
Page 4 of 29
“Verify news/message before you spread.”

This land may be used by Abbott Inc in the future to build a new factory on, or it may be sold to realize a profit,
depending on property prices in the coming years.

It is now the year end, June 30, 2019 and the financial controller is preparing Abbott’s financial statements for the
presentation to the Board. He knows that the land has fallen slightly in value to 950,000, but is unsure of how to
account for it.

Required: Advise the financial controller about the correct accounting treatment.

A. land is to be classified as Investment property in the financial statements. According to IAS 40 land held for
currently undetermined future use be classified as Investment Property (means if an entity has not determined
that whether it will use the land as owner occupied property (IAS 16) or for short term sale in the ordinary course
of business (IAS 2), the land is regarded as held for capital appreciation)
As per IAS 40 the company has a choice of using the cost model or the fair value model. If cost model is used then
land will be carried as cost.

If the company uses fair value model then a fair value loss of 50,000 should be accounted for in the financial
statements for the year ended June 30, 2019 and land will be carried at 950,000 in the statement of financial
position.

Example: D Ltd. owns a factory with an opening carrying amount of 40 Million. At 1 Jan 2019, the directors
decided to sell the property but have continued to use the factory for manufacturing during the year.

They wish to classify the building as an investment property and recognize a gain of 32 Million in the income
statement based on the market value at 31 Dec 2019 of 72 Million. It is estimated the factory has a remaining life
of 20 years on 01.01.2019. D Ltd. Apply the cost model for owner occupied properties (IAS 16).

Required: Discuss the director’s proposed accounting treatment of the property.


A. For a property to be classified as an investment property, following conditions in the definition must be met.
B. It is a property (land or building or part of a building or both) held by the owner to earn rentals or for
capital appreciation or both.

In addition IAS 40 specifically states that owner occupied property awaiting disposal is not an Investment
property.

In light of the above, it is quite evident that D Ltd. Is currently using the property as owner occupied therefore
they will continue to recognize the said property under IAS 16 using cost model. Therefore there is no question of
recording the fair value gain.

85
Page 5 of 29
Why investment properties are treated differently from other properties
An investment property is held primarily because it is expected to increase in value over time (capital
appreciation) or it is held to earn rentals. It generates economic benefits for the entity because it might earn
regular stream of income in the form of rentals or might be sold at a profit. An investment property also differs
from owner-occupied properties (IAS 16) because it generates cash flows that are largely independently of other
assets held by an entity.

The most relevant information about an investment property is its fair value (the amount for which it could be
sold). Therefore it is appropriate to re-measure an investment property to fair value each year and to recognise
gains and losses in profit or loss for the period.

86
Page 6 of 29
IAS 40 Practice Questions

Question 1. An entity owns two investment properties X and Y, the fair value of which are:
31 December 2006 31 December 2007
million Million
Property X 15 20
Property Y 10 8

The original cost of the properties was 9 million each when they were acquired on 1 january 2006. The entity uses
the fair value model to value all its investment properties.

Required ;
How will these transactions be accounted for in the financial statements for the year ended 31.12.2006 and 2007.

Question 2
An entity purchased an investment property on 1 January, 2014 for a cost of 400,000. The property has a useful
life of 50 years, with no residual life and at 31 December 2014 had a fair value of 560,000. On 1 January, 2015 the
property was sold for net proceeds of 540,000.

Required:
How will the disposal be treated using the Cost Model and the Fair Value Model

Question 3
XYZ plc. owns three investment properties X, Y, Z, the value of which are:
31 December 2002 31 December 2003
Million Million
Property X 27 32
Property Y 18 24.5
Property Z 26 23.5

The original cost of the properties was 20 million each when they were purchased on 1st January 2001. The entity
uses the fair value model to value all of its investment properties.

Required:
Show the financial Statement extract for each of the three years ended 31 st December, 2003.

Answer 1
Statement of profit or loss Extracts for the year ended 31/12 2006 2007
Fair value income M M
Property X 6 5
Property Y 1 (2)
7 3
Statement of financial position Extracts as at 31/12 2006 2007
Non-current assets: M M
Investment Property X 15 20

87
Page 7 of 29
Investment Property Y 10 8
25 28

Answer 2
Cost Model Fair Value Model

Cost 400,000 Cost 400,000


31.12.2014 31.12.2014
Useful life 50 No concept of depreciation
Annual depreciation Fair value 560,000
(400,000/50 yrs.)
8,000
Fair value gain 160,000
Income Statement Extracts Income Statement Extracts for
for the year ended the year ended 31/12/2014
31/12/2014

Depreciation 8,000 Fair value Income 160,000

SOFP Extract as at 31/12/2014 SOFP Extract as at 31/12/2014


Non-Current Assets Non-Current Assets
Investment Property (400,000- 392,000 Investment Property 560,000
8,000)

Upon Disposal on 1/1/2015 Upon Disposal on 1/1/2015


Bank 540,000 Bank 540,000
Gain (I.S) 148,000 Loss (I.S) 20,000
Investment Property 392,000 Investment Property 560,000

Answer 3
Statement of profit or loss Extract for the year ended 2001 2002 2003
31/12
Fair Value Income M M M
Property X - 7.0 5.0
Property Y - (2.0) 6.5
Property Z - 6.0 (2.5)
- 11.0 9.0

SOPF Extract as at 31/12 2001 2002 2003


Non-Current Assets M M M
Investment Property X 20.0 27.0 32.0
Investment Property Y 20.0 18.0 24.5
Investment Property Z 20.0 26.0 23.5
Total 60.0 71.0 80.0

88
Page 8 of 29
“If you want that Allah love you then obey Allah.”

FAR-1 QB
IAS-40

Question:
V Ltd owns several properties and has a year end of 31 December. Wherever possible, V Ltd carries investment
properties under the fair value model.

Property 1 was acquired on 1 January 2011. It had a cost of Rs. 1 million, comprising Rs. 500,000 for land and Rs.
500,000 for buildings. The buildings have a useful life of 40 years. V Ltd uses this property as its head office.

Property 2 was acquired many years ago for Rs. 1.5 million for its investment potential. On 31 December 2017 it
had a fair value of Rs. 2.3 million. By 31 December 2018 its fair value had risen to Rs. 2.7 million. This property has
a useful life of 40 years.

Property 3 was acquired on 30 June 2012 for Rs. 2 million for its investment potential. The directors believe that
the fair value of this property was Rs. 3 million on 31 December 2017 and Rs. 3.5 million on 31 December 2018.
However, due to the specialised nature of this property, these figures cannot be corroborated (means not reliable
figures). This property has a useful life of 50 years.

Required
a) For each of the above properties briefly state how it would be treated in the financial statements of V Ltd for
the year ended 31 December 201 8, identifying any impact on profit or loss.
b) Produce an analysis of properties of V Ltd showing movement in opening and closing balances for the year
ended 31 December 201 8, showing each of the above properties separately (means a schedule).

Solution:
a) Treatment in the financial statements for the year ended 31 December 2018

Property 1
This is used by V Ltd as its head office and therefore cannot be treated as an investment property. It will be stated
at cost minus accumulated depreciation in the statement of financial position, unless revalued under IAS 16 the
depreciation for the year will be charged in the statement of profit or loss.

Property 2
This is held for its investment potential and should be treated as an investment property. It will be carried at fair
value, V Ltd policy of choice for investment properties. It will be re-measured to fair value at each year end and
any resultant gain or loss taken to the statement of profit or loss (Rs. 400,000 gain in 2018) (2.7M – 2.3M)

Property 3
This is held for its investment potential and should be treated as an investment property. However, since its fair
value cannot be measured reliably it will be held at cost minus accumulated depreciation in the statement of
financial position. The depreciation for the year will be an expense in the statement of profit or loss.

89
Page 9 of 29
This situation provides the exception to the rule whereby all investment properties must be held under either the
fair value model, or the cost model.

b) Analysis of property, plant and equipment for the year ended 31 December 2018
Investment
Head Officeproperty held atInvestment property
Property (W1) fair value held at cost (W2) Total

Rs. Rs. Rs. Rs.


Cost/valuation
On 1 January 2018 1,000,000 2,300,000 2,000,000 5,300,000
Revaluation - 400,000 - 400,000
On 31 December 2018 1,000,000 2,700,000 2,000,000 5,700,000

Accumulated depreciation
On 1 January 2018 (W-1) 87,500 - (W-2) 220,000 307,500
Charge for the year (W1) 12,500 - (W-2) 40,000 52,500
On 31 December 2018 100,000 - 260,000 360,000
Carrying amount

On 31 December 2018 900,000 2,700,000 1,740,000 5,340,000


On 31 December 2017 912,500 2,300,000 1,780,000 4,992,500

*Note: There is no concept of charging depreciation of Investment properties if carried at Fair value Model of IAS-
40

Workings
(1) Depreciation on Property 1 Rs.
Brought forward (500,000 ÷ 40 x 7) 87,500
2018 (500,000 ÷ 40) 12,500
(2) Depreciation on Property 3
Rs.
Brought forward (2,000,000 ÷ 50 x 5.5) 220,000
2018 (2,000,000 ÷ 50) 40,000

90
Page 10 of 29
And whoever relies upon Allah – then He is sufficient for him. (A promise of Allah)

Introduction of other comprehensive income

Change in use of property


The entity may, during the current reporting period, change the use of the property.

There are following possible changes:

From To
Owner-occupied property (IAS 16) Investment property (IAS 40)
Inventories (IAS 2) Investment property (IAS 40)
Investment property(IAS 40) Owner-occupied property (IAS 16)
Investment property(IAS 40) Inventories (IAS 2)

If the entity uses the cost model (for properties in IAS 40)
If the entity uses the cost model, a change in use will not change the carrying amount of the property because (a)
investment property, (b) property plant and equipment and (c) inventory are all carried at their cost.

If the entity uses the fair value model (for properties in IAS 40)
If the entity uses the fair value model then there may be measurement implications.

a) Change from owner-occupied property to investment property


The entity must first revalue the property, plant and equipment to its fair value and the resultant increase or
decrease is recognised in accordance with the standard on property, plant and equipment (IAS 16):

An increase is:
o first credited to profit or loss (only where it reverses a previous loss); and
o then credited to evaluation surplus (in other comprehensive income) as in IAS 16;

A decrease is:
• first debited to the revaluation surplus (if the revaluation surplus account has a balance as a result of previous
revaluations); and
• the excess is then debited to profit or loss (revaluation loss).

Example 1: change from owner-occupied to investment property


Fantastic Limited had its head office located in Murree. During a heavy landslide on 30 June 2015, a building
nearby, which it was renting to Honda Limited, was destroyed. As Honda Limited was a valued customer, Fantastic
Limited decided to lease 80% of the head office to them as a ‘replacement’ (assume portions cannot be sold
separately).

The head office was purchased on the 1 January 2015 for 600 000 (total useful life: 10 years)

91
Page 11 of 29
On the 30 June 2015, the fair value of the head office was 800 000. There was no change in fair value at 31
December 2015.

Fantastic Ltd uses:


• the fair value model to measure its Investment Property; and
• the cost model to measure its property, plant and equipment.

Required:
Provide the journal entries in the books of Fantastic Ltd for the year ended 31 December 2015.

b) Change from inventories to investment property

Property that is intended for sale in the ordinary course of business is carried according to IAS 2. In case it is
classified as an investment property then the property is transferred to investment property at its carrying
amount in accordance with IAS 2 (i.e. lower of cost and NRV) and then revalued to fair value in accordance with
IAS 40 at the date of transfer, with any difference going to profit or loss.

Example 2:
Marvelous Limited has a building purchased with the intension of sale. On 30.06.2015 the management decided
to lease it out to earn rentals. On 30.06. 2015 the fair value was 1,300,000. The cost of purchase on 01.01.2015
was 1,000,000. There was no change in fair value at 31 December 2015.

Marvelous Limited uses the fair value model to measure its Investment Property

Required:
Provide the journal entries for the year ended 31 December 2015.

c) Change from investment property to owner-occupied property or inventories

The entity must first adjust the property’s carrying amount to fair value on the date of change. The resultant
change must be taken to the profit or loss as a gain or loss caused by a fair value adjustment in accordance with
IAS 40. The fair value on date of transfer, determined in accordance with IAS 40, will be treated as cost of the
owner-occupied property or inventory for future valuation. If the investment property is classified as owner-
occupied, it will then be depreciated over the remaining useful life and measured in terms of IAS 16: Property,
plant and equipment. If the investment property is classified as inventory, it will then be measured in terms of IAS
2: Inventories i.e. at lower of cost and NRV.

Example 3: change from investment property to owner occupied property


Super Limited owned and leased out a building in Peshawar, which was correctly classified as an investment
property on 31 December 2014 and carried at fair value model.

During an earthquake the head office of Super Limited was destroyed, with the result that Super Ltd had to
relocate its head office into the Peshawar Building. The tenants of this building were forced to move out as of 30
June 2015.

The fair value of the building on 31 December 2014 was 200 000.

92
Page 12 of 29
On the 30 June 2015 the buildings fair value was 260 000 and had a remaining useful life of 10 years.

Required:
Provide the journal entries in Super Limited’s records for the year ended 31 December 2015.

Answer:
Solution to example 1
1 January 2015
Head-office building: (PPE) 600,000
Bank/ liability 600,000
30 June 2015
Depreciation 30,000
Acc depreciation 30,000
Depreciation to date of change in
use (600 000 / 10 x 6 / 12 months)
30 June 2015
Acc depreciation 30,000
Head-office building: (PPE) 30,000
30 June 2015
Head-office building: (PPE) 230,000
Revaluation surplus 230,000
Revaluation of head office to fair
value on date of change in use
(800 000 – (600 000 – 30 000))
30 June 2015
Investment property 800,000
Head-office building: (PPE) 800,000
31.12.2015 No further depreciation and no fair value gain or loss.
31 december 2015
Revaluation surplus 12,105
Retained earnings 12,105
[230,000/9.5x6/12]

Solution to example 2
01.01.2015
Inventory 1,000,000
Bank 1,000,000
30.06.2015
Investment property 1,000,000
Inventory 1,000,000
30.06.2015
Investment property 300,000
Fair value gain 300,000
(1,300,000 -1,000,000)

93
Page 13 of 29
Solution to example 3
30.06.2015
Investment property 60,000
Fair value gain 60,000
(260,000 -200,000)
Head-office building: (PPE) 260,000
Investment property 260,000
31.12.2015
Depreciation 13,000
Acc depreciation 13,000
Depreciation to date of change in use (260
000 / 10 x 6 / 12 months)

Practice questions:
Question-1: (Investment property to owner-occupied property)

ABC limited company owns two buildings:


• A head office building located in Quetta; and
• Another office building located in Karachi.

The office building located in Quetta is used as ABC Limited’s head office. A minor earthquake, on 30 June 2015
destroyed this building.
• The building in Quetta was purchased on 1 January 2015 for 1,200,000 (total useful life 10 year and residual
value: nil).

The property in Karachi was leased to a tenant, Spider limited. After the earthquake, ABC limited urgently needed
new premises for its head office. Since spider limited was always late in paying their lease rentals, ABC limited
decided to immediately evict them and move their head office to this building situated in Karachi.
• The building in Karachi was purchased on the 1 January 2015 for 500,000
• On the 30 June 2015 the fair value of the building in Karachi was 950,000
• There was no change in fair value at 31 December 215
• The total useful life was estimated to be 10 years from date of purchase and the residual value was estimated
to be nil.

ABC limited uses:


• The cost model to measure its property, plant and equipment; and
• The fair value model for its investment properties.

Required:
Journalize the above transactions in the books of ABC limited for the year ended 31 December 2015.

Question-2: (Owner occupied property to investment property)


XYZ Limited owns an office block.
• XYZ Limited had occupied the office block from date of purchase until 30 June 2015.
• The office block had cost 1,000,000 on 1 January 2014

94
Page 14 of 29
• Its residual value is estimated to be nil and total useful life is estimated to be 10 years respectively (both
estimates have remained unchanged).
• On 30 June 2015, XYZ limited moved out of the office block and thereafter rented it to tenants under short-
term operating leases.
• The fair value of the office block was equal to its carrying amount on 30 June 2015
• The fair value of the office block was 800,000 on 31 December 2014 and 1,500,000 on 31 December 2015.

XYZ limited measure owner-occupied property using the cost model and investment property using the fair value
model.

Required:
Show all journals relating to the office block in the books of XYZ limited for the year ended 31 December 2015

Solution-1:
Debit Credit
1 January 2015
Quetta building: cost (PPE) 1,200,000
Bank/liability 1,200,000
Purchase of Quetta building (owner-occupied)
Karachi building: cost (Investment property) 500,000
Bank/liability 500,000
Purchase of Karachi building (leased to a tenant)
30 June 2015
Depreciation (1,200,000/10 x 6/12 months) 60,000
Accumulated deprecation 60,000
Depreciation of building (PPE) to date of destruction
Acc. Dep 60,000
loss (bal.) (1,200,000 - 60,000) 1,140,000
Quetta building: Acc. Dep and impairment losses 1,200,000
(PPE)
Write-off after earthquake
Karachi building: cost (investment prop) (950,000 – 500,000) 450,000
Fair value gain (P&L) 450,000
Revaluation of investment property prior to change in use
Karachi building: PPE 950,000
Karachi building (Investment property) 950,000
Transfer from investment property to property plant and equipment
31 December2015
Depreciation 950,000/9.5 x 6/12 months 50,000
Karachi building: accumulated depreciation 50,000
Depreciation to year end Karachi building (PPE)

95
Page 15 of 29
Solution-2:
a) Office Block
Debit Credit
30 June 2015
Depreciation (1,000,000 – 0)/ 10y x 6/12 50,000
Office block: Acc. Dep (PPE) 50,000
Depreciation on the owner-occupied office block to date of change in use
Acc. dep. O/bal (1,000,000 – 0)/10 y+ 150,000
20X5 Dep 50,000 (above)
Office block: PPE 150,000
No revaluation surplus or loss
30.06.2015
Investment property 850,000
Head office building 850,000

Transfer from property plant and equipment to investment property on the date of
change in use
31 December 2015
Office block: (Investment property) (1,500,000 – 850,000) 650,000
Fair value adjustment to investment property 650,000
(income)
Investment property re-measured to fair value at year-end

Gain or loss on disposal

Gains or losses on disposals of investment properties are included in profit or loss in the period in which the
disposal occurs.

Example: Disposal of investment property


On 1 January 2011 Entity P purchased a building for its investment potential. The building cost Rs. 1 million with
transaction costs of Rs. 10,000.

The depreciable amount of the building component of the property at this date was Rs. 300,000.

The building element has a useful life of 50 years.

At the end of 2011 the property’s fair value had risen to Rs. 1.3 million.

96
Page 16 of 29
Every success is based on the deeds do great deeds for great success.”

Answer:
Cost model
The amounts which would be included in the financial statements of Entity P at 31 December 2011, under the cost
model are as follows:

The property will be included in the statement of financial position as follows:

Rs.
Cost (1,000,000 + 10,000) 1,010,000
Accumulated depreciation (300,000 ÷ 50 years) (6,000)

Carrying amount 1,004,000

The statement of profit or loss will include depreciation of Rs. 6,000. Fair value of Rs. 1.3 millions will be disclosed
in notes to financial statements.

Fair value model


The amounts which would be included in the financial statements of Entity P at 31 December 2011, under the fair
value model are as follows:

The property will be included in the statement of financial position at its fair value of Rs. 1,300,000.

The statement of profit or loss will include a gain of Rs. 290,000 (Rs. 1,300,000 – Rs. 1,010,000) in respect of the
fair value adjustment.

Disposal of investment property:


Suppose the above investment property is sold on 01.01.2012 for Rs. 1,550,000, selling costs are Rs. 50,000.

The amount that would be included in the statement of profit or loss for the year ended 31.12.2012 in respect of
this disposal under the cost model is as follows:

Cost model Rs.


Sale value 1,550,000
Selling costs (50,000)

Net disposal proceeds 1,500,000


Minus: Carrying amount (1,004,000)

Gain on disposal 496,000

97
Page 17 of 29
The amount that would be included in the statement of profit or loss for Year 2 in respect of this disposal under
the fair value model is as follows:

(Fair value model Rs.


Sale value 1,550,000
Selling costs (50,000)

Net disposal proceeds 1,500,000


Minus: Carrying amount (1,300,000)

Gain on disposal 200,000

Disclosure requirements
The following disclosures are required by IAS 40 in the notes to the accounts.

Disclosure requirements applicable to both the fair value model and the cost model

whether the fair value model or the cost model is used

the methods and assumptions applied in arriving at fair values

the extent to which the fair value of investment property was based on a valuation by a qualified,
independent valuer with relevant, recent experience

amounts recognised as income or expense in the statement of profit or loss for:
1. rental income from investment property
2. operating expenses in relation to investment property

details of any restrictions on the ability to realise investment property or any restrictions on the remittance of
income or disposal proceeds

the existence of any contractual obligation to purchase, construct or develop investment property or for
repairs, maintenance or enhancements (future commitments)

Disclosure requirements applicable to the fair value model only


There must be a reconciliation, in a note to the financial statements, between opening and closing values for
investment property, showing:

additions during the year

net gains or losses from fair value adjustments

This reconciliation should show separately any amounts in respect of investment properties included at cost
because their fair values cannot be estimated reliably.

For investment properties included at cost because fair values cannot be estimated reliably, the following should
also be disclosed:

a description of the property

an explanation as to why fair values cannot be determined reliably

if possible, the range within which the property’s fair value is likely to lie.

98
Page 18 of 29
Disclosure requirements applicable to the cost model only

the depreciation methods used

the useful lives or depreciation rates used

gross carrying amounts and accumulated depreciation at the beginning and at the end of the period

A reconciliation between opening and closing values showing:
I. additions
II. depreciation
III. impairment losses
IV. transfers

When the cost model is used, the fair value of investment property should also be disclosed. If the fair value
cannot be estimated reliably, the same additional disclosures should be made as under the fair value model.

Extra practice questions (before this question revise cash flow questions Q. 6,7,8 and Drum limited )

Q. Following are the extracts from the financial statements of Sunday Traders Limited (STL) for the year
ended 30 June 2019:

Statement of financial position as on 30 June 2019

2019 2018 2019 2018


Assets Rs. in million Equity & liabilities Rs. in million
Property, plant and equipment 8,555 7,240 Share capital (Rs. 100 each) 4,650 3,450
Investment property 1,800 1,120 Share premium 1,600 1,240
Stock in trade 4,800 4,500 Retained earnings 1,652 (655)
Prepayments 184 268 Long term loans 6,024 6,523
Trade receivables 3,800 3,600 Trade payables 3,422 5,390
Contract liability (advance
Cash 194 480 from customer) 250 40
Accrued liabilities 310 180
Interest payable 135 110
Current maturity of long
term loans 850 700
Provision for taxation 440 230
19,333 17,208 19,333 17,208

Statement of profit or loss for the year ended 30 June 2019


Rs. in million
Sales 29,700
Cost of sales (15,750)
Gross profit 13,950
Distribution cost (6,185)
Administrative cost (2,302)
Other income 404

99
Page 19 of 29
Profit before interest and tax 5,867
Interest expense (1,210)
Profit before tax 4,657
Tax expense (1,150)
Profit after tax 3,507

Additional information:
(i) 72% of sales were made on credit.
(ii) Depreciation expense for the year amounted to Rs. 750 million which was charged to distribution and
administrative cost in the ratio of 3:1.
(iii) Distribution cost includes:
▪ Rs. 40 million in respect of loss on disposal of equipment. The written down value at the time of
disposal was Rs. 152 million.
▪ impairment loss on vehicles amounting to Rs. 24 million.
(iv) Loan instalments (including interest) of Rs. 1,984 million were paid during the year.
(v) Other income comprises of:
▪ increase in fair value of investment property amounting to Rs. 220 million.
▪ rent received from investment property amounting to Rs. 184 million.
(vi) During the year, STL issued right shares at premium.

Required:
Prepare STL’s statement of cash flows for the year ended 30 June 2019 using direct method. (19)

A. Sunday Traders Limited


Statement of Cash Flows
For the year ended 30 June 2019

Cash flows from operating activities Rs. in million


Cash receipts from customers (8,316 + 21,394) (W-1) 29,710
Cash receipts from tenants 184
Cash paid to suppliers (W-2) (18,018)
Cash paid to other vendors (W-3) (7,459)
Cash generated from operations 4,417
Interest paid (1,185)
Income taxes paid (940)
Net cash inflow from operating activities 2,292

Cash flows from investing activities


Purchase of property, plant and equipment (2,241)
Proceeds from disposal of property, plant and equipment 112
Purchase of investment property (460)
Net cash outflow from investing activities (2,589)

Cash flows from financing activities


Proceeds from issue of shares (1,200 + 360) 1,560

100
Page 20 of 29
Dividend paid (1,200)
Repayment of loans (799)
New loans acquired 450
Net cash inflow from financing activities 11
Net decrease in cash and cash equivalents (286)
Cash and cash equivalent at the beginning of the year 480
Cash and cash equivalent at the end of the year 194

Workings
Property Plant and Equipment
b/d 7,240 Depreciation 750
Cash 2,241 Disposal 152
Impairment 24
c/d 8,555

Investment property
b/d 1,120
F.V gain 220
Cash 460
c/d 1,800

Stock
b/d 4,500 COS 15,750
Purchases 16,050

c/d 4,800

Prepayments + Accrued liabilities


b/d 268 b/d 180
Cash 7,459 Expense 7,673
(6,185+2,302-750-40-24)
c/d 310 c/d 184

Trade receivable + Advance from customer


b/d 3,600 b/d 40
Sales 21,384 Cash 21,394
(29,700×72%)
c/d 250 c/d 3800

101
Page 21 of 29
Share Capital
b/d 3,450
Cash 1,200

c/d 4,650

Share premium
b/d 1,240
Cash 360

c/d 1,600

Retained Earnings
b/d 655
Dividend 1,200 PAT 3,507

c/d 1,652

Loan
b/d 6,523
Cash 799 b/d 700
(1,984-1185) Cash 450
c/d 6,024
c/d 850

Trade payables
b/d 5,390
Cash 18,018 Purchases 16,050

c/d 3,422

Interest payable
b/d 110
Cash 1,185 Expense 1,210

c/d 135

Tax payable
b/d 230

Cash 940 Expense 1,150


c/d 440

102
Page 22 of 29
Disposal
PPE 152
Cash 112
Loss 40

Extra practice question (IAS 16 and IAS 23)


Q. The following information pertains to Monday Limited (ML):

(i) The balances of property, plant and equipment as on 1 January 2018:

Cost/Revalued Accumulated
Assets amount depreciation
----------- Rs. in million -----------
Office building 240 36
Equipment 190 60

Revaluation surplus related to the office building as at 1 January 2018 amounted to Rs. 8.5 million.
(ii) On 1 September 2018, a new equipment was acquired by making payment of
Rs. 70 million to the supplier. An old equipment was also given in exchange to the supplier. The
fair values of old and new equipment were assessed at Rs. 21 million and Rs. 93 million
respectively. The old equipment had been acquired at a cost of Rs. 40 million on 1 July 2016. Cost
incurred on installing the new equipment amounted to Rs. 5 millions.
(iii) On 1 January 2018, ML commenced construction of a manufacturing plant. The whole process of
assembling and installation was completed on 31 October 2018. However, the work was stopped
from 16 to 31 August 2018 due to unexpected rains.

The total cost of Rs. 660 million incurred on the plant was paid as under:

Description Payment date Rs. in million


1st payment 1 February 2018 140
2nd payment 1 April 2018 214
3rd payment 1 September 2018 146
4th payment 1 December 2018 160

The plant was financed through a bank loan of Rs. 500 million obtained on 1 March 2018. The loan
carries a mark-up of 18% payable annually. The surplus funds available from the loan were
invested in a saving account and earned Rs. 17 million during capitalization period.

(iv) On 31 December 2018, the revalued amount of office building was assessed at Rs. 178 million by
Precise Valuers, an independent valuation firm.

103
Page 23 of 29
(v) Other relevant details are as follows:
Assets Depreciation Method Life/rate Subsequent
Measurement
Office building Straight Line 20 years* Revaluation
Equipment Reducing Balance 20% Cost
Manufacturing Plant Straight Line 15 years Cost

* Remaining life at the date of last revaluation

ML accounts for revaluation on net replacement value method and transfers the maximum
possible amount from revaluation surplus to retained earnings on an annual basis.

Required:
In accordance with IFRSs, prepare a note on ‘Property plant and equipment’ for inclusion in ML’s financial
statements for the year ended 31 December 2018.
(Comparatives figures are not required) (17)

Answer
Monday Limited
Notes to the financial statements
For the year ended 31-12-2018
Rs. in million
Office Building Equipment Plant Total
Gross carrying amount
Balance as on 1-1-2018 240 190 - 430
Addition - 96 699.25 (W) 795.25
(21+70+5)
Disposal - (40) - (40)
Elimination (48) - - (48)
Revaluation (W) (14) - - (14)
Closing balance 178 246 699.25 1,123.25
Accumulated depreciation
Balance as on 1-1-2018 36 60 96
Depreciation 12 30.48* 7.77 50.25
(240/20) (699.25/15
X2/12)
Disposal - (15.04)** - (15.04)
Elimination (48) - - (48)
Closing balance - 75.44 7.77 83.21
Carrying Amount 178 170.56 691.48 1,040.04

104
Page 24 of 29
Allah wants to forgive us, We need to repent often and ask Allah to forgive us.

Office building Equipment Plant


Measurement basis Revaluation model Cost model Cost model
Useful life/depreciation 20 years 20% 15 years
rate
Depreciation method Straight line Reducing balance Straight line

The last revaluation was performed on 31.12.2018 by Precise Valuers, an independent firm of valuers.

The carrying amount of office building has the cost model being used instead:
240-10(revaluation surplus) = 230
Depreciation per annum = 230/20 = 11.5
230-(11.5 x 4 up to 31.12.2018) = 184

* Depreciation:
Opening WDV [190-60] 130
Less: opening WDv of disposal (28.8)
101.2
X 20% 20.24
+ 28.8 x 20% x 8/12 3.84
+96 x 20% x 4/12 6.40
30.48

**Accumulated depreciation of disposal:


Cost (1-7-2016) 40
X 20% X 6/12 (4)
31-12-2016 36
X 20% (7.2)
31-12-2017 28.8
X 20% X 8/12 (3.84)
1-9-2017 24.96
Acc. Dep [40-24.96] 15.04

Cost on WIP 660


(140+214+146+160)

Borrowing Cost : 56.25


500 X 18% X [(8-0.5)/12]
Less: Investment Income (17)
39.25
Total 699.25

105
Page 25 of 29
Detail of payments
Owned Loan Change yourself to please ALLAH but not to please the people.
1-2 140 140 -
1-4 214 - 214
1-9 146 - 146
1-12 160 20(bal) 140

Revaluation [31-12-2018]
WDV [240- (36+240/20)] 192
FV 178
Revaluation Loss 14

R. Surplus 8 R. Loss 6
(8.5-0.5)

Rev. Surplus on 1-1-2018


240/20 = 12 X 3 = 36 [Three year after revaluation]

8.5 is the remaining balance after three years. So 8.5/17 = 0.5 will be transferred in current year.

Total surplus on last revaluation date would have been 8.5/17 x 20 = 10

Q.1[IAS 8 with investment property]


Following information have been extracted from the financial statements of Fakhr Limited (FL) for the year
ended 31 December 2019:

(i) 2019 2018 2017


Draft Audited Audited
--------- Rs. in million ---------
Net profit 84 98 72
Revaluation surplus arising during the year* 25 (14) -

*Transfer to retained earnings is made upon de-recognition of related asset.

(ii) Share capital and reserves as at 1 January:

2018 2017
----- Rs. in million -----
Share capital (Rs. 10 each) 300 300
Revaluation surplus 102 102
Retained earnings 348 276

106
Page 26 of 29
Change yourself to please ALLAH but not to please the people.

(iii) On 1 March 2018, FL declared a final cash dividend of 10% for the year ended 31 December 2017. On 1
November 2018, FL issued 40% right shares to its ordinary shareholders at Rs. 24 per share. On 1 August
2019, an interim bonus of 15% was declared.

Following matters need to be incorporated in the draft financial statements of FL:


(i) To provide more relevant and reliable information about investment property, it has been decided
to change the measurement basis for investment property from cost model to fair value model.
The only investment property of FL is a building purchased on 1 January 2016 at a cost of Rs. 150
million. 60% of the cost represents building component having estimated useful life of 20 years and
residual value of Rs.
10 million. The depreciation is included in the above draft financial statements. The fair value of the
investment property has increased by 6% in each year since acquisition.

Required:
Prepare FL’s statement of changes in equity (including comparative figures) for the year ended 31 December
2019. (‘Total’ column is not required) (18)

A.1 Fakhr Limited


Statement of Changes In Equity
For the year ended 31 December 2019

Ordinary share Share Revaluation Retained


capital premium surplus earnings
---------- Rs. in million ----------
Balance as at 31 December 2017 (As 300.00 - 102.00 348.00
given)
Effect of change in accounting policy - - - 26.54
[ 13 + 13.54 ] (Workings)
Balance as at 31 December 2017 – 300.00 - 102.00 374.54
Restated
Final cash dividend @ 10% for 2017 - - - (30.00)
( 300 x 10% )
Right issue @ 40% 120.00 168.00 - -
( 300 / 10 x ( 300 / 10 x
40% x 10) 40% x 14)
– Net profit: Restated (98 +14.11) - - - 112.11
(Workings)
- Revaluation Surplus - - (14.00) -
Balance as at 31 December 2018 – 420.00 168.00 88.00 456.65
Restated
Interim bonus dividend @ 15% for 63.00 (63.00) - -
2019 (420 x 15%)

107
Page 27 of 29
Total comprehensive income for the year
ended 31 December 2019
– Net profit (84 + 14.72) - - - 98.72
- Revaluation Surplus - - 25.00 -
Balance as at 31 December 2019 483.00 105.00 113.00 555.37

(i) Change in Policy of Investment Property :

At Cost Model : At FV Model :


1-1-2016 1-1-2016
Investment Property 150 Investment Property 150
Cash 150 Cash 150
31.12.2016 31.12.2016
Depreciation 4 X
Accumulated Depreciation 4
(150 x 60% - 10/12)
31.12.2016
Investment Property 9
X FV Gain 9
FV [150 X 1.06] = 159
C.A = 150
FV Gain = 9

31.12.2017 X
Depreciation 4
Accumulated Depreciation 4
31.12.2017
Investment Property 9.54
X FV Gain 9.54
FV [ 159 x 1.06 ] = 168.54 C.A
= 159.00
9.54
31.12.2018
Depreciation 4 X
Accumulated Depreciation 4
31.12.2018
Investment Property 10.11
X FV Gain 10.11
FV [168.54 x 1.06] = 178.65
C.A = 168.54
10.11
31.12.2019
Depreciation 4 X
Accumulated Depreciation 4

108
Page 28 of 29
31.12.2019
Investment Property 10.72
X FV gain 10.72
FV [ 178.65 x 1.06 ] = 189.369 C.A
= 178.65
10.72

Effect on Profits :
2016 2017 2018 2019
Reversal of Dep 4 Cr. 4 Cr. 4 Cr. 4 Cr.
Recording of gain 9 Cr. 9.54 Cr. 10.11 Cr. 10.72 Cr.
Net effect on Profit 13 Cr. 13.54 Cr. 14.11 Cr. 14.72 Cr.

109
Page 29 of 29
“Stay close to anything that reminds you of ALLAH”

Concepts Capital Maintenance

It simply means at least Capital/Equity at the beginning of the period should be equal to capital/Equity at the
end of the period. E.g. Rs. 10 millions.

It means companies should only pay dividends out of profits to maintain the capital.

Equity or capital = Total assets – Total liabilities


Discussion of capital account

There are two main concepts of capital maintenance:


❑ A financial concept of capital;
❑ A physical concept of capital.

Example:
• A Company commenced business on 01.01.2010 with 100,000 capital.
• It purchased 20,000 units of inventory at 5 per unit on 01.01.2010 at a cost of Rs. 100,000.
• It sold all inventory during the year for Rs. 120,000 in cash
• Therefore it resulted into profit of Rs. 20,000.
• If this profit is taken out in the form of drawings/dividends then it means closing equity is equal to the
equity at the start of the business, i.e. 100,000 (which means financial capital is maintained)
• However let’s assume that inventory cost 5.4 per unit at the end of the period due to inflation. Entity can
only buy 18,519 units (100,000/5.4) with 100,000, which means company has not maintained its operating
capacity and therefore physical capital is not maintained.
• Physical capital would have been maintained had the closing equity be equal to 20,000 units x 5.4
=108,000 (which would require profit of Rs.8,000 being retained in equity and therefore dividend should
not exceed 12,000)

Comparing the two concepts


In practice, almost all entities use financial capital maintenance, but both concepts can provide useful
information.

Financial capital maintenance is likely to be the most relevant to investors (external users) as they are
interested in maximising the return on their investment and therefore its purchasing power.

Physical capital maintenance is likely to be most relevant to management and employees (internal users) as
they are interested in assessing an entity’s ability to maintain its operating capacity. This is particularly true for
manufacturing businesses, where management may need information about the ability of the business to
continue to produce the same or a greater volume of goods.

1. Financial concept of capital:


In this concept we simply compare whether the closing equity is equal to or greater than the opening equity in
the statement of financial position.

110 Page 1 of 16
It also has two sub divisions:
a. Money financial capital maintenance. Under this concept, an entity makes a profit when its closing equity
(net assets) exceeds its opening equity (net assets) (without any inflation adjustment). Simply take the
amounts from the statement of financial position (after adjusting for capital raised or distributed). It is
most commonly used in financial accounting.
b. Real financial capital maintenance. Under this concept, an entity makes a profit when it’s closing equity
exceeds opening equity re measured by using general inflation rate (after adjusting for capital raised or
distributed). This is achieved by a simple double entry.

Adjustment to maintain opening equity

Credit
Statement of profit or loss/profit X

Inflation reserve X

This entry simply means transferring profits to equity to maintain the same purchasing power as the
entity had at the start of the period.

2. Physical concept of capital [in this concept it is seen that whether at the end of the period we have resources to
acquire same quantity of similar physical assets (e.g. inventories, property, plant and equipment, intangible assets
etc) which we had at the beginning of the period]

With a physical concept of capital maintenance, a profit is not earned during a period unless (after adjusting for
capital raised or distributed) the operating capability of the business (production or trading capability) is greater
at the end of the period than at the beginning of the period.

This concept simply compares the closing equity with opening equity re measured using specific rates of inflation
that apply to the individual components of the net assets of the business. Again, this is achieved by the same
simple double entry
Adjustment to maintain opening equity

Credit

Statement of profit or loss/profit X

Inflation reserve X
.
This entry simply means transferring profits to equity to maintain the same operating capacity as the entity
had at the start of the period.

111 Page 2 of 16
“No matter how hurt you are, You will always find comfort with ALLAH”

The following example should help you to understand this.

Example: Capital maintenance concepts


X Limited commenced business on 1 January with a single item of inventory which cost Rs.10, 000.
During the year it sold the item for Rs.14, 000 (cash).
During the year general inflation was 5% but the inflation specific to the item was 10%.

Profit is calculated under each concept in the following ways.


Inventory 10,000
Capital 10,000
Cash 14,000
Sale 14,000
Cost of sale 10,000
Inventory 10,000

Financial capital maintenance (Money terms)


Income statement:
Sale 14,000
COS (10,000)
G.P 4000

Statement of financial position


N.C.A -
C.A
Cash 14,000
Equity
Capital 10,000
Profit 4,000 14,000
14,000
Financial capital maintenance (real terms)
Income statement
Sale 14,000
COS (10,000)
4,000
Inflation reserve (10,000 x 5%)[opening capital x general rate of inflation] (500)
Remaining profit 3,500

112 Page 3 of 16
Statement of financial position
N.C.A -
C.A 14,000
14,000
Equity
Opening capital 10,000
Inflation reserve 500
10,500
Profit 3,500
14,000
Liability -
14,000
10,000 x 5% = 500
Profit 500
Inflation reserve 500
[Like general or specific reserve]

Capital maintenance physical


Income statement
Sale 14,000
COS (10,000)
Gross profit 4,000
Inflation reserve(10,000 x 10%)[opening capital x specific rate of inflation] (1,000)
Remaining profit 3,000

Statement of financial position


N.C.A -
C.A
Cash 14,000
14,000
Equity
Opening capital 10,000
Inflation reserve 1,000
11,000
Net Profit 3,000
14,000

10,000 x 10% = 1,000


Profit 1,000
Inflation reserve 1,000

Commentary on example:

113 Page 4 of 16
“When Love is for the sake of ALLAH, It never dies.”

Money financial capital maintenance


Under historical cost accounting, the profit is Rs.4, 000. If the business paid this out as a dividend it would have
Rs.10, 000 left.

Rs.10, 000 is the opening equity expressed as a number of units of currency. This means that the company
would have maintained its equity expressed as a number of units of currency. However, inflation in the period
has caused the purchasing power of the currency to decline. This means that Rs.10, 000 no longer has the same
purchasing power that it had a year ago. The company has not maintained its capital in real terms.

Real financial capital maintenance


To maintain its equity in real terms the company would have to ensure that it had the same purchasing power
at the year-end as it had at the start. Inflation was 5% so the company would need Rs.10, 500 at the year-end in
order to have the same purchasing power as it had at the start of the year. The company can achieve this by
transferring Rs.500 from profit and loss into an inflation reserve. Profit would then be reported as Rs.3, 500.

If the business paid out Rs.3, 500 as a dividend it would have Rs.10, 500 left. This is not enough t o buy the
same asset that it had at the start of the year. The asset has been subject to specific inflation of 10% therefore
the company would need Rs.11, 000 at the year-end in order to buy the same asset.

This means that the company would not have the same capacity to operate as it had a year ago.

Physical capital maintenance


To maintain its equity in physical terms the company would have to ensure that it had t h e same ability to
operate at the year-end as it had at the start. In other words it would need to have Rs.11, 000. The company
can achieve this by transferring Rs.1, 000 from profit and loss into an inflation reserve. Profit would then be
reported as Rs.3, 000. Therefore if operating capacity is to be maintained, then company should only pay a
dividend of 3,000.

Summary of capital maintenance:


financial capital financial capital Physical capital
maintenance( Money maintenance (Real terms) maintenance
terms)
Inflation adjustment No Yes; based on general Yes; based on specific
of opening capital inflation rate inflation rate
Also referred to as Historical cost accounting Constant purchasing Current cost accounting
power accounting
Most relevant to Investors (as they are Investors (as they are Management and
interested in maximizing interested in maximizing employees(as they are
return on investment) return on investment) interested in assessing
entity’s ability to maintain
operating capacity)

MCQs number 9 to 18

114 Page 5 of 16
Concepetual framework for financial reporting
Purpose of conceptual framework
The Conceptual Framework for Financial Reporting (Conceptual Framework) describes the objective of, and
the concepts for, general purpose financial reporting.

The purpose of the Conceptual Framework is to assist:


• IASB to develop IFRSs that are based on consistent concepts;
• preparers to develop consistent accounting policies when no Standard applies to a particular transaction
or other event, or when a Standard allows a choice of accounting policy; and
• all parties to understand and interpret the Standards.

The Conceptual Framework contributes to the stated mission of the IFRS Foundation and IASB i.e. to develop
Standards that bring transparency, accountability and efficiency to financial markets around the world.
The Conceptual Framework provides the foundation for Standards (IASs and IFRSs) that:
• contribute to transparency by enhancing the international comparability and quality of financial
information, enabling investors and other market participants to make informed economic decisions.
• strengthen accountability by reducing the information gap between the providers of capital and
management. IFRSs and Conceptual Framework are also source of information for regulators.
• contribute to economic efficiency i.e. the use of a single, trusted accounting language derived from
Standards based on the Conceptual Framework lowers the cost of capital and reduces international
reporting costs.

Contents of conceptual framework


The Conceptual framework is divided into eight chapters, namely:
Chapter 1 — The Objective Of General Purpose Financial Reporting
Chapter 2 — Qualitative Characteristics Of Useful Financial Information
Chapter 3 — Financial Statements And The Reporting Entity
Chapter 4 — The Elements Of Financial Statements
Chapter 5 — Recognition And Derecognition
Chapter 6 — Measurement
Chapter 7 — Presentation And Disclosure
Chapter 8 — Concepts Of Capital And Capital Maintenance

Status of conceptual framework


The Conceptual Framework is not a Standard and nothing in the Conceptual Framework overrides any
Standard (IASs or IFRSs) or any requirement in a Standard.

Elements of financial statements


The elements of financial statements defined in the conceptual framework are:
• Assets, liabilities and equity, which relate to a reporting entity’s financial position; and
• Income and expenses, which relate to a reporting entity’s financial performance

115 Page 6 of 16
These elements are linked to the economic resources, claims and changes in economic resources and claims
and are explained as under:
Item discussed Elements Definition or description
Economic resource Asset A present economic resource controlled by the entity as a result of
past events.
An economic resource is a right that has the potential to produce
economic benefits.
Claim Liability A present obligation of the entity to transfer an economic resource as
a result of past events.
Equity The residual interest in the assets of the entity after deducting all its
liabilities.
Changes in economic Income Increases in assets, or decreases in liabilities, that result in increase in
resources and claims, equity, other than those relating to contributions from holders of
reflecting financial equity claims.
performance Expenses Decreases in assets, or increases in liabilities, that result in decreases
in equity, other than those relating to distributions to holder of
equity claims.
Other changes in - Contributions from holders of equity claims, and distributions to
economic resources them.
and claims - Exchanges of assets or liabilities that do not result in increases or
decreases in equity.

Information to be useful for decision making


If financial information is to be useful, it must be relevant and faithfully represent what it purports to
represent. The usefulness of financial information is enhanced if it is comparable, verifiable, timely and
understandable.

It means information must have certain characteristics in order for it to be useful for decision making. The IASB
Conceptual Framework describes:
• fundamental qualitative characteristics; and
• enhancing qualitative characteristics

Fundamental qualitative characteristics


The fundamental qualitative characteristics are
• relevance; and
• faithful representation

Relevance
Relevant financial information is capable of making a difference in the decisions made by users. Information
may be capable of making a difference in a decision even if some users choose not to take advantage of it or
are already aware of it from other sources.

116 Page 7 of 16
The relevance of information is affected by its materiality. Information is material if omitting it or misstating it
could influence decisions that users make on the basis of financial information about a specific reporting
entity.

Faithful representation (True and fair view)


Financial reports represent economic phenomena in words and numbers. To be useful, financial information
must not only represent relevant phenomena, but it must also faithfully represent the substance of the
phenomena that it purports to represent. Although, in many circumstances, the substance of an economic
phenomenon and its legal form are the same, an accountant should be careful to identify when this might not
be the case.

To be a perfectly faithful representation, a depiction would have three characteristics. It would be complete,
neutral and free from error. Of course, perfection is seldom, if ever, achievable. The objective is to maximise
those qualities to the extent possible.

Enhancing qualitative characteristics


The qualitative characteristics that enhance the usefulness of information that is relevant and a faithful
representation are:
• comparability;
• verifiability
• timeliness; and
• understandability

Comparability
Comparability enables users to identify and understand similarities in, and differences among, items.
Information about a reporting entity is more useful if it can be compared with similar information about other
entities and with similar information about the same entity for another period or another date.

Consistency is related to comparability but is not the same. Consistency refers to the use of the same methods
for the same items, either from period to period within a reporting entity or in a single period across entities.
Consistency helps to achieve the goal of comparability.

Verifiability
This quality helps to assure users that information faithfully represents the economic phenomena it purports
to represent. Verifiability means that different knowledgeable and independent observers could reach
consensus that a particular depiction is a faithful representation. Quantified information need not be a single
point estimate to be verifiable. A range of possible amounts and the related probabilities can also be verified.
Verification can be direct or indirect.
• Direct verification means verification through direct observation, e.g. by counting cash or inventory.
• Indirect verification means checking the inputs to a model, formula or other technique and recalculating
the outputs using the same methodology. For example, the carrying amount of inventory might be verified
by checking the inputs (e.g. costs) and recalculating the closing inventory using the same assumption (e.g.
FIFO).

117 Page 8 of 16
Timeliness
This means having information available to decision-makers in time to be capable of influencing their
decisions. Generally, the older the information is the less useful it is.

Understandability
Information is made understandable by classifying, characterising and presenting it in a clear and concise
manner. Some phenomena are inherently complex and cannot be made easy to understand, however,
excluding the relevant information is not justified in such circumstances.

Financial reports are prepared for users who have a reasonable knowledge of business and economic activities
and who review and analyse the information diligently.

Cost constraint on useful information


Cost is a pervasive constraint on the information that can be provided by financial reporting. Reporting
financial information imposes costs, and it is important that those costs are justified by the benefits of
reporting that information.

The benefits obtained from financial information should exceed the cost of obtaining and providing it.
Information should not be provided if the cost is not worth the benefit.

Recognition criteria
Only items that meet the definition of an asset, a liability or equity are recognised in the statement of financial
position. Similarly, only items that meet the definition of income or expenses are recognised in the
statement(s) of financial performance.

However, not all items that meet the definition of one of those elements are recognised. Not recognising an
item that meets the definition of one of the elements makes the statement of financial position and the
statement(s) of financial performance less complete and can exclude useful information from financial
statements. On the other hand, in some circumstances, recognising some items that meet the definition of one
of the elements would not provide useful information.

An asset or liability is recognised only if recognition of that asset or liability and of any resulting income,
expenses or changes in equity provides users of financial statements with information that is useful, i.e. with:
• relevant information about the asset or liability and about any resulting income, expenses or changes in
equity; and
• a faithful representation of the asset or liability and of any resulting income, expenses or changes in
equity.

Items that fail to meet the criteria for recognition should not be included in the financial statements. However,
some of these items may have to be disclosed as additional details in a note to the financial statements.

118 Page 9 of 16
Recognition links elements of financial statements

Measurements of elements of financial statements


Elements recognised in financial statements are quantified in monetary terms. This requires the selection of a
measurement basis. A measurement basis is an identified feature, for example, historical cost, fair value or
fulfilment value, of an item being measured.

Applying a measurement basis to an asset or liability creates a measure for that asset or liability and for
related income and expenses. Consideration of the qualitative characteristics of useful financial information
and of the cost constraint is likely to result in the selection of different measurement bases for different assets,
liabilities, income and expenses.

Historical cost
Historical cost measure provides monetary information about assets, liabilities and related income and
expenses, using information derived, from the price of the transaction or other event gave rise to asset or
liability.

The historical cost of an asset, when it is acquired or created is the value of the cost incurred in acquiring or
creating the asset, comprising the consideration paid to acquire or create the asset. The historical cost of a
liability when it is incurred is the value of consideration received to incur liability.

The historical cost of an asset is updated overtime to depict, if applicable:


o The consumption of part of the economic resource that constitute the asset depreciation or amortization;
o Payments received that extinguish part or all of the asset (any receiveable);
o The effect of events that cause part or all the historical cost of the asset to be no longer
recoverable(impairment);

119 Page 10 of 16
o Accrual of interest to reflect any financing component of the asset.

The historical cost of a liability is updated over time to depict, if applicable:


• Fulfilment of part or all of the liability (payment);
• The effect of events that increase the value of the obligation (estimate change); and
• Accrual of interest to reflect any financing component of the liability.

Current value
Current value measures provide monetary information about assets, liabilities and related income and
expenses, using information updated to reflect conditions at the measurement date. Because of the
updating, current values of assets and liabilities reflect changes, since the previous measurement date, in
estimate of cash flows and other factors reflected in those current values. Unlike historical cost, the current
value of an asset or liability is not derived, from the price of the transaction or other event that gave rise to the
asset or liability.

Current value measurement bases include:


o Fair value
o Value in use for assets and fulfillment value for liabilities
o Current cost

Fair value
Fair value is the price that would be received to sell an asset, or paid to transfer a liability, in an orderly
transaction between market participants at the measurement date.

Fair value reflects the perspective of market participants—participants in a market to which the entity has
access. The asset or liability is measured using the same assumptions that market participants would use when
pricing the asset or liability if those market participants act in their economic best interest.

In some cases, fair value can be determined directly by observing prices in an active market. In other cases, it
is determined indirectly using measurement techniques, for example, cash-flows-based measurement
techniques reflecting all the following factors:
o estimates of future cash flows
o possible variations.
o The time value of money
o The price for bearing the uncertainty inherent in the cash (a risk premium or risk discount).
o Other factors; e.g liquidity, if market participants would take those factors into accounts in the
circumstances.

Value in use (for assets) and fulfillment value (for liabilities)


Value in use is the present value of the cash flows that an entity expects to derive from the use of an asset and
from its ultimate disposal. Fulfillment value is the present value of the cash that an entity expects to be obliged
to transfer as it fulfills a liability.

120 Page 11 of 16
Because value in use and fulfillment value are based on future cash flows they don’t include transaction cost
incurred on acquiring an asset or taking on a liability. However, value in use and fulfillment value include the
present value of any transaction cost that entity expects to incur on the ultimate disposal of the asset or on
fulfilling the liability.

Value in use and fulfillment value reflect entity specific assumptions rather than assumptions by market
participants.

Value in use and fulfillment value cannot be observed directly and or determined using cash flow based
measurement techniques. Value in use and fulfillment value reflect the same factors described for fair value
earlier, but from an entity specific perspective rather than from a market participant perspective.

Current cost
The current cost of an asset is the cost of an equivalent asset at the measurement date comprising the
consideration that would be paid at the measurement date plus the transaction cost that would be incurred
at that date. The current cost of a liability is the consideration that would be received for an equivalent liability
at the measurement date that would be incurred at that date. Current cost, like historical cost is an entity
value; it reflects prices in the market in which the entity would acquire the asset or would incur the liability.
Hence it is different from fair value in use and fulfillment value, which are exit value. However, unlike historical
cost, current cost reflects conditions at the measurement date.

In some cases, current cost cannot be determined directly by observing prices and in an active market and
must be determined directly by other means. For example, if prices are available only for new asset the
current cost of a used asset might need to be estimated by adjusting the current price of a new asset to reflect
the current age and condition of the asset held by the entity (depreciated replacement cost).

Example: Measurement Bases


Question: Adeel Limited (AL) owns a machine which it purchased two years ago for Rs. 200,000. The
accumulated depreciation on the machine to date is Rs. 80,000 based on 5 years life using straight line
method.

The machine could be sold in the market for Rs. 100,000 but there would be dismantling costs of Rs. 10,000.
The cash flows from the existing machine are estimated to be Rs. 50,000 for the next two years followed by Rs.
40,000 in the last year. Relevant discount rate is 10%.

To replace the machine with a new version would cost Rs. 220,000.

Required:
Measure the machine using different measurement bases for AL using the above information.
Answer:
Historical cost Rs.
Cost 200,000
Less: Accumulated depreciation (80,000)
120,000

121 Page 12 of 16
Fair value
The fair value is market value (exit price) of Rs. 100,000 without deducting cost to sell of Rs. 10,000.

Value in use Rs.


-1
Year 1 Rs. 50,000 x 1.1 45,455
Year 2 Rs. 50,000 x 1.1-2 41,322
Year 3 Rs. 40,000 x 1.1-3 30,053
116,830

Rs.
Current cost
Cost of new asset 220,000
Less: Accumulated depreciation* Rs. 220,000 / 5 x 2 years (88,000)
132,000
*The replacement cost is of new machine and needs to be adjusted for two years usage.

Practice questions:
Example 01: CRITERIA FOR RECOGNITION
❑ A manufacturing unit valuing Rs. 5 million, owned and controlled by the Company
❑ A fleet of trucks valuing Rs 100 million, controlled by another company
❑ A highly skilled workforce, getting an annual compensation of Rs. 12.5 million

Required: Which of the above assets will be recognized in the financial statements of a company in
accordance with the recognition criteria?

Answer:
The assets will be recognized in the financial statements of company in accordance with conceptual
framework as under:
❑ A manufacturing unit valuing Rs. 5 million, owned and controlled by the Company
❑ The fleet of truck will not be recognized because it is not controlled by the entity.
❑ Similarly, workforce will not be recognized by the entity because there is no certainty about the probability
of future economic benefits from workers as they can quit the entity at any time.

Example 03: CARRIE


Question: Carrie starts in business on 1 January Year 1. Carrie’s sole shareholder contributed capital of Rs.
1,000. Carrie purchased one item of inventory for Rs. 1,000 and sold that inventory for cash of Rs. 1,400. At the
end of Year 1 the replacement cost of the same item of inventory is Rs. 1,100. General inflation during the year
was 7%.

Required
Calculate the profit for the year and set out a summary statement of financial position as of 31 December Year
1 under the following capital maintenance concepts.
(a) Physical capital maintenance
(b) Financial capital maintenance
i. Historical cost accounting
ii. Constant purchasing power accounting

122 Page 13 of 16
Answer:
(a) Physical
Capital
Maintenance (b) Financial CapitalMaintenance

(i) Historical ii)Constant


cost Purchasing
accounting power
accounting

Profit for the year Rs. Rs. Rs.


Sales 1,400 1,400 1,400
Cost of sales (1,000) (1,000) (1,000)
Inflation adjustment
- Specific (1,100 – 1,000) (100) - -
- General (1,000 7%) - - (70)
------------- ------------- -------------
Profit 300 400 330
------------- ------------- -------------
Balance sheet as at 31 December Year 1
Cash at bank 1,400 1,400 1,400
------------- ------------- -------------
Share capital (1,000 + 100) (1,000 + 70) 1,100* 1,000 1,070*
Reserves 300 400 330
------------- ------------- -------------
1,400 1,400 1,400
------------- ------------- -------------
Tutorial note
Share capital at the year end is restated under the physical capital maintenance concept for an increase in
specific price changes and under Constant Purchasing Power accounting for general price changes. This is the
other side of the entry to the inflation adjustments in the statement of profit or loss.

Example 04:
Question: Read the following statements:
A. In case of conflict between requirements of conceptual framework and IFRS, the requirements of
conceptual framework shall prevail.
B. Conceptual framework is not an International financial reporting standard (IFRS)
C. HR related cost is recognized as an asset in the financial statements since economic benefit is probable
from human resource
D. Internally generated goodwill is recognized as asset and measured at fair value in the financialstatements
E. When economic benefits arise over several accounting periods, and the association with income can only
be decided in broad terms, expenses should be recognized in profit and loss of each accounting period on
the basis of systematic and rational allocation procedure

123 Page 14 of 16
F. When an item of expenditure is not expected to provide any future economic benefit, it is recognized as
an asset in the financial statements
G. In fair value method, assets are measured at the amount that would be paid to purchase the same or a
similar asset currently

Required:
Analyse the above statements as true or false along with reasons for the selected answer.

Answer:
A. False. In case of conflict between requirements of conceptual framework and IFRS, the requirements of
IFRS shall prevail being an established principle that specific law requirements prevail over general law
requirements.
B. True. Conceptual framework provides foundation for the IFRSs
C. False.HR related cost can never be capitalized as it does not meet the definition criteria of asset
“controlled by the entity”
D. False. Internally generated goodwill can never be recognized as it does not meet one of the basic
recognition criteria i.e. “The item should have a cost or value that can be measured reliably”
E. True, because of matching principle
F. False. For any item to be recognized as an asset, it must be probable that an item shallprovide future
economic benefits to the entity.
G. False. In current cost method assets are measured at the amount that would be paid topurchase the same
or a similar asset currently

Example 05:
Question: ABC received Rs. 160,000 in cash on 20 December 2004 from RM in return for having provided
financial advice during the 2004 financial year.

Required:
(a) Explain, with reference to the relevant definitions, which elements should possibly be recognized in the
2004 financial year.
(b) Briefly identify whether and/ or how your answer would change if the cash received had been received for
financial advice to be provided in the 2005 financial year.
Answer:
Part (a)
The cash received meets the definition of an asset i.e. present resource now controlled by the entity and entity
may spend it as it may wish. Services have already been provided, therefore, there is no obligation (no change
in liability). It shall be recognised as an income.

An asset and an income shall be recognised in year 2004.

Part (b)
The cash received meets the definition of an asset i.e. present resource now controlled by the entity and entity
may spend it as it may wish. Services have not been provided and there is present obligation to provide
services, resulting in increase in liability. No income can be recognised as there is no equity increase.
An asset and a liability shall be recognised in year 2004.

124 Page 15 of 16
Example 06:
Question: Read the following scenarios:
1. An amount paid to landlord totalling Rs.120,000 on 1st January 2012 against the rent for the year ended
31st December 2012. Year end of the entity is 30 June 2012.
2. An expenditure incurred on repairs and maintenance of plant amounting Rs.300,000.
3. There has been legal dispute between the entity and its customer and company expects the outflow of
Rs. 200,000 in order to settle the dispute.
4. Entity purchased goods costing Rs. 20,000 for trading purposes and the same was sold for Rs. 25,000.

Required:
Which of the above, would be recognized as expense &/or asset in the financial statements of a company in
accordance with the criteria given in conceptual framework.

Answer:
1. Increase in asset (advance rent: Future benefits) Rs. 60,000 and decrease in asset (Cash) Rs. 120,000
resulting in net decrease in equity is Rent expense (Rs. 60,000).
2. Decrease in asset (Cash) Rs. 300,000 and no increase in other assets (unless increase in present
resources) resulting in net decrease in equity is Repair expense (Rs. 300,000).
3. Increase in liability (obligation to settle) Rs. 200,000 and no increase in any assets resulting in net
decrease in equity is Expense (Rs. 200,000).
4. When entity purchased inventory, it was a present economic resource and recognised as an asset.
When sold, it becomes expense (cost of sales) due to decrease in assets resulting in decrease in equity.

Example 07:
Question: Read the following scenarios
1. Advance received from customer amounting Rs. 50,000 against the goods to be delivered after 6 months
2. Services provided to ABC and Co. on credit amounting Rs.30,000.
3. Account Receivables already written off in previous years amounting Rs. 30,000 were received during the
year.

Required:
Which of the above, would be recognized as income &/or liability in the financial statements of a company in
accordance with the criteria given in conceptual framework.

Answer:
1. Increase in asset (Cash) Rs. 50,000 and also an increase in liability (obligation to deliver) Rs. 50,000 and
there is no income as no increase in equity.
2. Increase in asset (Right to receive) Rs. 30,000 and no increase in liability (services already provided) and
resulting net increase in equity Rs. 30,000 recognised as income.
3. Increase in asset (cash) Rs. 30,000 but no decrease in asset (RA was already written off) resulting in Net
increase in equity is Income.

125 Page 16 of 16
“We worry about tomorrow As if it is guaranteed.”

EXTRA PRACTICE QUESTION

Revaluation
Q. 1 following information pertained to a building acquired by Sk limited on 01.07.2012 for Rs. 360 million:
(i) The building is being depreciated on straight-line basis over 10 years.
(ii) SKL uses revaluation model for subsequent measurement of buildings. It accounts for revaluation on net
replacement value method. The details of revaluations as carried out by independent values are as follows:

Fair value
Revaluation date
(Rs. in million)
31 December 2013 323
31 December 2015 208
31 December 2017 167

(iii) There is no change in useful life of the building.


(iv) SKL transfers the maximum possible amount from the revaluation surplus to retained earnings on an annual
basis.
(v) SKL’s financial year ends on 31 December.

Required:
Prepare entries to record revaluation surplus / loss on each of the above revaluation date. (12)

Q.2 Following information pertains to a building acquired by SK Limited (SKL) on 1 July 2012 for Rs. 360 million:
The building is being depreciated on straight-line basis over 10 years.

(i) SKL uses revaluation model for subsequent measurement of buildings. It accounts for revaluation on net
replacement value method. The details of revaluations as carried out by independent valuer are as follows:

Fair value
Revaluation date
(Rs. in million)
31 December 2013 323
31 December 2015 208
31 December 2017 167

(ii) There is no change in useful life of the building.


(iii) SKL transfers the maximum possible amount from the revaluation surplus to retained earnings on an annual
basis.
(iv) SKL’s financial year ends on 30 June.

Required:
Prepare entries to record revaluation surplus / loss on each of the above revaluation date. (12)

126
Page 1 of 29
Answer: 1

“ Rs in Million”
Date Particulars Debit Credit
For the year ended 31-12-2012
1-7-2012 Building 360
Cash/Payable 360
31-12-2012 Depreciation 18
Accumulated depreciation (360+10) x 6/12 18
For the year ended 31-12-2013
31-12-2013 Depreciation 36
Accumulated depreciation 36
31-12-2013 Accumulated depreciation 54
Building 54
31-12-2013 Building 17
Revaluation Surplus (W-1) 17
For the year ended 31-12-2014
30-6-2014 Depreciation 38
Accumulated depreciation (325/8.5) 38
30-6-2014 Revaluation Surplus 2
Retained earnings (17/8.5) 2
For the year ended 31-12-2015
31-12-2015 Depreciation 38
Accumulated depreciation 38
31-12-2015 Revaluation Surplus 2
Retained earnings (17/8.5) 2
31-12-2015 Accumulated depreciation 76
Building 76
31-12-2015 Revaluation Surplus 13
Revaluation Loss (Bal) 26
Building (W-2) 39
For the year ended 31-12-2016
31-12-2016 Depreciation 32
Accumulated depreciation (208+6.5) 32
For the year ended 31-12-2017
31-12-2017 Depreciation 32
Accumulated depreciation (208+6.5) 32
31-12-2017 Accumulated depreciation 64
Building 64
31-12-2017 Building 23
Reversal of loss 18
Reversal Surplus (W-1) 5

127
Page 2 of 29
(W-1) 31-12-2013
WDV = 360 – 18.36 = 306
FV = 323
R. Surplus = 17

(W-2) 31-12-2015
WDV = 323 – 76 = 247
FV = 208
R. Loss = 39

(W-3) 31-12-2017
WDV = 208 – 64 = 144
FV = 167

R. Surplus = 23

Reversal of loss 26 R. Surplus 5


Extra depreciation to be charged (4 x 2) (8)
Net reversal of loss 18

If Ledgers are prepared.

Building
1-7-2012 Cash/Payable 360
c/d 360
1-1-2013 b/d 360
31-12-2013 Revaluation Surplus 17 31-12-2013 Accumulated depreciation 54
c/d 323
1-7-2014 b/d 323
c/d 323
1-1-2015 b/d 323
31-12-2015 Accumulated depreciation 76
Revaluation surplus 13
Revaluation loss 26
c/d 208
1-1-2016 b/d 208
c/d 208
1-1-2017 b/d 208
31-12-2017 Reversal of loss 18 31-12-2017 Accumulated depreciation 64
31-12-2017 Revaluation surplus 5 c/d 167

128
Page 3 of 29
Accumulated Depreciation
31-12-2012 Depreciation (360/10 x 6/12) 18
c/d
18
1-1-2013 b/d 18
31-12-2013 Building 54 31-12-2013 Depreciation(360/10) 36
c/d -
1-1-2014 b/d -
31-12-2014 Depreciation(323/8.5) 38
c/d 38
1-1-2015 b/d 38
31-12-2015 Building 76 31-12-2015 Depreciation 38
c/d -
b/d -
31-12-2016 Depreciation (208/6.5) 32
30-6-2017 c/d 32

1-1-2017 b/d 32
Building 64 31-12-2017 Depreciation 32
c/d -

Revaluation Surplus
b/d -
c/d -
1-1-2013 b/d -
31-12-2013 Building 17
30-6-2014 c/d 17
1-1-2014 b/d 17
31-12-2014 Retained Earnings(17/8.5) 2
c/d 15
1-1-2015 b/d 15
31-12-2015 Retained Earnings 2
31-12-2015 Building 13
c/d -
1-1-2016 b/d -
c/d -
1-1-2017 b/d -
c/d 5 31-12-2017 Building 5

(W-1) 31-12-2013
WDV = 360 – 18.36 = 306
FV = 323
R. Surplus = 17

129
Page 4 of 29
“Don’t take salah as a burden. Allah gifted us salah as a relief from burden.”

(W-2) 31-12-2015
WDV = 323 – 76 = 247
FV = 208
R. Loss = 39
(W-3) 31-12-2017
WDV = 208 – 64 = 144
FV = 167
R. Surplus = 23

Reversal of loss 26 R. Surplus 5


Extra depreciation to be charged (4 x 2) (8)
Net reversal of loss 18

Answer: 2

“ Rs in Million”
Date Particulars Debit Credit
For the year ended 30-6-2013
1-7-2012 Building 360
Bank 360
30-6-2013 Depreciation 36
Accumulated depreciation (360/10) 36
For the year ended 30-6-2014
31-12-2013 Depreciation 18
Accumulated depreciation (360/10) x 6/12 18
31-12-2013 Accumulated depreciation – Building (36+18) 54
Building 54
31-12-2013 Building 17
Revaluation Surplus (W-1) 17
30-6-2014 Depreciation 19
Accumulated depreciation (323/8.5) x 6/12 19
30-6-2014 Revaluation Surplus 1
Retain Earning (17/8.5 x 6/12 1
For the year ended 30-6-2015
30-6-2015 Depreciation 38
Accumulated depreciation (323/8.5) 38
30-6-2015 Revaluation Surplus 2
Retained earnings (17/8.5) 2
For the year ended 30-6-2016
31-12-2015 Depreciation 19

130
Page 5 of 29
Accumulated depreciation (323/8.5) x 6/12 19
31-12-2015 Revaluation Surplus 1
Retained earnings (17/8.5) x 6/12 1
31-12-2015 Accumulated depreciation – Building 76
Building 76
31-12-2015 Revaluation Surplus (17 – 4) 13
Revaluation Loss 26
Building (W-2) 39
For the year ended 30-6-2017
30-6-2016 Depreciation 16
Accumulated depreciation (208/6.5) x 6/12 16
No transfer of surplus
30-6-2017 Depreciation 32
Accumulated depreciation (208/6.5) 32
For the year ended 30-6-2018
31-12-2017 Depreciation 16
Accumulated depreciation (208/6.5) x 6/12 16
31-12-2017 Accumulated depreciation (16+32+16) 64
Building 64
31-12-2017 Building 23
Reversal of revaluation loss (W-3) 18
Revaluation Surplus 5
30-6-2018 Depreciation 18.53
Accumulated depreciation (167/4.5) x 6/12 18.56
30-6-2018 Revaluation Surplus 0.56
Retain Earning (5/4.5) x 6/12 0.56

(W-1) 31-12-2013
WDV = 360 – 36 – 18 = 306
FV = 323
R. Surplus = 17

(W-2) 31-12-2015
WDV = 323 – 19 – 38 – 19 = 247
FV = 208
R. Loss = 39

131
Page 6 of 29
(W-3) 31-12-2017
WDV = 208 – 16 – 32 – 16 = 144
FV = 167
R. Surplus = 23

Reversal of loss 26 R. Surplus 5


Extra depreciation to be charged (2+4+2) (8)
Net reversal of loss 18

[Ledgers not required. For extra information only.]


Building Account
1-7-2012 Cash 360
30-6-2013 c/d 360
1-7-2013 b/d 360
31-12-2013 Revaluation Surplus 17 31-12-2013 Accumulated depreciation 54
30-6-2014 c/d 323
1-7-2014 b/d 323
30-6-2015 c/d 12,700
1-7-2015 b/d 323
31-12-2015 Accumulated depreciation 76
31-12-2015 Revaluation surplus 14
31-12-2015 Revaluation loss 25
30-6-2016 c/d 208
1-7-2016 b/d 208
30-6-2017 c/d 208
1-7-2017 b/d 208
31-12-2017 Reversal of loss 18 31-12-2017 Accumulated depreciation 64
31-12-2017 Revaluation surplus 5 30-6-2018 c/d 167

Accumulated Depreciation Account


1-7-2012 b/d -
30-6-2013 Depreciation 36
30-6-2013 c/d 36
1-7-2013 b/d 36
31-12-2013 Building 54 31-12-2013 Depreciation 18
31-12-2013 Depreciation 19
30-6-2014 c/d 19
1-7-2014 b/d 19
30-6-2015 Depreciation 38
30-6-2015 c/d 57
1-7-2015 b/d 57

132
Page 7 of 29
31-12-2015 Building 76 31-12-2015 Depreciation 19
31-12-2015 Depreciation 16
30-6-2016 c/d 16
1-7-2016 b/d 16
30-6-2017 Depreciation 32
30-6-2017 c/d 48
1-7-2017 b/d 48
31-12-2017 Building 64 31-12-2017 Depreciation 16
30-6-2018 Depreciation 19
30-6-2018 c/d 19

Revaluation Surplus
1-7-2012 b/d -
30-6-2013 c/d -
1-7-2013 b/d -
30-6-2014 Retained Earnings 1 31-12-2013 Building 17
30-6-2014 c/d 16
1-7-2014 b/d 16
30-6-2015 Retained Earnings 2
30-6-2015 c/d 14
31-12-2015 Retained Earnings 1 1-7-2015 b/d 14

31-12-2015 Building 13
30-6-2016 c/d -
1-7-2016 b/d -
30-6-2017 c/d -
1-7-2017 b/d -
30-6-2018 Building 0.56 31-12-2017 Building 5
30-6-2018 c/d 4.44

133
Page 8 of 29
“There is nothing heavier in the scales than good character”

Additional Practice Questions

Questions

PREPARATION OF FINANCIAL STATEMENTS

1 Question
MK Corporation Limited, an entity listed in Pakistan Stock Exchange is in the business of manufacturing and
sale of yarn products. Company year-end is December. Below is the relevant information given?

Opening balances as at January 01, 2018

Description Rs.

Opening Share Capital (at par value of Rs. 10 per share) 25,000,000

Share Premium 7,500,000

Opening General Reserves 750,000

Opening RE 18,250,000

Revaluation Surplus 1,500,000

Following events have taken place in year 2018 and 2019:


1. On March 31, 2018, Company issued Right shares for Rs. 20 per share. Right shares were issued in the
proportion of 1 right share against 5 ordinary shares held.
2. Board of Directors of the Company approved Interim dividend of Rs. 2.25 per share for the half year
ended June 30, 2018.
3. Annual profit for the year ended December 31, 2018 is Rs. 10,250,000.
4. The Board of Directors of the Company recommended annual dividend of Rs. 4.25 per share on
February 15, 2019, which was duly approved by the Shareholders on March 21, 2019.
5. The Board of Directors approved Bonus Shares of 20% of the outstanding shares on June 30, 2019
which were duly credited in Shareholders account on August 31, 2019.
6. Board of Directors of the Company approved Interim dividend of Rs. 1.25 per share for the third
quarter ended September 30, 2019.
7. Annual profit for the year ended December 31, 2019 is Rs. 12,500,000.
8. The Board of Directors of the Company recommended annual dividend of Rs. 5 per share on February
15, 2020, which was duly approved by the Shareholders on March 21, 2020.
9. The Company has a policy to transfer 5% of the Annual Profit to General Reserves.

134
Page 9 of 29
10. Company revalued fixed assets on December 31, 2017 resulting in Revaluation Surplus of Rs.
1,500,000. Remaining useful life of the Asset is 10 years and Company has a straight line method for
Depreciation.

Required
Make Statement of Changes in Equity for the year ended December 31, 2018 and 2019. Ignore taxation impact
if any.

2 Question
HMK Corporation Limited, an entity listed in Pakistan Stock Exchange is in the business of manufacturing and
sale of Cars. Company year-end is December. Below is the relevant information given?

Opening balances as at January 01, 2018


Description Rs.
Opening Share Capital (at par value of Rs. 10 per share) 100,000,000
Share Premium 50,000,000
Opening General Reserves 5,000,000
Opening RE 55,000,000

Following events have taken place in year 2018 and 2019:


1. On February 28, 2018, the Company issued Initial Public Offering, thereby offering 5 million shares for Rs.
25 each. All shares were subscribed and company received the subscription money by March 31, 2018.
2. The Board of Directors of the Company recommended annual dividend for the year ended December 31,
2017 of Rs. 5 per share on February 15, 2018, whereas Shareholders only approved Rs. 4 per share on
March 31, 2018.
3. Board of Directors of the Company approved Interim dividend of Rs. 1 per share for the third quarter
ended September 30, 2018.
4. Annual profit for the year ended December 31, 2018 is Rs. 130,250,000.
5. The Board of Directors of the Company recommended annual dividend of Rs. 4.25 per share on February
15, 2019, whereas Shareholders approved Rs. 6 per share on March 31, 2019.
6. The Board of Directors approved Bonus Shares of 10% of the outstanding shares on June 30, 2019 which
were duly credited in Shareholders account on August 31, 2019.
7. Board of Directors of the Company approved Interim dividend of Rs. 1.5 per share for the third quarter
ended September 30, 2019.
8. Annual profit for the year ended December 31, 2019 is Rs. 175,000,000.
9. The Board of Directors of the Company recommended annual dividend of Rs. 5.5 per share on February
15, 2020, which was duly approved by the Shareholders on March 31, 2020.
10. The Company has a policy to transfer 5% of the Annual Profit to General Reserves.

Required
Make Statement of Changes in Equity for the year ended December 31, 2018 and 2019. Ignore taxation impact
if any.

135
Page 10 of 29
“Forgive others as quickly as you expect Allah (God) to forgive you.”

3 Question
HMK Corporation Limited, an entity listed in Pakistan Stock Exchange is in the business of manufacturing and
sale of Cars. Company year-end is December. Below is the relevant information given?

Opening balances as at January 01, 2018

Description Rs.

Opening Share Capital (at par value of Rs. 10 per share) 100,000,000

Share Premium 50,000,000

Opening General Reserves 5,000,000

Opening RE 55,000,000

Following events taken place in year 2018 and 2019:


1. On February 28, 2018, the Company issued Initial Public Offering, thereby offering 5 million shares for Rs. 25
each. Only 75% shares were subscribed and company received the subscription money by March 31, 2018.
2. The Board of Directors of the Company recommended annual dividend for the year ended December 31, 2017
of Rs. 5 per share on February 15, 2018, whereas Shareholders only approved Rs. 4 per share on March 31,
2018.
3. Board of Directors of the Company approved Interim dividend of Rs. 1 per share for the third quarter ended
September 30, 2018.
4. Annual profit for the year ended December 31, 2018 is Rs. 175,000,000.
5. The Board of directors has declared a final dividend for the year ended 31.12.2018 at the rate of 1.65 per
share on February 15, 2019, which was duly approved by Shareholders in annual general meeting on March
21, 2019.
6. The Board of Directors approved Bonus Shares of 10% of the Outstanding shares on June 30, 2019 which were
duly credited in Shareholders account on August 31, 2019.
7. Board of Directors of the Company approved Interim dividend of Rs. 1.5 per share for the third quarter ended
September 30, 2019.
8. Annual profit for the year ended December 31, 2019 is Rs. 185,250,000.
9. The Company has a policy to transfer 5% of the Annual Profit to General Reserves.

Required
Make Statement of Changes in Equity for the year ended December 31, 2018 and 2019. Ignore taxation impact if
any.

136
Page 11 of 29
IAS 16: PROPERTY, PLANT AND EQUIPMENT
1 Question
Abbas Limited (AL) is engaged in the business of manufacturing near the Karachi-Hyderabad Motorway. Its
Property, Plant and Equipment comprises of land and buildings, plant and machinery, and equipment and
fittings.

Details for the period up to 30 June 2018 are as follows:


1. The balances of the Property, Plant and Equipment as at 30 June 2018 are given below:

Assets Gross Carrying Amount Accumulated Depreciation (Rs.


(Rs. Million) Million)

Land 12 N/A

Buildings 125 38

Plant and Machinery 500 300

Equipment 100 36

2.The relevant information pertaining these assets is given below:

Assets Depreciation Method Subsequent Measurement Model

Land N/A Fair Value

Buildings Straight-line Cost

Plant and Machinery Units of Production Cost

Equipment Written down value @ 20% Cost

3. Abbas Limited uses proportionate policy to depreciate its Property, Plant and Equipment.
4. All of the plant and machinery pertains to factory use whereas all the equipment pertains to office use.
However floor areas occupied by factory and office are in the ratio 60:40 respectively.
5. The equipment was purchased on 1 July 2016. No disposals and acquisitions took place in the period up to 30
June 2018.
6. Until 30 June 2018, 12,000 units had been produced by Abbas Limited in its factory. The plant and machinery
does not have any residual value. No additions or disposals of plant and machinery took place till this date.
7. The buildings were acquired on 1 July 2014 with a residual value of Rs. 11 million. No additions and disposals
took place till 30 June 2018.
8. The land had actually cost Rs. 15 million on the date of its acquisition.
9. It is assumed that value of land and buildings is spread evenly across the area occupied.

137
Page 12 of 29
The following information pertains to the year ended 30 June 2019:
1. On 1 July 2018, land was revalued to Rs. 20 million. The value was determined by an independent firm M/s
Ashfaq & Co. Chartered Accountants.
2. This year, 5,000 units were produced in the factory of AL.
3. On January 1, 2019, AL disposed 25% of its area comprising of land and buildings at a price of Rs. 90 million.
The portion of land was sold at its fair value as determined on 1 July 2018. The legal costs of drafting transfer
agreements were Rs. 0.1 million. It is assumed that this disposal will not affect the proportion of areas
occupied by factory and office.
4. Further equipment costing Rs. 60 million was acquired on 1 November 2018.
5. In the meeting of its board of directors, it was decided to open a new factory premises near Lahore-Islamabad
motorway. An expenditure of Rs. 20 million was spent of the construction of the factory on 1 December 2018,
financed by a loan obtained from the bank at the rate of 12% per annum. The construction had not been
completed at the end of the year.
6. Moreover, the directors also made a contract with M/s Uni Power& Co. to purchase plant and machinery
worth Rs. 35 million once the construction of factory building is completed.

Required:
a) Prepare journal entries to record the revaluation of land and disposal of land and buildings.
b) Prepare the disclosure under IAS 16 in relation to Property, Plant and Equipment in the notes to the published
accounts for the year ended 30 June 2019.

2 Question
Games Limited (GL) commenced a business of preparing and burning video game CDs on 1 July 2015.

The following information pertains to the year ended 31 March 2016:


1) GL purchased 30 computers on the date of commencement of business at a cost of Rs. 20,000 each, purely for
the task of burning CDs. The management of GL estimates that since the computers are subject to obsolescence,
more of its benefit can derived in its early life therefore use reducing balance method. The total useful life at
the date of acquisition was estimated to be 4 years and residual value was estimated to be Rs. 4,802 for each
computer.GL decided to adopt historical cost model for subsequently measurement of computers.
2) GL purchased an office building at the date of start of business worth Rs. 3 million. GL decided to adopt fair
value model due to fluctuations in property prices. 80% of the building is occupied by computer labs, whereas
20% is used by administrative and selling departments. The useful life is estimated to be 10 years at the date of
acquisition with no residual value, and the economic benefits are expected to be derived evenly over its useful
life. At the end of the year, the fair value of office buildings was assessed to be Rs. 3,237,500.
3) GL also purchased fittings for its administrative and selling departments, costing Rs. 120,000 on 1 July 2015. It
is to be depreciated over 10 years using the straight-line method, with no residual value.
4) GL made a contractual commitment with Al-Karim Computers to purchase 6 computers of Rs.20,000 each to be
delivered at GL’s premises on 1 May 2016.

The following information pertains to the year ended 31 March 2017:


1) The computers were delivered at the GL’s premises by Al-Karim Computers at the said date. It was decided to
use the same method and same rate to depreciate these computers. However, no further space was utilised by
the computer labs.

138
Page 13 of 29
2) At the end of the year, the fair value of office building was assessed to be Rs. 2 million. At the year-end GL
mortgaged entire building with JS Bank to obtain a loan worth Rs. 1.75 million for prospective investments in
other divisions.
3) Fittings with a cost of Rs. 30,000 were disposed of for Rs. 22,000 on 1 January 2017. The Suzuki
Driver was paid Rs. 1,000 to transfer the fittings to customer’s premises.

The fair values of the office building were determined byan independent firm M/s Hafeez Yasir Chartered
Accountants& Co. Moreover, GL uses proportionate policy to depreciate its assets.

Required:
(a) Prepare the disposal account to record the sale of fittings on 1 January 2017.
(b) Prepare the disclosure under IAS 16 in relation to Property, Plant and Equipment in the notes to the published
accounts for the year ended 31 March 2017 (comparatives are required).

ANSWERS

PREPARATION OF FINANCIAL STATEMENTS

1 Answer

Share Share General Retained Revaluation


Description Capital Premium Reserves Earnings Surplus Total

Opening Balance 25,000,000 7,500,000 750,000 18,250,000 1,500,000 53,000,000


Issuance of Right
Shares
(25,000,000 / 10 x 1 / 5)
x 10 5,000,000 5,000,000 - - - 10,000,000
Interim Dividend
(25,000,000 +
5,000,000)/ 10 x 2.25 (6,750,000) (6,750,000)

Profit for the year 10,250,000 10,250,000


Transfer
(5% x 10,250,000) 512,500 (512,500) -

Transfer of Rev Surplus


to RE
(1,500,000 / 10) 150,000 (150,000) -
Balance at the end of
the year 30,000,000 12,500,000 1,262,500 21,387,500 1,350,000 66,500,000

Annual Dividend (12,750,000) (12,750,000)

139
Page 14 of 29
(30,000,000 / 10 x 4.25)
Bonus Shares
(30,000,000 x 20% ) 6,000,000 (6,000,000) - -
Interim Dividend
(36,000,000 / 10 x 1.25) (4,500,000) (4,500,000)

Profit for the year 12,500,000 12,500,000


Transfer
(12,500,000 x 5% ) 625,000 (625,000) -

Transfer of Rev Surplus


to RE
(1,500,000 / 10 ) 150,000 (150,000) -
Balance at the end of
the year 36,000,000 6,500,000 1,887,500 16,162,500 1,200,000 61,750,000

140
Page 15 of 29
2 Answer

141
Page 16 of 29
“Allah does not burden a soul beyond that it can bear.” Quran 2:286
3 Answer

142
Page 17 of 29
PROPERTY, PLANT AND EQUIPMENT

1. Solution (a):

Date Particulars Dr. Cr.


Rs. 000 Rs.000

2018

July 1, Land 8,000


Revaluation Surplus 5,000
Reversal of Revaluation Loss (Other Income) (15-12) 3,000
(20 - 12)
2019

Jan 1, Cash (90m - 0.1m) 89,900


Accumulated Depreciation – Buildings (W1) 10,687.5
Buildings (125 x 25%) 31,250
Land (20 x 25%) 5,000
Gain on Disposal (Other Income) 64,337.5

Jan 1, Revaluation Surplus 1,250


Retained Earnings 1,250
(5,000 x 25%)= 1,250 [transfer on disposal]

(W1) Rs. 000

At the start of year (38 × 25%) 9,500


During the year [(125 – 11) × 25% ÷ 12* x6/12] 1,187.5

Accumulated Depreciation of Buildings Disposed 10,687.5

*38 /4 (01.07.2014 to 30.06.2018) = 9.5 per annum


(125 - 11) / x = 9.5
X = 114/ 9.5 =12

143
Page 18 of 29
“Taking pains to remove the pains of others is the true essence of generosity.”

Solution (b)

Abbas Limited
Notes to Financial Statements
For the year ended 30 June 2019
Property, Plant and Equipment:

Abbas Limited uses the following subsequent measurement bases to value its Property, Plant and Equipment, and
methods to calculate its depreciation.
Subsequent
Depreciation Useful Life/Residual
Assets Measurement
Method Value/Rate

Fair Value
N/A N/A
Land
Useful life of 12 years with a Cost less Accumulated
residual value of 8.8% (11 / 125) of
Straight-line cost.
Buildings Depreciation
(W1)
Cost less Accumulated
Units of
Plant and Rs. 25,000 per unit (W2)
Depreciation
production
Machinery
Cost less Accumulated
Written down
Rate of 20%
Equipment Depreciation
value

Furthermore, Abbas Limited uses proportionate policy to depreciate its assets.

Schedule of Movement in Property, Plant and Equipment

144
Page 19 of 29
For the year ended 30 June 2019 (in Rs. 000)

Plant and Total


Land Buildings Equipment
Machinery

Cost / Revalued amount


12,000 125,000 500,000 100,000 737,000
At 1 July 2018
- - - 60,000 60,000
Acquisitions
8,000 - - - 8,000
Revaluations
(5,000) (31,250) - - (36,250)
Disposals(10,687.5)
15,000 93,750 500,000 160,000 768,750
At 30 June 2019

Accumulated Depreciation:
- 38,000 300,000 36,000 374,000
At 1 July 2018 125,000
(5,000 x
- 8,312.5(W4) 25,000) 20,800(W5) 154,112.5
Depreciation charge for the
- - - - -
Year
- - -
Revaluation
Disposals (10,687.5)
- 35,625 425,000 56,800 517,425
At 30 June 2019

Carrying Amount at 30 15,000 58,125 75,000 103,200 251325


June 2019

Carrying Amount at 1 July 12,000 87,000 200,000 64,000 363,000


2018

An amount of expenditure of Rs. 20 million was incurred on the construction of a factory near Lahore-Islamabad
Motorway on 1 December 2018. This amount was capitalised as capital work-in-progress.

A further borrowing costs of Rs. 1.4 million (W6) were capitalised in respect of interest on loan obtained from the
bank to finance this project.

A contract was made with M/s UniPower& Co. to purchase plant and machinery worth Rs. 35 million once the
construction of factory building is completed.

145
Page 20 of 29
“Taking pains to remove the pains of others is the true essence of generosity.”

The following depreciations are either made part of cost of goods manufactured or operating expenses in statement
of profit or loss:

Assets Factory Operating Expense Total


Rs.000 Rs.000 Rs.000

4,987.5 3,325 8,312.5


Buildings (60%) (40%)

125,000 - 125,000
Plant and Machinery

- 20,800 20,800
Equipment

129,987.5 24,125 154,112.5


Total

Revaluation Disclosures:
(i) The revaluation of land took place on 1 July 2018. The value was determined by an
independent firm M/s Ashfaq& Co. Chartered Accountants.
(ii) The carrying amount of land had the revaluation not taken place:

Rs. million

15
At 1 July 2018
(3.75)
Disposals during the year (15 x 25%)
11.25
At 30 June 2019

146
Page 21 of 29
(iii) Revaluation Surplus
Rs. million

-
At 1 July 2018
5
Revaluation of land
(1.25)
Transfer to retained earnings (5 x 25%)
3.75
At 30 June 2019

A further reversal of revaluation loss of Rs. 3 million was reversed during the year.

(W1) (125 – 11) /38 x4 = 12 years

(W2)
300,000,000 / 12,0000 =Rs. 25,000 per unit

(W4) Depreciation during the year Rs. 000

Disposals [(125-11) x 25% /12 x 6/12] 1,187.5


During the year [(125 – 11) × 75% ÷ 12] 7,125

Depreciation Expense 8,312.5

(W5) Depreciation during the year Rs. 000

Acquisitions (60 × 0.2 × 8/12) 8,000


Remaining Assets (100 - 36)x 20% 12,800

Depreciation Expense 20,800

(W6) 20 million x 12% x 7


/12 = 1.4 million

147
Page 22 of 29
2 Solution (a):

30,000/10 x 9/12 =2,250+30,000/10 x 9/12 =2,250=4,500

Solution (b):

Games Limited

Notes to the financial statements

For the year ended 31 March 2017

Property, plant and equipment:

Games Limited (GL) uses the following subsequent measurement bases to value its Property, Plant and Equipment,
and methods to calculate its depreciation.
Subsequent
Depreciation
Assets Rate
Measurement
Method

Fair Value
Straight-line 10%
Buildings
Cost less Accumulated
Written-down
30% (W1)
Computers Depreciation
value

Straight-line 10%(useful life 10years) Cost less Accumulated

148
Page 23 of 29
Fittings Depreciation

Furthermore, Games Limited uses proportionate policy to depreciate its assets.

149
Page 24 of 29
Games Limited

Schedule of Movement in Property, Plant and Equipment

For the year ended 30 March 2016 (in Rs.)

Buildings Computers Fittings


Cost / Revalued amount
At start of year - - -
Acquisitions 600,000
3,000,000 (20,000x30) 120,000
Revaluations 462,500 - -
Elimination (225,000) - -
Disposal - - -
At end of year 3,237,500 600,000 120,000

Accumulated depreciation - - -
At start of year - - -
Depreciation charge for the
year (W-2) 225,000 135,000 9,000
Elimination (225,000) - -
Disposal - - -
At end of year - 135,000 9,000
Carrying amount at start of
year - - -
Carrying amount at end of
year 3,237,500 465,000 111,000

For the year ended 31-03-2017


Cost / Revalued amount
At start of year 3,237,500 600,000 120,000
Acquisitions - 120,000 -
Revaluations (887,500) - -
Elimination (350,000)
Disposal - - (30,000)
At end of year 2,000,000 720,000 90,000

Accumulated depreciation - - -
At start of year - 135,000 9,000
Depreciation charge for the
year (W-2) 350,000 172,500 11,250

150
Page 25 of 29
Elimination (350,000) - -
Disposal - - (4,500)
At end of year - 307,500 15,750
Carrying amount at start of
year 3,237,500 465,000 111,000
Carrying amount at end of
year 2,000,000 412,500 74,250

The entire office building was mortgaged with JS Bank on 31 March 2017, to obtain a loan worth Rs. 1.75 million for
prospective investments in other divisions.

No contractual commitments were made during the year ended 31 March 2017 to purchase Property, Plant and
Equipment.

A contract was made with Al-Karim Computers during the year ended 31 March 2016 to purchase 6 computers of
Rs. 20,000 each to be delivered on 1 May 2016.

The following depreciations are either made part of inventory or expensed out in statement of profit or loss:

Assets 2017 2016

OperatingEx OperatingE
Factory pense Total Factory xpense Total
Rs. Rs. Rs. Rs. Rs. Rs.

280,000 70,000 350,000 180,000 45,000 225,000


Buildings (80%) (20%) (80%) (20%)
172,500 - 172,500 135,000 - 135,000
Computers
- 11,250 11,250 - 9,000 9,000
Fittings
452,500 81,250 533,750 315,000 54,000 369,000
Total

Revaluation Disclosures:

(iv) The revaluations of office buildings took place on 31 March 2017 and 31 March 2016 respectively. The fair values
of the office building were determined by an independent firm M/s Hafeez Yasir Chartered Accountants & Co.

151
Page 26 of 29
“No one will reap except what they sow.” Quran 6:164

(v) The carrying amount of buildings had the revaluation not taken place:

2017 2016
Rs. Rs.
Cost:
3,000,000 -
At start of year
- 3,000,000
Acquisitions
- -
Disposals
3,000,000 3,000,000
At end of year

Accumulated Depreciation:
225,000 -
At start of year
300,000 225,000
Depreciation charge for the year (3,000/10)
- -
Disposals

2017 2016
Rs. Rs.

525,000 225,000
At end of year

Carrying Amount at year start 2,775,000 -

Carrying Amount at year end 2,475,000 2,775,000

152
Page 27 of 29
(vi) Revaluation Surplus

2017 2016
Rs. Rs.

462,500 -
At start of year
- 462,500
Revaluation of Buildings (Surplus)
(50,000) -
Transfer to Retained Earnings (462,500/9.25)
Revaluation building (loss) (412,500)
- 462,500
At end of year

Furthermore, a revaluation loss of Rs. 475,000 was recognized at 31 March 2017.

(W1)
4 4,802
𝑟 =30%𝑟 = 1 − √
20,000

Accumulated depreciation - Building 350,000


Building 350,000
(Reversal of prior year depreciation)
Surplus on revaluation of fixed assets(OCI) 412,500
Revaluation loss(PL) 475,000
Building 887,500

(W2) 2017 2016

3,000,000 ÷ 10 ×
9
12
Buildings 3,237,500 ÷ 9.25 = 350,000
*Remaining useful life = 225,000

Rs.
Acquisitions (120,000 ×30% × 11/12) 6,000,000 × 30%
33,000 9⁄12
= 135,000
Computers

Remaining [(600,000 - 135,000) × 139,500


30%]

153
Page 28 of 29
Total 172,500

Rs.

Disposals (30,000 × 10% × 9/12) 2,250


Fittings 120,000 ÷ 10 ×9⁄12
Remaining (90,000 × 10%) 9,000 = 9,000
Total 11,250

154
Page 29 of 29
“He Knows what is in every Heart – Surah Mulk {67:13}”

Autumn 2020

Q.2
Ratios are computed by using numerical values from financial statements to gain meaningful information about an
entity. However, due to inherent limitations of ratio analysis, it may not reflect the correct financial situation.

Required:
Briefly explain any four limitations of ratio analysis.
(06)

Q.3
On 1 July 2014, Indus Pharma Limited (IPL) received a government grant of Rs. 280 million to setup a plant in an
under-developed rural area. The grant is repayable in full if the conditions attached to the grant are not met for a
period of five years from the date of commencement of the production. At the inception, it was highly probable
that IPL would comply with the conditions for the required period.

IPL incurred total cost of Rs. 630 million on plant and it started production on 1 January 2015. Useful life of
the plant was estimated at 7 years. IPL deducted government grant in arriving at the carrying amount of the asset.

In January 2019, IPL showed its inability to comply with the conditions attached to the grant and regulatory
authority issued a notice to IPL for repayment of the grant in full. Accordingly, the grant was repaid by IPL.

In view of repayment of the grant, IPL carried out an impairment review of the plant on 31 December 2019. Net
annual cash inflows for the remaining life of the plant have been estimated at Rs. 90 million and Rs. 80 million for
2020 and 2021 respectively. These cash inflows are net of annual interest and maintenance cost of Rs. 10 million
and Rs. 6 million respectively for both years. Applicable discount rate is 12%.

On the date of impairment review, the existing plant can be sold in the local market for Rs. 160 million.
Estimated cost of disposal would be Rs. 5 million.

Required:
Prepare journal entries for the year ended 31 December 2019 in respect of the above information. (Show all
necessary workings. Narrations are not required)

(08)

155
Page 1 of 14
Q.4
Select the most appropriate answer from the options available for each of the following Multiple Choice
Questions.
i. Which of the following statements is correct about financial statements based on historical cost in times
of rising prices?
(a) Profits will be overstated and assets will be understated
(b) Assets will be overstated
(c) Profits as well as assets will be understated
(d) Depreciation will be overstated
(01)
ii. Under IAS 40 Investment property’, which of the following disclosures is NOT required to be made under
cost model?
(a) Fair value of the property
(b) Depreciation method
(c) Reconciliation of carrying amounts at the beginning and end of a period
(d) Residual value of the property
(01)

iii. Which of the following would cause negative net cash flow from operating activities?
(a) Decrease in depreciation expense
(b) A substantial investment in fixed assets
(c) A significant increase in credit sales
(d) Repayment of a long-term loan
(01)

iv. Alpha Club’s financial year ends on 31 December. Following information pertain to its members'
subscription:
Rupees
Subscription received in 2018 for 2019 180,000
Subscription received in 2019 for 2018 90,000
Subscription received in 2019 for 2019 1.400.000
Subscription received in 2019 for 2020 200,000
Subscription for 2018 outstanding as on 31 December 2018 150,000
Subscription for 2019 outstanding as on 31 December 2019 325,000

Subscription income for the year ended 31 December 2019 is:


(a) Rs. 1,845,000
(b) Rs. 1,705,000
(c) Rs. 1.905,000
(d) Rs. 1,665,000
(02)

156
Page 2 of 14
v. A company has current ratio and quick ratio of 2.0 and 0.8 respectively. If the company uses its positive
cash balance to pay a creditor, it will:
(a) increase current ratio as well as quick ratio
(b) increase current ratio and decrease quick ratio
(c) have no effect on current ratio as well as quick ratio
(d) decrease current ratio as well as quick ratio

vi. Which of the following would increase gearing ratio?


(a) Issuance of shares at premium
(b) Issuance of shares at discount
(c) Issuance of bonus shares
(d) Declaration and payment of cash dividend

vii. Which of the following statements is correct in the context of capitalisation of borrowing costs?
(a) If funds have been arranged from various general borrowings, the amount to be capitalised is based
on the weighted average cost of borrowings
(b) Capitalisation always commences as soon as expenditure for the asset is incurred
(c) Capitalisation always continues until the asset is brought into us?
(d) Capitalisation always commences as soon as borrowing costs are incurred

Q.6 Statement of financial position of Taxila Limited (TL) as on 30 June 2020 is as follows:

Assets 2020 2019 Equity & liabilities 2020 2019


Rs. in million Rs. in million
Property, plant and equipment 1,619 1,200 Share capital (Rs. 100 each) 1,200 800
Investment property 290 120 Share premium 290 150
Inventories 205 180 Retained earnings 260 90
Trade receivables 342 291 Revaluation surplus 215 200
Prepayments and other receivables 14 20 Long-term loans 367 445
Short-term investments 60 48 Trade and other payables 144 120
Cash and bank balances 24 6 Current portion of
long-term loans 78 60
2,554 1,865 2,554 1,865

Additional information:
(i) Equipment having fair value of Rs. 240 million was acquired by issuing 2 million shares.
(ii) As a result of revaluation carried out on 30 June 2020, property, plant and equipment was
increased by Rs. 80 million out of which Rs. 35 million was credited to profit and loss account.
(iii) During the year, fully depreciated items of property, plant and equipment costing Rs. 36
million were sold for Rs. 8 million out of which Rs. 3 million is still outstanding.
(iv) Depreciation on property, plant and equipment for the year amounted to Rs. 290 million.
(v) An investment property was acquired for Rs. 180 million. TL applies cost model for subsequent
measurement of its investment property.
(vi) Financial charges for the year amounted to Rs. 45 million. Trade and other payables include
accrued financial charges of Rs. 12 million (2019: Rs. 17 million).

157
Page 3 of 14
(vii) Short-term investments amounting to Rs. 35 million are readily convertible to cash (2019: Rs. 20
million). Investment income for the year amounted to Rs. 6 million.

Required:
Prepare TL’s statement of cash flows for the year ended 30 June 2020 in accordance with the
requirements of IFRS. (17)

Q.8 Following information pertain to property, plant and equipment of Harappa

Industries Limited (HIL) for the year ended 30 June 2020:

(i) Balance as on 30 June 2019


Cost/revalued Accumulated Revaluation Depreciation Useful
Assets
amount depreciation surplus method life/rate
----------- Rs. in '000 -----------

Land* 100,000 - - - Infinite


Buildings 70,000 14,000 16,000 Straight line 20 years
Plant 180,000 60,000 - Straight line 15 years
Vehicles 8,800 4,000 - Reducing 20%
balance

*An amount of Rs. 12 million had been charged to profit or loss upon previous revaluation

(ii) On 30 June 2020, the revalued amounts of the land and buildings were assessed by Smart Consultant
at Rs. 120 million and Rs. 35 million respectively.
(iii) Setting up of a new plant was commenced on 1 July 2019 and substantially completed on 29 February
2020. The plant was available for use on 1 April 2020 and immediately put into use. Useful life of the
plant was estimated at 10 years. Details of the cost incurred are as under:

Description Payment date Rs.in '000


1st payment 1 August 2019 12,000

2nd payment 1 October 2019 48,000

3rd payment 29 February 2020 48,000

4th payment 31 July 2020 12,000


120,000

The cost of the plant was financed through an existing running finance facility with a limit of Rs. 200 million
carrying mark-up of 12% per annum. A government grant of Rs. 20 million related to the plant was received
on 1 January 2020. The grant amount was used for repayment of the running facility

(iv) One of the vehicles had an engine failure on 1 January 2020 and its engine had to be sold as scrap for
Rs. 0.1 million. The vehicle had been acquired on 1 January 2018 at a cost of Rs. 2.5 million. 40% of
the cost is attributable to its engine. Though the engine of similar capacity was available at a cost of
Rs. 1.2 million, the old engine was replaced on 1 January 2020 with a higher capacity engine at a cost
of Rs. 1.8 million.

158
Page 4 of 14
“The Dunya (World) is not the resting place, it is the testing place.”

(v) HIL uses cost model for subsequent measurement of property, plant and equipment except for land and
buildings.
(vi) HIL accounts for revaluation on net replacement value method and transfers the maximum possible
amount from revaluation surplus to retained earnings on an annual basis.
(vii) HIL deducts government grant in arriving at the carrying amount of the asset.

Required:
In accordance with IFRSs, prepare a note on ‘Property, plant and equipment’ for inclusion in HIL’s financial
statements for the year ended 30 June 2020. (20)

(Comparatives figures and column for total are not required)

Answers

A.2 Limitations of ratio analysis:


(i) Historical
All information used in ratio analysis is derived from actual historical results. This does not mean
that the same results will carry forward into the future. However, ratio analysis can be used on pro
forma information and compare it to historical results for consistency.

(ii) Historical versus current cost


The information on the income statement is stated in current costs (or close to it), whereas many
elements of the balance sheet are stated at historical cost (which could vary substantially from
current costs). This disparity can result in unusual ratio results.

(iii) Inflationary effect


If the rate of inflation has changed in any of the periods under review, this can mean that the
numbers are not comparable across periods. For example, if the inflation rate was 100% in one
year, sales would appear to have doubled over the preceding year, when in fact sales did not
change at all.

(iv) Aggregation
The information in a financial statement line item that is used for a ratio analysis may have been
aggregated differently in the past, so that running the ratio analysis on a trend line does not
compare the same information through the entire trend period.

159
Page 5 of 14
A.3
Journal Entries:
Rs in ‘Millions’
Dr. Cr.
1-7-2014
Cash 280
Deferred Grant income 280
1-1-2015
Plant 630
Cash / Payable 630
1-1-2015
Deferred grant income 280
Plant 280
31-12-2015
Depreciation 50
Accumulated depreciation 50
[ 630 – 280 ] / 7 = 50
31-12-2016
Depreciation 50
Accumulated Depreciation 50
31-12-2017
Depreciation 50
Accumulated Depreciation 50
31-12-2018
Depreciation 50
Accumulated Depreciation 50
1-1-2019
Plant 280
Cash (Repayment of grant) 280
1-1-2019
Cumulative depreciation to be charged immediately
Depreciation 160
Accumulated depreciation 160
[ 280 / 7 x 4 ] = 160
31-12-2019
Depreciation 90
Accumulated Depreciation 90
[ 630 / 7 ] = 90

31-12-2019
Impairment Test:
Carrying Amount = 180
[630 – 50 x 4 – 160 – 90]
Or
[630 – (90 x 5)]

160
Page 6 of 14
“Kindness is a mark of faith, and whoever is not kind has not faith.”

Recoverable Amount:
Higher of:
FV less CTS 155
(160 – 5)
Value in Use (W) 161.04
161.04

Working: Value in use


[90 + 10] x ( 1 + 0.12 ) -1 = 89.29
[80 + 10] x ( 1 + 0.12 ) -2 = 71.75
Total = 161 .04

Impairment Loss :
[180 – 161.04] = 18.96

Impairment Loss ( P.L ) 18.96


Accumulated Impairment Loss 18.96

A.4
i. a) Profits will be overstated and assets will be understated
ii. b) Residual value of the property
iii. c) A significant increase in credit sales
iv. e) Rs. 1,905,000 (W.1)
v. f) Increase current ratio and decrease quick ratio (W.2)
vi. g) Declaration and payment of cash dividend (W.3)
vii. h) If funds have been arranged from various general borrowings, the amount to be capitalised is based on
the weighted average cost of borrowings

W.1:
Subscription A/c
b/d 150,000 b/d 180,000
Cash 90,000
Bad debts 60,000
Income 1,905,000 (150,000 – 90,000)
Remaining receivable of 2018

Cash 1400,000
Cash 200,000
c/d 200,000 c/d 325,000

161
Page 7 of 14
W.2: For Example
Current Asset Current Liability Quick Assets Current Liabilites
200,000 100,000 80,000 100,000
Existing ratios 2 1 0.8 1
If creditors are (50,000) (50,000) (50,000) (50,000)
paid in cash
150,000 50,000 30,000 50,000
Revised ratios 3 1 0.6 1

W.3:
(a) Issuance of shares at premium (Equity will increase)
(b) Issuance of shares at discount (Equity will increase)
(c) Issuance of bonus shares (No effect of equity)
(d) Declaration and payment of cash dividend (Equity will decrease and therefore gearing ratio will
increase)

A.6
Taxila Limited
Statement of cash flows for the year ended 30 June 2020
Rs. in million
Cash flows from operating activities
Profit before tax (as no tax) 140
Adjustments for:
Depreciation on property, plant and equipment 290
Depreciation on investment property 120 + 180 + 290 10
Gain on disposal of property, plant and equipment (8)
Revaluation gain (35)
Interest expense 45
Investment income (6)
Operating profit before working capital changes 436
Changes in working capital:
Increase in inventory (25)
Decrease in prepayments and other receivables 9
Increase in trade receivables (51)
Increase in trade and other payables 29
Cash generated from operations 398
Interest paid (50)
Net cash flows from operating activities 348

Cash flows from investing activities


Purchase of property, plant and equipment (389)
Purchase of investment property (180)
Proceeds from disposal of property, plant & equipment 5
Disposal of short term investment (As no other information) 3

162
Page 8 of 14
Investment income recieved 6
Net cash flows used in investing activities (555)

Cash flows from financing activities


Proceeds from issue of shares at premium (200 + 100) 300
Repayment of long term loan (60)
Net cash flows from financing activities 240
Net increase in cash and cash equivalents 33
Cash and cash equivalents at beginning of the year 26
Cash and cash equivalents at the end of the year 59

Workings:

Cash and Cash equivalents:


2020 2019
ST. Investment 35 20
Cash & Bank 24 6
59 26

PPE
b/d 1,200
SC 240 Disposal -
R.S 45 Depreciation 290
Reversal of loss(P.L) 35
Cash 389 c/d 1,619

Inventories
b/d 180
Increase 25
c/d 205

Investment Property
b/d 120
Cash 180 Dep 10
c/d 290

Trade Receivables
b/d 291
Increase 51
c/d 342

163
Page 9 of 14
Prepayments & others
b/d 20
Disposal 3 Decrease 9
c/d 14

S.T Investment
b/d 28
(48-20)
Disposal 3
c/d 25
(60-35)

S.C
b/d 800
PPE 200
(2 x 100)
Cash 200
c/d 1,200

S.P
b/d 150
PPE 40
(240-200)
Cash 100
c/d 290

R.E
b/d 90
PAT 140
R.E 30
c/d 260

Revaluation Surplus
b/d 200
R.E 30
PPE 45
c/d 215

164
Page 10 of 14
Trade Payable
b/d 103
(120 - 17)
Decrease 29
c/d 132
(144 - 12)

Loan
b/d 505
(445 + 60)
Cash 60
c/d 445

Financial Charges Payable


b/d 17
Cash 50
Expense 45
c/d 12

Receivable 3
Cash 5
PPE (36 - 36) 0
Gain (P.L) 8

A. 8
Harappa Industries Limited
Notes to Financial Statements
For the year ended 30-06-2020
Property, Plant and Equipment:
Land Building Plant Vehicles
-------- Rs. In”000” ---------
Gross Carrying Amount:
Opening balance 100,000 70,000 180,000 8,800
Additions - - 103,240 (W-1) 1,800
Disposals - - - (1,000)
Elimination - (17,500) (W-5) - -
Revaluation 20,000 (W-6) (17,500) (W-5) - -
Closing balance 120,000 35,000 283,240 9,600
Accumulated depreciation:
Opening balance - 14,000 60,000 4,000

165
Page 11 of 14
Depreciation - 3,500 14,581 (W-2) 1,068 (W-3)
[70,000 / 20]
Disposal - - - (352) (W-3)
Elimination - (17,500) - -
Closing balance - - 74,581 4,716
Carrying Amount 120,000 35,000 208,659 4,884

Disclosures:
Land Building Plant Vehicles
Measurement Basis Revaluation Revaluation Model Cost Model Cost Model
Model
Useful life/Rate - 15 Years 15/10 years 20%
Method of Depreciation - Straight Line Straight Line Reducing Balance

The last revaluation was performed by Smart Consultants on 30-06-2020, an independent firm of valuer. Had the
land and building been at cost model, carrying amount would have been on 30-06-2020
112 million (100 + 12) and 37.5 million (1.1) respectively.

(1.1)
16,000 / 16 x 20 = 20,000 should be surplus on 1-7-2015 (four years ago) therefore WDV on that date would be
70,000 – 20,000 = 50,000.

Afterwards WDV should be depreciated over remaining useful life (As original cost and total life is not available).
50,000 / 20 x 5 = 12,500
So 50,000 – 12,500 = 37,500

W-1) Cost of New Plant:


Cost [12,000 + 48,000 + 48,000 + 12,000] 120,000
Less: Government Grant 20,000
100,000

Capitalization of borrowing cost:


12,000 x 12% x 7/12 (From Aug to Feb) 840
48,000 x 12% x 5/12 (From Oct to Feb) 2,400
Total 3,240
Total Cost of plant 103,240

W-2) Depreciation of Plant:


Opening balance [180,000 / 15] 12,000
On new plant [103,240 / 10 x 3/12] 2,581
Total 14,581

166
Page 12 of 14
“You will die the way you lived – Prophet Muhammad SAW”

W-3) 01-01-2020 for Vehicle:

Vehicles
b/d 8,800 1-1-2020
Disposal (2.5 x 40%) 1,000
1-1-2020
Cash 1,800
c/d 9,600

Accumulated Depreciation
1-1-2020 b/d 4,000
Disposal 352
Depreciation 1,068
c/d 4,716

Accumulated Depreciation of Engine disposed on 1.1.2020:


Cost (2,500 x 40%) [1-1-2018] 1,000
x 20% x 6/12 (100)
30-06-2018 900
x 20% (180)
30-06-2019 720
x 20% x 6/12 (72)
1-1-2020 648
Accumulated Depreciation (100 + 180 + 72) = 352

Depreciation:
Opening WDV (8,800 – 4,000) 4,800
Less: Opening WDV of Disposal (720)
4,080
x 20% 816
+ 720 x 20% x 6/12 72
+ 1,800 x 20% x 6/12 180
1,068

W-4) Revaluation Surplus – Building:


Opening balance [It is not the surplus on the date of last revaluation] 16,000
Incremental Depreciation [Transfer of surplus] [ 16,000 / 16 (w-4.1)] (1,000)
Remaining Surplus on 30-06-2020 15,000

167
Page 13 of 14
(W-4.1)
Depreciation per Annum [70,000 / 20 = 3,500]
No. of years completed [14,000 / 3,500 = 4 upto 30-06-2019]
Remaining life on 1-7-2019 [20-4] = 16

W-5) Revaluation of buildings on 30-06-2020


Carrying amount [70,000 – 14,000 – 3,500] 52,500
Fair value 35,000
Revaluation Loss 17,500

Dr. Cr.
Accumulated Depreciation [14,000 + 3,500] 17,500
Building 17,500
Revaluation Surplus (OCI) 15,000
Reversal of Loss (P.L) 2,500
Building 17,500

W-6) Revaluation of Land:


Dr. Cr.
Land 20,000
Reversal of Loss (P.L) 12,000
Revaluation Surplus (OCI) 8,000

168
Page 14 of 14
Q.1 Following information pertains to Astrazenca Limited (AL):
(i) Shareholders' equity as on 1 January 2020:

Rs. in million
Share capital (Rs. 100 each) 250
Share premium 138
Retained earnings 142
Revaluation surplus: Land 25
Buildings 20
(ii) Profit and transfer of incremental depreciation as per the draft financial statements for the year ended
31 December 2020 amounted to Rs. 45 million and Rs. 5 million respectively.

(iii) Dividends for the last two years:

For the year ended *Interim cash dividend Final bonus dividend
31 December 2019 10% 20%
31 December 2020 12% 15%

*Declared with half yearly accounts


(iv) AL uses revaluation model for subsequent measurement of its land and buildings only. The
revalued amounts of land and buildings have been assessed at 31 December 2020 but
not incorporated in draft financial statements. The relevant details are as under:

Land Buildings
--- Rs. in million ---
Balances as on 31 December 2020 before revaluation:
Cost 75 240
Accumulated depreciation - 60
Revalued amounts assessed at 31 December 2020 65 158

Required:
Prepare AL’s statement of changes in equity for the year ended 31 December 2020. (08)
(Column for total and comparative figures are not required)

Q.4 Select the most appropriate answer from the options available for each of the following Multiple Choice
Questions (MCQs).

(i) Which of the following future cash flows should NOT be included in the calculation of
value in use of an asset?
(a) Cash flows on maintaining the asset’s performance
(b) Cash flows on enhancing the asset’s performance
(c) Cash flows from continuing use of the asset
(d) Cash flows from disposal of the asset (01)

169
Page 1 of 15
(ii) When an impairment review is carried out, an impaired asset is measured at:
(a) fair value less cost to sell
(b) value in use
(c) cost
(d) recoverable amount (01)

(iii) Which of the following would be an external indicator that an asset of an entity may be impaired?

(a) Increase in central bank discount rates


(b) Decline in economic performance of an asset
(c) Physical obsolescence of an asset
(d) Future restructuring plan of an asset (01)

(iv) Which of the following is NOT a measurement base for assets as referred in theConceptual
Framework?

(a) Value in use (b) Fulfilment value


(c) Current cost (d) Fair value (01)

(vi) An entity recognises revenue over time if:

(a) entity’s performance does not create an asset with an alternative use
(b) entity’s performance creates an asset whose control will be transferred at the endof contract

(c) customer simultaneously receives and consumes the benefit provided by theentity’s
performance
(d) entity has an enforceable right to payment for performance completed to-date (01)

(vii) An entity made a profit of Rs. 550,000 for the year 2020 based on historical cost accounting principles.
It had opening capital of Rs. 1,500,000. During 2020, specific prices indices increased by 15% while
general price indices increased by 10%. How much profit should be recorded for 2020 under physical
capital maintenance concept?
(a) Rs. 325,000 (b) Rs. 400,000
(c) Rs. 467,500 (d) Rs. 495,000 (01)

(viii) In order to survive in the long run, a business must generate positive net cash flowfrom:

(a) investing activities


(b) operating activities
(c) financing activities
(d) both (a) and (b) (01)

Q.5 A fire broke out in the office of Moderna Sports Club (MSC) and burnt all the accounting records. The accountant
was able to retrieve a burnt copy of financial statements of MSC for the year ended 31 December 2020. However, few
information (as indicated by capital alphabets) were unreadable. The retrieved copy is as follows:

170
Page 2 of 15
Balance sheet as on 31 December 2020
Funds and liabilities Rs. in '000 Assets Rs. in '000
2020 2019 2020 2019
General fund: Fixed assets - net 1,403 1,300
Opening balance A 1,586 Members’ subscription 270 158
Excess of income over expenditure B C Misc. supplies 13 10
Tuck-shop rent E 37
Tennis court fund 260 200 Advance salaries 18 15
Bank F 530
Liabilities:
Members’ subscription 20 25
Salaries 52 41
Utilities 25 D
Annual sports event 10 -

Income and expenditure account for the year ended 31


December 2020
Expenditur Rs. in '000 Income Rs. in '000
e
Salaries G Members’ subscriptions 919
Utilities 221 Tuck-shop rent 252
Misc. supplies H Donation - sports equipment 70
Members’ subscription written off 12 L M
Annual sports event I
J K
Disposal of fixed assets 8
Repair and maintenance 40
Excess of income over expenditure B

Receipts and payments account for the year ended 31


December 2020
Receipts Rs. in '000 Payment Rs. in '000
s
Opening balance 530 Salaries 560
N O Fixed assets 92
Tennis court fund P Annual sports event 180
Contribution for annual sports event 49 Misc. supplies 132
Entrance fee - annual sports event 86 Utilities 214
Sale of fixed assets 21 Repair and maintenance Q
Tuck-shop rent 248 Construction of tennis court 131

171
Page 3 of 15
Scrap sale 15 Closing balance F

Required: Determine the missing information as indicated by capital alphabets.

(Redrafting of Financial Statements is not required) (18)

Q.6 Epivac Limited is considering to take some of the following measures during the last week of
the year ending 31 March 2021 in order to show better financial performance;
(I) Pay balance of a major supplier from bank overdraft facility and avail 5% discount.
(ii) Sell slow moving stock items at a price equal to cost.
(iii) Recover debtors’ balances by offering cash discounts of 10%.
(iv) Offer extended credit terms of 90 days which would increase sales at existing margins.
(v) Dispose-off some non-current assets at gain.
Required:
State the effect (increase, decrease, no effect) of each of the above measure on the financialratios as per
following format:
RATIOS: (I) (ii) (iii) (iv) (v)
(a) Gross profit margin

(b) Net profit margin

(c) Current ratio

(d) Stock turnover (times)

(e) Return on non-current assets (17)

(f) Quick ratio

Q.7 You have recently joined as the finance manager of Corv Limited (CL). While reviewing the draft
financial statements for the year ended 31 December 2020 prepared by the junior accountant,
you have noted the following:

(i) In January 2020, Government allotted an industrial plot to CL at a prime location subject
to the condition that CL will establish a factory. CL constructed the factory building which
was available for use on 1 October 2020. Due to delay in recruitment of key factory
employees, the production activities will commence on 15 March 2021.
The accountant has not recorded the land as it was given free of cost. While the factory
building is still appearing in capital work in progress as production activities will commence
on 15 March 2021. (06)

172
Page 4 of 15
(ii) CL acquired a three story building on 1 March 2020. CL uses the ground floor for its
marketing department while remaining two floors were in excess of CL’s need and
therefore were rented out. The first floor was rented out on 1 June 2020 and the second
floor was rented out on 1 December 2020.
The accountant has recorded the building as property, plant and equipment. The
depreciation on ground, first and second floors has been computed from 1
March 2020, 1 June 2020 and 1 December 2020 respectively. (05)

(iii) CL is constructing a power generation plant for its factory. The project started on1
February 2020 and would complete on 30 November 2021. The work remained suspended
for 3 months. The project is financed through long term loan, acquired specifically on 1
January 2020. The unutilized amount of loan is kept in a separate saving account.

The accountant has deducted income of separate saving account from full year’s interest
on loan and presented the net amount as finance cost in the statement of profit or loss.
(05)

The accounting policy of CL is to carry land and building at fair value (wherever permitted by
IFRS).

Required:
Discuss how the above issues should be dealt in the financial statements of CL for the year
ended 31 December 2020 in accordance with the requirements of IFRSs.

Q.8 Sputnik Sea Limited (SSL) runs a cruise business across oceans. Following information in
respect of one of SSL’s cruise ship is available:
(i) SSL bought a cruise ship on 1 March 2018. After completing all the requiredformalities,
the ship was ready to sail on 1 April 2018.
(ii) Details regarding components of the ship are as under:

Cost Estimated
Component (Rs. in million) Useful life residual value
(Rs. in million)
Engine 840 50,000 hours 40
Body 535 25 years 35
Dry-docking (overhaul) 60 5 years -

(iii) On 1 May 2019, the ship suffered an accident which damaged its body. Repair work
took 2 months and costed Rs. 26 million. The repair work did not change useful life and
residual values of the components.
(iv) The average monthly sailing of the ship during the last three years are as under:

Year Hours
2018 360
2019 480

173
Page 5 of 15
2020 600
(v) SSL uses revaluation model for subsequent measurement. SSL accounts for revaluation on net
replacement value method and transfers the maximum possible amount from the revaluation surplus
to retained earnings on an annual basis.
(vi) The revalued amounts of the ship as at 31 December 2019 and 2020 were determined as Rs. 1,400
million and Rs. 1,000 million respectively. Revalued amounts are apportioned between the
components on the basis of their book values before the revaluation.

Required:
Prepare necessary journal entries to record the above transaction from the date of acquisition of the ship to the year
ended 31 December 2020. (17)

A.1 Astrazenca Limited


Statement of changes in equity
For the year ended 31 December 2020
Share Share Retained Revaluation
capital premium earnings surplus
--------------- Rs. in million ---------------
Balance as at 1 January 2020 (As given) 250 138 142 45
(25+20)
Final bonus dividend @ 20% for 2019 (250×20% 50 (50)
)
Interim cash dividend @ 12% for (250+50)
2020 ×12 (36)
%)
Total comprehensive income for the year:
Profit for the year 45–7 (W-1) 38
Other comprehensive income (10+15)(W-1) (25)
Transfer of incremental depreciation 5 (5)
Balance as at 31 December 2020 300 88 149 15

W-1: Revaluation of building Building Land


Rs. in million
Revalued amount 158 65
Carrying value (240–60) 180 75
Revaluation loss (22) (10)
Available surplus (20–5) 15 25
Revaluation loss taken to profit or loss 7 -

174
Page 6 of 15
A.4
(i) (b) Cash flows on enhancing the asset’s performance
(ii) (d) Recoverable amount
(iii) (a) Increase in central bank discount rates
(iv) (b) Fulfilment value
(v) (c) Customer simultaneously receives and consumes the benefit provided by the
entity’s performance
(vii) (a) Rs. 325,000[550,000 - (1,500,000x15%)]
(viii) (b) Operating activities

A.5
Balance sheet as on 31 December 2020
Funds and liabilities Rs. in '000 Assets Rs. in '000
2020 2019 2020 2019
General fund: (w-1) Fixed assets - net 1,403 1,300
Opening balance 1,766 1,586 Members’ subscription 270 158
Excess of income over expenditure 62 180 Misc. supplies 13 10
1,828 1,766 Tuck-shop rent (w-9) 41 37
Tennis court fund (w-2) 260 200 Advance salaries 18 15
Bank 450 530
Liabilities:
Members’ subscription 20 25
Salaries 52 41
Utilities(w-4) 25 18
Annual sports event 10 -
2,195 2,050 2,195 2,050

Income and expenditure account for the year ended 31 December 2020
Expenditur Rs. in '000 Income Rs. in '000
e
Salaries (w-7) 568 Members’ subscriptions 919
Utilities 221 Tuck-shop rent 252
Misc. supplies (w-8) 129 Donation - sports equipment 70
Members’ subscription written off 12 Scrap sale 15
Annual sports event (w-5) 55
DEP. (w-6) 161
Disposal of fixed assets 8
Repair and maintenance 40
Excess of income over expenditure (w- 62
1)
1256 1,256

175
Page 7 of 15
Receipts and payments account for the year ended 31 December 2020
Receipts Rs. in '000 Payment Rs. in '000
s
Opening balance 530 Salaries 560
Membership subscriptions (w- 790 Fixed assets 92
3)
Tennis court fund (w-2) 60 Annual sports event 180
Contribution for annual sports event 49 Misc. supplies 132
Entrance fee - annual sports event 86 Utilities 214
Sale of fixed assets 21 Repair and maintenance 40
Tuck-shop rent 248 Construction of tennis court 131
Scrap sale 15 Closing balance 450
1,799 1,799

176
Page 8 of 15
177
Page 9 of 15
Annual sports event:

Contribution 49
Entrance fee 86
Exp (190)
Net loss (55)
A.6

S. No. Ratios Measures


(i) (ii) (iii) (iv) (v)
(a) Gross profit % increase Decrease Decrease No effect No effect
(b) Net profit % Increase Decrease Decrease Increase Increase
(c) Current ratio Increase No effect Decrease Increase Increase
(d) Stock turnover (times) decrease Increase No effect Increase No effect
(e) Return on non-current assets Increase No effect Decrease Increase Increase
(f) Quick ratio Increase Increase Decrease Increase Increase

Workings:
Suppose Existing Figures are:
a) Statement of Profit or loss
Rs. In’‘000’’
Sales 1,000
Cost of sales (800)
Gross profit 200
Expenses (150)
Net profit 50

b) Statement of Financial Position


Non-current assets 500
Current assets :
Stock 225
Debtors 100
Cash 50
375
Total Assets 875
Equity & Liabilities
Equity: Share Capital 300
Retained Earnings 125
425
Non-Current Liabilities: 150
Current Liabilities:
Creditors 150
Bank Over Draft 150
300

178
Page 10 of 15
Total Equity & Liabilities: 875

(C) Existing Ratios:


Gross Profit = 200/1000 x 100 = 20%
Net Profit = 50/1000 x 100 = 5%
Current Ratio = 375/300 = 1.25 Times
Stock Turn Over = 800/225 = 3.56 times
Return on Non-Current Assets 50/500 x 100 = 10%
Quick Ratio = 375 - 225/300 = 0.5 times

Analysis:
(1) Creditors 100
Bank Overdraft 95
Discount Received 5
a) Gross Profit Ratio decrease
b) Net Profit Ratio {50+5} / 1000 x 100 = 5.5% Increase
c) Current Ratio 375/(300-100+95) = 1.27 times Increase
d) Stock Turn Over decrease
e) Return Non-Current Asset {50+5} / 500 x100 = 11% Increase
f) Quick Ratio (375-225} / {300-100+95} = 0.51 times Increase

2) Cash 100
Sales 100
Cost of Sales 100
Stock 100

a)Gross Profit = (200+100-100) / (1000+100)x100 = 18.18% Decrease


b)Net Profit = (50+100-100) /(1000+100) x100 = 4.55% Decrease
c)Current Ratio = (375+100-100) /300 = 1.25 times no effect
d) Stock Turn Over = (800+100)/(225-100) = 7.2 times increase
e) Return on Non – Current Assets =(50+100-100)/500x100= 10% no effect
f) Quick Ratio =(375+100-100-(225-100))/300= 0.83 time increase

3) Cash 90
Discount Allowed 10
Debtor 100
a)Gross Profit decrease
b)Net Profit = (50-10)/1000 x 100 = 4% Decrease
c)Current Ratio = (375+90-100)/3000 = 1.22 times Decrease
d)Stock Turn Over = 800/225 = 3.56 times No effect
e)Return on Non-Current Assets = (50-10)/500 x100 = 8% Decrease
f)Quick Ratio =( 375+90-100)-225/300 = 0.46 times Decrease

179
Page 11 of 15
4) Debtor 100
Sales 100
Cost of sales 80
Stock 80

a)Gross Profit = (200+100-80)/(1000+100)X100 = 20% no effect


b)Net Profit = (50+100-80)/ (1000+100)X100 = 6.36% Increase
c)Current Ratio = (375+100-80)/300 = 1.32 times increase
d)Stock Turn Over = (800+80)/(225-80) = 6.07 times increase
e)Return on Non-Current Assets = (50+100-80)/500X100 = 14% increase
f)Quick Ratio = {( 375+100-80) -(225-80)or(100+50+100)}/300 0.83 times increase

5) Cash 100
Non-Current Asset

Gain (P.L) 10
a)Gross Profit = (200/1000)x100 = 20% no effect
b)Net Profit = (50+10)/1000x100 = 6% increase
c)Current Ratio = (375+100)/300 = 1.58 times increase
d)Stock Turn Over = 800/225 = 3.56 times no effect
e)Return on Non-Current Assets = (50+10)/(500-90)x100 = 14.63% increase
f)Quick Ratio = (375+100)-225/300 = 0.83 times increase

A.7
(i) The accounting treatment adopted by accountant for not recording land is incorrect. Allotment of land by
Government is a transfer of a non-monetary asset and should be considered as a government grant. Such non-
monetary grant may be recorded at fair value or at a nominal value. As per CL’s policy, fair value of the land should
be assessedand reported in the financial statements under the head property, plant and equipment (PPE). The
grant was made subject to construction of factory so the resulting deferred income should be recognized in
income on a systematic basis over the useful life of the factory building.

The factory building should also be transferred from capital work in progress to PPE account as the building is
available for use on 1 October 2020. Further depreciation should also be charged from same date i.e. 1 October
2020 (for three months)

(ii) The accounting treatment adopted by accountant to record complete building under PPE head is incorrect
(assuming portions can be sold separately). Two floors which have been leased/rented separately so should be
accounted for as investment property (under IAS-40). While ground floor used by marketing department should
be recorded as property, plant and equipment under IAS 16 and depreciated over its useful life.

As per CL policy, investment property should be recorded at fair value and changes in fair value should be taken
to statement of profit or loss. Any depreciation already charged on these floors should be reversed.

180
Page 12 of 15
If however portions cannot be sold separately then entire building should be classified as investment property as
significant portion (2 floors out of 3 floors) is on rent, under IAS-40. The whole building should then be recorded
at fair value and changes in fair value should be taken to statement of profit or loss. In such a situation whole
depreciation should be reversed.

(iii) The accounting treatment adopted by accountant to expense out borrowing cost is incorrect as some
borrowing cost is eligible for capitalization. Power generation plant falls under definition of qualifying asset as its
construction involves substantial period.

Construction of the power plant is financed through specific borrowing so actual borrowing cost incurred less
temporary investment income on the borrowings would be capitalized. However, the borrowing cost will be
capitalized from the date when construction actually started i.e. 1 February 2020 rather than 1 January 2020.
Further, the capitalization of borrowing costs should be suspended and charged to the statement of profit or loss
during the three months when work was suspended.

In the statement of profit or loss, borrowing cost on loan and interest income earned from saving account should
be presently separately (related to the months of January, three months of suspension and December).

A.8
Sputnik Sea Limited
General Journal Rs. in million
Date Description Debit Credit
01-03-2018 Cruise ship (840+535+60) 1,435.00
Bank 1,435.00

31-12-2018 Depreciation expense (W-1) 75.84


Accumulated depreciation - Cruise ship 75.84

01-05-2019 Repair cost 26.00


Bank 26.00

31-12-2019 Depreciation expense (W-1) 108.80


Accumulated depreciation - Cruise ship 108.80

31-12-2019 Accumulated depreciation - Cruise ship 184.64


Cruise ship 75.84+108.80 184.64

31-12-2019 Cruise ship 149.64


Revaluation surplus 149.64

31-12-2020 Depreciation expense (W-1) 165.82


Accumulated depreciation - Cruise ship 165.82

31-12-2020 Revaluation surplus 165.82–147.20 (W-2) 18.62

181
Page 13 of 15
Retained earnings 18.62

31-12-2020 Accumulated depreciation - Cruise ship 165.82


Cruise ship 165.82

31-12-2020 Revaluation surplus (W-3) 131.02


Revaluation loss (W-3) 103.16
Cruise ship 1,234.18–1,000 234.18

W-1: Rs. in million


Date Description Engine Ship's Body Dry Docking Total
1/3/2018 Acquisition cost 840.0 535.0 60.0 1,435
31/12/18 Depreciation expense (51.84) (15.0) (9.0) (75.84)
(840– (535– 60÷5×9/12
40)÷50,000×(360×9) 35)÷25×9÷12
31/12/19 Depreciation expense (76.80) (20.0) (12.0) (108.80)
(840–40) ÷ 50,000 × (535– 60÷5
(480×10) 35)÷25

(12 – 2)
Repair
months
31/12/19 Carrying value 711.36 500.0 39.0 1,250.36
Revaluation surplus 85.13 59.84 4.67
(Bal.) 149.64
31/12/19 Revalued amount 796.49 559.84 43.67 1,400.00
[711.36/1250.36 [500/1250.36 [39/1250.36
x1400] x1400] x1400]
31/12/20 Depreciation expense (129.81) (22.57) (13.44) (165.82)
(796.49–40) ÷ (50,000–(559.84– 43.67÷(5–
3240 35)÷(25–1.75) 1.75)
–4,800)×(600×12)
31/12/20 Carrying value 666.68 537.27 30.23 1,234.18
31/12/20 Revaluation loss (126.50) (101.95 (5.74) (234.18)
)
31/12/20 Revalued amount 540.18 435.33 24.49 1,000
[666.68/1234.18 [537.27/1234.1 [30.23/1234.
x1000] 8 x1000] 18 x1000]

182
Page 14 of 15
W-2:
31.12 .2020 Transfer of surplus to retained earnings:

Engine: 85.13/(50,000 – 3,240 – 4,800) x (600 x 12) = 14.61


Ships body: 59.84/(25 – 1.75) = 2.57
Dry Dockings: 4.67/(5 – 1.75) = 1.44

Total 18.62
W-3
31 .12. 2020 Adjustment of revaluation loss:
Revaluation Transfer of Remaining Revaluation Adjustment Revaluation loss
surplus on surplus on surplus on loss as on against (P.L)
31.12.2019 31.12.2020 30.6.2020 31.12.2020 surplus(OCI)
Engine 85.13 14.61 70.52 126.50 70.52 55.98
Ships 59.84 2.57 57.27 101.95 57.27 44.68
body
Dry 4.67 1.44 3.23 5.74 3.23 2.51
Dockings
Total 149.64 18.62 131.02 234.18 131.02 103.17

183
Page 15 of 15
Earnings per share (IAS 33)
Introduction
‘Earnings per share’ is a ratio used in the financial analysis of a set of financial statements. This ratio is, however, so
useful and popular that the standard, IAS 33, had to be developed to control the method of calculation thereof. This
standard sets out how to calculate:
• the numerator: earnings; and
• the denominator: the number of shares

Objective of IAS 33
The objective of IAS 33 is to set out principles for:

the calculation of EPS; and

the presentation of EPS in the financial statements.

The purpose of standardizing the calculation and presentation of EPS is to make it easier for the users of financial
statements to compare the performance of:

different entities in the same reporting period; and

the same entity for different reporting periods over time.

Scope of IAS 33
IAS 33 applies only to publicly-traded entities or those which are about to be publicly traded. A publicly-traded
entity is an entity whose shares are traded on a stock exchange.

Price earning (P/E) RATIO AND EARNINGS PER SHARE (EPS)


Earnings per share
Earnings are profits available for equity holders (ordinary shareholders). Earnings per share (EPS) is a measure of
the amount of earnings in a financial period for each equity (ordinary) share.

As its name implies, EPS is calculated as earnings divided by the number of ordinary shares in issue.

The price/earnings ratio


The price/earnings ratio (P/E ratio) is also a key stock market ratio. It is a measure of the company’s current share
price (market price) in relation to the EPS. The P/E ratio is calculated as follows:

Formula: Price earnings ratio


P/E ratio = Market value of share / Earnings per share
= 46 / 23 = 2 times

Interpretation: This ratio shows the payback period of investment.

EPS is used by investors as a measure of the performance of companies in which they have invested or might
possibly invest. Investors are usually interested in changes in a company’s EPS over time.

EPS should therefore be calculated by all companies in a standard way, so that investors can obtain a reliable
comparison between the EPS and P/E ratios of different companies.

184
Page 1 of 40
Definition of ordinary share
An ordinary share is an equity instrument that is subordinate to all other classes of equity instruments.

The ordinary shares used in the EPS calculation are those entitled to the residual profits of the entity, after
dividends relating to all other shares have been paid.

Types of shareholders
Ordinary shareholders
Ordinary shareholders buy a share in a company to earn dividends, (when this payment is considered prudent) and
for capital growth. These dividends fluctuate annually depending on profits and available cash reserves etc. As the
terms ‘ordinary’ and ‘preference’ implies, the ordinary shareholders have fewer rights than the preference
shareholders. By way of illustration, assume that a company with both preference and ordinary shareholders is
liquidated (wound up): the preference shareholders will have their capital returned first and only if there are
sufficient funds left over, will the ordinary shareholders have their capital paid out.

Preference shareholders
Preference shareholders have more rights than ordinary shareholders. Not only do they have preference on
liquidation, but they also have a fixed amount paid out each year in dividends (as opposed to ordinary shareholders
whose dividends are at the discretion of the entity and are largely dependant on profits and available cash reserves).
The rate of dividends paid out is based on the preference share’s rate (e.g. 10%). A shareholder owning 1 000
preference shares with a par value of 2 each and a rate of 10% will expect dividends of 200 per year (2 x 1 000 x
10%). The shareholder’s rights to dividends depend on whether his shares were:
• cumulative; or
• non-cumulative.

Cumulative preference shares indicate that if a dividend was not paid out in a particular year, (perhaps due to
insufficient funds), these arrear dividends must be paid out in the future to the holders of these shares when funds
become available. No dividend may be paid out to ordinary shareholders until the arrear preference dividends have
been paid. Non-cumulative preference shares are those where, if a dividend is not paid out in a year, these unpaid
dividends need never be paid.

There is a further variation with regard to preference shares; the shares may be:
• redeemable; or
• non-redeemable(Irredemable)

Redemption of a share involves the company returning the amount invested by the shareholder to this shareholder
at some stage in the future. Shares that are redeemable should be classified as a liability instead of as equity, in
which case the related dividends will be recognised as ‘finance charges’ in the statement of comprehensive income
instead of as ‘dividends’ in the statement of changes in equity (as in case of ordinary shares)

Point to remember:
As suggested already, some preference shares are recognised as liabilities rather than as equity and their dividends
are recognised as finance charges instead of as dividends. In these instances, even if the dividend has not yet been
declared as at the end of the reporting period, the dividend will be recognised as a finance charge on accrual basis
(just like interest expense on a loan)

185
Page 2 of 40
There are two types of Earnings per share
1. basic earnings per share
2. diluted earnings per share

1. Basic earnings per share

Earnings
Number of shares
Detailed Formula: Basic EPS

Net profit (or loss) attributable to ordinary shareholders during a period


Weighted average number of ordinary shares in issue during the period

Important points to remember:


• This profit should be before deducting ordinary dividend (if any)
• In the event that the entity reports a loss instead of a profit, the earnings per share will simply be reported as a
loss per share instead.
• This ratio is different from dividend per share.

Basic earnings
In order to calculate the earnings attributable to the ordinary shareholders, one should start with the ‘ net profit
for the period’ as per the statement of comprehensive income and deduct the profits attributable to the
irredeemable preference shareholders (if any).

Profit (or loss) for the period (Profit (or loss) after tax xxx
Less preference dividends related to irredeemable preference shares (based on (xxx)
Earnings
the rate) attributable to ordinary shareholders xxx

Preference dividends are, in fact, not always deducted. Deciding whether or not to deduct the preference dividends
depends on whether the shares are cumulative or non-cumulative. The following guidelines should be helpful when
dealing with irredeemable preference shares:
• in respect of non-cumulative preference shares, deduct only the preference dividends that are declared in
respect of that period; and
• in respect of cumulative preference shares, deduct the total required preference dividends for the period (in
accordance with the preference share’s rate), whether or not these dividends have been declared.

It should be borne in mind that where the preference shares are classified as a liability in case of redeemable
preference shares, their dividends would be treated as finance costs, in which case these dividends would have
already been deducted in the calculation of ‘net profit for the period’. These must obviously not be deducted again
when calculating ‘earnings attributable to the ordinary shareholders’.

Note: It is always assumed before the calculation of EPS that all accounting entries should have been correctly made
and recorded in the financial statements, unless there is any indication.

186
Page 3 of 40
A. When there are only ordinary shares
If there are only ordinary shareholders, the entire profit or loss of the company belongs to the ordinary shareholders
(owners).

Example 1: ordinary shares only


A company has 10 000 ordinary shares in issue throughout 2011. The company earns a profit after tax of
100 000.
Required:
Calculate the basic earnings per ordinary share.
Answer:
Calculation of the earnings per ordinary share:

= Earnings
Number of ordinary shares

= 100 000
10 000

= 10 per share

B. When there are ordinary and preference shares


If there are both ordinary and preference shareholders, some of the profit for the year must be set aside for the
preference shareholders’ preference dividends.

Example 2: ordinary and preference shares


A company has 10 000 ordinary shares and 10 000 non-cumulative, non-redeemable 10% Rs. 2 each
preference shares in issue throughout 2011.
The company earns a profit after tax of 100 000.
The company declared the full 2011 dividends owing to the preference shareholders.
Required: Calculate the basic earnings per ordinary share.

Answer: Calculation of earnings attributable to ordinary shareholders:


Profit (or loss) for the year 100 000
Less fixed preference dividends (10 000 x 2 x 10%) (2 000)*
Earnings attributable to ordinary shareholders 98 000

*if this dividend would not have been declared then it would be ignored because preference shares are non
cumulative.
Calculation of the earnings per ordinary share:

= Earnings belonging to ordinary shareholders


Number of ordinary shares

= 98 000
10 000

= 9.80 per share

187
Page 4 of 40
Example 3: preference shares and preference dividends – equity versus liability

A company has 10 000 ordinary shares and 10 000 10% Rs. 2 each preference shares in issue throughout
2012. The profit after tax was 100 000 in 2012.
Required:
Calculate the basic earnings in 2012, assuming that the preference shares are:
A) non-cumulative and non-redeemable (i.e. equity) and the dividend is declared.
B) non-cumulative and non-redeemable (i.e. equity) and the dividend is not declared.
C) cumulative and redeemable (i.e. liability) and the dividend is declared.
D) cumulative and redeemable (i.e. liability) and the dividend is not declared.

Answer: 3A Calculation of earnings attributable to ordinary shareholders:


Profit (or loss) for the year 100 000
Less preference dividends: 10 000 x 2 x 10% (2 000)
Earnings attributable to ordinary shareholders 98 000
3B Calculation of earnings attributable to ordinary shareholders:
Profit (or loss) for the year 100 000

3C Calculation of earnings attributable to ordinary shareholders:


Profit (or loss) for the year 100 000
Less preference dividends (see comment below) (0)
Earnings attributable to ordinary shareholders 100 000

Comment: Preference shares that are cumulative and redeemable are treated as liabilities. The dividends on these
preference shares are therefore recognized as interest. This dividend has therefore already been deducted in
calculating the profit for the period.

3D Calculation of earnings attributable to ordinary shareholders:


Profit (or loss) for the year 100 000
Less preference dividends (see comment below) (0)
Earnings attributable to ordinary shareholders 100 000

Comment: Preference shares that are cumulative and redeemable are treated as liabilities. The dividends on these
preference shares are therefore recognized as interest, irrespective of whether or not the dividend has been formally
declared. This dividend should therefore already been deducted on accrual basis in calculating the profit for the
period.

Example: Cumulative preference shares


In the year ended 31 December 2011, Entity G made profit after tax of Rs. 3,500,000.

Entity G has Rs. 1,000,000 10% cumulative irredeemable preference share capital in issue. (This would entitle
investors to receive a dividend of Rs. 100,000 (10% of Rs. 1,000,000)

Entity G had 1 million ordinary shares in issue throughout the year.

188
Page 5 of 40
Entity G’s basic EPS for the year ended 31 December 2011 is calculated as follows:

EPS = 3,500,000 – 100,000 / 1,000,000 = 3.4 per share

Note that Rs. 100,000 deducted above would be deducted irrespective of whether a dividend had been declared or
not. However, if these preference shares had been non-cumulative then the dividend would have been deducted
only in case of declaration by the company.

Basic number of shares


Overview
In the event that there was no movement of shares during the year, (i.e. the balance of shares at the beginning of
the year equals the balance of shares at year-end, say 10 000), then the denominator in the earnings per share
calculation is simply 10 000 shares.

If, however, there was movement in the number of shares during the year, then the number of shares to be used in
the calculation will need to be adjusted or weighted. The movement could result into an increase or a decrease in
the number of shares.

Increase in number of shares could come in the form of:


• issue for value (e.g. shares issued at their market price);
• issue for no value (e.g. bonus shares and share split); and
• combination issue (e.g. shares issued at less than their market value). This share issue is also called as right
shares.

Decreases in the number of shares could come in the form of:


• share buy-backs/repurchase of shares (a for-value reduction); and
• share consolidations (a not-for-value reduction).

Each of these types of movements will now be dealt with separately.

1.Issue for value (means shares issued at full market value)


Time apportionment to find weighted average shares

On 1 January 2020 a company had 5,000,000 ordinary shares in issue. On 1 April 2020, 1,000,000 new shares were
issued.

On 1 July 2020 an extra 1,000,000 shares came into existence. On 1 November 2020 500,000 more shares were
issued.

Assume all issues were at full market price.

189
Page 6 of 40
The weighted average number of shares is calculated as follows.

Weighted
Number of Time average
Date shares factor number
1 January 5,000,000 X 12/12 5,000,000
New issue on the 1 April 1,000,000 X 9/12 750,000
New issue on the 1 July 1,000,000 X 6/12 500,000
New issue on the 1 November 500,000 X 2/12 83,333

7,500,000 6,333,333

Practice question 1
Company B has a financial year ending 31 December.
On 1 January 2013, there were 9,000,000 ordinary shares in issue.
On 1 May 2013, Company B issued 1,200,000 new shares at full market price.
On 1 October 2013, it issued a further 1,800,000 shares, also at full market

price.

Total earnings in Year 3 were Rs. 36,900,000.

Required
Calculate the EPS for the year to 31 December 2013.

SOLUTION
Weighted
Number of Time average
Date shares factor number
1 January 9,000,000 × 12/12 9,000,000
New issue on 1 May 1,200,000 × 8/12 800,000
New issue on 1 October 1,800,000 × 3/12 450,000

12,000,000 10,250,000
EPS = Rs. 36,900,000/10,250,000 = Rs. 3.6

2.Issue for no value (bonus shares and share split)


Issues for no value involve an entity effectively giving away shares. Examples of this include capitalization issues
(bonus issues or scrip issue) and share splits (will be discussed later on). Capitalization issues frequently occur when
a company has a shortage of cash with the result that shares are issued instead of paying cash dividends to the
shareholders.

190
Page 7 of 40
Since there has been no increase in capital resources (there is no cash injection), an increase in profits cannot be
expected. If the earnings in the current year are the same as the earnings in the prior year and there is an increase
in the number of shares in this current year as a result of bonus shares or share split, the earnings per share in the
current year will, when compared with the earnings per share in the prior year, indicate deterioration in the
efficiency of earnings relative to the available capital resources. Comparability would thus be jeopardized
(adversely affected) unless an adjustment is made.

The adjustment made for an ‘issue for no value’ is made to the prior year, (note: an ‘issue for value’ is adjusted for
in the current year based on time). This adjustment has the effect that it appears that the shares issued in the current
year had already been in issue in the prior periods. This adjustment is not time weighted as well as like issue for
value.

Example 8: issue for no value


A company had 10 000 ordinary shares in issue during the previous year. There was a capitalisation issue of 10 000
ordinary shares during the current year. The earnings in the previous year were 20 000, and thus the earnings per
share in the previous year were 2 per share (20 000/ 10 000 shares). Earnings in current year are 30,000.
Required:
Calculate EPS of current year.

Answer: Although the capital base doubled in the current year, the resources available to the entity remained the
same and thus the user could not reasonably expect an increase in profits.

The earnings per share in the current year will be disclosed at 1.50 per share (30 000/ 20 000 shares) and the earnings
per share in prior period presented will be restated: the prior period will disclose an EPS of 1 per share (20 000/ 20
000 shares (10,000 + 10,000)).
Point to remember:
Please note that the adjustment is not time-weighted and therefore issues for no value made during the year, (either
at the beginning or end of the year or at any other point), are all dealt with in the same way (by increasing the current
period shares and adjusting the prior period number of shares).

Share split (example can be Rs. 1,000 note divided into 10 notes of Rs. 100) is a situation where a share is
subdivided into two or more shares. E.g. denomination of share is reduced (e.g. Govt. has changed the
denomination of shares); means Rs. 10 each share is divided into 2 shares of Rs. 5 each etc. In EPS the treatment of
share split is similar to the treatment of bonus shares (as it is also a not for value issue).

Example 8A:

A company had 10 000 ordinary shares in issue on 1.01 2011. On 1 April 2012, there was a share split
whereby every 2 shares became 5 shares. On 1 June 2012, 12 000 shares were issued at market value of
5 per share. The basic earnings were 150 000 in 2011 and 261 250 in 2012.

Required:
Calculate the basic earnings per share for the years ended 31 December 2012 with comparatives. Also calculate
EPS of 2011.

191
Page 8 of 40
Answer:
Earnings per share: [2011]

150 000
10 000
= 15 per share
Earnings per share: [2012]
2012 2011

Earnings 261 250 150 000


Number of shares = 32 000 25 000

= 8.16 per share 6 per share


Workings:

Number of shares Actual Current year Prior year


(weighted) (adjusted)
Opening balance 10 000 10 000 10 000
Issue for no value (bal) [01.04] 15 000 15 000 15 000
[10,000/2x5] 25 000 25 000 25 000
Issue for value [01.06] 12 000 7 000(2) 0
Closing balance 37 000 (1) 32 000 25 000

(1) 22 000 / 2 x 5 = 55 000

(2) 12 000 x 7 / 12 = 7 000

Example 9: issue for no value after an issue for value (a share split with issue of shares with value )

A company had 10 000 ordinary shares in issue on 1.01 2011. On 1 April 2012, 12 000 shares were issued at market
value of 5 per share. On 1 June 2012, there was a share split whereby every 2 shares became 5 shares. The basic
earnings were 150 000 in 2011 and 261 250 in 2012.

Required:
Calculate the basic earnings per share for the years ended 31 December 2012 with comparatives. Also calculate
EPS of 2011.

Answer:
Earnings per share: [2011]

150 000
10 000
= 15 per share

192
Page 9 of 40
Earnings per share: [2012]

2012 2011

Earnings 261 250 150 000


Number of shares = 47 500 25 000

= 5.5 per share 6 per share

Workings:

Number of shares Actual Current year Prior year


(weighted) (adjusted)
Opening balance 10 000 10 000 10 000
Issue for value [01.04] 12 000 9 000(2) 0
22 000 19 000 10 000
Issue for no value[01.06] 33 000 28 500(3) 15 000(4)
55 000 (1) 47 500 25 000

(2) 22 000 / 2 x 5 = 55 000

(2) 12 000 x 9 / 12 = 9 000


(3) 33 000 / 22 000 x 19 000 = 28 500
(4) 33 000 / 22 000 x 10 000 = 15 000

Practice question 2

Company D has a 31 December year end and had 2,000,000 ordinary shares in issue on 1 January 2012.

On 31 March 2012, it issued 500,000 ordinary shares, at full market price.

On 1 July 2012, Company D made a 1 for 2 bonus issue. In 2011, the EPS had been calculated as Rs. 30 per share. In
2012, total earnings were Rs. 85,500,000.

Required
Calculate the EPS for the year to 31 December 2012, and the comparative EPS figure for 2011.

Solution:

Number of shares Actual Current year Prior year


(weighted) (adjusted)
Opening balance 2,000,000 2,000,000 2,000,000
Issue for value 31.3 500,000 375,000 (500,000 0
x9/12)

193
Page 10 of 40
2,500,000 2,375,000 2,000,000
Issue for no value (bonus shares 1/2) 1,250,000 (1) 1,187,500 1,000,000
(2) (3)
3,750,000 3,562,500 3,000,000

(1) 2,500,000 x 1 / 12 = 1 250 000

(2) 1 250 000 / 2 500 000 x 2 375 000 = 1 187 500


(3) 1 250 000 / 2 500 000 x 2 000 000 = 1 000 000

EPS in 2012 = Rs. 85,500,000/3,562,500 = Rs. 24 per share.

The 2011 EPS restated as: Rs. 60,000,000 / 3,000,000 = 20 per share
In 2011 EPS original will be: 60,000,000 / 2,000,000 = 30 per share

3.Combination issues (right shares)


A combination issue is one that involves an issue at less than market value. A right issue occurs when shares are
offered to existing shareholders at a specified reduced price, (less than the market price) in proportion to existing
shares.

The issue price of the new shares in a rights issue is generally below the current market price. This means that they
include a bonus element which must be taken into account in the calculation of the weighted average number of
shares.

While calculation we can subdivide it as: in effect an issue for value (i.e. some of the shares are assumed to have
been sold at full market value) and an issue for no value (some of the shares are assumed to have been given away).
There are two methods of calculating the number of shares: one involves the use of a table (where the principles
are those used in the previous examples) and the other involves the use of formula. Both will give you the same final
answer.
Example 10: rights issue
A company had 10 000 shares in issue at the beginning of the current year (01.01.2012), and 3 months before the
year-end, the company had a rights issue of 1 share for every 5 shares held. The exercise (issue) price was 4 per
share when the fair value immediately before the rights issue was 5 per share (i.e. market value cum rights). Earnings
of prior period are 100 000.

All the shares offered in terms of this rights issue were taken up.

Required:
Calculate the weighted average number of shares in issue for the purposes of calculating earnings per share in the
•financial
The number of shares
statements for theissued in terms
year ended 31of the rights2012.
December issue:Alongwith
10 000/5 comparative
x 1 share = 2 000 shares
•Answer
The Table
cash received
form: from the rights issue: 2 000 shares x 4 = 8 000
• The number of shares that are issued may be split into those shares that are effectively sold and those that are
effectively given away:

194
Page 11 of 40
Number
Shares sold (issue for value): proceeds/ market price cum rights = 8 000/ 5 1,600
Shares given away (issue for no value): [total shares issued – shares sold = [2 000 400
shares – 1 600 shares ]
2,000

The weighted and adjusted average number of shares may then be calculated:

Number of shares (weighted & adjusted) Actual Current year Prior year
(weighted) (adjusted)
Balance: 1/1 10 000 10 000 10 000
Issue for value 1/10 1 600 400 0
(1 600 x 3/12)
11 600 10 400 10 000
Issue for no value 1/10 400 359* 345**

(400/ 11 600 (400/ 11 600 x


x 10400) 10 000)

Balance: 31/12 12 000 10 759 10 345

Solution to example 10: using the ‘formula approach’(if table cannot be used )

Theoretical ex-rights value per share (means market price of the share that ought to be in theory after the
right issue):

(Shares before right issue x Actual cum rights price) + (Right shares x issue price)
ights issue Number of shares in issue after the rights issue

= 10 000 shares x 5 + 2 000 shares x 4


10 000 + 2 000 000

= 58 000
12 000

= 4.833 per share

Adjustment factor:

Actual cum rights price


Theoretical ex-right value per share

= 5
4.833

= 1.0345

195
Page 12 of 40
Number of shares:
Current period: (number of shares before the right issue x adjustment factor x fraction of the year before the
right issue + total shares after the right issue x fraction of the year after the right issue)

Prior period: (number of shares before the right issue x adjustment factor)

= Current year (10 000 shares x 1.0345 x 9/12 + 12 000 x 3/12) 10 759
= Prior year (10 000 shares x 1.0345) 10 345

Or last year EPS can be adjusted as: original EPS x Theoretical ex-right value per share/ Actual cum rights
price:100 000/10 000 =10 x 4.833 /5 = 9.67 or 100 000/10 345 = 9.67

Comment: Notice that the current year calculation of the number of shares is weighted for the number of months
before the issue and after the issue, whereas the prior year is not weighted at all.

Practice question 3
Company F had 3 million ordinary shares in issue on 1 January 2017.
On 1 April 2017, it made a 1 for 2 rights issue of 1,500,000 ordinary shares at Rs. 20 per share.
The market price of the shares prior to the rights issue was Rs. 50.
An issue of 400,000 shares at full market price was then made on 1 August 2017.
In the year to 31 December 2017, total earnings were Rs. 17,468,750. In 2016 EPS had been reported as Rs. 3.5
(means prior period earnings were (3,000,000 x 3.5 = 10,500,000).

Required
Calculate the EPS for the year to 31 December 2017, and the adjusted EPS for 2016 for comparative purposes.

Solution:
The number of shares issued in rights issue: 1,500,000 shares
The cash received from the rights issue: 1,500,000 shares x 20 = 30,000,000
Number
Shares sold (issue for value): proceeds/ market price 30,000,000 / 50 600,000
Shares given away (issue for no value): total shares issued – shares sold = 900,000
1,500,000 shares – 600,000 shares
1,500,000
The weighted and adjusted average number of shares may then be calculated:

Number of shares (weighted & Actual Current year Prior year


adjusted) (weighted) (adjusted)
Balance: 1/1 3,000,000 3,000,000 3,000,000
Issue for value 1 / 4 600,000 450,000 0
(600,000 x9/12)
3,600,000 3,450,000 3,000,000
Issue for no value 1 / 4 900,000
862,500 750,000
(900 / 3,600 x 3,450) (900 / 3,600 x 3,000)

196
Page 13 of 40
1/4 4,500,000 4,312,500 3,750,000
1/8 400,000 166,667 0
(400 x 5 / 12)

31 / 12 4,900,000 4,479,167 3,750,000

EPS for 2017: 17,468,750 / 4,479,167 = 3.9 per share


EPS for 2016 (Restated): 10,500,000 / 3,750,000 = 2.8 per share

Example 11: various issues over three years


Numbers Ltd has a profit of 100 000 for each of the years 2013, 2014 and 2015. There are no preference shares. On
1 January 2013, there were 1 000 ordinary shares in issue, all of which had been issued at 10 each equal to nominal
amount, after which, the following issues took place:

• 30 June 2014: 1 000 ordinary shares were sold for 35 each (their market price);
• 30 September 2014: there was a capitalisation issue of 1 share for every 2 shares in issue
on this date, utilising the share premium account;
• 30 June 2015: 2 000 ordinary shares were sold for 40 each (their market price); and
• 31 August 2015: there was a share split whereby every share in issue became 3 shares.
Required:
A) Journalise the issues for the years ended 31 December 2014 and 2015.
B) Calculate the basic earnings per share to be disclosed in the financial statements of Numbers Ltd for the year
ended 31 December 2015 (with comparative figures of 2014 and 2013)

C) Calculate the basic earnings per share as disclosed in the financial statements of Numbers Ltd for the year
ended 31 December 2014 (with comparative figures of 2013).

Solution to example 11A: journals

30/6/2014 Debit Credit


Bank (1,000 x 35) 35 000
Ordinary share capital 10 000
Share premium 25 000
Issue of 1 000 ordinary shares at 35 (market price)
30/9/2014
Dividend (share premium) 10 000
Ordinary share capital 10 000
Capitalisation issue: (1 000 + 1 000) / 2 x 1 x 10

30/6/2015
Bank (2,000 x 40) 80 000
Ordinary share capital (2 000 x 10) 20 000
Share premium (2 000 x 30) 60 000
Issue of 2 000 ordinary shares at 40 (market price)
31/8/2015

197
Page 14 of 40
There is no journal entry for a share split (the authorized and issued number of shares are
simply increased accordingly). However, remember that entry is made for bonus shares.

Solution to example 11B:


EPS for 2013:
EPS = 100,000 / 1,000 = 100 per share

Actual Current year Prior year


(weighted) (adjusted)
Balance: 1/1 1,000 1,000
No movement - -
Balance: 31/12 1,000 1,000

EPS for 2014:


EPS 2014 = 100,000 / 2,250 = 44.44 per share
EPS 2013 [Restated]= 100,000 / 1,500 = 66.67 per share

Actual Current year Prior year


(weighted) (adjusted)
2014 2013
Balance: 1/1 1,000 1,000 1,000
Issue for value 30 / 06 1,000 500 0
(1,000 x 6/12)
2,000 1,500 1,000
Issue for no value 30 / 09 1,000 750 500
(bonus issue) (2,000 x 1/2) (1,000 / 2,000 x 1500) (1,000 / 2,000 x
3,000 2,250 1,000)
1,500

EPS for 2015:


EPS 2015 = 100,000 / 12,000 = 8.33 per share
EPS 2014 [Restated]= 100,000 / 6,750 = 14.81 per share
EPS 2013 [Restated]= 100,000 / 4,500 = 22.22 per share

Actual Current year Prior year Prior year


(weighted) (adjusted) (adjusted)
2015 2014 2013
Balance: 1/1/2015 3,000 3,000 2,250 1,500
Issue for value 30 / 06 2,000 1,000 0 0
(2,000 x 6/12)
5,000 4,000 2,250 1,500
Issue for no value 31 / 08 10,000 8,000 4,500 3,000
(share split) (5,000 x 3/1) (10,000 / 5,000 x (10,000 / 5,000 (10,000 / 5,000
15,000 4,000)
12,000 X 2,250)
6,750 x 1,500)
4,500

198
Page 15 of 40
4.Share buy-backs
A share buy-back involves a decrease in the capital base of the entity through the entity repurchasing shares from
its shareholders. A buy-back involves a reduction of the capital base (fewer issued shares exist after the buy-back)
and a reduction in the money/ resources of the entity (the entity pays the shareholders for the shares).

Since the entity pays the shareholders for their shares, the share buy-back is a for-value reduction. The treatment
of a for-value reduction is very similar to that of a for-value issue with the exception that the number of shares
involved is subtracted rather than added.
Example 12: share buy-back
A company had 10 000 ordinary shares in issue during 2012.
There was a share buy-back:
• of 5 000 ordinary shares (at market price)
• 60 days before the end of the current year (year-end: 31 December 2013).
The basic earnings in 2012 were 20 000 and 17 000 in 2013

Required:
Calculate the earnings per share in 2013 along with comparatives.
Answer:

W1: Denominator: number of shares Actual Current year Prior period


(weighted) (Adjusted)
2013
2012
Opening balance: 1/1 10 000 10 000 10 000
Reduction for value: 1/11 (5 000) (822) 0
( 5 000 x 60
/365)
5 000 9 178 10 000

W2: earnings per share for inclusion in 20X3 financial statements


Basic earnings per share: 2013 Restated
2012

Basic earnings 17 000 20 000


Weighted average number of shares 9 178 10 000

1.85 2.00

5.Share consolidation (Also called as reverse share split)


A share consolidation is the combination of 2 or more shares into 1. This transaction requires none of the entity’s
resources and thus it is treated as a not-for-value reduction. The treatment of a not-for-value reduction is very
similar to that of a not-for-value issue with the exception that the number of shares involved is subtracted rather
than added.

199
Page 16 of 40
Example 13: share consolidation
A company had 10 000 ordinary shares in issue during 2012.
During 2013, the company consolidated its shares:
• such that every 2 shares were consolidated into 1 share.
• 60 days before the end of the current year (year-end: 31 December 2013).

The basic earnings in 2012 were 20 000 and 17 000 in 2013

Required:
Calculate the earnings per share in 2013 along with comparative.

Answer:

W1: Denominator: number of shares Actual Current year Prior period


(weighted) (Adjusted)
2013
Opening balance: 1/1 10 000 10 000 102012
000
Reduction for no value: 1/11 (5 000) (5 000) (5 000)
( 5 000 x 10 000/ ( 5 000 x 10 000/
10 000) 10 000)
5 000 5 000 5 000

W2: earnings per share for inclusion in 2013 financial statements

Basic earnings per share: 2013 2012


Basic earnings 17 000 20 000
Weighted average number of shares 5 000 5 000

3.40 4.00*
* The 2012 financial statements would have reflected earnings per share figure of 2 (20 000 / 10 000).

Extra discussion points to be considered:


Total earnings
The total earnings figure is the profit or loss from continuing operations after deducting tax and preference
dividends.

When there is a net loss, total earnings, and therefore, the EPS are negative.

Earnings from discontinued operations (means if a segment of business is closed. E.g. Dubai operations) are dealt
with separately. An EPS from any discontinued operations must also be disclosed separately.

The total earnings figure must be adjusted for the interests of preference shareholders before it can be used in EPS
calculations.

Preference shares
Preference shares must be classified as either equity or liability

200
Page 17 of 40
If a class of preference shares is classified as equity (means irredeemable preference shares), any dividend relating
to that share is recognized in equity. Any such dividend must be deducted from the profit or loss from continuing
operations.

If a class of preference shares is classified as liability (redeemable preference shares), any dividend relating to that
share is recognized as a finance cost in the statement of profit or loss. It is already deducted from the profit or loss
from continuing operations and no further adjustment need be made.

Rs. 3,000,000* Rs. 3 per share (EPS from

= =
1,000,000 continued operations)

Rs. 500,000 Rs. 0.5 per share (EPS from

= 1,000,000 = discontinued operations)

*preference dividend on preference shares classified as a liability should have already been deducted from profit
after tax.

Example: Basic EPS

In the year ended 31 December Year 1, Entity G made profit after tax of Rs. 3,500,000.

Of this, Rs. 3,000,000 was from continuing operations and Rs. 500,000 from discontinued operations.

201
Page 18 of 40
It paid ordinary dividends of Rs. 150,000 and preference dividends of Rs. 65,000.

The preference shares were correctly classified as equity.

Entity G had 1 million ordinary shares in issue throughout the year.

Entity G’s basic EPS for the year ended 31 December 2011 is calculated as follows:
EPS =

Rs. 3,000,000 - Rs. 65,000 Rs. 2.935 per share

= =
1,000,000 (continued operations)

Rs. 500,000 Rs. 0.5 per share

= 1,000,000 = (discontinued operations)

As EPS from discontinued operations must also be disclosed but this does not have to be disclosed on the face of
statement of profit or loss. Instead it may be shown in notes to financial statements.

Retrospective adjustments
If the number of ordinary shares outstanding increases as a result of a capitalisation, bonus issue or share split, or
decreases as a result of a reverse share split (means share consolidation), the calculation of basic EPS for all periods
presented shall be adjusted retrospectively.

If these changes occur after the reporting period but before the financial statements are authorised for issue, the
per share calculations for those and any prior period financial statements presented shall be based on the new
number of shares.

The fact that per share calculations reflect such changes in the number of shares shall be disclosed.

In addition, basic earnings per share of all periods presented shall be adjusted for the effects of errors and
adjustments resulting from changes in accounting policies accounted for retrospectively.

Important note: If both the announcement of shares and entitlement dates of shares are available, then use
entitlement date in the working of EPS.
Example: Home Dynamics Limited
Question: The following information pertains to the financial statements of Home Dynamics Limited (HDL), a
listed company, for the year ended 31 December 2016:
(i) Profit after tax for the year: Rs. in
million
Profit from continuing operations – net of tax 765
Profit from discontinued operations – net of tax 155
Profit after tax 920
(ii) Shareholders’ equity as on 1 January 2016 comprised of:

202
Page 19 of 40
▪ 10 million ordinary shares of Rs. 10 each, having market value of Rs. 25 each.
▪ 4 million cumulative preference shares of Rs. 10 each entitled to a cumulative dividend at 10%.
(iii) On 31 March 2016, HDL announced 40% right shares to its ordinary shareholders at Rs. 25 per share.
The entitlement date of right shares was 31 May 2016. The market price per share immediately
before the announcement date and entitlement date was Rs. 28 and Rs. 32 respectively.
(iv) On 2 August 2016, HDL announced 20% bonus issue. The entitlement date of bonus shares was 31
August 2016.
(v) On 1 February 2017, the board of directors announced 20% cash dividend and 10% bonus issue being
the final dividend to the ordinary shareholders and 10% cash dividend for preference shareholders.
Required:
Calculate basic earnings per share for inclusion in HDL’s financial statements for the year ended 31 December
2016. Show all relevant calculations.

Answer:
Basic earnings per share for the year ended 31 December 2016.
Rupees in millions
Continuing Discontinuing
operations operations
Profit attributable to ordinary shareholders 761 155
(765-4)(4 x 10 x
10%)
Weighted average number of shares in issue during the 16.65 16.65
year (W.1) [shares in millions]
Earnings per share 45.71 9.31
.
Workings: [W.1] shares in 000
Actual Current year
(weighted)

Opening balance 1.1 10,000 10,000

Issue for value 31.05 (Note 1) 3,125 1,823


(4,000,000 x 1/4) (3,125 x 7/12)
13,125 11,823

Issue for no value 31.05 [note 1] 875 788


(875/13,125 x 11,823)

14,000 12,611
Issue for no value 31.8 2,800 2,522
(14,000 x 20%) (2,800/14,000 x 12,611)
Closing balance 31.12 16,800 15,133
Bonus issue after the year No entry 1,513 (10%)
end(Final) (10%)[no value] [1,680/16,800x15,133]

203
Page 20 of 40
16,646
Note 1:
For value: 10,000 x 40% =4,000 x 25 = 100,000/32 = 3,125 shares
For no value: 4,000 – 3,125 = 875 shares

Practice Questions
1. AIRCON LTD
Mr Hamad, currently owns 20 million shares in Aircon Ltd. He recently received the published financial statements
of Aircon Ltd for the year ended 31 March 2016. Mr Hamad is not sure how the performance of the company
during the year will affect the market value of the entity’s shares but he is aware that the earnings per share
statistics are often used by analysts in assessing the performance of listed companies.

Extracts from these published financial statements and other relevant information are given below.

Statement of profit or loss for the period ended 31 March 2016

2016 2015
Rs.’m Rs.’m
Revenue 18,000 15,300
Cost of sales (11,340) (9,180)
Gross profit 6,660 6,120
Operating expenses (3,420) (3,240)
Operating profit 3,240 2,880
Interest expense (540) (576)
Profit before tax 2,700 2,304
Taxation (846) (720)
Profit after tax 1,854 1,584

Statement of financial position as at 31 March 2016


2016 2015
Rs.’m Rs.’m Rs.’m Rs.’m
Intangible assets 5,400 -
Tangible assets 7,200 6,660
──── ────
12,600 6,660
Current Assets
Inventory 2,340 1,800
Receivables 2,700 2,160
Cash in bank 180 5,220 162 4,122
──── ──── ──── ────
17,820 10,782

204
Page 21 of 40
──── ────
Capital and Reserves
Share Capital 2,700 900
Share Premium 4,860 900
Retained Earnings 1,620 1,206
──── ────
9,180 3,006

Current Liabilities
Trade Payables 3,060 2,160
Taxation 900 756
Bank Overdraft 1,080 1,260
──── ────

5,040 4,176
──── ────
14,220 7,182
15% Loan Note 3,600 3,600
──── ────
17,820 10,782
──── ────

The following information is also relevant:


(i) The share capital of the company comprises Rs. 1 equity shares only.
(ii) On 1 October 2015, the company made a rights issue to existing shareholders of two new shares for every
one share held at a price of Rs. 5.94 per share.
(iii) The market price of shares immediately before the rights issue was Rs. 6.30 per share.
(iv) No other changes took place in the equity capital of Aircon Ltd in the year ended 31 March 2016.

Required
(a) Compute EPS for the year and the comparative figures that will be included in the published financial
statements of Aircon Ltd for the year ended 31 March 2016.
(b) Using the extracts you have been provided with, write a report to Mr Hamad identifying the key factors
which led to the change in the EPS of Aircon Ltd since the year ended 31 March 2016.

Comment on the relevance of the EPS statistics to shareholders.

Q.2 Following are the balance sheets relating to United Limited (UL) as at December 31 st:
2016 2015
Rs. In Millions
Share capital (Rs.10 each) 2,772 2,000
Share premium - 188
Revaluation surplus 120 100

205
Page 22 of 40
Retained earnings 813 405
Trade payable 250 190
Tax payable 825 1,055
Property, plant, equipment 3,450 3,200
Capital work in progress 877 400
Trade receivables 323 228
stock 130 110

Additional information relating to year 2016:


i. Further expenditure incurred on capital work in progress was Rs. 847 million.
ii. UL made a right issue of 1 for 5 on April 1,2016 at premium of Rs. 42 million when cum-right market price per
share was Rs.14.
iii. Sales for the year amount to Rs. 4,300 million. All goods have been sold at a profit margin of 40% during the
year.
iv. Depreciation for the year is charged to admin expenses.
v. Tax rate is 30%.
vi. It is UL’s policy to maintain a provision for doubtful debts at 5% of debtors. Bad debts written off during the
year amount to Rs. 80 million.
vii. Profit after tax as reported in financial statements of 2015 was Rs. 420 million.
viii. Dividend paid / issued during 2016 as follows:
Cash Bonus
Final dividend (2015) 5% 5%
Interim dividend (2016) 0 10%

Required:
a) Prepare “cash flow from operating activities” section of cash flow statement for the year ending December
31,2016 using direct method (show interest and dividend paid under operating activities)
(12)

b) Prepare “statement of changes in equity” for the year ending December 31,2016 [comparatives not required]
(05)

c) Calculate basic EPS for 2016 and 2015 (restated) assuming that final dividend was paid on January 31,2016 and
interim dividend was paid on July 31,2016.
(06)

206
Page 23 of 40
1. Aircon Ltd

(a) Earnings Per Share

EPS for 2016: 1,854/1,817.5 (W 1) = 1.02 per share

EPS for 2015 [Restated]: 1,584/936 = 1.7 per share

Workings
• The number of shares issued in rights issue (900 / 1 x 2) 1,800 shares
• The cash received from the rights issue: 1,800 shares x 5.94 = 10,692

Shares sold (issue for value): proceeds/ market price [10,692 / 6.3] 1,697
Shares given away (issue for no value): total shares issued – shares sold 103
= 1,800 shares – 1,697 shares
1,800

The weighted and adjusted average number of shares may then be calculated:

Note 1: Please remember that issues for value during the year require weighting of the number of shares to take
into account how long the extra capital was available to the entity.
Note 2: Please note that an issue for no value will not cause an increase in the profits and therefore, in order to
ensure comparability, the prior year shares are adjusted as if the issue for no value had occurred in the prior year.

Number of shares (weighted & adjusted)


Actual Current year Prior year
(weighted) (adjusted)
Balance: 1/4/2015 900 900 900
Issue for value 1/10/2015 1,697 848.5 (1,697 x 6/12) 0
2,597 1,748.5 900
Issue for no value 1/10/2015 103 69 36
(103/2,597 x 900) (103/2,597 x
900)
31/03/2016 2,700 1,817.5 936

(b) Report

To: Mr Hamad

From: Accountant

Date: 15 April 2016

Subject: Evaluating the changes in EPS of Aircon Ltd

207
Page 24 of 40
Calaculation of EPS is based on data of income statement therefore the key factors which has led to changes in
the EPS of Aircon Ltd. are as follows:

Revenue has increased by Rs. 2,700 million which means 18% increase over the last year (2,700/15,300 x 100).
However, as the capital employed is increased by a higher amount of 6,174 therefore, asset turnover ratio decreased
from last year.

The gross profit ratio has declined which is an indication of increase in cost of sales.
There is an increase in operating expenses therefore operating profit ratio and net profit ratio remains approximately
constant.

Return on assets, return on capital employed and return on equity all indicate increase in total assets, capital
employed and equity but the profit has not increased proportionately.

Conclusion:
Assets are increased because of increase in intangible assets by Rs. 5,400 millions. The company has generated cash
flow from shares of Rs. 5,760 (7,560 – 1,800) which looks like invested in assets. Therefore, company can generate
increased profits in the future periods by utilizing the assets acquired during the period.

However, as these assets have yet to start generating income during the year, the EPS has therefore fallen from Rs.
1.69 in 2015 to Rs. 1.01 in 2016.

Signed

Accountant

APPENDIX TO THE REPORT

Relevant ratios

Total Asset Turnover Net Sales___


(Utilization of Total Assets) Avg. Capital Employed
(Answer is in times)
2016: 18,000 / 9,180 + 3,600 = 1.41 times
2015: 15,300 / 6,606 = 2.31 times

Gross profit Ratio Gross Profit x 100 =…...%


Net Sales
(Answer in %)
2016: 6,600 / 18,000 x 100 = 37 %
2015: 6,120 / 15,300 x 100 = 40 %

Operating Profit Ratio Operating Profit x 100


(Operating profit is Profit before interest & Tax) Net Sales
(Answer in %)

208
Page 25 of 40
2016: 3,240 / 18,000 x 100 = 18 %
2015: 2,880 / 15,300 x 100 = 19 %

Expense Ratio
Operating Expenses x 100
Net Sales
(Answer in %)

2016: 3,420 / 18,000 x 100 = 19 %


2015: 3,240 / 15,300 x 100 = 21 %

Net Profit after tax Ratio


Profit After tax x 100
Net Sales
(Answer in %)

2016: 1,854 / 18,000 x 100 = 10 %


2015: 1,584 / 15,300 x 100 = 10 %

Return on Capital Employed (ROCE) Profit before interest & tax x100
Avg Capital Employed

(Answer is in %)
2016: 3,240 / 9,180 + 3,600 x 100 = 25 %
2015: 3,880 / 6,660 x 100 = 44 %

Return on Equity PAT – *Preference Dividend x100


Equity= (Share capital + Reserve) Average Equity
(Answer is in %)
*If preference shares are non-redeemable

2016: 1,854 / 9,180 x 100 = 20.2 %


2015: 1,584 / 3,006 x 100 = 52.7 %

Average is only calculated if it can be for all given relevant periods.

Relevance of EPS to shareholders


• The EPS is used to compute the price earning (P/E) ratio, a major market indicator to determine how successful
a company has been operating. The price earning figure is a multiple of the EPS, where the multiple represents
the number of years’ earnings required to recoup the price paid for the share (pay back period).
• Rising trend in EPS is a more accurate performance indicator than rising trend in profit after tax (because of
considering only profit from continuing operations).
• EPS is a measure of performance from the existing and potential investors’ perspective.
• EPS show the amount available to each ordinary shareholder thereby indicating the potential returns on
individual investment.

209
Page 26 of 40
EPS is used to compare the activities of two entities in the same industry (however EPS has some limitations as well).

A.2 (a)
United Limited
Statement of Cash Flow
For the year ended 31-12-2016
Cash flow from operating activities: Rs. In Million
Receipts from Customers (w-1) 4,120
Payments to suppliers (w-2) (2,540)
Payment for other expenses (w-3) (595)
Cash generated from operations 985
Tax paid (w-4) (500)
Dividend paid (100)
Net cash from operating activities 385

Dividend paid will be classified in financing activities.

Workings:
w-1)
Debtor Account
b/d 240 Bad debts 80
(228 / 95%)
Cash 4,120
sales 4,300
c/d 340 [323 / 95%]

w-2)
Creditor Account
Cash 2,540 b/d 190

Purchases 2,600
c/d 250

Stock Account
b/d 110 COS 2,580
[4,300 / 100 X 60]
purchases 2,600
c/d 130

210
Page 27 of 40
W-3) Other Expenses / Payment:
Gross profit [4,300 – 2,580] 1,720
Depreciation [w-3.1] (140)
Bad debts [80+5] (85)
Other expenses (bal.) (595)
Profit before tax [630 (b) / 70%] 900

As there are no expenses payable / prepaid expenses; therefore Expenses = Payments

W-3.1)
PPE
b/d 3,200 Depreciation 140

Revaluation Surplus 20
[120 – 100]
CWIP 370 c/d 3,450
W-3.2)
Capital WIP
b/d 400 PPE (bal) 370

Cash 847
c/d 877
W-3.3)
Provision for Bad Debts
b/d 12
(240 x 5%)
Expense 5
c/d (340 x 5%) 17

W-4)
Tax Payable
Cash 500 b/d 1,055

Exp [900 x 30%] 270


c/d 825

211
Page 28 of 40
b)
United Limited
Statement of changes in Equity
For the year ended 31-12-2016
Rs in ‘Million’
Share Capital Share Premium Retained Revaluation Total
Earnings surplus
Balance as on 1-1-2016 2,000 188 405 100 2,693
Final div. cash [2,000 x 5%] - - (100) (100)
Final div. bonus [2,000 x 100 (100) -
5%]
Right issue [2,100 x 1/5] 420 42 - 462
(bal.)
Revaluation 20 20
Interim div. bonus 252 (130) (122) -
[2,772/110 x 10]
Profit after tax (bal.) - - 630 630
Balance as on 31-12-2016 2,772 - 813 120 3,705

C) Calculation of Basic EPS:


‘Rs in Million’
2016 2015 (Restated)
= 630 / 267.79 (W-1) = 420 / 239.56 (W-1)
= 2.35 = 1.75

Workings:
W-1
Actual Current (weighted) Prior (Adjusted)
1-1 200 200 200
31-1 [bonus shares] 10 10 10
[5% issued for no value] (10 / 200 x 200) (10 / 200 x 200)
210 210 210
1-4 issued for value 33 24.75 -
[w-1.1] (33 x 9/12)
243 234.75 210
1-4 [issue for no value] 9 8.69 7.78
[w-1.1] (9 / 243 x 234.75) (9 / 243 x 210)
252 243.44 217.78
31-7 [issue for no value] 25.2 24.34 21.78
Bonus shares 10% (25.2/252 x 243.44) (25.2/252 x 217.78)
31-12 277.2 267.78 239.56

W-1.1) 210 x 1/5 =42 x 11 [(420 + 42) / 42 = 11]


=462/14 = 33 [for value]
For no value [42 – 33] = 9
212
Page 29 of 40
Extra practice questions:
Example 1 share issuance with share consolidation
At 1 January 20X5 an entity had 10,000,000 ordinary shares in issue. It issued 3,000,000 new shares on 30.04.20X5
for cash at full market value. On 31.08.20X5 it went through a share reconstruction by consolidating the shares in
issue, on the basis of one new share for two old shares. The year ned of the company is 31.12.

Required: Calculate the weighted average number of shares in issue during the year.

Solution:
Actual Current year
(weighted)
Opening balance 1.1 10,000,000 10,000,000
Issue for value 30.04 3,000,000 2,000,000
(3,000,000
x 8/12)
13,000,000 12,000,000
Issue for no value 31.08 (6,500,000) (6,000,000)
(share consolidation) (6,500,000/
(13,000,000 13,000,000
x 1/2) x 12,000,000)
Opening balance 31.12 6,500,000 6,000,000

Example 2 cash and right issue


At 1 January 20X4 and 20X5 an entity had 14,000,000 ordinary shares in issue. The financial year end of the entity
is 31.12.20X5.It issued further shares as follows:

It issued 4,000,000 new shares on 01.04.20X5 for cash at full market value.

On 01.07.20X5 it made a right issue of one for six at 15 each when market price was 20 each.

A profit of 17,000,000 attributable to ordinary equity holders was reported for 20X5 and 14,000,000 for 20X4.

Required: calculate the earnings per share for 20X5 including comparative figure of 20X4.

Solution:
EPS [2005] = 17,000,000 / 18,796,296 = 0.94 per share
EPS [2004] [Restated] = 14,000,000 / 14,518,518 = 0.96 per share

Workings:
Actual Current year Prior year
(weighted) (adjusted)
Opening balance 1.1 14,000,000 14,000,000 14,000,000
Issue for value 1.4 4,000,000 3,000,000 0
(4,000,000 x 9/12)
18,000,000 17,000,000 14,000,000
Issue for value 1.7[Right shares(W)] 2,250,000 1,125,000 0
(2,250,000 x 6/12)

213
Page 30 of 40
20,250,000 18,125,000 14,000,000
Issue for no value 1.7(W) 750,000 691,296 518,518
(750/20,250 x (750/20,250 x
18,125,000) 14,000,000)

Closing balance 31.12 21,000,000 18,796,296 14,518,518

Working :
Issued for value = 18,000,000 x 1/6 = 3,000,000 x 15 = 45,000,000/20 = 2,250,000 shares
Issued for no value = 3,000,000 – 2,250,000 = 750,000 shares

Example 3 cash and right issue


At 1 January 2016 there were 13,000 ordinary shares in issue. It issued further shares as follows:
It issued 8,000 new shares on 01.04.2016 for cash at full market value.
On 01.06.2016 it made a bonus issue of one new share for every three shares held.
On 01.11.2016 it issued 8,500 new shares at full market value.

Profit of 1,000,000 attributable to ordinary equity holders was reported for 2016 and 613,000 for 2015.

Required: calculate the basic earnings per share for the year ended31.12. 2016 and 2015(including comparative
figures).

Solution:
For the year ended 2015:
EPS [2015] = 613,000 / 13,000 = 47.15 per share
For the year ended 2016:
EPS [2016] = 1,000,000 / 26,750 = 37.38 per share
EPS [2015] [Restated]= 613,000 / 17,333 = 35.37 per share

Workings:
Actual Current year Prior year
(weighted) (adjusted)

Opening balance 1.1 13,000 13,000 13,000


Issue for value 1.4 8,000 6,000 0
(8,000 x 9/12)

21,000 19,000 13,000


Issue for no value 1.6 7,000 6,333 4,333
(bonus issue) (21,000 x 1/3) (7/21 x 19,000) (7/21 x 13,000)
28,000 25,333 17,333
Issue for value 1.11 8,500 1,417 0
(8,500 x 2/12)

Opening balance 31.12 36,500 26,750 17,333

214
Page 31 of 40
Example 4 cash and right issue
At 1 January 2016 there were 6,000 ordinary shares in issue. It issued further shares as follows:
It issued 5,000 new shares on 01.03.2016 for cash at full market value.

On 01.04.2016 it made a 2 for 3 rights issue at 5 per share. The market price of shares prior to the rights issue was
15.
On 01.09.2016 it issued 12,000 new shares at full market value.
Profit of 200,000 attributable to ordinary equity holders was reported for 2016 and 70,000 for 2015.

Required: calculate the basic earnings per share for the year ended31.12. 2016 and 2015 (including comparative
figures).

Solution:
For the year ended 2015:
EPS [2015] = 70,000 / 6,000 = 11.67 per share

For the year ended 2016:


EPS [2016] = 200,000 / 20,363 = 9.82 per share
EPS [2015] [Restated] = 70,000 / 8,180 = 8.56 per share

Workings:
Actual Current year Prior year
(weighted) (adjusted)

Opening balance 1.1 6,000 6,000 6,000


Issue for value 1.3 5,000 4,167 0
(5,000 x 10/12)

11,000 10,167 6,000


Issue for value 1.4[Right issue (W)] 2,444 1,833 0
(2,444 x 9/12)
13,444 12,000 6,000
Issue for no value 1.4 (W) 4,888 4,363 2,181
(4,888/13,444 (4,888/13,444
x 12,000) x 6,000)
18,332 16,363 8,181
Issue for value 1.9 12,000 4,000 0
(12,000 x 4/12)

Opening balance 31.12 30,332 20,363 8,180

Working :
Issued for value = 11,000 x 2/3 = 7,333 x 5 = 36,666/15 = 2,444 shares
Issued for no value = 7,333 – 2,444 = 4,888 shares

215
Page 32 of 40
ICAP EXAMPLES
Example 01: Ghalib Limited
Question: In the year ended 31 December 2011, Ghalib Limited (GL) made profit after tax of Rs. 3,500,000. Of this,
Rs. 3,000,000 was from continuing operations and Rs. 500,000 from discontinued operations.

GL paid ordinary dividends of Rs. 150,000 and preference dividends of Rs. 65,000. The preference shares were
correctly classified as liabilities in accordance with IAS 32 (means redeemable preference shares).

GL had 1 million ordinary shares of Rs. 10 each in issue throughout the year.

Required:
Calculate basic EPS for the year ended 31 December 2011.

Answer:
Basic EPS Rs. 3,000,000
= = Rs. 3 per share
(continued operations) 1,000,000 shares

Basic EPS Rs. 500,000


= = Rs. 0.5 per share
(discontinued operations) 1,000,000 shares

Example 02: Jazib Limited


Question: In the year ended 31 December 2011, Jazib Limited (JL) made profit after tax of Rs.
3,500,000. Of this, Rs. 3,000,000 was from continuing operations and Rs. 500,000 from discontinued
operations.
JL paid ordinary dividends of Rs. 150,000 and preference dividends of Rs. 65,000. The preference
shares were correctly classified as equity in accordance with IAS 32 (means irredeemable preference
shares).
JL had 1 million ordinary shares of Rs. 10 each in issue throughout the year.

Required:
Calculate basic EPS for the year ended 31 December 2011.
Answer:
Basic EPS Rs. 3,000,000 – 65,000 Rs. 2.94 per
= =
(continued operations) 1,000,000 shares share

Basic EPS Rs. 500,000 Rs. 0.5 per


= =
(discontinued operations) 1,000,000 shares share
.

216
Page 33 of 40
Example 03: Aqeel Limited
Question: In the year ended 31 December 2011, Aqeel Limited (AL) made profit after tax of Rs.
3,500,000.
AL has Rs. 1,000,000 10% cumulative preference shares in issue. This would entitle the investors to
receive a dividend of Rs. 100,000 (10% of Rs. 1,000,000).
AL had 1 million ordinary shares of Rs. 10 each in issue throughout the year.

Required:
Calculate basic EPS for the year ended 31 December 2011.
Answer:
Rs. 3,500,000 – 100,000
Basic EPS = = Rs. 3.4 per share
1,000,000 shares

Note that the Rs. 100,000 deducted above would be deducted irrespective whether a dividend has
been declared or not.

Example 04: Adeel Limited


Question: In the year ended 31 December 2011, Adeel Limited (AL) made profit after tax of Rs.
3,500,000.
AL has Rs. 1,000,000 10% non-cumulative preference shares in issue. This would entitle the investors
to receive a dividend of Rs. 100,000 (10% of Rs. 1,000,000) if declared). The preference dividend was
declared during the year ended 31 December 2011.
AL had 1 million ordinary shares of Rs. 10 each in issue throughout the year.

Required:
Calculate basic EPS for the year ended 31 December 2011.
Answer:
Rs. 3,500,000 – 100,000
Basic EPS = = Rs. 3.4 per share
1,000,000 shares

Example 05: Kashif Limited


Question: In the year ended 31 December 2011, Kashif Limited (KL) made profit after tax of Rs.
3,500,000.
KL has Rs. 1,000,000 10% non-cumulative preference shares in issue. This would entitle the investors
to receive a dividend of Rs. 100,000 (10% of Rs. 1,000,000) if declared). The preference dividend was
not declared during the year ended 31 December 2011.
KL had 1 million ordinary shares of Rs. 10 each in issue throughout the year.

Required:
Calculate basic EPS for the year ended 31 December 2011.
Answer:
Rs. 3,500,000
Basic EPS = = Rs. 3.5 per share
1,000,000 shares
.

217
Page 34 of 40
Example 06: Friday Limited
Question: Friday Limited (FL) has a financial year ending 31 December.
On 1 January 20X1, there were 6,000,000 ordinary shares (Rs. 10 each) in issue. On 1 April 2011, it
issued 1,000,000 new shares at full market price.
Total earnings for the year ended 31 December 2011 were Rs. 27,000,000.

Required:
Calculate basic EPS for the year ended 31 December 2011.
Answer:
Rs. 27,000,000
Basic EPS = = Rs. 4 per share
6,750,000 shares

Number of Time Weighted


Date
shares factor average number
1 January to 31 March 6,000,000 x 3/12 1,500,000
New issue on 1 April 1,000,000
1 April to 31 December 7,000,000 x 9/12 5,250,000
6,750,000
.

Example 07: Sunday Limited


Question: Sunday Limited (SL) has a financial year ending 31 December.
On 1 January 2013, there were 9,000,000 ordinary shares in issue. On 1 May 2013, SL issued
1,200,000 new shares at full market price. On 1 October 20X3, SL issued a further 1,800,000 shares,
also at full market price.
Total earnings in 2013 were Rs.36,900,000.

Required:
Calculate basic EPS for the year ended 31 December 2013.
Answer:
Rs. 36,900,000
Basic EPS = = Rs. 3.6 per share
10,250,000 shares

Number of Time Weighted


Date
shares factor average number
1 January to 30 April 9,000,000 x 4/12 3,000,000
New issue on 1 May 1,200,000
1 May to 30 September 10,200,000 x 5/12 4,250,000
New issue on 1 October 1,800,000
1 October to 31 December 12,000,000 x 3/12 3,000,000
10,250,000
.

218
Page 35 of 40
Example 08: Pink Limited
Question: Pink Limited (PL) has a 31 December financial year end.
On 1 January 2014 it has 4,000,000 shares in issue. There were no share issues in Year 2014. On 1 July 2015 it
made a 1 for 4 bonus issue.

◼ Basic EPS reported in 2014 was: Rs. 20,000,000/4,000,000 shares = Rs. 5 per share

◼ Earning attributable to ordinary shareholders for the year 2015 are Rs. 24,000,000

Required:
Calculate basic EPS for the year ended 31 December 2015 (including comparative for 2014).
Answer:
For the year ended 2015:
EPS [2015] = 2,400,000 / 5,000,000 = 4.80 per share
EPS [2014] [Restated] = 2,400,000 / 5,000,000 = 4.00 per share

Workings:
Actual Current year Prior year
(weighted) (adjusted)

Opening balance 1.1 4,000,000 4,000,000 4,000,000

Issue for no value 1.7 1,000,000 1,000,000 1,000,000


(4,000,000 x 1/4)

Closing balance 31.12 5,000,000 5,000,000 5,000,000

Example 09: Red Limited


Question: Red Limited (RL) has a 31 December financial year-end and had 2,000,000 ordinary shares in issue
on 1 January 2012.
On 31 March 2012, it issued 500,000 ordinary shares, at full market price.
On 1 July 2012, RL made a 1 for 2 bonus issue.

In Year 2011, the EPS had been calculated as Rs. 30 per share.
In Year 2012, total earnings were Rs. 85,500,000.

Required:
Calculate the EPS for the year to 31 December 2012, and the comparative EPS figure for 2011.
Answer:
For the year ended 2012:
EPS [2012] = 85,500,000 / 3,562,500 = 24.00 per share
EPS [2011] [Restated] = 2,000,000 x 30 = 60,000,000/ 3,000,000 = 20.00 per share

219
Page 36 of 40
Workings:
Actual Current year Prior year
(weighted) (adjusted)

Opening balance 1.1 2,000,000 2,000,000 2,000,000

Issue for value 31.03 500,000 375,000 0


(4,000,000 x 1/4) (500,000 x 9/12)
2,500,000 2,375,000 2,000,000

Issue for no value 1.7 1,250,000 1,187,500 1,000,000


(2,500,000 x 1/2) (1,250,000/2,500,000 (1,250,000/2,500,000
X 2,375,000) X 2,000,000)

Closing balance 31.12 3,750,000 3,562,500 3,000,000

Example 10: Peach Limited


Question: Peach Limited (PL) has a 31 December financial year-end.
PL had 3,600,000 shares in issue on 1 January 2012.
It made a 1 for 4 rights issue on 1 June 2012, at a price of Rs. 40 per share. (After the rights issue, there will
be 1 new share for every 4 shares previously in issue). The share price just before the rights issue was Rs. 50.
Total earnings in the financial year to 31 December 2012 were Rs. 25,125,000.
The reported EPS in Year 2011 was Rs. 6.4 per share.

Required:
Calculate the EPS for the year to 31 December 2012, and the comparative EPS figure for 2011.
Answer:

Answer:
For the year ended 2012:
EPS [2012] = 25,125,000 / 4,187,500 = 6.00 per share
EPS [2011] [Restated] = 3,600,000 x 6.40 = 23,040,000/ 3,750,000 = 6.14 per share
Workings:
Actual Current year Prior year
(weighted) (adjusted)

Opening balance 1.1 3,600,000 3,600,000 3,600,000

Issue for value 1.6 720,000 420,000 0


(4,000,000 x 1/4) (720,000 x 7/12)
4,320,000 4,020,000 3,600,000

220
Page 37 of 40
Issue for no value 1.6 180,000 167,500 150,000
(180,000/4,320,000 (180,000/4,320,000
X 4,020,000) X 3,600,000)

Closing balance 31.12 4,500,000 4,187,500 3,750,000


Note 1:
For value: 3,600,000 x 1/4 = 900,000 x 40 =3,600,000/ 50 = 720,000 shares
For no value: 900,000 – 720,000 = 180,000 shares

Example 11: Grapes Limited


Question: Grapes Limited (GL) has a 31 December financial year-end.
GL had 3 million ordinary shares in issue on 1 January 2017.
On 1 April 2017, it made a 1 for 2 rights issue of 1,500,000 ordinary shares at Rs. 20 per share. The market
price of the shares prior to the rights issue was Rs. 50.
An issue of 400,000 shares at full market price was then made on 1 August 2017.
In the year to 31 December 2017, total earnings were Rs. 17,468,750.
In Year 2016, EPS had been reported as Rs. 3.5 per share

Required:
Calculate the EPS for the year to 31 December 2017, and the adjusted EPS for 2016 for comparative purposes.
Answer:
Answer:
For the year ended 2017:
EPS [2017] = 17,468,750 / 4,479,167 = 3.90 per share
EPS [2016] [Restated] = 3,000,000 x 3.50 = 10,500,000/ 3,750,000 = 2.80 per share

Workings:
Actual Current year Prior year
(weighted) (adjusted)

Opening balance 1.1 3,000,000 3,000,000 3,000,000

Issue for value 1.4 600,000 450,000 0


(4,000,000 x 1/4) (600,000 x 9/12)
3,600,000 3,450,000 3,000,000

Issue for no value 1.4 900,000 862,500 750,000


(900,000/3,600,000 (900,000/3,600,000
X 3,450,000) X 3,000,000)

4,500,000 4,312,500 3,750,000


Issue for value 1.8 400,000 166,667 0
(4,000,000 x 1/4) (400,000 x 5/12)
Closing balance 31.12 4,900,000 4,479,167 3,750,000

221
Page 38 of 40
Note 1:
For value: 1,500,000 x 20 =3,000,000/ 50 = 600,000 shares
For no value: 1,500,000 – 600,000 = 900,000 shares

Example 12: Pacific Limited


Question: Pacific Limited (PL) has a 31 December financial year end.
On 1 January 2014 it has 4,000,000 shares in issue. There were no share issues or other changes in Year 2014.
On 1 July 2015 it made a 5 for 2 share split.
Basic EPS reported in 2014 was: Rs. 20,000,000/4,000,000 shares = Rs. 5 per share
Earning attributable to ordinary shareholders for the year 2015 are Rs. 24,000,000

Required:
Calculate basic EPS for the year ended 31 December 2015 (including comparative for 2014).
Answer:
For the year ended 2015:
EPS [2015] = 24,00,000 / 10,000,000 = 2.40 per share
EPS [2014] [Restated] = 20,000,000 / 10,000,000 = 2.00 per share

Workings:
Actual Current year Prior year
(weighted) (adjusted)

Opening balance 1.1 4,000,000 4,000,000 4,000,000

Issue for no value 1.7 (bal) 6,000,000 6,000,000 6,000,000

Closing balance 31.12 10,000,000 10,000,000 10,000,000


(4,000,000/2 x 5)

Example 13: Atlantic Limited


Question: Atlantic Limited (AL) has a 31 December financial year end.
On 1 January 2014 it has 4,000,000 shares in issue. There were no share issues or other changes in Year 2014.
On 1 July 2015 it made a 2 for 5 share consolidation.
Basic EPS reported in 2014 was: Rs. 20,000,000/4,000,000 shares = Rs. 5 per share
Earning attributable to ordinary shareholders for the year 2015 are Rs. 24,000,000

Required:
Calculate basic EPS for the year ended 31 December 2015 (including comparative for 2014).
Answer:
For the year ended 2015:
EPS [2015] = 24,00,000 / 1,600,000 = 15.00 per share
EPS [2014] [Restated] = 20,000,000 / 1,600,000 = 12.50 per share

222
Page 39 of 40
Workings:
Actual Current year Prior year
(weighted) (adjusted)

Opening balance 1.1 4,000,000 4,000,000 4,000,000

Issue for no value 1.7 (bal) 2,400,000 2,400,000 2,400,000

Closing balance 31.12 1,600,000 1,600,000 1,600,000


(4,000,000/5 x 2)

223
Page 40 of 40
Diluted EPS
A company might have potential ordinary shares in issue.

Potential ordinary share

Definition
A potential ordinary share is a financial instrument that may entitle its holder to ordinary shares (at some time in
the future).

Examples of potential ordinary shares include:


• Convertible bonds (debentures)
• Convertible preference shares
• Share options or warrants

1. Convertible bonds are fixed interest debt securities (means a loan) which give the holder the right to convert the
bond into ordinary shares of the company. The conversion (e.g after 3 years every Rs. 20 is convertible into 1 ordinary
share) takes place at a pre-determined rate and on a pre-determined date. If the conversion does not take place the
bond will run its full life and be redeemed in cash on maturity.

Once converted into ordinary shares, convertible securities cannot be converted back into loan.

2. Convertible preference shares have similar characteristics as convertible bonds.

3a. Share Options


Definition:
A share option gives the holder the right, but not the obligation to purchase the shares in future.

The holder of an option sometimes pays an amount in exchange for the option (Fee for option).

If the holder of the option takes up the option, this is called ‘exercising’ the option.

For example, let’s say an employee or any other investor pays a premium of Rs. 30 for the option to buy a share in
Honda limited for Rs. 200 in 3 months’ time. Rs. 200 is called the ‘strike price’(also called as Exercise price).

If the price of shares in Company is Rs. 250 in 3 months’ time then the holder of the option will exercise his right to
buy a share at Rs. 200. He could immediately sell that share for Rs. 250 in the open market making a profit of Rs. 50
(less the original option premium of Rs. 30)(net gain is 20).

However, if in 3 months’ time the market price of shares in company is only Rs. 180. In this case the holder of the
option will not exercise his option to buy for Rs. 200 as he can buy shares in Company at that time in the open market
for Rs. 180. In this case the option ‘lapses’ (i.e. is not exercised).

When the market price of a share is such that to exercise the option would enable the option holder to make a profit
this option is called ‘in the money’. If the market price is such that to exercise the option would lose money, the option
is said to be ‘out of the money’.

224
Page 1 of 13
So in the previous example the option with a strike price of Rs. 200 is ‘in the money’ when the market price of the
shares is above Rs. 200, but ‘out the money’ when the market price of the shares is trading below Rs. 200.

Example: On 1 January 2019, Marigold Enterprises (ME) purchased an option for Rs. 10,000 allowing ME to buy 5,000
shares of Aroma Limited (AL) at a price of Rs. 140 per share, during the next two months. On 12 February 2019, ME
purchased the shares at the agreed price when the market value of AL's shares was Rs. 180 per share.

Required: Briefly explain what is the meaning of in the money and out of the money options and link the terms to the
above scenario.

Ans. When the market price of the share is such that by exercising the option, the option holder makes a profit, the
option is said to be ‘in the money’.

When the market price of the share is such that by exercising the option, the option holder suffers a loss, the option
is said to be ‘out the money’.

By exercising the option, ME made a profit of Rs. 5,000 x 40 (180–140) = 200,000, therefore, the option is said to be
‘in the money’. However net gain will be Rs. 200,000-10,000 = 190,000.

3b.A warrant (a loan and share option to purchase company’s share in future) is similar to a convertible bond in that
the warrant allows the holder to buy shares at a set price i.e at exercise price (rather than convert the bond into
shares). The share option part of a warrant can be separated from the bond and traded on its own whereas a
convertible bond cannot be separated.

Impact of potential ordinary shares:


Potential ordinary shares do not impact calculation of basic EPS but diluted EPS might differ from basic EPS when
there are potential ordinary shares in existence.

If potential shares become actual ordinary shares, the earnings figure will be shared with a larger number of
ordinary shares. This might dilute the EPS. The literal meaning of ‘dilution’ is ‘reduction in strength. IAS 33 defines
‘dilution’ as follows:
Definition of dilution:
Dilution is a reduction in earnings per share resulting from the assumption that convertible instruments are
converted, or that options or warrants are exercised.

Overview
With respect to earnings per share, dilution would occur if the same earnings have to be shared amongst
more shares than are currently in existence. Many entities at year-end have potential shares outstanding,
which if converted into shares, would dilute the earnings per share. Diluted earnings per share shows the
lowest earnings per share possible assuming that potential ordinary shares are converted into ordinary
shares. The diluted earnings per share shows users the maximum potential dilution of their earnings in the
future (i.e. the worst case scenario) assuming the potential shares currently in existence are converted into
ordinary shares. It logically follows that diluted earnings per share can never be higher than basic earnings
per share.

225
Page 2 of 13
Objective of diluted EPS
The objective of diluted EPS is consistent with that of basic earnings per share, that is, to provide a measure of the
performance of each ordinary share taking into account dilutive potential ordinary shares outstanding during the
period.

calculation of diluted EPS:


Always calculate the basic EPS before the diluted EPS and then consider adjustment if any, on basic earnings and
basic number of shares.

1.Share options
Options are granted to individuals allowing them to acquire a certain number of shares in the company at a
specified price per share (the strike price or exercise price) in the future. This is usually lower than the average
market price (fair value) of the share, which therefore encourages the option holder to buy the share.

When the option is exercised it results in both a ‘for value issue’ (relating to the cash received) and a ‘not for
value issue’ (relating to the bonus element, being the difference between what should have been received
based on the market price and what was received).

The possible conversion of options into ordinary shares will result into increase in number of shares.

The amount that would be received on exercise of the options is treated as cash received from selling shares at full
price with the remaining shares having been given away. The shares sold at full price are not considered to be dilutive
as any cash received would be invested to earn some return. It is only the free shares that are dilutive.

Example 1: options to acquire shares


2015
Profit before tax 800 000

Income tax expense (390 000)


Profit for the year 410 000

There are 200 000 ordinary Rs. 2 shares in issue.


During 2015 the company’s shares had an average market value of Rs.6.

The company’s directors hold option to purchase 100 000 shares, at a strike price of Rs. 2 each.

Required:
Calculate the basic and diluted earnings per share figures for 2015.

226
Page 3 of 13
Answer:

Basic earnings per share


2015
Basic Earnings = 410 000
Weighted number of shares outstanding 200 000

Basic earnings per share = 2.05

Diluted earnings per share (2015):

Diluted earnings 410 000


Weighted number of ordinary shares(W) 266 667

Diluted earnings per share = 1.5375

W.1
Weighted number of shares:
Basic number of shares 200,000
Not for value shares(W1.1) 66,667
Diluted number of shares 266,667

(W1.1) Diluted earnings per share:


Effective number of shares that would be sold
100 000 x 2 = 200,000/6 = 33,333 shares
100 000 – 33,333 = 66,667 shares (sold for no
value)
After that example 5

2.Potential ordinary shares other than share options (e.g. convertible bonds (debentures) or convertible
preference shares):
If such potential ordinary shares become actual ordinary shares, not only there would be change in number of shares
but earnings may also change as entity will no longer be paying interest on convertible bonds or dividend on
convertible preference shares.

For the purpose of calculating diluted EPS, an entity shall adjust profit or loss attributable to ordinary equity holders,
and the weighted average number of shares outstanding, for the effects of all dilutive potential ordinary shares.

The effect of a conversion will be:


◼ an increase in the expected earnings; and
◼ an increase in the number of shares on conversion.

227
Page 4 of 13
Example 2: convertible debentures
There are:
• 100 000 ordinary Rs. 2 shares in issue
• 200 000 Rs. 2 convertible debentures in issue (the conversion rate is: 1 ordinary share for each debenture).

Profit for the year ended 2015 was 279 000, which included finance costs on the convertible debentures of 30
000. Tax is levied at 30%

Required:
Calculate basic earnings and diluted earnings per share to be included in the statement of comprehensive
income for the year ended 31 December 2015.

Answer:
Basic earnings per share:

Basic earnings 279 000


Weighted number of ordinary shares in issue 100
= 000

Basic earnings per share 2.79


=

Convertible bonds
Interest saving resulting into increase in earnings = Rs. 30,000
Increase in tax increased earnings = Rs. 30,000 x 30% = Rs. 9,000
Incremental earnings = Rs. 30,000 – 9,000 = Rs. 21,000
Incremental shares = 200,000 shares(one share for each debenture)
Incremental EPS = Rs. 21,000 / 200,000 shares = Rs. 0.105

These are potentially dilutive since incremental EPS is less than basic EPS.
Diluted EPS Rs. 279,000 + 21,000
= = Rs. 1.00 per share
(2012) 100,000 + 200,000

If the holder of the instrument is faced with more than one conversion option, the entity (being the issuer of the
instrument) must use the most dilutive option in the diluted earnings per share calculation. For example, if the
holder of a debenture has the option to convert the debenture into an ordinary share or to redeem it for cash, the
entity must assume that the holder will choose the ordinary shares since this will increase the number of shares and
therefore decrease dilutive earnings per share.

Number of shares
The weighted average number of shares is increased, by adding the maximum number of new shares that would be
created if all the potential ordinary shares were converted into actual ordinary shares.

IAS 33 provides for use of most dilutive option when multiple conversion options are available.

228
Page 5 of 13
Example 3: Gold Limited
Question: Gold Limited (GL) has 12,000,000 ordinary shares and Rs. 4,000,000 5% convertible bonds in issue as at
31 December 2012, there have been no new issues of shares or bonds for several years.

The bonds are convertible into ordinary shares in 2013 or 2014, at the following rates:
▪ At 30 shares for every Rs. 100 of bonds if converted at 31 December 2013
▪ At 25 shares for every Rs. 100 of bonds if converted at 31 December 2014

Total earnings for the year to 31 December 2012 were Rs. 36,000,000. Tax is payable at a rate of 30% on profits.

Required:
Calculate basic EPS and diluted EPS for the year ended 31 December 2012.

Answer:
Basic EPS Rs.36,000,000
= = Rs. 3 per share
(2012) 12,000,000 shares

Convertible bonds
Interest saving resulting into increase in earnings = Rs. 4,000,000 x 5% = Rs. 200,000
Increase in tax on increased earnings = Rs. 200,000 x 30% = Rs. 60,000
Incremental earnings = Rs. 200,000 – 60,000 = Rs. 140,000
Incremental shares (maximum) = Rs. 4,000,000 / 100 x 30 shares = 1,200,000 shares
Incremental EPS = Rs. 140,000 / 1,200,000 shares = Rs. 0.1167

These are potentially dilutive since incremental EPS is less than basic EPS.

Diluted EPS Rs. 36,000,000 + 140,000


= = Rs. 2.74 per share
(2012) 12,000,000 + 1,200,000 shares

The additional number of shares is calculated on the assumption that they were in issue from the beginning of the
year or from the date of issue whichever is later.

If new convertibles are issued during the course of the year, the additional number of shares and the earnings
adjustment are included only from the time that the convertibles were issued.

Example 4: Silver Limited


Question: Silver Limited (SL) has 10,000,000 ordinary shares in issue on 1 January 2015. There has been no new issue
of shares for several years. However, the company issued Rs. 2,000,000 of convertible 6% convertible redeemable
preference shares on 1 April 2015.

These are convertible into ordinary shares at the following rates:


▪ On 31 March 2018: 25 shares for every Rs. 100 of convertible redeemable preference shares
▪ On 31 March 2019: 20 shares for every Rs. 100 of convertible redeemable preference shares

In the financial year to 31 December 2015 total earnings were Rs. 40,870,000. Tax is at the rate of 30%.

229
Page 6 of 13
Required:
Calculate basic EPS and diluted EPS for the year ended 31 December 2015.

Answer:
Basic EPS Rs. 40,870,000
= = Rs. 4.09 per share
(2015) 10,000,000 shares

Convertible Preference shares


Preference dividend saving results into increased earnings = Rs. 2,000,000 x 6% x 9/12 = Rs. 90,000
Increase in tax on increase in earning 90,000 x 30% = 27,000
Incremental earnings = 90,000 – 27,000 = 63,000
Incremental shares (maximum) = Rs. 2,000,000 / 100 x 25 shares x 9/12 = 375,000 shares
Incremental EPS = Rs. 63,000 / 375,000 shares = Rs. 0.168

These are potentially dilutive since incremental EPS is less than basic EPS.
Diluted EPS Rs. 40,870,000 + 63,000
= = Rs. 3.95 per share
(2015) 10,000,000 + 375,000 shares

Example 5: Bronze Limited(share options)


Question: Bronze Limited (BL) had total earnings during Year 2013 of Rs. 25,000,000. It has 5,000,000 ordinary shares
in issue.

There are outstanding share options on 400,000 shares, which can be exercised at a future date, at an exercise price
of Rs. 25 per share.

The average market price of shares in BL during Year 2013 was Rs. 40.

Required:
Calculate basic EPS and diluted EPS for the year ended 31 December 2013.

Answer:
Basic EPS Rs. 25,000,000
= = Rs. 5 per share
(2013) 5,000,000 shares

Share options
There is no effect of conversion of shares into ordinary shares on earnings. Only consider issue for no value in
number of shares.

These are dilutive since exercise price of Rs. 25 is less than share market price of Rs. 40 i.e. in the money options.
For value: 400,000 options x 25 = 10,000,000 / 40 = 250,000 shares
For no value: 400,000 – 250,000 = 150,000 shares
Diluted EPS Rs. 25,000,000 + 0
= = Rs. 4.85 per share
(2013) 5,000,000 + 150,000 shares

Anti dilution:
Not all potential ordinary shares are dilutive, they may be anti-dilutive.

230
Page 7 of 13
Definition
Anti-dilution is an increase in earnings per share or a reduction in loss per share resulting from the assumption that
convertible instruments are converted, that options or warrants are exercised, or that ordinary shares are issued
upon the satisfaction of specified conditions.

When potential ordinary shares are anti-dilutive


When potential ordinary shares are anti-dilutive, they are disregarded in the calculation of diluted EPS. The following
summary is helpful:

Options and warrants


Anti-dilutive, when Options are ‘out of money’. [it means they will not be expected to convert into ordinary shares]
Options are ‘out of money’ when exercise price of the option exceeds market price. Nobody would pay an exercise
price of Rs. 100 for something worth only say Rs. 80.

Convertible bonds/debentures/redeemable preference shares


Incremental EPS = Interest or dividend for current period (net of tax) / ordinary shares on conversion
Anti-dilutive, when:
Incremental EPS > (greater) basic earnings per share

Convertible irredeemable preference shares


Incremental EPS = Dividend for current period / ordinary shares on conversion
Anti-dilutive, when:
Incremental EPS > basic earnings per share

Multiple dilutive instruments


Many companies have more than one type of dilutive instruments in issue. Some of these instruments will be more
dilutive than others. If you recall, the objective of dilutive earnings per share is to show the most dilutive option
or ‘worst case scenario’. In order to achieve this all instruments must be ranked (most dilutive to least dilutive) and
the correct combination that lowers dilutive earnings per share the most must be chosen. The instrument that has
the lowest incremental earnings per share is the most dilutive and is ranked first.

In the case, the diluted EPS is calculated in two steps:


1. Rank in order of Dilution.
2. Testing whether or not dilutive and select the most dilutive combination.

In the money options, which have no effect on earnings but do have an effect on the number of shares, will
thus have a zero incremental earnings per share and will always be the most dilutive instrument. (and therefore
always ranked first)

Example 6: Olympics Limited


Question: The following information relates to Olympics Limited (OL) for the year ended 31 December 2015.
Number of ordinary shares in issue 2,000,000
Profit after rax Rs. 6,000,000
Average market price of shares during the year Rs. 80

231
Page 8 of 13
Potential ordinary shares:
Options 600 ,000 options, with an exercise price of Rs. 60
1,000,000 7% convertible Each preference share is convertible in 2018 into ordinary shares at
preference shares of Rs. 10 the rate of 3 ordinary share for every 10 preference shares
each
4% convertible bonds of Each bond is convertible in 2019 into ordinary shares at the rate of
Rs. 5,000,000 20 new shares for every Rs. 100 of bonds.

Tax rate is 30%.


Required:
Calculate basic EPS and diluted EPS for the year ended 31 December 2015.

Note: if there is no information then assume that preference shares are irredeemable.(as in ratios)

Answer:
Basic EPS Rs. 6,000,000 -700,000
= = Rs. 2.65 per share
(2015) 2,000,000 shares
Diluted basic earnings per
share(W.1) 1.94

W.1) Step 1:
Ranking in order of
dilution Ranking:
Increase in earnings/Increase in 5,000,000x4%x70%/ 0.14
Convertible bonds shares 5,000,000/100x20 dilutive 2
Convertible Increase in earnings/Increase in 1,000,000x10x7%*/ 2.33
preference shares shares 1,000,000/10x3 dilutive 3
Options Note 1 0.00*
0/150 000 dilutive 1
*This will always be zero.

*irredeemable preference dividend is not tax deductible.

Note 1:
For value: 600,000 options x 60 = 36,000,000 / 80 = 450,000 shares
For no value: 600,000 – 450,000 = 150,000 shares

Step 2: Testing whether dilutive or


not
6 000 000-
Basic earnings 700, 000 2.65
Basic number of shares 2 000 000

232
Page 9 of 13
Adjust for:

1. Options 5 300 000 + 0 5 300 000 2.46


2 000 000 + 150 000 2 150 000 Dilutive

2. Options &
convertible bonds 5 300 000 + 0 + 140 000 5 440 000 1.72
2 150 000 + 1 000 000 3 150 000 Dilutive

3. Options,
convertible bonds & 5 440 000 + 700 000 6 140 000 1.77
convertible preference shares 3 150 000 + 300 000 3 450 000 Anti-
dilutive
On the basis of above working our diluted EPS is 1.94 per share.
Example 7: multiple dilutive instruments
The following information relates to ABC Limited for the year ended 31 December 2015:
Basic earnings: 1,000,000
Basic number of shares: 995,500

The following potential shares are applicable on 31 December 2015:


A. Convertible debentures (convertible into 20 000 ABC Ltd’s ordinary shares on 31 December 2019). If the
debentures are not converted into ordinary shares they will be redeemed on 31 December 2019. Finance
costs of 20 000 were expensed in arriving at the profit for 2015;
B. Convertible redeemable preference shares (convertible into 40 000 ABC Limited’s ordinary shares on 31
December 2019). If the shares are not converted into ordinary shares they will be redeemed on 31
December 2019. 100 000 finance cost were expensed in arriving at the profit for 2015; and
C. Options to acquire 100 000 ordinary shares in ABC Ltd on or after 31 December 2016 at a strike price of
7.50 per share. During 2015 the average market price of the shares was 10 per share.
Tax rate is 50%.

Required:
Disclose the earnings per share figures for inclusion in ABC Limited’s statement of comprehensive income for
the year ended 31 December 2015.

Answer

ABC Limited
Statement of comprehensive income (extracts) 2015
For the year ended 31 December 2015
Basic earnings per share 1 000 000 ÷ 995 500 1.0045

Diluted basic earnings per share


(W.1) 1 010 000 ÷ 1 040500 0.9707

233
Page 10 of 13
W.1) Step 1
Ranking in order of
dilution Ranking:
Convertible Increase in earnings/Increase in 20 000x50%/ 0.50
debentures shares 20 000 Dilutive 2
1.25
Convertible Increase in earnings/Increase in 100 000x50%/ Anti
preference shares shares 40 000 Dilutive 3
Options Note 1 0.00*
0/25 000 Dilutive 1
*This will always be zero.

Note 1:
For value: 100,000 options x 7.5 = 750,000 / 10 = 75,000 shares
For no value: 100,000 – 75,000 = 25,000 shares
Step 2 Testing whether dilutive or
not
Basic earnings 1 000 000 1.0045
Basic number of shares 995 500
Adjust for:
1. Options 1 000 000 + 0 1 000 000 0.9799
995 500 + 25 000 1 020 500 Dilutive

2. Options &
convertible debentures 1 000 000 + 0 + 10 000 1 010 000 0.9707
995 500 + 25 000 + 20 000 1 040 500 Dilutive

3. Options,
convertible debentures & 1 010 000 above + 50 000 1 060 000 0.9810
convertible preference shares 1 040 500 + 40 000 1 080 500 Anti-
dilutive

On the basis of above working our diluted EPS is 0.9707 per share.
Presentation requirements
An entity should present in the statement of profit or loss:
• the basic EPS and
• the diluted EPS
for the profit or loss from continuing operations.

The basic EPS and diluted EPS should be presented with equal prominence for all the periods presented (the current
year and the previous year). These figures are presented at the end of the statement of profit or loss.

234
Page 11 of 13
If there is a discontinued operation, the basic EPS and diluted EPS from discontinued operation should be shown
either on the face of the statement of profit or loss or in a note to the financial statements.

The basic and the diluted EPS should be presented, even if it is a negative figure (means even if it is a loss per
share).

If diluted earnings per share is reported for at least one period, it shall be reported for all periods presented, even
if it equals basic earnings per share. If basic and diluted earnings per share are equal, dual presentation can be
accomplished in one line in the statement of comprehensive income [means by writing basic and diluted EPS
combined].

Disclosure requirements
IAS 33 also requires disclosure in a note to the financial statements of the following:
• The total amounts used as the numerators (total earnings figures) to calculate the basic EPS and diluted EPS,
and a reconciliation of these numerator figures to the profit or loss for the period
• The total amounts used in the denominators (weighted average number of shares) to calculate the basic EPS
and diluted EPS, and a reconciliation of these two denominator figures to each other.

Additional measure of EPS


An entity may disclose, in addition to basic and diluted earnings per share, amounts per share using a reported
component of profit other than one required by IAS 33, for example, EPS based on operating profit or profit from
major segment of the business. In such case, the following requirements apply:
• such amounts shall be calculated using the weighted average number of ordinary shares determined in
accordance with IAS 33.
• Basic and diluted amounts per share relating to such a component shall be disclosed with equal prominence
and presented in the notes (not on face of profit or loss).
• An entity shall indicate the basis on which the numerator(s) is (are) determined, including whether amounts per
share are before tax or after tax.
• If a component of profit is used that is not reported as a line item in the statement of profit or loss, a
reconciliation shall be provided.

USEFULNESS OF EPS AS FINANCIAL INFORMATION


EPS is an important component of determining an entity’s P/E ratio i.e. calculated as market value per share divided
by earnings per share. P/E ratio is often used by investor in making stock market decisions. The reliable EPS means
reliable P/E ratio leading to better and informed decisions by investors.

Investors and their advisers pay close attention to an entity’s net profit for the period. However, profit for the period
can include large and unusual items and also the results of discontinued operations. This may make it volatile i.e.
liable to fluctuate rapidly up and down. Users can then find it difficult to assess trends in the profit figure or to use
the current year’s profit to predict an entity’s performance in future years.

Reasons as to why EPS is more reliable indicator of future performance or relevance of EPS to shareholders:
The trend (improvement or deterioration) in an entity’s published EPS figure can sometimes be a more reliable
indicator of future performance. There are a number of reasons for this.
• The standard version of both basic and diluted EPS is based on profit from continuing operations. This means
that the results of discontinued operations (which may distort total profit) are excluded.
235
Page 12 of 13
• An entity may also choose to present one or more alternative versions of EPS. These normally exclude large or
unusual items so that EPS is based on ‘normal’ recurring earnings (e.g. gain or loss on disposal of non-current
assets).
• EPS measures an entity’s performance from the viewpoint of investors. It shows the amount of earnings
available to each ordinary shareholder (mean every shareholder can calculate how much he has earned on his
investment).
• Diluted EPS can provide an ‘early warning’ of any changes to an investor’s potential return on their investment
due to future share issues.

Limitations of earnings per share


EPS is probably the single most important indicator of an entity’s performance. It is a very useful measure when it is
used as the starting point for a more detailed analysis of an entity’s performance.
However, EPS can have serious limitations:

Not all entities use the same accounting policies*. It may not always be possible to make meaningful
comparisons between the EPS of different entities.

EPS does not take account of inflation, so that growth in EPS over time might be misleading (means e.g
suppose 2016 profit figure is inflated as compared to 2015;so an increase in EPS in 2016 might only have
covered inflation impact)

EPS measures an entity’s profitability, but this is only part of an entity’s overall performance. An entity’s
cash flow can be just as important as its profit (and more essential to its immediate survival).

Diluted EPS is often described as an ‘early warning’ to investors that the return on their investment may
fall sometime in the future. However, diluted EPS is based on current earnings, not forecast earnings. This
means that it may not be a reliable predictor of future EPS.

One of the main problems with EPS can be the way that it is used by investors and others. Users often rely on EPS as
the main or only measure of an entity’s performance. Management know this and try to make EPS appear as high as
possible. They may attempt to manipulate the figure by using ‘creative accounting’ (e.g. fake recording of invoices).
They may also make decisions which increase EPS in the short term but which damage the entity in the longer term.

*Accounting policies includes inventory valuation policy, property, plant and equipment valuation policy

236
Page 13 of 13
Extra Practice Questions
Question 1
You have recently been appointed the accountant of CP Products Limited. Your first assignment was to draw up the
financial statements of the company for the year ended 30 September 2014. This you have done, including earnings
and dividends per share.

The Managing Director, instead of praising you for your technical expertise as you expected, wants to know why you
changed last year’s number of shares when calculating the earnings per share for the comparative statement of
comprehensive income. He points out accusingly that last year’s statement of financial position reflects only 100,000
ordinary shares and that the 200,000 shares that you have reflected have only been in issue since half-way through
the current year. Furthermore, he wants to know why you included the profit of 80,000 made on the sale of
investments during the year. He believes that this should be excluded.

Required:
a) Explain to the managing director all the circumstances under which the previous year's comparative figures for
earnings per share should be restated. Give reasons why the restatement is necessary in each case.
b) Explain why the profit on the sale of investments was included in the amount of earnings used for the earnings
per share calculation.
c) Explain why the earnings per share figure is a better indicator of performance than
• dividends per share
• profit after tax

Question 2
A Limited was incorporated in 2010 with 150,000 9% cumulative preference shares of Rs. 1 each and 150 000 12%
non-cumulative preference shares of Rs.1 each and 300,000 ordinary shares of Rs. 1 each. The preference shares are
non-redeemable. The only changes to this structure were a new issue of 50,000 ordinary shares at a fair value of
Rs.1.50 on 1 January 2012 and a capitalization issue of 1 share for every 5 held on 15 June 2013.

The abridged statement of comprehensive income and statement of changes in equity of the company for the years
ended 30 September 2012 and 2013 are as follows:

A LIMITED
STATEMENT OF CHANGES IN EQUITY
FOR THE YEAR ENDED 30 SEPTEMBER
Retained earnings
2013 2012

Opening balance 940,000 950,000


Profit for the period 475,000 (10,000)
Capitalization issue of ordinary shares (70,000) 0

Dividends paid - 30 Sept 2013 (150,000) 0


Ordinary shares 105,000 0
9% preference shares (150,000 x 9% x 2) 27,000 0
12% preference shares 18,000 0
Closing balance 1,195,000 940,000

237
Page 1 of 11
Required:
Calculate the earnings per share and dividends per share and show how it would be disclosed in the financial
statements of A Limited for the year ended 30 September 2013 in accordance with International Financial Reporting
Standards.

Comparatives and notes to EPS required.


Question 3

The following information relates to Mitch Limited for the year ended 31 December 2011:
MITCH LIMITED “
EXTRACTS FROM TRIAL BALANCE AT 31 DECEMBER 2011
2011 2010
Profit for the period 100,000 80,000
Preference dividend declared - 31/12 5,000 5,000
Ordinary dividend declared - 31/12 10,000 6,000
Share capital details are as follows:
• Ordinary share capital balance at 1/1/2010: 200,000 (0.20 par value)
• 10% non-cumulative non-redeemable preference shares balance at 1/1/2010: 50,000 (1 par value)
• 500,000 ordinary shares were issued on 31/3/2011 at the market value of 0.20 per share.

Required:
Prepare extracts from the statement of comprehensive income and statement of changes in equity of Mitch Limited
for the year ended 31 December 2011 in terms of International Financial Reporting Standards.

Comparatives are required along with notes relating to EPS are required.

Question 4
The following information relates to Miles Limited for the year ended 31 December 2011:
MILES LIMITED
DRAFT RESULTS OF OPERATIONS
2011 2010
Profit before tax 503,000 403,000
Income tax expense (200,000) (180,000)
Profit after tax 303,000 223,000
Preference dividends declared (3,000) (3,000)
Ordinary dividend declared (30,000) (30,000)
Retained earnings for the year 270,000 190,000

238
Page 2 of 11
Additional information:

The balances in equity at 1 January' 2010 comprised:


• 1,000,000 ordinary shares of 0.20 each;
• 10,000 10% non-cumulative non-redeemable preference shares of 3 each;
• Share premium of 290,000; and
• Retained earnings of 60,000.

In terms of an agreement with the bank the company has undertaken to have a capitalisation issue in order to
capitalise excess reserves. In accordance with this agreement, there was a capitalisation issue of 1 for every 2 shares
held on 1 July 2011.

Required:
Prepare extracts from the statement of comprehensive income, statement of changes in equity and related notes
of Miles Limited in terms of International Financial Reporting Standards for the year ended 31 December 2011.
Comparatives are required

Question 5
LOYAL LIMITED
EXTRACTS FROM TRIAL BALANCE
AT 31 DECEMBER

2011 2010
250,000 280,000
Profit for the period
3,000 3,000
Non-cumulative preference dividend paid 31/12/2011
10,000 12,000
Ordinary dividend declared 31/12/2011

Additional information:
• The balances in equity at 1 January 2010 comprised:
• 100,000 ordinary shares of 1 each,
• 20,000 15% non-cumulative non-redeemable preference shares of 1 each.
• Retained earnings of l20,000.
• There was a share split on 1/7/2011 in which every 1 ordinary share became 2 shares.

Required:
Prepare extracts from the statement of comprehensive income and statement of changes in equity of Loyal Limited
in terms of International Financial Reporting Standards for the year ended 31 December 2011.
Comparatives are required along with earnings per share note.

239
Page 3 of 11
Question 6
ROGER LIMITED
DRAFT RESULTS OF OPERATIONS
FOR THE YEAR ENDED 31. DECEMBER 2018
2018 2017

Profit before tax 750,000 730,000


Income tax expense (400,000) (300,000)
Profit for the period 350,000 430,000
Ordinary dividend declared (40,000) (30,000)
Preference dividend declared (32,000) (32,000)
Retained earnings for the year 278,000 368,000
Retained earnings - beginning of the year 568,000 200,000
Retained earnings - end year 846,000 568,000

Additional information:

• The company's share capital at 31 December 2018 was as follows:


• 500,000 ordinary share capital at par value of 0.50 each.
• 200 000 8% non-cumulative non-redeemable preference shares of 2 each.
• On 30 September 2018, the company announced a rights issue of 1 ordinary share for
every 3 shares held at a price of 2.20. The market price at this date was 2.50. All the shareholders took up the
offer on this date. Prior to this date all shares issued were issued at par value.
• The normal tax rate for both years was 40%.

Required:
Prepare extracts from the statement of comprehensive income and statement of changes in equity of Roger
Limited in terms of International Financial Reporting Standards for the year ended 31 December 2018.

Comparatives and the earnings per shore note are required.

Question 7i
The following are the details of the share movements of a company called Anne Limited:
» There were 10,000 shares in issue throughout 2010.
• 10,000 shares were issued on 30 June 2011 at full market value.
• 10,000 shares were issued on 30 September 2012 at full market value.
• There was a rights issue on 30 June 2013, offering 2 shares for every 3 shares held on this date at a strike
(issue) price of 10 each when the market price was 15/ share, All shares were taken up;
• Shares were split on 30 September 2013 (splitting 2 shares into 3 shares).
• There were 100,000 Rs.10, 20%, non-redeemable, non-cumulative preference shares (i.e. treated as equity) in
issue throughout the three years. The preference dividends were declared in each of the three years.

240
Page 4 of 11
Details of Anne Limited’s profits are as follows:
2013 2012 2011
Profit for the year 700,000 900,000 800,000
The normal tax rate for both years was 40%.

Required:
Calculate and disclose earnings per share in the financial statements for the year ended:
a) 31 December 2013 (with comparatives)
b) 31 December 2012 (with comparatives)
c) 31 December 2011 (no comparatives are required for part (c)).

Question 8
T Limited was incorporated on 1 January 2012.
An extract of its previous year’s statement of financial position follows:
EXTRACTS OF THE STATEMENT OF FINANCIAL POSITION AS AT 31 DECEMBER 2013
2013 2012
EQUITY
Ordinary share capital: Rs.1 per shares 270,000 100,000
Preference share capital: Rs.2 per shares 100,000 100,000
Share premium 30,000 10,000
Retained earnings 250,000 100,000

Information regarding its share capital:


Authorized share capital:
• 400,000 ordinary shares of Rs.1 each and
• 300,000 10% non-redeemable, non-cumulative preference shares of 2 each.

Issued Preference share capital:


• 50,000 preference shares were issued on 1 January 2012.
• Preference dividends are always declared and paid on 30 December of each year.

Issued Ordinary share capital:


• 100,000 ordinary shares were issued on 1 January 2012 at market value.
• 170,000 ordinary shares were issued on 30 June 2013 at market value.
10,000 ordinary shares were offered to existing shareholders at Rs.6 each on 31 May 2014, when
the market price was Rs.9 each. All 10,000 shares offered were taken up on this day.
• On 30 November 2014, there was a capitalization issue of 2 ordinary shares for every 5 shares in
issue. The capitalization issue utilized the share premium as far as is possible.

Other information:
• Share issue expenses were 15,000 during 2014.
• The profit for 2014 was 180,000, 2013 was 150,000 and 2012 was 100,000.
• There are no other share issues or reserves other than those mentioned above.

241
Page 5 of 11
Required:
a) Disclose earnings per share in the statement of comprehensive income and notes to the
financial statements of Thomas Limited for the year ended 31 December 2014, showing both
2013 and 2012 as comparatives.
b) Calculate the basic earnings per share as it would have been disclosed in the financial
statements for the year ended 31 December 2013.
c) Show all journal entries relating to the transactions mentioned above for the year ended 31
December 2014.

Question 9 [Basic EPS with errors]


Hubbard Limited’s bookkeeper drew up the following draft statement of comprehensive income for the year:
HUBBARD LIMITED
STATEMENT OF COMPREHENSIVE INCOME
FOR THE YEAR ENDED 30 JUNE 2016

2016 2015
Sales 500,000 400,000
Cost of sales (250,000) (200,000)
Gross profit 250,000 200,000
Other expenses (110,000) (103,000)
Profit on sale of plant 7,000 0
Interest received 3,000 3,000
Profit before tax 150,000 100,000
Income tax expense - current (40,000) (35,000)
Profit for the period 110,000 65,000

The following are extracts from the draft statement of changes in equity for the year ended: I

HUBBARD LIMITED
STATEMENT OF CHANGES IN EQUITY
FOR THE YEAR ENDED 30 JUNE 2016
Retained earnings
2016 2015

Opening balance 81,000 25,000


Profit for the period 110,000 65,000
Transfer to reserves (7,000) 0
Ordinary dividends (10,000) (5,000)
Preference dividends (2,000) (4,000)
Closing balance 172,000 81,000

Additional information
• Before the 2016 financial statements were published, it was discovered that tax of 2015 was overstated
by 3,500.
• The company had operated with an ordinary share capital of 100,000 (200 000 shares of Rs.0.5 each) for
a number of years. On 31 March 2015, 50,000 new shares were issued at a premium of Rs.10 each. On
1 January 2016, the directors decided to split the share capital into shares of Rs.0.25 in order to improve
242
Page 6 of 11
the shares’ marketability.
• The dividends paid to the ordinary shareholders were declared as follows:
2016 2015
31 December 4,000 -
30 June 6,000 5,000
Total 10,000 5,000
• The preference dividends in 2015 include Rs. 2,000 dividends owing in respect of 2014. A dividend was
not declared in 2014 as a loss was incurred in that year. (Therefore it means cumulative preference
shares)

Required:
In so far as the information is available, prepare the statement of comprehensive income and column of
retained earnings in statement of changes in equity of Hubbard Limited for the year ended 30 June 2016 in
terms of International Financial Reporting Standards. Comparatives are required.

Question 10
Trini Limited operates in the retail sector and is listed on the PSE. The following extract of information is available
for its financial year ended 31 December 2018.
TRINI LIMITED
STATEMENT OF FINANCIAL POSITION
FOR THE YEAR ENDED 31 DECEMBER 2017
2017
Equity and Liabilities
Issued ordinary shares of Rs.2 par value each 1,000,000
Share premium 200,000
The correctly calculated net profit after tax amounted to Rs.3,220,000 for 2018 (2017: Rs 2,125,000)'

Additional information
On 30 April 2018, Trini Limited issued 125,000 shares at their market value of Rs.5 per share. Another issue of 30,000
shares took place on 30 November 2018 at their market value of Rs.7 per share. A further issue of 30,000 shares
took place on 20 January 2019 at their market value of Rs.7 per share.
Trini limited had a rights issue on 30 May 2018, the terms of which were as follows:
• One share was offered at an exercise price of Rs.3 for every 4 shares held on 30 May 2018, the market price
immediately before the issue was Rs.5 per share. All shares offered were taken up.
On 31 October 2018, Trini Limited consolidated its shares such that every 5 shares were consolidated into 2 shares.
An ordinary dividend of Rs.275,000 on 30 December 2018. On 29 December 2017 the ordinary dividend declared
was Rs.200,000.

Required:
Disclose the earnings per share in the Statement of comprehensive income and in the related note to the financial
statements of Trini Limited for the year ended 31 December 2018 in accordance with IFRS. (Comparatives are
required)

Question 11.
Sprog Limited had a profit for the year ended 2015 of Rs.20,000,000. Details regarding the company’s share capital
and potential share capital at 31 December 2015 are overleaf.

243
Page 7 of 11
• There are 1,000,000,000 authorised ordinary shares (with a par value of Rs.4.50), of which 10% are in issue.
• There are 500,000 convertible debentures in issue. These debentures may be converted into ordinary shares in
a ratio of 100 ordinary shares for every 1 debenture held, (at the option of the debenture holder), on the 31
December 2018. Any debentures not converted at this date will be redeemed. Finance charges on these
debentures of Rs. 1,505,000 were incurred during 2015.
• There were no movements in share capital during 2015.
• No dividends were declared in 2015.
• Ignore Tax.

Required:
Disclose earnings per share in Sprog Limited’s statement of comprehensive income for the year ended 31 December
2015.
Question 12
Details of Laser Limited’s profits (or losses) for 2015 and 2014 are as follows:
• Profit for the year: Rs.125,000 (2014: loss of 50,000).
Details of Laser Limited’s share capital and potential share capital include the following:
• At 1 January 2014 there were 100,000 ordinary shares with a par value of Rs.1.75 in issue.
• On 30 November 2014, 12,000 ordinary shares were issued at a premium of Rs.0.25 per share. There have been
no other issues since 30 November 2014.
• On 31-12-2014 there are 25,000 options in issue entitling the option holder to 1 ordinary share for each option
at a strike price of Rs.2.00 per share, (the average market price of an ordinary share for 2015: Rs.2.75).
Other information includes:
• An interim ordinary dividend of Rs.0.04 per share was declared and paid on the 30 June 2015. On 15 December
2015 a final ordinary dividend of Rs.2,800 was declared.
• No dividends were declared in 2014 due to the loss made in 2014.
• Normal company tax is levied at 35%.

Required:
Disclose earnings per share for the year ended 31 December 2015 in the Laser Limited’s statement of comprehensive
income along with comparative.

Question 13
Rebel Limited had the following draft statement of comprehensive income:
STATEMENT OF COMPREHENSIVE INCOME
FOR YEAR ENDED 31 DECEMBER 2015 (DRAFT)
2015 2014

Basic earnings per share 0.20 1.75


Basic diluted earnings per share 0.20 1.75
Dividends per share 0.05 0.00

The financial accountant of Rebel Limited resigned shortly before year end, leaving the bookkeeper to draw up the
draft annual financial statements. Numerous errors have been made by the bookkeeper. The errors are as follows:
• In both years, the irredeemable preference shareholders receive a fixed dividend of Rs.5,000 a year. This has
been ignored in the calculation of earnings per share.
• During further investigations, the bookkeeper revealed that no additional work had been done for dilutive

244
Page 8 of 11
earnings per share as the bookkeeper was under the assumption that the calculations were highly complex and
that in their small business the figures would be the same as basic earnings per share.

In your investigations you came across the following working paper:

Actual 2015 Weighted 2015 Adjusted 2014


1 January 20,000 20,000 20,000
Rights issue: ‘for value’ portion 60,000 60,000 0
Sub-total 80,000 80,000 20,000
Rights issue: ‘not for value’ portion 20,000 20,000 0
31 December 100,000 100,000 20,000

At 1 January 2014 there were 20 000 ordinary shares in issue. The rights issue (which took place on the 30 June 2015)
was on a ‘4 for 1’ basis, at a strike price of Rs. 1.50 per share. The market value per share immediately before the
rights issue was Rs.2.00.

The following potential ordinary shares are in issue:


Convertible debentures (convertible at the option of the debenture holders). These are convertible into 5,000
ordinary shares on 31 December 2017. If not converted, the debentures will be redeemed on 31 December 2017.
Finance costs of Rs.100 are incurred annually on these debentures. The convertible debentures were issued half way
through 2014.

No dividends were declared in 2014, but in the following financial year (early 2015, before the financial statements
were authorised and issued) a dividend of Rs.0.03 per share was declared. Other dividend declarations in 2015
included an interim dividend of Rs.0.05 per share (declared in June 2015) and a final dividend of Rs.0.02 per share
(declared in December 2015).

Ignore Tax

Required:
Recalculate the correct earnings per share figures and disclose them in the statement of comprehensive income of
Rebel Limited for the year ended 31 December 2015 (with comparatives)

Question 14
The following relates to Early Morning Limited for the year ended 31 December 2015:
1. Profit for the year Rs.500,000 (2014: Rs.337,500). This profit includes a profit from a discontinued operation
(after tax) of Rs.52,500 (2014: Rs.0).
2. On 1 January 2014 there were 250 000 ordinary shares each with a par value of Rs.5.00 in issue. On the 30
September 2014 there was a rights issue on a basis of 1 ordinary share issued for every 5 already held at a price
of Rs.6.00 The market value of the ordinary shares immediately before the rights issue was Rs.7.50 per share.
On 31 May 2015 there was an issue of 50,000 ordinary shares at market price (Rs.5 per share).
3. There are 25,000 options in existence, each of which allows the holder to acquire four shares at a strike price of
Rs.10.00 per share. The average market price per ordinary share for 2014 and 2015 was 12.00. These options
were in existence throughout 2014 and 2015.
4. Preference shares in issue are convertible (at the option of the preference shareholders) into 500 ordinary
shares on 31 December 2017. If not converted, the preference shares will be redeemed on 31 December 2017.

245
Page 9 of 11
Dividends of Rs. 1,000 are incurred annually on these preference shares (these have been correctly accounted
for as finance charges). The preference shares were in existence throughout 2014 and 2015.
5. Ignore tax.

Required:
Disclose earnings per share in the financial statements of Early Morning Limited for the year ended 31 December
2015. (with comparatives)

Question 15
L, G and M Limited is a listed company.
Details for their current reporting period is as follows:

The correctly calculated profit after tax for the year ended 31 December 2018 is Rs. 550,000 (2017: Rs. 400,000)
At 31 December 2017 the following were in issue:
1. 350,000 ordinary shares of Rs. 3 par value each
2. 300,000 10% redeemable, cumulative preference shares of Rs.3 each. The preference dividends for 2018 have
not yet been declared. The 2017 preference dividends were declared on 30 November 2017.
3. Options to acquire 40,000 ordinary shares in L, G and M limited after 30 November 2019 at a strike price of Rs.
6 per share. The average market price of the shares during 2018 was Rs.10 per share

The following transactions took place in 2018:


• 1 January 2018 200,000 12% convertible debentures of Rs.2 each were issued. These debentures are convertible
on 30 September 2019 at the option of the debenture holders into L, G and M limited ordinary shares at a rate
of one ordinary share for two debentures. If not converted into ordinary shares they will be redeemed on 30
September 2019.
• On 31 March 2018, there was a capitalization issue of 2 ordinary shares for every 5 shares in issue. The
capitalization issue utilized current share premium as far as is possible so as to minimize the impact of this issue
on retained earnings.
• On 30 September 2018 50,000 ordinary shares were issued at their market value on that date. Another such
issue of 10,000 shares took place on 30 November 2018 at their market value on that date.
• On 20 October 2018, 30,000 ordinary shares were offered to existing shareholders. The offer was fully
subscribed for at Rs.7 per share on 31 October 2018, when the market price was Rs.10 per share(entitlement
date).
• The corporate tax rate is 30%

Required:
Disclose the EPS in the statement of Comprehensive income and the notes to the financial statements of L, G and M
limited for the year ending 31 December 2018 in accordance with IFRS. (along with comparatives)

246
Page 10 of 11
Question 16
The following information is for the year ended 30.06.2021:
Profit from continuing operations 16,400,000
Less dividends on preference shares (6,400,000)
Profit from continuing operations attributable to ordinary equity holders 10,000,000
Loss from discontinued operations (4,000,000)
Profit attributable to ordinary equity holders 6,000,000
Ordinary shares outstanding 2,000,000
Average market price of one ordinary share during year 75

Potential ordinary shares:


Options 100,000 with exercise price of 60
Convertible preference shares 800,000 shares with a par value of 100 entitled to a cumulative dividend
of 8 per share. Each preference share is convertible to two ordinary
shares.
5% convertible bonds Nominal amount 100,000,000. Each 1,000 bond is convertible to 20
ordinary shares.
Tax rate 40%

Required: Calculate the basic and diluted Earnings per share for the year ended 30.06.2021.

247
Page 11 of 11
Solution 1
(a) The following situations will result in comparatives for earnings per share to be restated:
A capitalisation issue or a share split (i.e. a not for value issue) and share consolidation (i.e a not for value
reduction)

A change in accounting policy or a correction of an error (As per IAS-8)

With a capitalisation issue or a share split and share consolidation no new cash resources are available to the
company. The number of shares increases resulting in a decrease in the EPS. Therefore, the comparative EPS
must be restated to ensure that comparability is not lost. This adjustment applies not only to the prior period
but to the figures of all previous periods that are presented as comparatives as well.

A change in accounting policy or a correction of an error gives rise to a prior year adjustment in terms of IAS
8. With a prior year adjustment it is necessary to restate the previous year’s comparatives and the retained
earnings at the beginning of the prior year. If the adjustment has an impact on the ‘earnings’ used for the
earnings per share calculation, it will therefore be necessary to restate the earnings per share figure for the
previous year.

(b) IAS 33 requires earnings per share to be based on basic earnings, which is defined as the profit or loss for the
period attributable to ordinary shareholders after deducting preference dividends. In terms of IAS 8, profit for
the period should include all items of income and expense recognised in a period. The profit on sale of
investments is an other income should be included in profit for the period and therefore should be included in
basic earnings as well.

(c) Dividends per share depends on the dividend payout policy of a company, and not necessarily on the size of its
profits. It is not possible to judge a company’s performance on its dividends declared. In new or expanding
companies, for example, it would be irresponsible to adopt too high a dividend payout ratio. A low dividend
per share in such cases would not necessarily reflect poor performance - management may just be retaining
the profits in order to re-invest in the business. Earnings per share, on the other hand, is based on profit
earned by the business regardless of whether such funds are being paid out to the owners or are being re-
invested to increase the value of the business.

Profit after tax on its own does not tell shareholders the extent of the return on their investment. For example
if two companies both reflect profit after taxation for the year of Rs.100,000 but company A has 1,000 shares
and company B has 2,000 shares, it cannot be said that a shareholder with one share in each of the companies
has earned the same amount on each investment, even though the profits earned by each company are the
same. The one share held in company A has yielded a Rs.100 return whereas the one share in company B has
only yielded a Rs.50 return once the profits have been shared out amongst the owners. It is therefore more
meaningful to look at earnings per share than at total earnings.

248
Page 1 of 31
Solution 2

A LIMITED

EXTRACTS FROM THE STATEMENT OF COMPREHENSIVE INCOME FOR THE


YEAR ENDED 30 SEPTEMBER 2013
Notes 2013 2012

443 500 / 420 000; (23 500)/ 405


Basic earnings / (loss) per share 000 1.06 (0.06)

A LIMITED
NOTES TO THE FINANCIAL STATEMENTS
FOR THE YEAR ENDED 30 SEPTEMBER 2013

Earnings per share


The calculation of basic earnings per share is based on profit / (loss) of 443,500 (2012: loss of 23,500)
and on the weighted average of 420,000 shares in issue (2012: 405,000) after the capitalisation
issue on 15 June 2013. The earnings per share for 2012 have been adjusted accordingly.

Reconciliation of profit to earnings


2013 2012

Profit/(loss) for the


period 475 000 (10 000)
Preference dividends (13 500 + 18 000) (13 500) (31 500) (13 500)
Basic earnings 443 500 (23 500)

W.1) 2013:

2013 2013 Current 2012 Prior


Actual (Weighted) (Adjusted)
b/d 350,000 350,000 337,500 (w.2)
For no value (15.6) 70,000 70,000 67,500
(350,000/5 x 1) (70/350 x 337,500)
c/d 420,000 420,000 405,000

249
Page 2 of 31
W.2) 2012:
2012 2012 Current
Actual (Weighted)
b/d 300,000 300,000
For value (1.1.2012) 50,000 37,500
(50,000 x 9/12)
c/d 350,000 337,500

Solution 3

MITCH LIMITED
EXTRACTS FROM THE STATEMENT OF COMPREHENSIVE INCOME
FOR THE YEAR ENDED 31 DECEMBER 2011

Notes 2011 2010


Profit for the period 100,000 80,000
Basic earnings per ordinary share2011: 95,000 /
1,375,000
2010: 75,000 /
1,000,000 0.0691 0.0750

MITCH LIMITED
EXTRACTS FROM THE STATEMENT OF CHANGES IN EQUITY
FOR THE YEAR ENDED 31 DECEMBER 2011

Ordinary Preference Retained Total


Shares shares Earnings

Opening balance at 1/1/2010 200,000 50,000 Xxx


Profit for the period 80,000
Preference dividends (5,000)
Ordinary dividends (6,000)
Opening balance at 1/1/2011 200,000 50,000 Xxx
Share issue (500,000 x 0.2) 100,000
Profit for the period 100 000
Preference dividends (5,000)
Ordinary dividends (10,000)
Closing balance at 31/12/2011 300,000 50,000 Xxx

250
Page 3 of 31
MITCH LIMITED

EXTRACTS FROM THE NOTES TO THE FINANCIAL STATEMENT

FOR THE YEAR ENDED 31 DECEMBER 2011

Earnings per share


The calculation of earnings per share is based on earnings of 95,000 (2010: 75,000) and a weighted

average of 1,375,000 ordinary shares (2010: 1,000,000) in issue throughout the year.

Reconciliation of earnings 2011 2010

Profit for the year 100,000 80,000


Preference dividends (5,000) (5,000)
Basic earnings 95,000 75,000

Workings:

2011 Actual 2011 Current (Weighted) 2010Prior (Adjusted)


b/d (200,000 / 0.2) 1,000,000 1,000,000 1,000,000 (No Change in
2010)
For value (31.3) 500,000 375,000 0
(500,000 x 9/12)
1,500,000 1,375,000 1,000,000

Solution 4

MILES LIMITED

EXTRACTS FROM THE STATEMENT OF COMPREHENSIVE INCOME FOR THE


YEAR ENDED 31 DECEMBER 2011
Notes 2011 2010

Profit before tax 503,000 403,000


Income tax expense (200,000) (180,000)
Profit for the period 303,000 223,000

Basic earnings per ordinary share 2011: 300 000* / 1 500 5 0.200 0.147
000
2010: 220 000** / 1 500
000

251
Page 4 of 31
MILES LIMITED
EXTRACTS FROM THE STATEMENT OF CHANGES IN EQUITY
FOR THE YEAR ENDED 31 DECEMBER 2011

Ordinary Preference Share Retained


shares shares premium earnings Total

Opening balance at 1/1/2010 200,000 30,000 290,000 60 ,000 580,000


Profit before tax 223,000 223,000
Preference dividends (3,000) (3,000)
Ordinary dividends (30,000) (30,000)
Opening balance at 1/1/2011 200,000 30,000 290,000 250,000 770,000
Capitalisation issue 100,000 (100,000) 0
Profit before tax 303,000 303,000
Preference dividends (3,000) (3,000)
Ordinary dividends (30,000) (30,000)
Closing balance at 31/12/2011 300 000 30 000 190 000 520,000 1,040,000

MILES LIMITED

EXTRACTS FROM THE NOTES TO THE FINANCIAL STATEMENT

FOR THE YEAR ENDED 31 DECEMBER 2011

Earnings per share


Basic earnings per
share
The calculation of earnings per share is based on earnings of 300,000 (2010: 220,000) and
1 500,000 ordinary shares (2010: 1,500,000) after adjusting for the capitalisation issue during 2011.

Reconciliation of profit to earnings


2011 2010

Profit/(loss) for the period 303 000 223 000


Preference dividends (3 000) (3 000)
Basic earnings 300 000 220 000

252
Page 5 of 31
Workings:
W.1) 2011

Actual Current (Weighted) Prior


(Adjusted)
b/d 1,000,000 1,000,000 1,000,000
1.7 ( issue for no value) 500,000 500,000 500,000
1,000,000/2 x 1
c/d 1,500,000 1,500,000 1,500,000

Solution 5

LOYAL LIMITED

EXTRACTS FROM THE STATEMENT OF COMPREHENSIVE INCOME FOR THE


YEAR ENDED 31 DECEMBER 2011
2011 2010

Profit for the year 250 000 280 000


Basic earnings per ordinary share 2011: 247 000 / 200
000
2010: 277 000 / 200
000 1.235 1.385

LOYAL LIMITED
EXTRACTS FROM THE STATEMENT OF CHANGES IN EQUITY
FOR THE YEAR ENDED 31 DECEMBER 2011

Ordinary Preference Retained Total


shares Shares earnings

Opening balance at 1/1/2010 100 000 20 000 120 000 240 000
Profit for the year 280 000 280 000
Preference dividends (3,000) (3,000)
Ordinary dividends (12,000) (12,000)
Opening balance at 1/1/2011 100 000 20 000 385 000 505 000
Profit for the year 250 000 250 000
Preference dividends (3,000) (3,000)
Ordinary dividends (10,000) (10,000)
Closing balance at 31/12/2011 100 000 20 000 622 000 742 000

*No entry of share split in financial statements

253
Page 6 of 31
LOYAL LIMITED

EXTRACTS FROM THE NOTES TO THE FINANCIAL STATEMENT

FOR THE YEAR ENDED 31 DECEMBER 2011

Earnings per share


Basic earnings per
share
The calculation of earnings per share is based on earnings of 247 000 (2010: 277 000) and 200 000
ordinary shares (2010: 200 000) after adjusting for the share split during 2011.

Reconciliation of earnings 2011 2010

Profit for the year 250 000 280 000


Preference dividends (3 000) (3 000)
Basic earnings 247 000 277 000

Working: 2011
2011 2011 Current 2010 Prior
Actual (Weighted) (Adjusted)
b/d 100,000 100,000 100,000 (No change
in 2010)
1.7 No value issue bal. 100,000 100,000 100,000
c/d[1,000,000/1 x 2] 200,000 200,000 200,000

Solution 6

ROGER LIMITED

EXTRACTS FROM THE STATEMENT OF COMPREHENSIVE INCOME FOR THE


YEAR ENDED 31 DECEMBER 2018
Notes 2018 2017

Profit before tax 750 000 730 000


Income tax expense (400 000) (300 000)
Profit for the year 350 000 430 000

Basic earnings per ordinary share


0.3832 0.5147

254
Page 7 of 31
ROGER LIMITED

EXTRACTS FROM THE STATEMENT OF CHANGES IN EQUITY

FOR THE YEAR ENDED 31 DECEMBER 2018

Ordinary Preference Share Retained Total


shares shares premium earnings
Opening balance at 1/1/2017 375 000 400 000 0 200 000 975 000
(750,000 x 0.5)
Profit for the year 430 000 430 000
Preference dividends (32 000) (32 000)
Ordinary dividends (30 000) (30 000)
Opening balance at 1/1/2018
(W3) 375 000 400 000 0 568 000 1 343 000
Rights issue 125 000 425 000 550 000
250,000 x 0.5 = 125,000
250,000 x (2.2 – 0.5) = 425,000
Profit for the year 350 000 350 000
Ordinary dividends (40 000) (40 000)
Preference dividends (32 000) (32 000)
Closing balance at 31/12/2018 500 000 400 000 425 000 846 000 2 171 000

ROGER LIMITED

EXTRACTS FROM THE NOTES TO THE FINANCIAL STATEMENTS FOR THE


YEAR ENDED 31 DECEMBER 2018

Earnings per share


Basic earnings per share
The calculation of earnings per share is based on earnings of 318 000 (2017: 398 000) and a
weighted average of 829 897 ordinary shares (2017: 773 196) after adjusting for the rights issue
on 30th September 2018.

Reconciliation of profit to earnings 2018 2017

Profit/(loss) for the period 350 000 430 000


Preference dividends (32 000) (32 000)
Basic earnings 318 000 398 000

255
Page 8 of 31
EPS 2018 2017
350,000 – 32,000 430,000 – 32,000
829,897 773,196

0.3832 0.5147

W2. Number of Shares (rights issue) - an alternative calculation


Actual Current weighted Prior Adjusted
2018 2018 2017
b/d 1-1-18 (W 3) 750,000 750,000 750,000
issue for value (30.9) 220,000 55,000 0
(220,000 x 3/12)
970,000 805,000 750,000
Issue for no value (30.9) 30,000 24,897 23,196
(30/970 x 805000) (30/970 x 750000)
Balance 21-12-2018 1,000,000 829,897 773,196

750,000/3 x 1 = 250,000 x 2.2/2.5 = 220,000 ( for value)


250,000 – 220,000 = 30,000 ( for no value)

W3 Number of shares at beginning of the year

the number of shares at year-end = 500 000 / 0.50 = 1 000


000 shares let X = the number of shares in issue at the
beginning of the year then :

X + X/ 3 x 1 = 1 000 000 shares

1.3333X = 1 000 000

X = 1 000 000/1.3333 = 750 000 shares

check:

750 000 / 3 x 1 = 250 000 (issued through rights issue)

opening balance + rights issue = balance at year-end

750 000 + 250 000 = 1 000 000

256
Page 9 of 31
Solution 7

a)
ANNE LIMITED

EXTRACTS FROM THE STATEMENT OF COMPREHENSIVE INCOME FOR THE


YEAR ENDED 31 DECEMBER 2013

Notes 2013 2012

Profit for the year 700 000 900 000

Basic earnings per share (2013: 500 000 / 63 462) 7.88 17.98
(2012: 700 000 / 38 943)

ANNE LIMITED

EXTRACTS FROM THE NOTES TO THE FINANCIAL STATEMENT

FOR THE YEAR ENDED 31 DECEMBER 2013

Earnings per share


Basic earnings per share is calculated based on earnings of 500 000 (2012: 700 000) and a weighted
average number of shares of 63 462 (2012: 38 943).

Reconciliation of earnings:

Reconciliation of profit to earnings


2013 2012
Profit/(loss) for the period 700 000 900 000
Preference dividends (100,000 x 10 x 20%) (200 000) (200 000)
Basic earnings 500 000 700 000

W.1)
2013 Actual 2013 Current 2012 Prior
(Weighted) (Adjusted)
b/d 30,000 30,000 22,500
30.6 For value (W) 13,333 6,667 0
(13,333 x 6/12)
43,333 36,667 22,500
20.6 For no value (W) 6,667 5,641 3,462

257
Page 10 of 31
(6,667/43,333 x (6,667/43,333 x
36,667 22,500)
50,000 42,308 25,962
30.9 for no value bal. 25,000 21,154 12,981
(25/50 x 42,308) (25/50 x 25,962)
[50,000/2 x 3] 75,000 63,462 38,943

30,000/3 x 2 = 20,000 x 10 /15 = 13,333 [for value]


20,000 – 13,333 = 6,667 [for no value]

b)
ANNE LIMITED

EXTRACTS FROM THE STATEMENT OF COMPREHENSIVE INCOME FOR THE


YEAR ENDED 31 DECEMBER 2012

2012 2011

Profit for the year 900,000 800 000

Basic earnings per share (2012: 700 000 / 22 500) 31.11 40.00
(2011: 600 000 / 15 000)

ANNE LIMITED

EXTRACTS FROM THE NOTES TO THE FINANCIAL STATEMENT

FOR THE YEAR ENDED 31 DECEMBER 2012

Earnings per share

Basic earnings per share is calculated based on earnings of 700 000 (2011: 600 000) and a weighted
average number of shares of 22 500 (2011: 15 000).

Reconciliation of profit to earnings


2012 2011
Profit/(loss) for the period 900 000 800 000
Preference dividends (1,000,000 x 20%) (200 000) (200 000)
Basic earnings 700 000 600 000

258
Page 11 of 31
W.1) 2012
2012 2012 Current 2011 Prior
Actual (Weighted) (Adjusted)
b/d 20,000 20,000 15,000
30.9 issue for value 10,000 2,500 0
(10,000 x 3/12)
c/d 30,000 22,500 15,000

C)
ANNE LIMITED

EXTRACTS FROM THE STATEMENT OF COMPREHENSIVE INCOME FOR THE


YEAR ENDED 31 DECEMBER 2011

2011
Profit for the period 800 000

Basic earnings per share (2011: 600 000 / 15 000) 40.00

ANNE LIMITED

EXTRACTS FROM THE NOTES TO THE FINANCIAL STATEMENT

FOR THE YEAR ENDED 31 DECEMBER 2011

Earnings per share

Basic earnings per share is calculated based on earnings of 600 000 and 15 000 shares in
issue throughout the year.

Reconciliation of profit to earnings 2011

Profit/(loss) for the period 800 000


Preference dividends (100,000 x 10 x 20%) (200 000)
Basic earnings 600 000

259
Page 12 of 31
W.1) Weighted average number of shares:

2011 2011 Current


Actual (Weighted)
b/d 10,000 10,000
30.6 issue for value 10,000 5,000
(10,000 x 6/12)
c/d 20,000 15,000

260
Page 13 of 31
Solution 8
a)
T LTD

STATEMENT OF COMPREHENSIVE INCOME


FOR THE YEAR ENDED 31 DECEMBER 2014

2014 2013 2012

Profit for the year 180 000 150 000 100 000

Basic earnings per share 0.438 0.534 0.635

2014: 170 000 / 388 065; 2013: 140 000 / 262 121; 2012: 90 000 / 141 687

T LTD

NOTES TO THE FINANCIAL STATEMENTS


FOR THE YEAR ENDED 31 DECEMBER 2014

Earnings per share

Basic earnings per share

Basic earnings per share is calculated based on earnings of 170 000 (2013: 140 000 and 2012:
90 000) and a weighted average number of shares after taking into account the share issues of 388 064
(2013: 262 120 and 2012: 141 687).

Reconciliation of profits to earnings


per share
2014 2013 2012

Profit for the period 180 000 150 000 100 000
Preference dividends (100,000 x 10%) (10 000) (10 000) (10 000)
Basic earnings 170 000 140 000 90 000

261
Page 14 of 31
W.1)
(2014) (2014) (2013) (2012)
Actual Current (weighted ) Prior(Adjusted) Prior
Adjusted
b/d 270,000 270,000 185,000 100,000
31.5 For value 6,667 3,889 0 0
(6667×7/12)
276,667 273,889 185,000 100,000
31.5 For No value 3,333 3,300 2,229 1,205
(3,333/2676,667×273,889) (3,333/276,667×185,000)
280,000 277,189 187,229 101,205
30.11 For no value [280,000/5 ×2] 112,000 110,876 74,892 40,482
(112/280 ×277,189) (112/280 ×187,229) (112/280
×101,205)
C/d 392,000 388,065 262,121 141,687

10,000×6/9 =6,667 9(for value )


10,000-6,667= 3,333(for no value)

b) Earnings per share in 2013 financial statements

2013 2012

Basic
earnings 140 000 90 000

Number of
shares 185 000 100 000

0.757 0.900

W.1)
2013
Actual Current Prior (adjusted)
(2013) (weighted ) 2012
2013
b/d 100,000 100,000 100,000*
30.6[For value] 170,000 85,000 0
[170,000×6/12]
C/d 270,000 185,000 100,00

*issued at the start of on 1.1.2012

262
Page 15 of 31
c) Journal entries

Journals Debit Credit


Bank (10 000 x 6) 60 000
Ordinary shares (10 000 x 1) 10 000
Share premium (balancing) 50 000

Ordinary shares issued at 6 (market price of 9)

Share premium 15 000


Bank 15 000
Share issue expenses written off

Share premium (30K + 50K – 15K) 65 000


Retained earnings (bal) 47 000
Ordinary share capital (112 000 x 1) 112 000
Capitalisation issue of ordinary shares using the rest of the share
premium balance:
(270,000 + 10,000)/5 x 2 x 1

Preference dividends 10 000


Bank 10 000

Preference dividends paid: (100,000 x 10%)

Solution 9

HUBBARD LIMITED

EXTRACTS FROM THE STATEMENT OF COMPREHENSIVE INCOME FOR THE


YEAR ENDED 30 JUNE 2016
Notes 2016 2015

Revenue 500 000 400 000


Cost of inventory expense (250 000) (200 000)
Gross profit (2016: 7 000 + 250 000 200 000
Other income 3 000) 10 000 3 000
Other expenses (110 000) (103 000)
Profit before tax 150 000 100 000
Income tax expense (2016: 40 000 (40 000) (31 500)
2015: (35 000 – 3500)
Profit for the period 110 000 68 500

263
Page 16 of 31
Earnings per share 10 0.2160 0.1565

HUBBARD LIMITED
STATEMENT OF CHANGES IN EQUITY
FOR THE YEAR ENDED 30 JUNE 2016
Retained
earnings

Balance at 01.07.2014 25 000


Profit for the period [restated] 68 500
Ordinary dividend (5 000)
Preference dividend (4 000)
Balance at 30/6/2015 as restated 84 500
Profit for the period 110 000
Ordinary dividend ( 4,600 +
6,000) (10 000)
Preference dividend (2 000)
Transfer to reserves (7 000)
Balance at 30/6/2016 175 500

HUBBARD LIMITED
NOTES TO THE FINANCIAL STATEMENTS
FOR THE YEAR ENDED 30 JUNE 2016

Earnings per share

Basic earnings per share:


The calculation of earnings per share is based on earnings of 108,000 (2015: 66,500) and on the
weighted average of 500 000 shares in issue (2015: 425 000) after the share split on 1 January 2016.
Comparatives have been restated.

Reconciliation of profit to earnings 2016 2015

Profit for the period 110 000 68 500


Preference dividends (2 000) (2 000)
250
Basic earnings 108 000 66 500

264
Page 17 of 31
Workings of EPS:
2016 2015

110,000 – 2,000 66,500 – 2,000


500,000 425,000

0.216 0.1565
Prior period Error: There was an error in calculation of tax during the year 2015. Comparatives have been
appropriately restated.

The effect of the correction is as follows: 2015

Effect on the statement of comprehensive income

Increase in tax expenses 3,500


Increasee in profits 3,500

Effect on the statement of financial position

Decrease in tax payable 3 500


Increase in Retained
earnings 3 500

WORKINGS :

W.1 2016:
Actual 2016 2015
Current Prior
(weighted) (Adjusted)
b/d 250,000 250,000 212,500(W.2)
For no value (1.1) 250,000 250,000 212,500
(250/250
×212,500)
c/d 500,000 500,000 425,000
[250,000/1 ×2]

W. 2 2015
(2015) (2015)
Actual Current
(weighted)
b/d 200,000 200,000
For value (31.3) 50,000 12,500
(50,000×3/12)
c/d 250,000 212,500

265
Page 18 of 31
Solution 10

TRINI LIMITED

STATEMENT OF COMPREHENSIVE INCOME


FOR THE YEAR ENDED 31 DECEMBER 2018
2018 2017

Profit for the year 3 220 000 2 125 000


Basic earnings per share (W.2) 11.50 9.78

TRINI LIMITED

EXTRACTS FROM THE NOTES TO THE FINANCIAL STATEMENT

FOR THE YEAR ENDED 31 DECEMBER 2018

Earnings per share


Basic earnings per
share
The calculation of basic earnings per share is based on earnings of 3 220 000 (2017: 2 125 000)
and a weighted average of 279 900 (2017: 217 391) ordinary shares in issue during the year.

Reconciliation of earnings 2018 2017

Profit for the year 3 220 000 2 125 000


Basic earnings 3 220 000 2 125 000

Workings: W.1 2016:


Actual 2018 2017
Current (weighted) Prior (Adjusted)

Balance 1/1/18 500 000 500 000 500 000


30 April for value

125 000 83 333 -


625 000 583 333 500 000
30 May rights issue for value 93 750 54 688 -
718 750 638 021 500 000
30 May rights issue for no value 62 500 55 480 43 478
781 250 693 501 543 478
31 Oct share consolidation (bal.) (468 750 ) (416 101) (326 087)

266
Page 19 of 31
(781,250/5 x 2)

312 500 277 400 217 391


30 Nov for value issue 30 000 2 500 -
c/d [250,000/1 ×2] 342,500 279,900 217,391

625,000/4 x 1 x 3/5 = 93,750 [For Value]


625,000/4 = 156,250 – 93,750 = 62,500 [For Value]

W.2

Basic earnings per share 3 220 000 2 125 000


279 900 217391
= 11.50 = 9.78
Solution 11

SPROG LIMITED

EXTRACTS FROM THE STATEMENT OF COMPREHENSIVE INCOME FOR THE


YEAR ENDED 31 DECEMBER 2015
2015

Profit for the year 20 000 000

Earnings per share:


Basic 20 000 000 ÷ (10% x 1 000 000 000) 0.20
Diluted basic (W-1) 0.14

Workings

W1: Dilutive earnings per share


Incremental EPS = 1,505,000/50,000,000* = 0.0301 Dilutive
*500,000/1 x 100

Revised earnings 20 000 000 + 1 505 000


Revised shares = 100 000 000 + 50 000 000

= 21 505 000 / 150 000 000


= 0.1434

267
Page 20 of 31
Solution 12

LASER LIMITED

EXTRACTS FROM THE STATEMENT OF COMPREHENSIVE INCOME FOR THE


YEAR ENDED 31 DECEMBER 2015
2015 2014

Profit/ (loss) for the year 125 000 (50 000)

Earnings per share: (125,000/112,000)


Basic earnings (50,000/101,000) 1.116 (0.495)
(125,000/118,818)
Diluted basic (50,000/107,818) 1.052 (0.464)

Workings

W1: Basic earnings 2015 2014


Profit for the year 125 000 (50 000)
Preference dividends 0 0
Basic Earnings 125 000 (50 000)

W-2) 2014:

2014 Actual 2014 Current


(Weighted)
b/d 100,000 100,000
30.11 for value 12,000 1000
(12,000 x 1/12)
c/d 112,000 101,000

W.2.1) 2015. No Change in Shares:


2015 Actual 2015 Current 2014 Prior
(Weighted) (Adjusted)
b/d 112,000 112,000 101,000 (W.2)
c/d 112,000 112,000 101,000

W3: Dilutive number of shares


Options (bonus/ free portion) 25 000 – (25 000 x 1 x 2.00) ÷ 2.75 = 6 818

Dilutive number of shares 2014: (101 000 + 6 818*) = 107 818


2015: (112 000 + 6 818) = 118 818

268
Page 21 of 31
*Options are in comparative year as on 31.12.2014.

Solution 13

REBEL LIMITED
EXTRACTS FROM THE STATEMENT OF COMPREHENSIVE INCOME FOR THE
YEAR ENDED 31 DECEMBER 2015
2015 2014
(15,000/62,500)
Basic earnings per share (30,000/25,000) 0.2400 1.2000
Basic diluted earnings per share (W4) 0.2237 1.0927

Workings:
W.1) Calculation of correct basic earnings
2015 2014
Incorrect basic earnings per
share Given 0.20 1.75
Incorrect number of shares Given 100,000 shares 20,000 shares
Incorrect basic earnings (0.2 x 100,000) (1.75 x 20,000) 20,000 35,000

W.2) Calculation of incorrect basic earnings


2015 2014

Incorrect basic earnings W-1 20,000 35,000


Preference shares dividends given (5,000) (5,000)
Correct basic earnings 15,000 30,000

W.3)
2015:
Actual Current 2015 Prior
(weighted) 2014(adjusted)
b/d 20,000 20,000 20,000 (W.3.1)

For value (30.6) 60,000 30,000(60,000 0


×6/12)
80,000 50,000 20,000

For no value (30.6) 20,000 12,500 5,000


(20/80 ×50,000) (20/80 ×20,000)
100,000 62,500 25,000
20,000/1×4= 80,000 × 1.5 = 120,000
120,000/2= 60,000 (for value)
80,000-60,000=20,000 (for no value)

269
Page 22 of 31
W.3.1 2014
Actual Current (weighted)
b/d 20,000 20,000
20,000 20,000

W-4) Dilutive earnings per share:


Whether dilutive or not
2014= [100 × 6/12] ÷ [5000 ×6/12]= 0.02 dilutive
2015= 100/5000= 0.02 dilutive

2015 2014
basic earning (15,000 + 100) 15,100 (30,000 + 30,050
(Financed costs saved) 100 x 6/12)
Dilutive number of shares (62,500 + 5,000) 67,500 (25,000 + 27,500
(Additional shares) 5,000 x
6/12)
Dilutive earnings per share (15,100 / 67,500) 0.2237 (30,500 / 1.0927
27,500)

Solution 14

EARLY MORNING LIMITED

EXTRACTS FROM THE STATEMENT OF COMPREHENSIVE INCOME FOR THE


YEAR ENDED 31 DECEMBER 2015
2015 2014
Profit for the year 500 000 337 500

Earnings per share


Basic 500 000 / 329 166 337 500 / 268 966 1.5190 1.2548
Continuing
operations 447 500 / 329 166 337 500 /268 966 1.3595 1.2548
Discontinued
operations 52 500 / 329 166 0.1595
Diluted 500 000/ 345 833 337 500 / 285 633 1.4458 1.1816
Continuing operations 447 500 / 345 833 337 500 / 285 633 1.2940 1.1816

Discontinued operations 52,500/345,833 0.1518

270
Page 23 of 31
EARLY MORNING LIMITED
EXTRACTS FROM THE NOTES TO THE FINANCIAL STATEMENTS
FOR THE YEAR ENDED 31 DECEMBER 2015

Earnings per share


The calculation of the following earning per share are based on the following amounts
2015
Earnings Weighted Average number of
shares
Basic 500 000 329 166
Diluted basic 500 000 345 833

2014
Weighted Average number
Earnings of
shares
Basic 337 500 268 966
Diluted basic 337 500 285 633

Reconciliation of profit for the


year 2015 2014
to basic earnings
Profit for the year (after tax)

Less preference dividends ( No adjustment because classified as 500 000 337,500


liability) 0 0
500 000 337,500
Basic earnings

Reconciliation of earnings to diluted basic


earnings
2015 2014
Basic Basic
Earnings 500 000 337 500
Options 0 0
Preference shares
(not adjusted for as anti-dilutive)
Diluted basic earnings 500 000 337 500

271
Page 24 of 31
EARLY MORNING LIMITED
EXTRACTS FROM THE NOTES TO THE FINANCIAL STATEMENTS
FOR THE YEAR ENDED 31 DECEMBER 2015

15. Earnings per share continued …


Reconciliation of basic number of shares to diluted number of
shares
2015 2014
Basic number of shares 329 166 268 966
Options 16 667 16 667
Preference shares (not adjusted for because anti-dilutive)
Diluted number of shares 345 833 285 633

Workings
2015: Weighted average no. Of shares

Actual Current Prior 2014


weighted (adjusted)
(2015)
b/d 300,000 300,000 268,966
(W.2)
(31.5) For value 50,000 29,167 0
(50,000 ×7/12)
350,000 329,167 268,966

W. 2 2014:
Weighted average no. Of shares
Actual (2014)
Current
weighted
b/d 250,000 250,000
(30.9) for value (W) 40,000 10,000
(40,000×3/12)
290,000 260,000
(30.9) for no value (W) 10,000 8,966
(10,000/290,00
0 ×260,000)
c/d 300,000 68,966

250,000/5 ×1=50,000×6= 300,000/7.5 =40,000[for value]


50,000-40,000 = 10,000[for no value]

272
Page 25 of 31
W2: Ranking of dilutive instruments Ranking:

0.000
Options No change to earnings 0 / 16,667(w-2.1) Dilutive 1
2.0
Preference shares Dividend 1 000/500 Anti Dilutive 2

W.2.1)
25,000×4=100,000
100,000×`10=1,000,000
1,000,000/12 =83,333(for value)
100,000-83,333= 16,667(for no value)

Point to remember:
IAS 33 requires the dilutive (or anti-dilutive) effect of potential ordinary shares to be
determined with reference to basic EPS from continuing operations.

Of the total profit for the year, Rs. 52 500 (after tax) was earned from a discontinued
operation. When testing to determine whether the potential ordinary shares are dilutive,
profit for the period excluding profit from discontinued operations is used.

The actual calculation of diluted earnings per share will include profit from discontinued
operations. The total diluted earnings per share is then split into diluted earnings per share
from continuing operations and diluted earnings per share from discontinued operations (see
extracts from the financial statements above).

W3: 2015 Test whether anti-dilutive


Basic earnings per share (500 000 -52 500) 447 500 = 1.3595
329 166

Options 447 500 + 0 447 500 = 1.2940 Dilutive


329 166 + 16 667 = 345 833
Options and Preference
shares 447 500 + 1 000 448 500 = 1.2950 Anti-
345 833 + 500 = 346 333 Dilutive

W4: 2014 Test whether anti-dilutive


Based on shares of 2014 in 2015
Basic earnings per share 337 500 = 1.2548
268 966

Options 337 500 + 0 337 500 = 1.1816 Dilutive


268 966 + 16 667 = 285 633
Options andPreference
shares 337 500 + 1 000 338 500 = 1.1830 Anti-
285 633 + 500 = 286 133 Dilutive

273
Page 26 of 31
Solution 15

L, G AND M LIMITED

EXTRACTS FROM THE STATEMENT OF COMPREHENSIVE INCOME FOR THE


YEAR ENDED 30 SEPTEMBER 2013
2018 2017

Basic earnings / (loss) 550 000 (W.1) ÷ 514 952(W.2) ;400 000 (W.1) ÷497
per share 861 (W.2) 1.068 0.8034
Diluted basic earnings 583 600 (W.3) ÷ 630 952 (W.4); 400 000 (W.3) ÷ 513
per share 861 (W.4 and reconciliation) 0.925 0.778

Notes to Financial Statements


Earnings per share

The calculation of basic earnings per share is based on profit / (loss) of 550 000 (2017: 400 000) and

on the weighted average of 514 952 shares in issue (2017: 497 861) at the end of the year.

The calculation of diluted basic earnings per share is based on profit / (loss) of 583 600 (2017:
400 000) and on the weighted average of 630 952 shares in issue (2017:513 861) at the end of
the year.

Reconciliation of profit to earnings

2018 2017

Profit/(loss) for the period 550 000 400 000


Already
Preference dividends deducted from 0 0
profit
Basic earnings 550 000 400 000

Basic earnings 550 000 400 000


Debentures finance
cost avoided (48,000 – 14,400) 33 600
Diluted basic
earnings 583 600 400 000

Reconciliation of basic number of shares to diluted


Basic share (w.1) 514 952 497 861
Not for value options (W.4.1) 16 000 16 000
Convertible debentures (200,000/2 x 1) 100 000 0
Diluted number of shares 630 952 513 861

274
Page 27 of 31
• Options were also as on 31-12-2017. Even if they are issued during the period they are adjusted
retrospectively as it is not for value.
• Convertible debentures were issued at the beginning of the current year, i.e. as on 1-1-2018. Therefore,
no effect on 2017.

WORKINGS:
W.1 ) Basic Earnings
2018 2017

Profit After Tax 550,000 400,000


Finance cost on preference shares-redeemable 0 0
(No Adjustment)
Basic Earnings 550,000 400,000

W.2) Numbers of shares :


Actual Current Prior
2018 (weighted ) (adjusted)
b/d 350,000 350,000 350,000
(31.3) [issue for no value ] 140,000 140,000 140,000
[350,000/5 ×2]
490,000 490,000 490,000

(30.9) [issue for value ] given 50,000 12,500 0


(50,000 ×3/12)
540,000 502,500 490,000
(31.10)[for value(W-2.1)] 21,000 3,500 0
(21,000×2/12)
561,000 506,000 490,000
(31.10) [for no value (W-2.1)] 9,000 8,118 7,861
(9/561 (9/561 ×
×506,000) 490,0000
570,000 514,118 497,861
(30.11) [for value ] given 10,000 834 0
(10,000×1/12)
c/d 580,000 514,952 497,861

W-2.1) 31.10.2018
30,000 ×7 = 210,000/10= 21,000 [For value]
30,000-21,000 = 9,000 [For no value]

275
Page 28 of 31
W.3)Diluted Earnings:
2018 2017
Basic Earnings 550,000 400,000
Finance cost on convertible bonds[200,000 ×2×12%] 48,000 0*
Tax impact due to increase in profit [48,000×30%] (14,400) 0
33,600
Diluted Earnings 583,000 400,000
*Convertible bonds are not in 2017
**Redeemable preference shares are not convertible in this question therefore no impact on the diluted EPS.

W.4) Diluted EPS


(I)Ranking in order of dilution Rank
Options [0÷16,000(w-4.1)] 0 (dilutive)* 1
Convertible debanture [33,600÷100,000(20,000/2 ×1) 0.336 2
(dilutive)*
*In comparison is with basic EPS

(ii) Testing whether dilutive [2018]


Basic EPS [550,000÷514,952] 1.068
Adjustment For
Options [(550,000+0) ÷ (514,952+16,000=530,952)] 1.0359 dilutive
Option as well as convertible debantures
[(550,000+0+33,600) ÷ (514,952+16,000+100,000=630,952)] 0.925 dilutive
Lowest will be selected; i.e 0.925
Testing whether dilutive or not [2017]
Basic EPS [Restated] [400,000 ÷497,861] 0.8039
Adjustment For
Options [(400,000+0) ÷ [497,861+16,000 = 513,861] 0.778

Answer 16:
i) Basic EPS:
From continuing operations = 10,000,000/2,000,000 = 5 per share
From discontinuing operations = (4,000,000)/2,000,000 = (2) per share
Overall = 6,000,000/2,000,000 = 3 per share

ii)Calculation of diluted Earnings per share: (W1)


From continuing operations [13,000,000/4,020,000] 3.23
From discontinued operations [(4,000,000)/4,020,000] (0.99)
Overall [9,000,000/4,020,000] 2.24

276
Page 29 of 31
W.1) Step 1
Ranking in order of dilution
Increase in Increase in number Earnings per RANK
earnings of ordinary shares incremental share
Options
Increase in earnings Nil
Incremental shares issued for no 20,000 0 1
considerations [100,000 – (100,000 x (dilutive)*
60/75)]
Convertible preference shares
Increase in profit 6,400,000
800,000 x 8
Incremental shares 2 x 800,000 1,600,000 4 3
(dilutive)*
5% convertible bonds
Increase in profit 3,000,000
100,000,000 x 0.05 x (1-0.40)
Incremental shares 100,000 x 20 2,000,000 1.5 2
(dilutive)*
*In comparison is with basic EPS from continuing operations:

The order in which to include the dilutive instruments is therefore:


1 Options
2 5% convertible bonds
3 Convertible preference shares

Point to remember: IAS-33 requires the dilutive or antidilutive effect of potential ordinary shares to be determined
with reference to basic EPS from continuing operations. (Means all are dilutive compared with basic EPS of 5) Of
the total profit for the year, loss of 4,000,000 is from a discontinued operation. When testing to determine
whether potential ordinary shares are dilutive, profit for the period excluding profit from discontinued is used.

The actual calculation of diluted earnings per share will include profit from discontinued operations then total is
splitted into diluted EPS from continued and diluted EPS from discontinued operations.

277
Page 30 of 31
Step 2) Test whether Dilutive or Antidilutive:
Profit from continuing operations Ordinary Per share
attributable to ordinary equity holders shares
of the entity
As reported 10,000,000 2,000,000 5
Options - 20,000
10,000,000 2,020,000 4.95 Dilutive
5% Convertible bonds 3,000,000 2,000,000
13,000,000 4,020,000 3.23 Dilutive
Convertible preference share 6,400,000 1,600,000
19,400,000 5,620,000 3.45 Antidilutive
Because diluted earnings per share is increased when taking the convertible preference shares into account (from
3.23 to 3.45), the convertible preference are antidilutive and are ignored in the calculation of diluted earnings per
share. Therefore, diluted earnings per share for profit from continuing operation is 3.23:

278
Page 31 of 31
Question 1:
The profit after tax earned by AAZ Limited during the year ended December 31, 2007 amounted to Rs. 127.83 million.
The weighted average number of shares outstanding during the year were 85.22 million.

Details of potential ordinary shares as at December 31, 2007 are as follows:


• The company had issued debentures which are convertible into 3 million ordinary shares. The debenture holders
will exercise the option on December 31, 2009. If the debentures are not converted into ordinary shares they
shall be redeemed on December 31, 2009.The interest on debentures for the year 2007 amounted to Rs. 7.5
million.
• Preference shares issued in 2004 are convertible into 4 million ordinary shares at the option of the preference-
shareholders. The conversion option is exercisable on December 31, 2010. The dividend paid on preference
shares during the year 2007 amounted to Rs. 2.45 million.
• The company has issued options carrying the right to acquire 1.5 million ordinary shares of the company on or
after December 31, 2007 at a strike price of Rs. 9.90 per share. During the year 2007, the average market price
of the shares was Rs. 11 per share.

The company is subject to income tax at the rate of 30%.

Required:
a) Compute basic and diluted earnings per share.
b) Prepare a note for inclusion in the company's financial statements for the year ended December 31, 2007 in
accordance with the requirements of International Accounting Standards.
(18)
Question 2:
The following information relates to Afridi Industries Limited (AIL) for the year ended December 31, 2008:
i. The share capital of the company as on January 1, 2008 was Rs. 400 million of Rs. 10 each.
ii. On March 1, 2008, AIL entered into a financing arrangement with a local bank. Under the arrangement, all the
current and long-term debts of AIL, other than trade payables, were paid by the bank. In lieu thereof, AIL issued
4 million Convertible Term Finance Certificates (TFCs) having a face value of Rs. 100, to the bank. These TFCs are
redeemable in five years and carry mark up at the rate of 8% per annum. The bank has been allowed the option
to convert the TFCs on the date of redemption, in the ratio of 10 TFCs to 35 ordinary shares.
iii. On April 1, 2008, AIL issued 30% right shares to its existing shareholders at a price which did not contain bonus
element.
iv. During the year, AIL earned profit after tax amounting to Rs. 78 million. This profit includes a loss after tax for a
discontinued operation, amounting to Rs. 13 million.
v. The applicable tax rate is 35%.

Required:
Prepare extracts from the financial statements of Afridi Industries Limited for the year ended December 31, 2008
showing necessary disclosures related to earnings per share and diluted earnings per share. (Ignore corresponding
figures)

279
Page 1 of 2
Question 3:
The following information pertains to ABC Limited, in respect of year ended March 31,2010
Rs .in ‘000’
Profit for the year 13,000
Dividend paid during the year to ordinary shareholders 4,000
Dividend paid on 10% Cumulative preference shares for the year 2009 2,000
Dividend paid on 10% Cumulative preference shares for the year 2010 2,000
Dividend declared on 12% non cumulative preference shares for the year 2010 2,400

i. The dividend declared on the non-cumulative preference shares, as referred above, was paid in April 2010.
ii. The cumulative preference shares were issued at the time of inception of the company.
iii. The company had 10 million ordinary shares at March 31, 2009 of Rs. 10 each.
iv. 12% non-cumulative preference shares having nominal value of Rs. 10 each are convertible into one ordinary
shares, on or before December 31, 2011.
v. 1.20 million right shares of Rs. 10 each were issued at a premium of Rs. 1.50 per share on October 1, 2009. The
market price on the date of issue was Rs. 12.50 per share.
vi. 20% bonus shares were issued on January 1, 2010.
vii. Due to insufficient profit no dividend was declared during the year ended March 31, 2009.
viii. The average market price for the year ended 31,2010 was Rs. 15 per share

Required: Compute basic and diluted earnings per share and prepare a note for inclusion in the consolidated
financial statements for the year ended March 31, 2010
(17)

Question 4:
The following information relates to Que Limited (QL) for the year ended 31 December 2011:
i. Issued share capital on 1 January 2011 consisted of 80 million ordinary shares of Rs. 10 each.
ii. Profit after tax amounted to Rs. 130 million. It includes a loss after tax from a discontinued operation, amounting
Rs. 40 million.
iii. On 30 September 2011, QL issued 20% right shares at a price of Rs. 11 per share. The market value of the share
immediately before the right issue was Rs. 12.50 per share.
iv. There are 25,000 share options in existence. Each option allows the holder to acquire 120 shares at a strike price
of Rs. 10 per share. The options have already vested and will expire on 30 June 2013. The average market price
of ordinary shares in 2011 was Rs. 12 per share.
v. QL had issued debentures in 2008 which are convertible into 6 million ordinary shares. The debentures shall be
redeemed on 31 December 2012. The conversion option is exercisable during the last six months prior to
redemption. The interest on debentures for the year 2011 amounted to Rs. 11 million.
vi. Preference shares issued in 2009 are convertible (at the option of the preference shareholders) into 4 million
ordinary shares on 31 December 2013. The dividend paid on preference shares during 2011 amounted to Rs.
5.75 million.
vii. The company is subject to income tax at the rate of 35%.

Required: Prepare extracts from the financial statements of Que Limited for the year ended 31 December 2011
showing all necessary disclosures related to earnings per share. (Ignore comparative figures).
(16)

280
Page 2 of 2
Daffodils Limited
(same question as in IAS – 8)
Basic earning per share :
Restated
2017 2016
Earnings 660.25 331.67
No. of shares 291.86(W-1) 255.04(W-2)
2.26 per share 1.30 per share

2017 2017 2106


Actual Current Prior
(weighted) (Adjusted)
b/d 1.1.2017 200 200 186.67(w.2)

1.4.2017 issue for no 20 20 18.67


value (10%) (20/200 X200) (20/200 X 186.67)
220 220 205.34
1.7.2017 issue for 30 15 0
value (right (30 x 6/12)
shares)(W-3)
250 235 205.34

1.7.2017 issue for no 20 18.85 16.43


value (20/250 X235) (20/250 X 205.34)
270 253.8 221.77
1.9.2017 issue for no 40.5 38.07 33.27
value (15%) (40.5/270 X 253.8) (40.5/270 X221.77)
310.5 291.86 255.04
(W-1) Weighted Average number of shares :

(W-2)
2016 2016
Actual Current weighted)
b/d 1.1.2016 160 160

1.5 issue for value (right shares but 40 26.67


no bonus element) (40×8/12)
200 186.67

(W-3) 1.7.2017
50 × 15/25 =30 [for value]
50-30 =20 [for no value]

281
Page 1 of 6
Answer 1:
AAZ Limited
Earning per share for the year end 31.12.2017
NOTE: Preference shares are assumed to be irredeemable as there is no other information .
‘’Rs in Millions’’
Basic earning per share:
127.83 – 2.45
85.22
125.38
=1.47 per share
85.22
Dilutive Earning per share (W.1)
127.83
=1.43
89.37
AAZ Limited
Notes to Financial statements
For the year ended 31.12.2007
Reconciliation of earning used for basic EPS to diluted EPS :
Earnings used in basic EPS 125.38
Preference dividend 2.45
Earning used in diluted EPS 127.83

Reconciliation of weighted Average number of shares used for basic EPS to dilute EPS :
Weighted Average number of shares used for basic EPS 85.22
Shares option 0.15
Irredeemable preference shares 4.00
Weighted Avg numbers of shares for dilute EPS 89.37

(W-1) Multiple potential ordinary shares :


Step 1 : Ranking
Incremental Earnings/Incremental ordinary shares
1. Debentures
7.5×70% 5.25
; = 1.75 Anti diluted
3 3
2. Convertible preference Shares
2.45
= 0.61 Dilutive
4

3. Options
0
= 0 dilutive
0.15∗

*[1.5 - (1.5 × 9.9/11)]


=0.15 [for no value]

Ranking:
1. Option
2. Convertible preference shares
3. Debentures

282
Page 2 of 6
Step 2 : Testing which combination is most dilutive
125.38
Basic earning per shares = 1.47
85.22
1. Option [125.38+0/85.22+0.15] ; [125.38/85.37] = 1.469 Dilutive
2. Option & Convertible preference Shares [125.38+0+2.45/85.22+0.15+4] ;
[127.83/89.37] = 1.4303 Dilutive

3.Options , preference shares & debenture [125.38+0+2.45+5.25/85.22+0.15+4+3] ;


[133.08/92.37] =1.4407 Anti Dilutive

Answer 2. Afridi Industries limited


Extracts of statement of Comprehensive Income For the year ended 31.12 2008
“Rs in Millions”

Continuing Operations:
Profit After tax (78+13) 91
Discontinuing Operations:
Profit After tax (13)
Total profit After tax 78
Earnings per share:
Basic EPS:
Continued Operations 1.86
Discontinued Operations (0.27)
overall 1.59
Dilutive EPS:
Continued Operations 1.78
Discontinued Operations (0.21)
Overall 1.57

Workings:
Basic EPS:
Continued Operations (91/49) (W-1 =1.86
Discontinued Operations (13)/49 =(0.27)
Diluted EPS:
Continued Operations (108.33/60.67) (W-2) =1.78
Discontinued Operations (13)/60.67 = (0.21)
(W-1)
Actual Current(weighted)
b/d 40 40
1.4 For value 12 9(12×9/12)
52 49

(W-2)
Only one potential Ordinary Share:
Incremental Earnings
Term Finance Certificate =
Incremental Ordinary Shares

283
Page 3 of 6
10
4×100×8%× ×65% 17.33
12
= 4 10 ; = 1.49 Dilutive
×35× 11.67
10 12
Diluted EPS:
91
Basics EPS (from Continued Operations) = 1.86
49
[91+17.33] 108.33
Term Finance Certificate ; = 1.79(Dilutive)
[49+11.67] 60.67

Answer 3.
ABC limited
Basic and diluted EPS(W-4)
Earnings Attributable to Ordinary Shareholders
Weighted Avg number of shares outstanding
8600 (W−1)
= 0.67 per share
12,774 (w 2)
(W-1)
Profit After tax 13,000
Less: Cumulative preference dividend (2,000)
Less: Non-cumulative. preference dividend (2,400)
Profit Attributable to ordinary shareholders 8,600

(W-2) Weighted Avg. Number of shares:

Actual Current (Weighted)


b/d 10,000 10,000
1.10 For Value(W) 1,104 552
(1,100×6/12)
11,104 10,552
1.10 For No Value 96 91
(96/11,104 × 10552)
11,200 10,643
1.10 For No Value (20%) 2,240 2,129
(2,240/11,200×10,643)
13,440 12,772
1,200×11.5/12.5 = 1,104 [ For Value]
1,200-1,104 = 96 [For No Value]

(W-3) Convertible Non-Cumulative preference shares:


Preference dividend 2,400
Preference dividend rate 12%
Value of preference shares (2,400/12%) 20,000
No. of preference shares (20,000 ÷ 10) 2,000

(W-4) Only one potential ordinary share:


2,400
Non- Cumulative preference shares = = 1.2 Anti-dilutive
2,000
Therefore, there is no expectation of dilution.

284
Page 4 of 6
Notes to Financial statements:
Reconciliation of earnings used for basic EPS to diluted EPS:
Earnings used for basic EPS 8,600
Add: Preference dividend on convertible preference shares (W-4) -
Earnings used for diluted EPS 8,600

Reconciliation of weighted Avg. number of shares used for basic EPS to diluted EPS:
Weighted Avg number of shares used for basic EPS 12,774
Add: Convertible preference shares (W-4) -
Weighted Avg. number of shares used for diluted EPS 12,774

Answer 4:

Que Limited
Extracts of statement of Comprehensive Income;
For the year ended 31.12.2014
“ Rs. in Million “

Continued Operation :
Profit After tax (130 + 40) 170
Discontinued Operation:
Loss After tax (40)
Total profit After tax 130
Earning per share:
Basic EPS:
From Continued Operations (164.25 ÷ 85.22) 1.93
From discontinued Operations ((40) ÷ 85.22) (0.46)
Overall Basic EPS 1.47
Diluted EPS :
From Continued Operations (177.15 ÷ 95.72) 1.85
From discontinued Operations ( (40) ÷ 95.72) (0.42)
Overall Diluted EPS 1.43

Reconciliation of earning used for basic EPS Continued Discontinued


to diluted EPS: Operation Operation

Earning used for basic EPS 164.25 (40)


Finance cost on debenture ( net of tax ) 7.15 -

Preference dividend 5.75 -


Earning used for diluted EPS 177.15 (40)

285
Page 5 of 6
Reconciliation of Weight Avg number of shares used for basic EPS to diluted EPS :

Weighted Avg number of shares used for basic EPS 85.22


Add : Share option 0.5
Add : Convertible debenture 6.0
Add : Preference dividend 4.0
Weighted Avg number of shares used for diluted EPS 95.72

Workings :

(W-1) Earnings for basic EPS


Continued Operations (170 – 5.75) 164.25
Discontinued Operations ( Given) (40)

(W-2)

Actual Current (weighted)


b/d 80 80
30.9 Issue for value (w) 14.08 3.52 (14.08 x 3/12)
94.08 83.52
30.9 Issue for No value 1.92 1.7 (1.92/94.08 x83.52)
96.0 85.22

80 x 20% = 16 x 11/12.5 = 14.08 (For Value)


(16-14.08) = 1.92 (For No Value)

Multiple Potential Shares :


Ranking: 𝑹𝒂𝒏𝒌𝒊𝒏𝒈
0
1.Options = 0 Dilutive (1)
0.5∗
10
*25,000 x 120= 3,000,000 x 2500,000 (For Value)
12
3,000,000 – 2,500,000 = 500,000 (For No Value) (0.5 in Million )
11 × 65% 7.15
Debentures ; =1.19 Dilutive (2)
6 6
5.75
Preference Shares = 1.44 Dilutive (3)
4

Testing :
164.25
Basic EPS = 1.93 per share
85.22

164.25+0 164.25
1. Options ; = 1.916 Dilutive
85.22+ 0.5 85.72

164.25+0+7.15 171.4
2. Options and Debentures ; =1.869 Dilutive
85.22+0.5+6 91.72

164.25+0+7.15+5.75 177.15
3. Options, debentures and Preference shares ; = 1.851 Dilutive
85.22+0.5+6+4 95.72

286
Page 6 of 6
Rectification of Errors
Types of errors:
1) Those errors that do not cause a difference In trail balance agreement.
2) Those errors that do cause a difference in trail balance.

Main Heads of accounts [types of accounts]


• Assets
• Liabilities
• Income
• Expenses
• Capital

1 Errors that do not cause any difference in trail balance include:

Sr # Reasons of errors Rectifying entry


Error of omission:
When financial information completely omitted for recording in the Debtors Account 1,000
books of original entry e.g. Sale invoice of Rs. 1000 issued before Sales Account 1,000
1
closing date but not recorded In the books of original entry (Sales
Journal)

Errors of Commission:
When correct accounting effect (Dr/Cr) is given in the wrong Computer a/c 5,000
accounting head but the main head [means type of account] remains Furniture a/c 5,000
2 correct E.g. Purchase of computer Rs. 5,000 for office was wrongly
debited to the furniture account (Both accounts relate to the assets
main head)

Error of Principle:
When correct accounting effect (Dr/Cr) is given in the wrong Computer a/c 5,000
accounting head as well as the wrong main head [means type of Salary a/c 5,000
account]. E.g. Purchase of computer Rs. 5,000 for office was wrongly
3
debited to the salary account, (both accounts belong to the different
main head like computer Is Asset and salary is Expense).
This error will also cause a different in financial performance and
financial position (Asset = Owner’s equity + Liability).

Error of Original entry (transposition error):


When the correct accounting entry is recorded in the books of Debtors a/c 270
accounts but the amount in both accounting effects is wrong although Bad debt a/c 270
the amount is same: E.g. Debtors to be written off for Rs. 250 as bad
4
debts were recorded in the correct accounts but the amount was
posted in both accounts was Rs. 520 causing a different of Rs. 270 on
both sides. It Is also known as transposition error

287
Page 1 of 31
Compensating Error:
When sum of more than One errors contra the accounting effect with Capital A/c 200
each other. E.g. Sales were less credited with Rs.260 and at the same Rent Expenses 60
time opening balance of capital account was brought forward with an Sales 260
5
amount that is Rs. 200 more than the correct amount and Rent
Expense owing of Rs.60 was although credited to rent payable account
but was not debited to the rent expense a/c.

Complete reversal of entry


Means correct accounts and amounts but sides (debit and credits) Cash 200,000
6
reversed. For example; obtained a loan of Rs. 100,000 from bank Loan 200,000
recorded as a reverse entry

Possible reasons for Difference in Trial Balance


When sum of debit column of trial balance exceeds sum of credit column the difference named as
suspense account will appear in credit column of trial balance and vice versa (unless errors are
corrected).

These types of errors are:


1. Casting error:
Incorrect addition in any account, i.e. under casting or overcasting.
2. Balancing error:
Opening balance not brought down or brought down on wrong side or with wrong amount.
3. Extraction error:
The ledger balance omitted or placed in trial balance at wrong side or with incorrect amount.
4. Posting error:
Part of the transaction not posted or transaction posted with incorrect amount or posting to
wrong side of the account.
5. Error of part omission:
A debit entry has been made but no corresponding credit entry or vice versa i.e. single sided
entry.
6. Error of different amounts:
Debit and credit entries have been made but at different amounts.

Sr# Reasons of errors Rectifying entry


Under/over casting of a ledger a/c maintained in main
ledger e.g.
• Sales income account was over casted by Rs. 200 Sales A/c 200
1
• Sales Income Account Was under Casted by Rs. Suspense a/c 200
500 Suspense a/c 500
Sales a/c 500
Omission of a balance from trail balance e.g. balance of a
bad debts account Rs. 700 is not appearing in the trail Bad debt a/c 700
2
balance. Suspense a/c 700

288
Page 2 of 31
Balance representing an account appearing in trail
balance with less or excess amount e.g. Building a/c Building a/c 630,000
3
balance c/f Rs. 700,000 wrongly appearing in the Trail Suspense a/c 630,000
balance with Rs. 70,000.

An account was given debit effect instead of credit effect Suspense a/c 1600
(causing difference with double amounts) e.g Sales of Rs. Sales a/c 1600
4
800 on credit was correctly debited in debtors a/c but
was also debited in sale a/c mistakenly
An account was given credit effect instead of debit effect
(causing difference with double amounts) e.g Sales of Rs. Debtor a/c 1,600
5
800 on credit was correctly credited in sales a/c but was Suspense a/c 1,600
also credited in debtors a/c mistakenly
Single accounting effect either Dr or Cr was recorded
/posted in the books of account e.g. sales of Rs.800 on Debtor a/c 800
6
credit was posted in sales a/c only. Suspense a/c 800

One of the accounting effects was given with wrong Debtor a/c 720
amount e.g. sales of Rs. 800 on credit was correctly Suspense a/c 720
7
credited in sales a/c but was wrongly debited to debtors
a/c with Rs.80 only.

CORRECTION OF ERRORS

1. GRANT
The accountant of Grant Company has prepared a trial balance, but has found that the total of debit
balances is Rs.864,600 and the total of credit balances is Rs.862,150.
On investigation, he discovers the following errors in the book-keeping:
(1) Total purchases in the period were recorded at Rs.100 below their correct value, although the
total value of trade payables was correctly recorded.
(2) Total telephone expenses were recorded at Rs.1,000 above their correct amount, although
the total value of the amounts payable was correctly recorded.
(3) Purchase returns of Rs.550 were recorded as a debit entry in the sales returns account, but
the correct entry had been made in the trade payables control account.
(4) Equipment costing Rs.2,000 had been recorded as a debit entry in the repairs and
maintenance account.
(5) Rental expenses of Rs.5,490 were entered incorrectly as Rs.5,940 in the expense account but
were entered correctly in bank account in the ledger.
(6) Bank charges of Rs.200 have been omitted entirely from the ledger.

Required:
Prepare journal entries for the correction of the errors.

Open a suspense account. Record the appropriate corrections in the suspense account, so that the
balance on this account is eliminated.
2. EASTERN PRODUCTS
289
Page 3 of 31
The following errors and adjustments were discovered:

(i) An invoice of Rs. 3,700 was debited to purchases but the goods were received after
year-end and were not included in the closing inventory.
(ii) Store equipment costing Rs. 8,100 and having a book value of Rs. 3,600 was sold for Rs.
2,500. Cash was debited and store equipment was credited by Rs. 2,500. No other
entries were made.
(iii) A cheque of Rs. 1,850 received from a customer was dishonoured on June 25, 2013 but
no entry was made in the books. Cash there against was received after year-end.
(iv) Purchase of office equipment costing Rs. 15,200 was entered in the purchases account.
Depreciation on office equipment is provided at the rate of 10%.
(v) A purchase invoice of Rs. 197 was debited to the supplier account as Rs. 917.
(vi) Purchase returns book was under-casted by Rs. 650.
(vii) The opening balance of furniture account was brought forward as Rs. 18,300 instead of
Rs.13,800. Depreciation on furniture is provided at the rate of 10%.
(viii) A balance of Rs.730 in the debtor account is to be offset against a balance of Rs. 880 in
the creditor account.

The profit for the year was calculated at Rs. 956,180 before the correction of above errors..

Required:
(a) Prepare journal entries to adjust the above items.
(b) Recalculate the net profit for the year.

3. CND

The bookkeeper has prepared a preliminary trial balance of CND for the year ended 31
December as follows.

Rs. Rs.
Capital account 110,000
Accumulated profit at 1 January 50,000
Bank loan 30,458
Trade receivables and payables 77,240 60,260
Cash in hand and bank overdraft 1,000 5,036
Inventories at 1 January 108,000
Non-current assets at cost and accumulated
depreciation at 31 December 161,879 60,943
Depreciation for the year 15,000
Purchases and revenues 300,297 402,000
Returns 4,370 4,630
Discounts allowed and received 9,760 6,740
Wages and salaries 22,000
Rent, rates and insurance 18,036
Postage, telephone and stationery 3,009

290
Page 4 of 31
Repairs and maintenance 2,124
Advertising 4,876
Packing materials 924
Motor expenses 2,000
Sundry expenses 1,000
Loan interest 4,000
Accrued expenses 6,478
Suspense account 1,030
736,545 736,545

When the bookkeeper discovered that the preliminary trial balance did not balance he
made it do so by opening a suspense account and entering the required amount on the
appropriate side. A subsequent investigation shows the following mistakes have been
made.

(1) A loan to the business of Rs.10,000 from the owner’s brother, X, has been added to
capital.
(2) Accrued interest on the bank loan of Rs.458 has been credited to the bank loan
account instead of being treated as a current liability.
(3) Bank charges of Rs.1,000 have been completely omitted from the books.
(4) In addition to allowing discount of Rs.240 and receiving discount of Rs.260, various
customers’ and suppliers’ accounts amounting to Rs.10,000 were set off by contra.
No entries whatever have been made in respect of these items.
(5) Trade receivables amounting to Rs.2,000 are bad and need to be written off.
(6) A debt of Rs.1,000 written off as bad in a previous year has been recovered in full.
The amount has been credited to the trade receivables ledger control account.
(7) Goods returned from a customer of Rs.630 have been correctly entered into the
receivable ledger control account, but by mistake were entered in the returns
outwards journal.
(8) A payment for stationery of Rs.234 was correctly entered in the cash book but
debited in the ledger as Rs.243.
(9) A payment of Rs.76 for packing materials has been correctly entered in the cash
book, but no other entry has been made.
(10) A payment of Rs.124 for advertising has been debited to repairs and maintenance.
(11) A cheque payment of Rs.26 for insurance has been recorded in all accounts as
Rs.62.
(12) A page in the purchase account correctly totalled Rs.125,124 was carried forward
to the top of the next page as Rs.125,421.
All entries other than those given above are to be assumed to have been made correctly.

Required:
(a) Show the correcting entries in journal form (i.e. showing accounts and amounts debited and
credited but no supporting narrative is required) in respect of each of the mistakes
mentioned above.
(b) Show the trial balance of the company at 31 December after these corrections have been
made. A working showing how the suspense account is cleared should be included.

291
Page 5 of 31
4. MR. FAWWAD

Mr. Fawwad owns a factory and closes his books on June 30. The trial balance prepared
by him, contained a difference which he kept in a suspense account. On scrutinising the
records, the following errors were detected:
a. A cheque of Rs. 10,800 was paid to a creditor who allowed 10% cash discount. The
payment was correctly entered in the bank book but was posted to purchase
account as Rs. 1,080 only. No other entry was made.
b. Sundry receivables include an amount of Rs. 15,000 which had proved
irrecoverable but was not written off. According to a consistent policy, a reserve
for bad debt was created @ 5% on closing receivables;
c. Commission of Rs. 3,500 was paid but was debited twice, once in the party’s
account and again in the commission account;
d. Purchases of Rs. 4,500 were entered as sales in the Sales Day Book. Rs. 600
collected from a party in respect of dues which had been written off as bad two
years ago, was credited to the receivables control account.
e. Goods invoiced at Rs. 4,600 were returned by a debtor. These were entered in the
purchase book and posted from there to debtor’s account as Rs. 6,400.
f. The discount allowed column in the cash book was short casted by Rs. 1,500.
g. A cash sale of Rs. 7,300 to Mr. Anwar was correctly entered in the cash book but
was posted to the credit of Mr. Anwar’s account
h. An amount of Rs. 17,400 was received in full and final settlement from a customer
after he was allowed a discount of Rs. 2,600. However, while writing the books,
the amount received was entered in the discount allowed column of the bank
book and the discount allowed was entered in the bank column.

Required:
Pass rectification entries (without narration) to correct the above errors.

5. MR REHAN
While closing his books on 30 June 2013, Mr. Rehan identified a difference in the trial
balance which he kept in a suspense account. He prepared his P & L account on the basis
of this trial balance and arrived at a profit of Rs. 679,000. While trying to reconcile the
trial balance he detected the following errors:
a. A cheque of Rs. 25,000 received from the insurance company in respect of loss of
inventory has been paid into the proprietor’s personal bank account and has not
been recorded in the books. No entry has been passed in respect of the loss.
b. Bill received from ABC Furnishings on 1 July 2012 for repairs to furniture Rs. 3,000
and for new furniture supplied Rs. 10,000 was entered in the purchase day book as
Rs. 11,000. Depreciation on furniture is provided @ 10 % per annum.
c. Goods worth Rs. 10,200 purchased from a creditor on 28 June 2013 had been
entered in the Purchase Day Book and credited to him but were not delivered till 5
July 2013. However, the title of the goods had passed on 28 June 2013.

d. A computer bought originally for Rs. 70,000 four years ago and depreciated to Rs.

292
Page 6 of 31
12,000 had been sold for Rs. 15,000 on the first day of the year. The amount
deposited was entered in the bank book but no other entry was passed.
e. Goods valuing Rs. 13,000 were returned by Zahid. These were entered in the
Purchase Day Book and posted to a supplier’s account as Rs. 31,000.
f. Discount of Rs. 3,700 was allowed but posted to the credit of discount received a/c
as Rs. 7,300.
g. A cheque of Rs. 10,800 was paid to a creditor who allowed 10% cash discount, but
the payment was wrongly posted to purchase account as Rs. 1,080 only without
any other entry.

Required:
(a) Pass rectification entries (without narration) to correct the above errors.
(b) Recalculate the profits after taking into account the above corrections.

6. AYUB BROTHERS

The trial balance of Ayub Brothers did not agree as at 31 December 2013 and the
difference was carried to a suspense account. On scrutinising the books of account, the
following types of errors were detected:
a. Receivables include Rs. 15,000 which are irrecoverable and need to be written off.
b. Goods invoiced at Rs. 4,600 were returned by a customer. It was entered in the
purchase book and posted from there to a creditor’s account as Rs. 6,400.
c. A cheque of Rs. 8,000 received from a customer was not posted to his ledger
account. Moreover, the corresponding sales invoice for Rs. 12,000 was incorrectly
passed through the sales day book as Rs. 2,000.
d. An amount of Rs. 3,800 owed by Zahid & Company for goods supplied was to be
adjusted against an amount of Rs. 8,500 owed to Zahid & Company. No entry has
been made in this regard.
e. A purchase of Rs. 15,100 was entered in the purchase day book as Rs. 1,500 and
posted to the supplier’s account as Rs. 5,100.
f. Goods invoiced at Rs. 23,000 and returned by Hamid Khan, a debtor, were entered
in the purchase day book and posted therefrom to Hammad Khan, a creditor, as
Rs. 32,000.
g. A supplier’s invoice for Rs. 12,300 had been entered in the purchase day book on
28 December 2013. However, the goods were received on 2 January 2014.
h. Ayub Brothers maintains a allowance of 5% of the gross amount of receivables.

Required:
Prepare journal entries to rectify the errors identified above. (Narrations are not
required.)

Answers:
1. GRANT

Transaction Debit Credit

293
Page 7 of 31
Rs. Rs.
1 Purchases 100
Suspense account 100
Correction of error: purchases under-stated by Rs.100.
2 Suspense account 1,000
Telephone expenses 1,000
Correction of error: telephone expenses over-stated by Rs.1,000.
3 Suspense account 1,100
Purchase returns 550
Sales returns 550
Correction of error. Purchase returns of Rs.550 incorrectly recorded as a debit entry in sales returns.

4 Equipment 2,000
Repairs and maintenance 2,000
Correction of error. Equipment purchase costs incorrectly recorded as repairs and maintenance
expenses

5 Suspense account 450


Rent expenses 450
Correction of error: rent expenses over-stated by Rs.450.

6 Bank Charges 200


Bank account 200

Transaction omitted from the ledger.


Suspense account
Rs. Rs.
Telephone expenses 1,000 Opening balance 2,450
Purchase returns 550 (864,600 – 862,150)
Sales returns 550 Purchases 100
Rent expenses 450

2,550 2,550

294
Page 8 of 31
2. EASTERN PRODUCTS

(a) Debit Credit


1 Creditor account 3,700
Purchases 3,700
2 Accumulated Depreciation 4,500
Loss on sale of store equipment 1,100
Store equipment a/c 5,600
3 Debtors a/c 1,850
Bank 1,850
4 Office Equipment 15,200
Purchases 15,200
Depreciation expense 1,520
Accumulated depreciation – office
equipment 1,520

5 Suspense a/c 1,114


Creditor a/c (197 + 917) 1,114
6 Creditors 650
Purchase Returns 650
7 Suspense a/c 4,500
Furniture a/c 4,500
Accumulated Depreciation 450
Depreciation expenses 450
8 Creditor a/c 730
Debtors a/c 730
(b) Recalculation of net profit for the year
Net profit as per question 956,180

Add: Reversal of purchases 3,700


Equipment wrongly debited to purchases 15,200
Purchase returns 650
Reversal of depreciation on furniture 450
976,180
Less: loss on sale of store equipment (1,100)
Depreciation on office equipment (1,520)
Adjusted net-profit 973,560

3. CND
(a) Correcting entries Dr Cr
Rs. Rs.
(1) Capital a/c 10,000
Loan a/c X 10,000
(2) Bank loan a/c 458

295
Page 9 of 31
Accrued expenses a/c 458
(3) Bank charges a/c 1,000
Bank overdraft a/c 1,000
(4) Trade payables a/c 10,260
Discounts allowed a/c 240
Trade receivables a/c 10,240
Discount received a/c 260
(5) Bad debts a/c 2,000
Trade receivables a/c 2,000
(6) Trade receivables a/c 1,000
Bad debts a/c 1,000
(7) Sales returns a/c 630
Purchases returns a/c 630
Suspense a/c 1,260
(8) Suspense a/c 9
Postage, telephone and stationery a/c 9
(9) Packing materials a/c 76
Suspense a/c 76
(10) Advertising a/c 124
Repairs and maintenance a/c 124
(11) Bank overdraft a/c (62-26) 36
Insurance a/c 36
(12) Suspense a/c (125,421 – 125,124) 297
Purchases a/c 297

(b) Trial balance at 31 December

Dr Cr
Rs. Rs.
Capital account Rs.(110 – 10) 100,000
Accumulated profits at 1 January 50,000
Bank loan Rs.(30,458 – 458) 30,000
Loan account – X 10,000
Trade receivablesRs. (77,240 – 10,240 – 2,000 + 1,000) 66,000
Trade payables Rs.(60,260 – 10,260) 50,000
Cash in hand 1,000
Bank overdraft Rs.(5,036 + 1,000 – 36) 6,000
Inventories at 1 January 108,000
Non-current assets at cost 161,879
Accumulated depreciation at 31 December 60,943
Depreciation for the year 15,000
Purchases Rs.(300,297 – 297) 300,000
Revenues 402,000
Sales Returns (4,370 +630) 5,000

296
Page 10 of 31
Purchase Returns(4,630 -630) 4,000
Discounts allowed Rs.(9,760 + 240) 10,000
Discounts received Rs.(6,740 + 260) 7,000
Wages and salaries 22,000
Rent, rates and insurance Rs.(18,036 – 36) 18,000
Postage, telephone and stationery Rs.(3,009 – 9) 3,000
Repairs and maintenance Rs.(2,124 – 124) 2,000
Advertising Rs.(4,876 + 124) 5,000
Packing materials Rs.(924 + 76) 1,000
Motor expenses 2,000
Sundry expenses 1,000
Loan interest 4,000
Bank charges 1,000
Bad debts (2,000 – 1,000) 1,000
Accrued expenses 6,478
Interest payable 458
726,879 726,879

Working

Suspense a/c
Rs. Rs.
Original balance 1,030 (7) Goods returned
(8) Stationery transposition mis posting 1,260
Error 9 (10) Materials payment
(12) Purchases transposition omitted 76
Error 297
1,336 1,336

4. MR. FAWWAD
Journal Entries
Debit Credit
Rs. Rs.
1. Creditor A/c (10,800+1,200) 12,000
Purchases A/c 1,080
Suspense A/c 9,720
Discount received (purchases) 1,200
2. Bad Debts expenses 15,000
Debtors A/c 15,000
Allowance for doubtful debts A/c 750
Bad debt expense account 750
3. Suspense A/c 3,500
Parties A/c 3,500
4. Sales A/c 4,500

297
Page 11 of 31
Debtors A/c 4,500
Purchase A/c 4,500
Payables A/c 4,500
5. Debtors (receivables control account) 600
Other income or bad debt expense A/c 600

6. Sales return A/c 4,600


Purchase A/c 4,600
Debtors A/c 1,800
Suspense A/c 1,800
7. Discount allowed(sales) 1,500
Debtors account 1,500
8. Mr. Anwar 7,300
Cash sales 7,300
9. Bank (17,400 – 2,600) 14,800
Discount allowed(sales) 14,800

5. MR REHAN

(a) S. Particulars Debit Credit


No. (Rs.) (Rs.)
(i) Drawings A/c 25,000
Purchases 25,000
(ii) Repairs 3,000
Furniture and Fittings 10,000
Purchases 11,000
ABC Furnishings 2,000
Depreciation 1,000
Acc. dep. on furniture 1,000
(iv) Goods-in-transit 10,200
Purchases 10,200
(v) Suspense Account 15,000
Accumulated depreciation - computers 58,000
Non-current assets 70,000
Gain on disposal of computers 3,000
(vi) Returns Inward 13,000
Payables 31,000
Purchases 13,000
Debtors 13,000
Suspense Account 18,000

(vii) Discount Allowed (sales) 3,700


Discount Received (purchases) 7,300
Suspense Account 11,000

298
Page 12 of 31
(viii) Creditor A/c 12,000
Suspense A/c 1,080
Purchases 1,080
Discount
received(purchases)
(Purchases) 1,200
Bank 10,800

Amount in Rupees
(b) Recalculation of Profit and Loss

(ii) Repair exp Unadjusted


3,000 Profit 679,000
(ii) Dep. on furniture 10% of 1,000 (i) Drawings a/c
10,000 (receipt from
ins co.) 25,000
(ii) Purchases
11,000

(v) Goods-in-
Transit 10,200
(vi) Gain on sale
of asset 3,000
(vii) Return inwards
13,000
(viii) Discount (vii) Purchases
allowed/received (3,700+7,300) 11,000 13,000
Adjusted profit for the (ix) Purchases
Year 715,480 1,080
(ix) Discount
Received 1,200

715,480 715,480

299
Page 13 of 31
6. AYUB BROTHERS
Dr. Cr.
1. Bad debts accounts 15,000
Debtors A/c 15,000
2. Sales return A/c 4,600
Payables 6,400
Purchase A/c 4,600
Suspense A/c 1,800
Debtors A/c 4,600
3. Suspense A/c 8,000
Debtor A/c 8,000
Debtor A/c 10,000
Sales 10,000
4. Payables A/c – Zahid & Co. 3,800
Debtors A/c – Zahid & Co. 3,800
5. Purchases A/c (15,100-1,500) 13,600
Payables A/c (15,100-5,100) 10,000
Suspense A/c 3,600

6. Returns Inwards/Sales returns 23,000


Payables A/c 32,000
Purchase A/c 23,000
Debtors A/c 23,000
Suspense A/c 9,000
7. Creditors 12,300
Purchases 12,300
8. Allowance for doubtful debts 2,220
Bad Debt Expenses 2,220
(-15,000-4,600-8,000+10,000-3,800-23,000)= -
44,400 × 5%

Extra practice questions


Q1. Draft income statement of Timothy Enterprises (TE) for the year ended 31 December 2017 shows gross
profit of Rs. 850,000 and net profit of Rs. 460,000.

During the review of the financial statements, following errors were noticed:
(i) An invoice of Rs. 3,700 was debited to purchases but the goods were received after year-end and
were not included in the closing inventory.
(ii) Transportation inward amounting to Rs. 2,000 was included in transportation outward.
(iii) The sub-total of a closing stock sheet had been carried forward as Rs. 21,830 instead of Rs. 21,380.
(iv) A receipt of Rs. 21,850 was credited to sales. The amount was received from a debtor as full and
final settlement of an outstanding balance of Rs. 23,000.
(v) Goods having sales value of Rs. 4,500 were used for office repairs. No entry has been made in the
books.

300
Page 14 of 31
(vi) Purchase of office computer on 1 April 2017 amounting to Rs. 42,000 was entered in the purchase
account.
(vii) An item of furniture was sold on credit for Rs. 3,000 and entered in the sales day book. The book
value of this item was Rs. 5,000.
(viii) Purchase return amounting to Rs. 6,700 has been recorded as sales return.
(ix) The owner had withdrawn goods costing Rs. 4,680 for personal use. No entry has been made in the
books.

TE uses periodic inventory method. Goods are sold at cost plus mark up of 25%.
Depreciation on office computer is provided at the rate of 25%.

Required:
Compute the corrected gross profit and net profit for the year. (14)
Ans.
Effect on Gross Profit
Given Profit 850,000 Cr
Purchases 3,700 Cr
Carriage Inwards 2,000 Dr
Cost of sales 450 Dr
Sales 21,850 Dr
Discount Allowed 1,150 Dr
Purchases 3,600 Cr
Purchases 42,000 Cr
Sales 3,000 Dr
Purchase return 6,700 Cr
Sale return 6,700 Cr
Purchases 4,680 Cr
Adjusted Gross Profit 888,930 Cr

Effect on Net Profit


Given Profit 460,000 Cr
Effect of change in gross profit [850,000 – 38,930 Cr
888,930]
Carriage Outward 2,000 Cr
Office Repairs 3,600 Dr
Depreciation 7,875 Dr
Loss 2,000 Dr
Adjusted Net Profit 487,455 Cr

Note:
Increase in closing stock increases the Gross Profit because of decrease in cost of sale while decrease in closing
stock decrease the gross profit because of increase in cost of sale.

301
Page 15 of 31
Workings:
1 Creditor 3,700
Purchases 3,700
2 Carriage Inwards 2,000
Carriage outwards 2,000
3 Cost of sales 450
Stock 450
4 Sales 21,850
Discount Allowed(sales) 1,150
Debtor 23,000
5 Office Repairs 3,600
Purchases 3,600
6 Office Computer 42,000
Purchases 42,000
Depreciation 7,875
Acc. Depreciation 7,875
7 Posted:
Debtor 3,000
Sale 3,000
Required:
Receivable 3,000
Loss 2,000
Furniture 5,000(WDV)
Rectifying:
Sale 3,000
Debtor 3,000
Receivable 3,000
Loss 2,000
Furniture 5,000(WDV)

8 Suspense 13,400
Purchase return 6,700
Sale Return 6,700
9 Drawings 4,680
Purchases 4,680

302
Page 16 of 31
Q.2 Financial statements of Zeta Traders (ZT) for the year ended 30 June 2019 is under preparation. Following
information has been gathered in this respect:

(i) Trade receivables as at 30 June 2019:

Rupees
Trade receivables 2,500,000
Provision for doubtful debts (400,000)
Net trade receivables 2,100,000

It was noted that:


▪ an old outstanding balance of Rs. 250,000 which was written off previously was settled during the year at 20%
discount. The amount received was credited to trade receivables.
▪ purchase return amounting to Rs. 500,000 was mistakenly debited to trade receivables instead of trade
payables.
▪ Rana and Sons having a balance of Rs. 80,000 due for more than one year was declared bankrupt and its
balance needs to be written off.

ZT maintains provision for doubtful debts:


▪ at 25% for balances outstanding for more than six months. As at 30 June 2019, such balances are aggregated
to Rs. 600,000 (excluding balance of Rana and Sons); and
▪ at 5% for the remaining balances

(ii) A cheque dated 25 June 2019 for Rs. 150,000 was received from an insurance company and deposited by the
owner in his personal bank account. The cheque was received in settlement of an inventory loss claim. Actual
inventory loss was determined at Rs. 180,000. No entries have been made for loss of inventory and insurance
claim.
(iii) The opening balance of accumulated depreciation was brought forward as Rs. 280,000 instead of Rs. 820,000.
The error was tried to be corrected with the difference by crediting accumulated depreciation and debiting
depreciation expense.
(iv) Goods amounting to Rs. 350,000 received from a supplier on 30 June 2019 were included in the year-end
physical inventory count but recorded in purchases day book on 1 July 2019.
(v) Third party stock of Rs. 500,000 lying on ZT premises has been included in ZT’s year-end inventory.
(vi) ZT uses periodic inventory method.

Required:
a) Prepare adjusting / correcting entries for the year ended 30 June 2019. (Narrations are not required)
b) Compute the net effect of the above on ZT’s profit for the year ended 30 June 2019

303
Page 17 of 31
A.2 (a) Zeta Traders
Adjusting/correcting entries
General journal
Description Debit Credit
------ Rupees ------
(i) Trade receivable (250,000 x 80%) 200,000
Bad debt 200,000
OR
Trade receivable (250,000 x 80%) 200,000
Discount allowed 50,000
Bad debts 250,000
Trade payable 500,000
Trade receivable 500,000
Bad debts 80,000
Trade receivable 80,000
Allowance for doubtful debts (W-1) 174,000
Bad debts 174,000

(ii) Abnormal loss 180,000


Purchases 180,000
Drawings 150,000
Abnormal loss 150,000
(iii) Suspense (820,000–280,000) 540,000
Depreciation expense 540,000
(iv) Purchases 350,000
Trade payables 350,000
(v) Cost of sales 500,000
Closing inventory 500,000

Debtors
Unadjusted b/d 2,500,000 Creditors 500,000
Bad debts 200,000 Bad debts 80,000
c/d 2,120,000

Allowance
Allowance 174,000 Unadjusted b/d 400,000
c/d (W.1) 226,000

W-1
1.Specific allowance
600,000 x 25% 150,000
2.Closing debtors 2,120,000
Less balance on which specific allowance is created (600,000)
Remaining Debtors 1,520,000

304
Page 18 of 31
3. General allowance @5% 76,000
4.Total allowance (150,000 + 76,000) 226,000

(b)
Increase / decrease in net profit
Recovery of old outstanding balance previously written off 200,000 Cr.
Bad debts 80,000 Dr
Bad debts 174,000 Cr
Loss of goods (180,000-150,000) 30,000 Dr
Purchases 180,000 Cr
Depreciation expense 540,000 Cr
Purchases 350,000 Dr
Cost of sales 500,000 Dr
Net effect on profit 134,000 Cr

305
Page 19 of 31
Test Questions Rectification of errors:
1.The accountant of BA Enterprises prepared a statement of comprehensive income for the year ended December
31, 2013 which showed gross profit of Rs. 1,050,000 and net profit of Rs. 650,000. The company sells goods at cost
plus mark-up of 20%.

The following errors/omissions were found on a detailed review of the financial statements.
1. Items not included in the statement of comprehensive income:
• Free samples costing Rs. 25,000 were sent to potential and regular customers.
• Goods costing Rs. 10,000 were taken by the owner for personal use and goods having sales value of Rs. 2,500
were used for office repairs.
• Unpaid salaries and transportation (inward) expenses payable, amounting to Rs. 20,000 and Rs. 10,000
respectively.
2. Old furniture items were sold on credit for Rs. 3,000 and entered in the sales day book. The bookvalue of
these items was Rs. 2,000.
3. Rs. 24,500 were paid to a creditor as full and final settlement of an amount of Rs. 25,000 and debited to
purchases.
4. The sales day book was overcast by Rs. 30,000.
5. An amount of Rs. 67,000 was carried forward in the purchase day book as Rs. 6,700.

Required:
Ascertain the correct amount of gross and net profit for the year.

Answer 1: BA Enterprises
Effect on Gross Profit
For the year ended 31-12-2013

Gross Profit ( Given ) 1,050,000 Cr.


Purchases 25,000 Cr.
Purchases 10,000 Cr.
Purchases 2,083 Cr.
Transportation Inwards 10,000 Dr.
Sales 3,000 Dr.
Purchases 24,500 Cr.
Discount received (Purchases) 500 Cr.
Sales 30,000 Dr.
Purchases 60,300 Dr.
Adjusted Gross profit 1,008,783 Cr.

306
Page 20 of 31
Effect on Gross Profit:
Given Net Profit 650,000 Cr.
Effect of change in gross profit [ 1,050,000 – 1,008,783] 41,217 Dr.
Free samples 25,000 Dr.
Office Repairs 2,083 Dr.
Salaries 20,000 Dr.
Gain on disposal 1,000 Cr.
Adjusted Net Profit 562,700 Cr.

Workings:
Dr. Cr.
1.
i) Free Samples [in operating expenses] 25,000
Purchases 25,000
ii) Drawings 10,000
Purchases 10,000
Office Repairs 2,083
Purchases 2,083
[ 2,500 / 120 x 100 ]
iii) Salaries 20,000
Payable 20,000
Transportation Inwards 10,000
Payable 10,000
2. Sales 3,000
Debtors 3,000
Receivable 3,000
Furniture 2,000
Gain (P.L) 1,000
3. Creditor 25,000
Purchases 24,500
Discount Received (purchases) 500
4.Sales 30,000
Debtor 30,000
5.Purchases 60,300
Creditor 60,300

307
Page 21 of 31
2. The bookkeeper has produced the following statement of financial position at 31 December for Smetena’s
Newsagents.

Rs. Rs.
Non-current assets 72,208
Current assets
Inventory 18,826
Trade receivables 26,216
Drawings 8,260
Suspense account 3,830
Cash 700
57,832

130,040

Capital account 50,224

Loan – L Franks 12% 20,000


Trade payables 26,782
Bank overdraft 14,634
Profit for year 18,400
130,040

Jan Smetena, the proprietor, is unhappy with the statement of financial position and asks you to revise it. You
discover the following.
1. The suspense account balance represents the difference on the trial balance.
2. The purchases day book total for October of Rs.4,130 was posted to the purchases account as Rs.4,310
although the correct entry was made to the payables ledger control account.
3. Inventory sheets were overcast by Rs.2,000.
4. Cash should be Rs.110.
5. Fixtures and fittings account balance of Rs.4,600 has been omitted from the trialbalance.
6. Interest for a half year on the loan account has not been paid and no provision hasbeen made for it.

Required:
a) Show the journal entries to correct the above errors.
b) Write up the suspense account.
c) Draw up a revised statement of financial position at 31 December. Clearly show theadjustments to profit.

308
Page 22 of 31
Answer:
Dr.bit Cr.
Suspense a/c 180
Purchases 180
(2) Correction of amount posted to purchases a/c arising from transposition error
Cost of Sales 2,000
Stock 2,000
(3) Correction of overcasting of Inventory-Sheets
Suspense a/c 590
Cash a/c 590
(4) Correction of overstatement of cash in hand
Fixtures & Fittings 4,600
Suspense a/c 4,600
(5) Correction of omission from the trial balance of fixtures & fittings
Interest a/c 1,200
Interest Payable 1,200
(6) Provision for interest due on loan not yet provided for [20,000 x 12% x 6/12]

(b) Suspense account


Rs. Rs.
DIFFERENCE IN TB 3,830 Fixtures & Fittings omitted from TB (5) 4,600
PURCHASES (2) 180
CASH IN TB (4) 590

4,600 4,600

(c) Statement of financial Position at 31 December

Rs. Rs.
Non-Current Assets (72,208+4,600) 76,808
Current Assets :
Inventory(18,826-2,000) 16,826
Trade Receivables 26,216
Cash(700-590) 110
43,152
Total assets 119,960

Opening capital 50,224


Add: Net Profit (working below) 15,380
65,604
Less: Drawings (8,260)
57,344
Non-Current Liabilities

309
Page 23 of 31
Loan – L Franks 20,000
Current Liabilities
Bank Overdraft 14,634
Trade Payables 26,782
Interest Payable 1,200

Total Capital and Liabilities 119,960

WORKING
Adjustment to Profit
Profit per draft statement of financial position 18,400
Less: Overstated closing inventory (3) (2,000)
Interest on loan a/c (6) (1,200)
Add: Overstated Purchases (2) 180
15,380

Question 3
Following draft statement of financial position as on 31 December 2019 of Naltar Establishment (NE) is under
review:

Assets Rs. in ‘000 Equity & liabilities Rs. in ‘000


Fixed assets – net 22,590 Opening capital 32,240
Current assets: Net profit for the year 9,360
Stock 15,320 41,600
Trade receivables 19,730 Current liabilities:
Drawings 1,400 Trade payables 17,332
Cash & bank 3,850 Other payables 2,680
40,300 Suspense account 798
Discount received 480
21,290
62,890 62,890

The following matters are identified:


(i) Goods costing Rs. 5,800,000 received on 31 December 2019 were included in the year-end physical count.
However, these goods were recorded in purchases day book on 5 January 2020 on receipt of the invoice.
(ii) Year-end physical count sheets include a third party stock of Rs. 1,320,000.
(iii) Goods sold on credit at a trade discount of 10% were recorded at gross amount of Rs. 6,400,000.
(iv) An unidentified credit of Rs. 294,000 appearing in the bank statement was accounted for in the suspense
account. It was discovered that the credit was a settlement of an old outstanding balance previously written
off. The amount was net of 2% bank charges.
(v) A cheque of Rs. 500,000 issued by a customer as an advance was dishonored and returned by the bank on
31 December 2019. No entry was made on return of cheque.
(vi) Operating expenses include insurance premium of Rs. 900,000 paid during the year, out of which Rs.
200,000 pertain to owner's residential premises. The policy is valid up to 30 June 2020.
(vii) An up gradation of a plant to improve quality and efficiency was completed on 1 July 2019 at a cost of

310
Page 24 of 31
Rs. 2,500,000. The cost was charged to repair and maintenance expense.
(viii) Total sales of 26 December 2019 as per sales day book was Rs. 167,000. This was posted in trade receivable
control account as Rs. 671,000. Trial balance was balanced by taking the difference to the suspense account.

Other information:
NE uses periodic stock method. The plant is depreciated at 20% using diminishing balance method.
Required:
Prepare corrected statement of financial position as on 31 December 2019. (17)

A.3 Naltar Establishment


Statement of financial position as at 31 December 2019
Rs. in '000
Assets
Fixed assets 22,590+2,500–250 24,840
Current assets:
Closing stock 15,320–1,320 14,000
Trade receivables 19,730–640–504 18,586
Prepaid Insurance 350
Cash & bank 3,850–500 3,350
36,286
61,126

Equity & liabilities


Opening capital 32,240
Net profit for the year (W-1) 5,174
Drawings 1,400+200 (1,600)
Net equity 35,814
Current liabilities:
Trade payables 17,332+5,800 23,132
Other payables 2,680–500 2,180
25,312
61,126

W-1: Corrected net profit for the year Rs. in '000


Balances as per draft financial statements (given) 9,360 Cr.
Corrections:
(i) Purchases of Dec. 2019 recorded in Jan. 2020 (5,800) Dr.
(ii) Cost of Sales (1,320) Dr.
(iii) Goods sold recorded at gross of 10% trade discount (640) Dr.
(iv) Clearance of unidentified credit taken to suspense account:
- Bank charges (6) Dr.
- Recovery of previously written off balance 300 Cr.
(vi) Insurance premium paid pertains to:
- Owner 200 Cr.
- Insurance 350 Cr.

311
Page 25 of 31
(vii) Upgradation cost of a plant charged to repair and maintenance:
- Repair and maintenance expense 2,500 Cr.
- Depreciation expense (2,500×20% x6/12) (250) Dr.
Discount received(already appearing in liabilities) 480 Cr.
Revised net profit 5,174

W-2
Purchases 5,800
Creditor 5,800
Cost of sale 1,320
Sock 1,320
Sales 640
Debtor 640
[6,400 x 10%]
Bank 294 Bank 294 Suspense 294
Bank charges 6 Suspense 294 Bank Charges 6
(294/98 x 2)
Bad debts 300 Bad debts 300
(Required) (Posted) (Rectifying)
Advance from customer 500
Bank 500
Drawings 200
Operating expense 200
350
Prepaid Insurance
Operating expense 350
[700 x 6/ 12]
Fixed assets 2,500
Repair and maintenance 2,500
Depreciation 250
Acc. Depreciation 250
(2,500 x 20% x 6/12)
Debtor 167 Debtor 671 Suspense 504
Sales 167 Sales 167 Debtor 504
Suspense 167
(Required) (Posted) (Rectifying)

Suspense a/c
Rs. Rs.
Original balance 798 1,030
Bad debts 294
Debtors 504
798 798

312
Page 26 of 31
Rectification of errors:
Q. While reviewing the draft financial statements of Sky Electronics (SE) for the year ended 31 December 2017,
following errors have been identified:

I. Computers costing Rs.240,000 purchased on 1 September 2017 for office use were debited to purchases
account. SE depreciates computers at 20% per annum using straight line method.
II. Furniture costing Rs. 1,200,000 and having a book value of Rs. 670,000 as on 31 December 2017 had
already been sold on 1 November 2017. The proceeds of Rs. 700,000 were credited to sales. SE
depreciates furniture at 10% per annum using straight line method.
III. On 1 April 2017, SE rented-out one of its premises at an annual rent of Rs. 900,000 payable in advance.
The rent received was credited to income.
IV. Trade receivables include a balance of Rs. 180,000 which is irrecoverable but has not been written-off.
Further, a recovery of Rs. 96,000 against receivables written off in prior years was credited to trade
receivables. As per SE's policy, provision for doubtful receivables has already been made at 5% on year-
end balance.
V. A cheque of Rs. 192,000 was received after a discount of 4% from a customer. However, in the cash book,
the amount received was entered in the discount allowed column and the amount of discount was
entered in the bank column.

Required:
a. Prepare rectification entries to correct the above errors. (Narrations are not required)
b. Post the effect of rectification entries in accounting equation

313
Page 27 of 31
A. (a)
Sky Electronics

Debit Credit
Date Description
Rupees

(i) Fixed assets (Computers) 240,000


Purchases 240,000
Depreciation expense [240,000×0.2÷12×4] 16,000
Accumulated depreciation 16,000
(ii) Accumulated depreciation (1,200,000×0.1×2÷12) 20,000
Depreciation expense 20,000
Sales 700,000
Accumulated depreciation [(1,200,000-670,000) - 20,000] 510,000
Fixed assets-Furniture
1,200,000
Gain on disposal (Balancing figure) 10,000
(iii) Rent income (900,000÷12×3) 225,000
Unearned rent income 225,000
(iv) Bad debt expense 180,000
Trade receivables 180,000
Trade receivable 96,000
Bad debt expense/Bad debt recovered 96,000

Provision for doubtful (180,000-96,000)×5% 4,200


Bad debt expense
(Reversal of allowance) 4,200
*(v) Bank 192,000- (192,000÷0.96×0.04) 184,000
Discount allowed (sales) 184,000

Accounting entries for correction of the errors

* Required Posted
Bank 192,000 Bank 8,000
Disc. Allowed 8,000 Disc. Allowed 192,000
(192,000/96 x 4) Debtor 200,000
Debtor 200,000

314
Page 28 of 31
Test question:
Q. Following information has been extracted from the draft financial statements of Lather Establishment (LE)
for the year ended 31 December 2020:

Statement of profit or loss


Rs. in '000
Revenue 3,500
Cost of sales (2,000)
Gross profit 1,500
Administrative expenses (800)
Selling expenses (550)
Operating profit 150
Other expenses (60)
Other income 200
Net profit 290

Summarised financial position


Rs. in '000
Total assets (including balance of suspense Rs. 132,000) 8,000
Total liabilities (3,000)
Net assets / equity 5,000

During the review following matters were identified:


(i) Inventory costing Rs. 440,000 received on 30 December 2020 were included in the closing
inventory. The invoice for the same was not received till year end.
(ii) Selling expenses include freight-in of Rs. 200,000. 75% of the freight relates to goods sold and
remaining relates to inventories in hand.
(iii) A selling expense amounting to Rs. 35,000 has been posted in the other expenses
account as Rs. 53,000.
(iv) Sales include an amount of Rs. 145,000 received from a customer on 20 December 2020. Goods
were dispatched on 6 January 2021.
(v) Payment of office rent expense amounting to Rs. 120,000 was recorded as a credit entry in the cash
book and also credited to rent income account.
(vi) A purchase of Rs. 352,000 was entered in the purchase day book as Rs. 325,000 and posted to the
creditor’s account as Rs. 235,000.
(vii) Rent payable for owner’s residence amounting to Rs. 250,000 was recorded as accrued office rent.
(viii) Laptop costing Rs. 120,000 purchased on 1 January 2019 was sold on 1 October 2020 for Rs. 65,000
to a supplier with agreement that supplier’s outstanding balance will be adjusted against the sale
proceed. No entry was posted for disposal. Depreciation expense was recorded on the laptop for full
year and included in administrative expenses. LE has a policy of depreciating laptop at straight line
method over a useful life of 3 years.

315
Page 29 of 31
LE uses periodic inventory method to record the inventory. Control accounts are not maintained for
debtors and creditors.

Required:
Prepare corrected statement of profit or loss for the year ended 31 December 2020. Also
determine the correct amounts of total assets and total liabilities as at that date. (17)

Ans
Lather Establishment
Statement of profit or loss
for the year ended 31 December 2020
Rs.in ‘000’
Revenue 3,500-145 3,355
Cost of sales 2,000+440+150+27 (2,617)
Gross profit 738
Other income 200-120+15 95
Admin Expenses 800+120-250-10 (660)
Selling expenses 550-200+35 (385)
Other expenses 60-53 (7)
Net loss (219)

Lather Establishment
Statement of Financial Position
as on 31-12-20
Rs.in 000’
Total Assets 8,000+50+18-240+90+10-120+70 7,878
Total Liabilities 3,000+440+145+117-250-65 (3,387)
Net Assets / Equity 4,491

Working
i) Purchases 440,000
Creditors 440,000

ii) Freight Inward (200x 75%) 150,000


Inventory (200x 25%) 50,000
Selling Expense 2,00000

iii) Selling Expense 35,000


Suspense A/C 18,000
Other Expense 53,000

iv) Sale 145,000


Unearned Income 145,000

316
Page 30 of 31
v) Suspense 240,000
Cash 120,000
Rent income 120,000 POSTED

Rent exp (Admin Expense) 120,000


Cash 120,000 REQUIRED

Admin Expense 120,000


Other income 120,000
Suspense 240,000 RECTIFYING

vi) Purchases (352-325) 27,000


Suspense 90,000
Creditor (352-235) 117,000

vii) Rent payable 250,000


Rent expense (Admin Expense) 250,000
[Reversal of rent payable of owner’s residence as it is not expense of business]

viii) Receivable 65,000


Acc. Depreciation 70,000
[120,000 /3 x 1.75]
Laptop 120,000
Gain (P&L) 15,000

Trade payable 65,000


Trade receivable 65,000

ix) Acc. Depreciation 10,000


Depreciation- [Admin Expense] 10,000
(120,000 / 3 x3/12)
Reversal of Depreciation

317
Page 31 of 31
Income and Expenditure Account
Many organizations exist, not in order to make profits. E.g.:
a) Clubs and societies
b) Charitable organizations
c) Trusts
d) NGOs
e) Hospitals

Terminologies used in non profit organizations:


a. Statement of Comprehensive Income Income and Expenditure account
b. Profit Surplus
c. Loss Deficit
d. Capital/Equity of business Equity Fund/ General fund / Accumulated fund
e. Receipt and Payment account Means a combined cash +bank a/c

Non-Profit Organization:
Primary purpose of these organization is to provide services rather than to make profit.
e.g.: charitable hospitals

Sources of Income:
▪ Membership fee or subscription fee
▪ Investment income from surplus funds
▪ Donation
▪ Legacy
▪ Supporting activities e.g.: sales of medicines, functions, cafeteria
▪ Life membership fee

Statement of Comprehensive Income income and expenditure account


Income and expenditure account
Expenses : Incomes:

Surplus (excess of income) OR Deficit (excess of expense)

Income and expenditure account can also be presented in statement form.


Capital Capital fund, General fund, Accumulated fund

In the questions of income and expenditure accounts a subscription account is usually required to be prepared,
which looks like as follows:
(related to members) Subscription account (Just like debtor control account)
b/d (receivable from members) 15,000 b/d (advance from members) 500
Cash 1,000
Bank 15,000
Subscription Income (Bal.) 19,000 Bad debt 2,000
Discount allowed 500

318
Page 1 of 40
c/d (closing advance) 10,000 c/d (closing receivable) 25,000
44,000 44,000

Example: Subscription account


Question: At 31 March 2016 a cricket club had membership subscriptions in arrears amounting toRs. 48,000 and had
received Rs. 12,000 subscriptions in advance.

During the year to 31 March 2017 the club received Rs. 624,000 including 26 memberships forthe year to 31 March
2018 at Rs. 1,200 per annum.

At 31 March 2017 16 members owed subscriptions of Rs. 1,200 each.

Half of the members who were in arrears at the end of the previous period still had not paid by 31March 2017. It
was decided to write these amounts off.

Required: How the above transactions would be recorded in the subscription’s ledger account for the year to 31
March 2017?

Answer:
Subscriptions Account
Rs. Rs.
Balance b/d: Balance b/d:
Members in arrears 48,000 Advance payments 12,000
Cash 624,000
Membership fees for the
year (to I&E) 600,000 Bad debts (1/2 x 48,000) 24,000
Balance c/d: Advance Balance c/d: Members in
payments (26 × 1,200) 31,200 arrears (16 × 1,200) 19,200
679,200 679,200

Q. 1 Following is the Receipt and Payment Accounts of Sehat club for the year ended 30 June 2011:
Receipts and Payments Account For the year ended 30 June 2011
Receipt Rupees Payment Rupees
Opening balance 15,000 Salaries 63,500
Subscription 201,000 Rent 34,000
Entrance fees 63,000 Travelling expenses 1,500
Donations ' 38,000 Printing and stationery 1,000
Interest 16,000 General charges 2,500
Receipt on disposal of furniture 500 Periodicals 500
Investments 200,000
Closing balance 30,500
333,500 333,500

319
Page 2 of 40
The club’s balance sheet as on 30 June 2010 was as follows:
Balance Sheet As on 30 June 2010
Liabilities Rupees Assets Rupees
General Fund 172,500 Furniture-net 40,000
Liabilities: Rent 11,000 Sports equipment – net 20,000
Salaries 17,500 Investments 100,000
Subscription receivable 15,000
Interest receivables 11,000
Bank balance 15,000
201,000 201,000
Other details for the year ended 30 June 2011 are as follows:
(i) Furniture purchased on 1 July 2009 costing Rs. 4,000 was disposed off on 1 January 2011 at a scrap value
of Rs. 500.
(ii) On 1 July 2010, furniture having written down value of Rs. 6,000 was traded-in with new furniture having
fair value of Rs. 6,700.
(iii) Depreciation is charged on diminishing balance basis at 20% on furniture and 15% on sports equipment.
(iv) Sports equipment worth Rs. 12,000 were received at year end as donation.
(v) Following amounts are receivable /outstanding as at 30 June 2011:
Rs.
Subscription receivable 8,000
Entrance fee receivable 3,000
Salaries outstanding 4,000
Rent outstanding 2,000

Required:
Prepare an income and expenditure account of Sehat Club for the year ended 30 June 2011 and its balance sheet
on that date.

2. GILTAN GOLF CLUB


The treasurer of the Giltan Golf Club has prepared the following receipts and payments account for the year ended
31 March 2016.
Rs.(000) Rs.(000)
Balance at 1 April 2015 682 Functions 305
Subscriptions 2,930 Repairs 146
Functions 367 Telephone 67
Sale of land 1,600 Extension of club house (building) 600
Bank interest 60 Furniture 135
Bequest (legacy) 255 Heat and light 115
Sundry income 46 Salary and wages 2,066
Sundry expenses 104
Balance at 31 March 2016 2,402

5,940 5,940

320
Page 3 of 40
(i) Subscriptions received included Rs.65,000 which had been in arrears at 31 March 2015 and Rs. 35,000
which had been paid for the year commencing 1 April 2016.
(ii) Land sold had been valued in the club's books at cost Rs.500,000.
(c) Accrued expenses
31 March 2015 31 March 2016
Rs.(000) Rs.(000)
Heat and light 32 40
Wages 12 14
Telephone 14 10
58 64

(iii) Depreciation is to be charged on the original cost of assets appearing in the books at 31 March 2016 as
follows:

Buildings 5%
Fixtures and fittings 10%
Furniture 20%
The following balances are from the club's books at 31 March 2015:
Rs.(000)
Land at cost 4,000
Buildings at cost 3,200
Buildings allowance for depreciation 860
Fixtures and fittings at cost 470
Fixtures allowance for depreciation 82
Furniture at cost 380
Furniture allowance for depreciation 164
Subscriptions in arrears (including Rs.15,000 irrecoverable - 80
member had emigrated)
Subscriptions in advance 30

Required:
Prepare an income and expenditure account for the year ended 31 March 2016 and a Statement of financial position
as at that date.

3. LANGTON HOCKEY CLUB


The Langton Hockey club does not have any formal accounting records but the following information is available.

(1) The payments that have been made by the club for the year ending 30 June 2016 are as follows:
Rs.(000)
Purchase of second hand table tennis table 250
Rent 600
Tea stall purchases 900
Annual fair expenses 1,450
Outings expenses 370
Prizes for whist evenings 90

321
Page 4 of 40
Repairs to snooker table 35
Refreshments at social evenings 240

(2)The club's income, apart from annual subscriptions, is as follows:

Rs.(000)
Contributions to outings 300
Takings at the annual fair 2,150
The club also run a tea stall in the village car park every Sunday in the summer months. This sells tea and coffee,
cakes, biscuits and ice creams etc. The profit margin on the tea stall is normally 20% of selling price.

(3) All the club's transactions are in cash but if there are any surplus funds they are banked in a local bank account.
The balance on the bank account was Rs.30,000 at 1 July 2015.

(4) The club has an annual subscription rate of Rs.20,000 per annum per person or Rs.50,000 per annum for a family
membership. Members are asked to pay their subscription in the July at the beginning of the club's accounting year.
There are 10 family members of the club. Of these two paid their 2016 subscription in June 2015 and all the rest
were received before 30 June 2016.

No individual members had paid their 2016 subscriptions in advance but at 30 June 2016 four members still owed
their subscriptions. They had been contacted and all four had promised to pay at the next evening social event. There
are in total 80 individual members.

The club has the following other assets and liabilities:


30 June 30 June
2015 2016
Rs. (000) Rs. (000)
Sports equipment 2,560 Note 6
Inventory for the tea stall 120 60
Payables for the tea purchases 110 190
Prepayment of rent 40 50

(6) The sports equipment is all depreciated at 20% per annum on net book value on the basis of the equipment held
at 30 June each year.

(7) The old table tennis table was sold during the year for Rs.40,000. Its value as recorded by the club at 30 June
2015 was Rs.30,000.

Required: You are required to prepare an income and expenditure account for the year ended 30 June 2016 and a
statement of financial position at that date. (20)

322
Page 5 of 40
Q. 4 The following balances have been obtained from the books of Gulshan Cricket Club:
June 30, 2007 June 30, 2008
Building 6,024,000 6,438,150
Furniture 3,012,000 2,710,800
Books 1,129,500 1,246,950
Sports equipment 1,807,200 1,595,200
Investments - 436,000
Advance subscription 86,000 92,000
Prepaid expenses 122,000 176,000
Expenses payable 186,900 207,600
Subscriptions receivable 326,000 357,000
Cash 1,204,800 1,586,500

The following information is also available in respect of year ended June 30, 2008:
i. Depreciation for the year has been credited directly to the asset accounts. The rates of depreciation are as
follows:
Building 5%
Furniture and books 10%
Sports equipment 20%

ii. The club had 600 members on June 30, 2008. No fresh members were admitted during the year but 10 members
left the club on January 1,2008. Subscription per member is Rs. 500 per month.

Required:
(a) Summary of receipts and payments made during the year ended June 30,2008.
(b) Income and Expenditure Account for the year ended June 30,2008.
(c) Statement of financial position as on June 30,2008.

323
Page 6 of 40
ANSWERS
A.1 Sehat Club
Income and Expenditure Account For the ye.ir ended 30 June 2011
Expenditure Amount (Rs.) Income Amount (Rs.)
Salaries (63.5+4-17.5) 50,000 Subscriptions (201+8-15) 194,000
Rent (34+2-11) 25.000 Entrance fees (63+3) 66,000
Travelling expenses 1,500 Donation (38+12) 50,000
Printing and stationary 1,000 Interest(16-11) 5,000
General charges 2,500 Gain on trade in of furniture 700
Periodicals 500
Depreciation on furniture 7,820
Depreciation on sports equipment 3,000
Loss on furniture disposed off (2,880- 2.380
500)
Excess of income over expenditure 222.000
315,700 315,700

Sehat Club
Balance sheet As on 30 |une 2011
Liabilities Rupees Assets Rupees
General fund: Furniture 30,000
Opening balance 172,500 Sports equipment (20-3+12) 29,000
Add excess of income over expense 222,000 Investments (100+200) 300,000
394,500 Subscription receivable 8,000
Liabilities : Entrance fee receivable 3,000
Salaries payable 4,000 Bank balance 30,500
Rent payable 2,000
400,500 400,500

Furniture
b/d 40,000 Disposal (4,000-800-320) 2,880
Addition 01.07.2010 6,700 Assets exchanged (01.07.2010) 6,000
Depreciation expense 7,820*
c/d 30,000
46,700 46,700

*Depreciation:
(40,000 – 3,200 – 6,000) x 20% = 6,160
3,200 x 20% x 6/12 = 320
6,700 x 20% = 1,340
7,820

324
Page 7 of 40
Disposal
Furniture 2,880 Cash 500
Loss (bal) 2,350
2,880 2,850

Salary payable
Cash 63,500 b/d 17,500
c/d 4,000 Expense (bal) 50,000
67,500 67,500

Rent payable
Cash 34,000 b/d 11,000
c/d 2,000 Expense(bal) 25,000
36,000 36,000

Entrance fee receivable


b/d - Cash 63,000
Income (bal) 66,000 c/d 3,000
66,000 66,000

Interest receivable
b/d 11,000 Cash 16,000
Income(bal) 5,000 c/d -
16,000 16,000

Equipment
b/d 20,000 Depreciation (20,000 x 15%) 3,000
Donation (30-06-2011) 12,000 c/d 29,000
32,000 32,000

2. GILTAN GOLF CLUB


Income and expenditure account for Giltan Golf Club for year ending 31 March 2016
Income Rs.(000) Rs.(000)
Functions surplus (367 - 305) 62
Sale of land (1,600 - 500) 1,100
Bank interest 60
Bequest 255
Sundry income 46
Subscriptions (W1) 2,860
4,383
Expenditure
Bad debts 15
Repairs 146
Telephone (67 - 14 + 10) 63

325
Page 8 of 40
Heat and light (115 - 32 + 40) 123
Salaries and wages (2,066 - 12 + 14) 2,068
Sundry expenses 104
Depreciation – building 190
Depreciation – furniture 103
Depreciation - fixtures and fittings 47 (2,859)
Surplus for the year 1,524

Giltan golf club: Statement of financial position as at 31 March 2016


Non-current assets Cost Accumulated Carrying
depreciation Amount
Rs.(000) Rs.(000) Rs.(000)
Land (4,000 - 500) 3,500 - 3,500
Buildings (W3) 3,800 (1,050) 2,750
Fixtures and fittings (W4) 470 (129) 341
Furniture (W5) 515 (267) 248
8,285 (1,446) 6,839
Current assets
Bank 2,402
9,241
Accumulated Fund (W2) 7,618
Surplus for the year 1,524
9,142
Current liabilities
Accruals 64
Subscriptions in advance 35
9,241
Workings

W1 Subscriptions account
Rs.(000) Rs.(000)
Subs in arrears b/d 80 Subs in advance b/d 30
Income and expenditure 2,860 Bank 2,930
Subs in advance c/d 35 Bad debts 15
2,975 2,975

W2 Opening statement of affairs 2015


Assets Rs (000)
Bank 682
Subscriptions in arrears 80
Land 4,000
Buildings (3,200 - 860) 2,340
Fixtures (470 - 82) 388
Furniture (380 - 164) 216
Liabilities 7,706

326
Page 9 of 40
Accruals (58 + 30) (88)
7,618

W3 Buildings
Cost Acc. Depreciation
Rs.(000) Rs.(000)
Balance b/d 860
3,200
Extension to club house 600
Depreciation (3,800 x 5%) 190
3,800 1,050

W4 Fixtures and fittings


Cost Acc. Depreciation
Rs.(000) Rs.(000)
Balance b/d 470 82
Depreciation (10% x 470) 47
470 129
W5 Furniture
Cost Acc. Depreciation
Rs.(000) Rs.(000)
Balance b/d 380 164
Additions 135
Depreciation (20% x 515) 103
515 267

Telephone payable account


Rs.(000) Rs.(000)
cash 67 b/d 14
Expense (bal) 63
c/d 10
77 77
Heat and light payable
Rs.(000) Rs.(000)
cash 115 b/d 32
Expense (bal) 123
c/d 40
155 155
Telephone payable account
Rs.(000) Rs.(000)
Cash 2,066 b/d 12
Expense (bal) 2,068
c/d 14
2,070 2,070

327
Page 10 of 40
3. LANGTON HOCKEY CLUB
Income and Expenditure Account for the year ended 30 June 2016
Income Rs.(000) Rs.(000)
Profits from tea stall (W1) 260
Profit from annual fair (2,150 - 1,450) 700
Subscriptions (W4) 2,100
Profit on sale of table tennis table (40 - 30) 10
3,070
Expenditure
Rent (600 + 40 - 50) 590
Net expense of outings (370 - 300) 70
Prizes for whist evenings 90
Repairs to snooker table 35
Refreshments 240
Depreciation (W2) 556 (1,581)
Excess of income over expenditure 1,489

Statement of financial position as at 30 June 2016


Rs.(000) Rs.(000)
Assets

Non-current assets

Sports equipment 2,224


Current assets
Inventories for tea stall 60
Subscriptions due (4 x 20) 80
Prepayments – rent 50
Bank (W3) 1,805 1,995

Total assets 4,219


Equity and liabilities
Accumulated fund b/f (W5) 2,540
Excess of income over expenditure 1,489
4,029
Current liabilities
Trade payables (tea stall) 696
Total equity and liabilities 4,219

328
Page 11 of 40
Workings
(W1) Tea stall
Inventory
Rs.(000) Rs.(000)
b/d 120 Cost of sale 1,040

Purchases 980 c/d 60


1,100 1,100

Trade payables
Rs.(000) Rs.(000)
Cash 900 b/d 110
Expense (bal) 980
c/d 190
1,090 1,090

Sports equipment
Rs.(000) Rs.(000)
b/d 2,560 Disposal 30
Depreciation (2,560 + 250) x
20% 556
Cash 250 c/d 2,224
2,780 2,780
Prepaid rent
Rs.(000) Rs.(000)
b/d 40 Expense (bal) 590

Cash 600 c/d 50


640 2,780

Bank + Cash account


Rs.(000) Rs.(000)
Opening balance 30 Table tennis table 250
Contribution to outings 300 Rent 600
Annual fair takings 2,150 Tea stall purchases 900
Tea stall sales (W1) 1,300 Annual fair 1,450
Subscriptions (1,520 + Outings 370
400) 1,920 Prizes 90
Sale of table tennis 40 Repairs 35
Table Refreshments 240
Bal c/f (bal fig) 1,805
5,740 5,740

329
Page 12 of 40
Subscriptions account
Rs.(000)
Rs.(000)
Bal. b/f - Family (2 x 50,000) 100
Income and 2,100 Bank - Family (8 x Rs.50,000) 400
expenditure (bal fig)
Bank - Individual (76 x 20,000) 1,520
Bal. c/f - Individual (4 x 20,000) 80
2,100 2,100

Opening accumulated fund


Rs.(000)
Sports equipment 2,560
Inventory for tea stall 120
Subscriptions in advance (2 x 50,000) (100)
Rent prepaid 40
Bank 30
Payables for the tea stall (110)
2,540

A. 4 Gulshan cricket club


(a) Receipt and payment account for the year ended June 30, 2008
b/d 1,204,800 Building (W-4) 753,000
Book (W-5) 256,000
Subscription (W-2) 3,605,000 Shop equipment (W-6) 186,800
Investment 436,000
Expenses (Bal.) 1,591,500
c/d 1,586,500
4,809,800 4,809,800

(b) Income and expenditure account for the year ended June 30, 2008
Expenses (W-8) 1,558,200
Depreciation: Subscription Income (W-1) 3,630,000
Building 338,850
Furniture 301,200
Books 138,550
Sports equipment 398,800
Surplus (bal) 894,400
3,630,000 3,630,000

330
Page 13 of 40
(c) Gulshan cricket club
Statement of financial position as on June 30,2008.
Non current assets:
Building 6,438,150
Furniture 2,710,800
Books 1,246,950
Sports equipment 1,595,200
Investments 436,000 12,427,100
Current assets:
Prepaid expenses 176,000
Subscriptions receivable 357,000
Cash 1,586,500 2,119,500
14,546,600
Equity and liabilities:

Capital fund 13,352,600*


Surplus 894,400 14,247,000
Current liabilities:
Advance subscription 92,000
Expenses payable 207,600 299,600
14,546,600

*Note: Opening capital fund: 6,024,000 + 3,012,000 +1,129,500 + 1,087,200 – 86,000 +122,000 -186,900 + 326,000
+ 1,204,800 = 13,352,600

Workings:
W-1) Subscription income
[600 x 500 x 6] + [610 x 500 x 6]

=3,630,000

(W-2) Subscription account


b/d 326,000 b/d 86,000
Subscription income 3,630,000 Cash (bal) 3,605,000
c/d 92,000 c/d 357,000
210,000 210,000

(W-4) Building account


b/d 6,024,000 Depreciation (6,438,150/95x5) 338,850
Cash(bal) 753,000 c/d 6,438,150
6,777,000 6,777,000

331
Page 14 of 40
(W-5) Books
b/d 1,129,500 Depreciation (1,246,950/90x10) 138,550
Cash(bal) 256,000 c/d 1,246,950
1,385,500 1,385,500

(W-6) Sports Equipment


b/d 1,807,200 Depreciation (1,595,200/80x20) 398,800
Cash(bal) 186,800 c/d 1,595,200
1,994,000 1,994,000

(W-7) Furniture
b/d 3,012,000 Depreciation (2,710,800/90x10) 301,200
c/d 2,710,800
3,012,000 3,012,000

(W-8) Expenses prepaid + Expenses payable


b/d 122,000 b/d 186,900
Cash 1,591,500 Expenses (bal) 1,558,200
c/d 207,600 c/d 176,000
1,921,100 1,921,100

332
Page 15 of 40
Extra practice questions of Income and expenditure account:
Q. 1
Seaview Club started its operations on 1 February 2015. Sponsor of the club contributed Rs. 50 million towards
general fund for the start of operations and placed the amount in the bank. Following is the receipts and payments
summary for the period from 1 February 2015 to 31 December 2015:
Receipts Rs. In ‘000’ Payments Rs. In ‘000’
Sponsor’s contribution 50,000 Furniture & fixtures 1,200
Joining fees 20,800 Van 1,500
Subscription from members 29,952 Salaries 1,000
Sale of beverages 1,500 Rent 3,600
Utilities 570
Insurance 120
Repairs and maintenance 275
Purchase of beverages 1,367
Advance for plot of land 65,000
Balance 27,620
102,252 102,252

Additional information:
(i) The joining fee for award of membership is Rs. 50,000. Annual subscription is Rs. 24,000. All new members
pay three years subscription in advance. The memberships were awarded as follows:
Month March June September December
No. of members 112 98 101 105

(ii) The club sells beverages at a gross profit margin of 20%. All sales are billed in the first week of the next month
and the payment is received in the same month. Sale of beverages during December 2015 amounted to Rs.
150,000.
(iii) 25% of total purchases of beverages made during the year remained unsold at year-end.
(iv) Salaries are paid on the first day of next month. The amount of salaries includes an advance amounting to Rs.
10,000 paid to an employee on 1 December 2015. the advance is repayable on 1 February 2016.
(v) Rent for three years was paid in advance on 1 February 2015.
(vi) Presently the club is operating on rental premises. However, a plot of land has been purchased on which
construction would commence shortly. Title of land would be transferred after completion of legal
formalities.
(vii) Payments for utilities include security deposit paid to utility companies amounting to Rs. 20,000. Utility bills
are paid on the 7th day of the next month.
(viii) Insurance premium was paid on 1 February 2015 covering a period of 12 months.
(ix) Repairs and maintenance include an advance of Rs. 100,000 paid to a contractor for construction of a parking
shed. Repair bills amounting to Rs. 7,000 were outstanding at year-end.
(x) Furniture & fixtures and van were purchased on 1 February 2015. Depreciation on these assets is to be
charged at 10% and 20% respectively.

Required:
Prepare statement of financial position as at 31 December 2015 and income & expenditure account of Seaview Club
for the period ended 31 December 2015. (20)

333
Page 16 of 40
Question 2
The accountant of Leisure Club was terminated on account of charges of fraud on 31 December 2016 and Mr. Emad
has been appointed in his place. Emad has gathered the following information in respect of the year ended 31
December 2016:
(i) The club has 3,300 members and the membership fee is Rs. 10,000 per annum. The fee payable by each
member becomes due on the first day of the quarter in which he became a member. The fee received in each
quarter was as follows:
Quarter First Second Third Fourth
Subscription received (Rs.) 9,900,000 8,250,000 5,500,000 9,350,000

Last year the fee was Rs. 9,000 per annum. However, the number of members was the same.

(ii) A summary of the bank account for the year is shown below;
Deposits Rupees Withdrawals Rupees
Balance as at 1 Jan. 2016 3,700,500 Insurance 175,000
Cash deposited into bank 37,848,500 Rent and rates 4,200,000
Written off amount recovered 1,860,000 Utilities 4,365,000
Disposal of fixed assets 750,000 Freehold land purchased 17,000,000
Members subscription received Cash withdrawals from bank 6,120,000
directly in bank account 19,800,000 Payment to creditors 18,155,000
Repairs and maintenance 700,000
Exercise equipment 7,350,000
Balance as at 31 Dec. 2016 5,894,000
63,959,000 63,959,000

(iii) Amounts paid from petty cash were as follows:


Rupees
Salaries 2,300,000
Sundry expenses 640,000

(iv) The club has a tuck shop which earns a profit margin of 20% of sales. All sales of tuck shop are made on cash.
During the year, stock costing Rs. 500,000 was destroyed by fire.
(v) The opening WDV of fixed assets was Rs. 28,000,000. Exercise equipment was purchased on 1 October 2016.
Fixed assets having opening WDV of Rs. 800,000 were disposed off on 31 March 2016. Fixed assets are
depreciated @ 20% under the reducing balance method.
(vi) The opening and closing balances of cash in hand were Rs. 300,000 and Rs. 25,000 respectively.
(vii) The following balances have been extracted through a scrutiny of the available records:

2016 2015
Rupees
Creditors 3,330,000 2,500,000
Prepaid rent 175,000 168,000
Stock- tuck shop 2,500,000 2,300,000

334
Page 17 of 40
Required:
(a) Determine the amount of loss incurred by the club due to fraud committed by the previous accountant. (09)

(b) An income and expenditure account for the year ended 31 December 2016. (05)
(c) Statement of financial position as at 31 December 2016. (06)

Life membership fee


A club should have an accounting policy for these. Possible policies include:

Recognition as income when received.

Recognition as income over a specified period.

Recognition as income when received

Illustration:

Debit Credit
Bank (cash received) X
Income and expenditure account X

Recognition as income over a specified period

Illustration:

On receipt: Debit Credit


Bank (cash received) X
Deferred income ( a credit account on the face of the
statement of financial position) X

Each year over the a specified future period:


Deferred income X
Income and expenditure account X

This treatment recognises the amount received as income over several years as given in question.

Special funds
An organisation might also have other funds in addition to the accumulated fund.These “special”
funds arise in a number of circumstances including:
• when an organisation receives cash for a designated purpose (e.g. cash for purchase of building); or
• when an organisation sets aside resources for a designated purpose (like creation of reserve from accumulated
funds)

335
Page 18 of 40
Illustration: Receipt of cash for a specified purpose

Debit Credit
Cash X
Special fund X

An organisation itself might set aside funds for a particular purpose.

Illustration : Set up fund for a specified purpose


Debit Credit
Accumulated fund X

X
Special fund

Illustration: Special fund


A social club in a small town has managed to accumulate a significant balance on its accumulatedfund over the
years.
Its members have decided that the club should establish a fund to contribute to the school fees ofchildren of
high promise from the town. Parents of such children would apply to the club for a grant of Rs. 50,000.
Rs. 1,500,000 is to be set aside for this purpose.This would
be accounted for as follows:

Setting up the fund Debit Credit


Accumulated fund 1,500,000
Special fund (Education fund) 1,500,000

On the award of a grant. Debit Credit


Special fund (Education fund) 50,000
Cash 50,000

Answers to extra practice questions:


A.1
Seaview Club
Income & Expenditure Account
For the period ended 31 December 2015
Expenditure Rs. In ‘000 Income Rs. In ‘000
Salaries and wages (1,000 – 10 + 99) 1,089 Joining fees (Given) 20,800
Rent (3,600/3 × 11/12) 1,100 Subscription income (W-1) 4,630
Utilities (570 – 20 + 55) 605 Profit on sale of beverages (W-2) 330
Insurance (120/12 × 11) 110
Repairs and maintenance (275–100+7) 182
Depreciation expense

336
Page 19 of 40
(1,200×10%×11/12+1,500×20%11/12(110+275) 385
Excess of income over expenditure 22,289
25,760 25,760

Seaview Club
Statement of Financial Position
As at 31 December 2015
Assets Rs. In ‘000
Non-Current Assets
Land Advance 65,000
Furniture & fixtures (1,200 – 110) 1,090
Van (1,500 – 275) 1,225
Advance for parking shed 100
Long term deposits – Utility 20
Long term prepayment – Rent 1,300
68,735

Current Assets
Stock (W-2) 440
Debtors for beverages (credit sale) 150
Advance & prepayments (W-3) 1,220
Bank 27,620
29,430
Total Assets 98,165

General Fund & Liabilities Rs. In ‘000


General fund 50,000
Excess of income over expenditure 22,289
72,289

long term advance (W-1) 15,338


Current Liabilities
Creditors (1,760 – 1,367) 393
Accrued expenses (7 + 55 + 99) 161
Advance subscription (W-1) 9,984
10,538

Total General Fund & Liabilities 98,165

337
Page 20 of 40
W-1: Subscription Income
Subscription for 3 years is Rs. 72,000 so subscription for 1 year is Rs. 24,000 or Rs. 2,000 per month.
Month No. of No. of Subscription Income for Advance subscription
members Months the year income
A B A × B × 2,000 A × (36 – B) × 2,000
-------------------- Rupees -----------------------
March 112 10 2,240,000 5,824,000
June 98 7 1,372,000 5,684,000
September 101 4 808,000 6,464,000
December 105 1 210,000 7,350,000
4,630,000 25,322,000
Less: short term [(112+98+101+105)×24,000] (9,984,000)
Long term 15,338,000

or
Months
March 112 × 24,000 = 2,688,000 × 3 = 8,064,000
June 98 × 24,000 = 2,352,000 × 3 = 7,056,000
September 101 × 24,000 = 2,424,000 × 3 = 7,272,000
December 105 × 24,000 = 2,520,000 × 3 = 7,560,000
29,952,000
[29,952,000 – 4,630,000] = 25,322,000

W-2: Beverage Sale Results Rs. In ‘000’


Sales (1,500 + 150) 1,650
Less: Cost of sales
Purchases (1,320/0.75) 1,760
Closing stock (1,760 × 25%) (440) 1,320
330
Debtors
b/d -- Cash 1,500
Sales 1,650
c/d 150
Payable
b/d --
Cash 1,367 Purchases 1,760
c/d 393

Stock
b/d -- Cost of sales 1,320
Purchases (bal) 1,760 (1,650 / 100 x 80)
c/d (1,320 / 75 x 25) 150

338
Page 21 of 40
A.2
(a) Determination of Amount of Loss incurred due to fraud [it means a Cash Account in a Statement form to find
out the missing cash]

Opening balance 300,000


Receipts:
Collections from members [(3,300 × 10,000) – 19,800,000] 13,200,000
Bank withdraws 6,120,000
Tuck shop sales (W) 22,856,250
42,176,250
Payments:
Salaries (2,300,000)
Sundry Expenses (640,000)
Cash Deposited in Bank (37,848,500)
(40,788,500)
Closing balance that should have been 1,687,750
Closing Cash – Actual (Given) 25,000
Difference – Loss due to fraud 1,662,750
(b) Income and Expenditure Account
Incomes:
Subscription Income (W) 31,817,500
Income from tuck shop [22,856,250 – 18,285,000] 4,571,250
Other income – bad debts recovered 1,860,000
38,248,750
Expenditures:
Salaries (2,300,000)
Insurance (175,000)
Rent expense (4,193,000)
Utilities (4,365,000)
Repair & Maintenance (700,000)
Depreciation (W) (5,847,500)
Sundry Expenses (640,000)
Loss on disposal (10,000)
Loss of inventory due to fire (500,000)
Loss due to fraud [from (a)] (1,662,750)
(20,393,250
Surplus 17,855,500

339
Page 22 of 40
(c) Statement of Financial Position
Non-Current Assets:
Fixed Assets – WDV 45,742,500
Current Assets:
Stock 2,500,000
Prepaid rent 175,000
Cash at bank 5,894,000
Cash in hand 25,000
54,336,500
Fund and Liabilities:
Accumulated Fund – Opening [168 + 2,300 + 28,000 + 300 + 3,700.5 – 2,500 – 10,642.5] 21,326,000
Add: Surplus 17,855,500
39,181,500
Liabilities:
Creditors 3,330,000
Unearned subscription Income (W) 11,825,000
54,336,500
Workings:
(W-1) Subscription Income:
Opening balance of unearned subscription [11,825,000/10,000 × 9,000] 10,642,500
Add: Receipt for the year (3,300 × 10,000) 33,000,000
Less: Closing unearned subscription (W – 1.1) (11,825,000)
Subscription Income 31,817,500

(W-1.1) Closing Balance:


Quarter 1 --
Quarter – 2 [8,250,000 × 3/12] 2,062,500
Quarter – 3 [5,500,000 × 6/12] 2,750,000
Quarter – 4 [9,350,000 × 9/12] 7,012,500
11,825,000
(W-2) Creditors
Bank 18,155,000 b/d 2,500,000
Purchases 18,985,000
c/d 3,330,000
(W-3) Prepaid Rent

b/d 168,000 Expense 4,193,000


Bank 4,200,000
c/d 175,000
(W-4) Stock
b/d 2,300,000 Loss 500,000
Cost of sales 18,285,000
Purchases 18,985,000
c/d 2,500,000

340
Page 23 of 40
Sales from cost of sales:
18,285,000
× 100 = 22,856,250
80

(W-5) Fixed Assets Account


b/d 28,000,000 31-3 Disposal (W) 760,000
1-10 Bank 7,350,000 Depreciation (W) 5,847,500
Bank (Land) 17,000,000
c/d 45,742,500
(W-6) Disposal Account
Fixed Asset 760,000 Bank 750,000
Loss 10,000

Depreciation for the year:


[28,000,000 – 800,000] × 20% = 5,440,000
800,000 × 20% × 3/12 = 40,000
7,350,000 × 20% × 3/12 = 367,500
5,847,500
WDV of Disposal:
Opening WDV = 800,000
Depreciation [800,000 × 20% × 3/12] = (40,000)
760,000

341
Page 24 of 40
Extra practice questions
Q.1 Violin Family Club was formed in 2016. Following are the details of assets and liabilities of the club as on 31
December 2017:

Assets Rs. in '000 Liabilities Rs. in '000


Subscription in arrears: Bank overdraft 181
2016 15 Subscription in advance for 2018 45
2017 90 Accrued electricity 23
Advance rent 24 Canteen wages 11
Canteen stock 215 Canteen creditors 118
Snooker tables 960
Furniture & equipment

Additional information:
(i) Some of the balances as on 31 December 2018 are as follows:
(ii)
Assets Rs. in '000 Liabilities Rs. in '000
Subscription in arrears for 2018 30 Accrued electricity 35
Canteen stock 247 Canteen creditors 142

(iii) Break-up of the subscription received during 2018 is as follows:

Related to year Rs. in '000


2017 60
2018 920
2019 75

The club’s management has decided to write-off the remaining subscription in arrears relating to the
year 2016 and 2017.
(iv) A scheme was introduced in 2016 under which a person is awarded life time membership upon
payment of Rs. 120,000. Life memberships received in the years 2016, 2017 and 2018 were 5, 8 and
6 respectively. Life memberships are credited to ‘Life Membership Fund’ upon receipt and are
transferred to income equally over 10 years, starting from the year of admission.
(v) The club operates a canteen. Till last year, the canteen earned a gross profit of 20% of sales. Effective
1 January 2018, selling prices were increased by 10%.
(vi) Details of some payments during 2018 are as follows:

Rs. 000
Canteen creditors 512
Salaries 285
Equipment 66
Electricity 263
(vii) Equipment acquired during the year is only 30% paid and the remaining amount is payable
in February 2019.

342
Page 25 of 40
(viii) Wages of canteen staff are paid on 5th of each month.
(ix) The club operates from a rented place. The rent is paid quarterly in advance on 1 March, 1 June,
1 September and 1 December. As per agreement, annual rent was increased by Rs. 6,000 with effect
from 1 September 2018.
(x) Balance of snooker tables as at 31 December 2017 represents the book value of 5 similar tables
purchased in 2016. One of the tables was sold to a member for cash during the year for Rs. 212,000.
(xi) Snooker tables are depreciated at 12.5% on straight line method while furniture & equipment are
depreciated at 20% using reducing balance method. Full year depreciation is charged in the year of
addition whereas no depreciation is charged in the year of disposal.

Required:
(a) Prepare income and expenditure account for the year ended 31 December 2018.
(12)
(b) Prepare statement of financial position as on 31 December 2018.
(09)

343
Page 26 of 40
Ans.1 Violin Family Club

(a) Income and expenditure account for the year ended 31 December 2018

Rs. in '000
Income
Subscription (W-1) 995
Gain on disposal of table 20
Profit from canteen 57
Life membership (W-2) 228
1,300
Expenditures
Rent 146
Salaries 285
Electricity 275
Depreciation – snooker tables 128
Depreciation – furniture & equipment 188
Subscription written off (Bad Debts) 45
(1,067)
Excess of income over expenditure 233

Canteen trading account for the year ended 31 December 2018


Rs. in '000
Sales 504×110÷80 693
Cost of goods sold
Opening stock 215
Purchases 536
Closing stock (247)
504
Gross profit 189
Expenses
Wages 11×12 (132)
Profit from canteen 57

W-1: Subscription Rs. in '000


Opening arrears: Opening advance 2018 45
2016 15 Receipts (60+920+75) 1,055
2017 90 Write off (15+30) 45
Income balance (bal) 995 Closing arrears 30
Closing advance 75
1,175

1,175

344
Page 27 of 40
W-2: Life membership Rs. in '000
Income
[(5+8+6)×120÷10] 228 Opening balance
(5×120×8÷10)+(
Closing balance 1,836 8×120×9÷10) 1,344
Receipt (6×120) 720
2,064 2,064

(b) Violin Family Club

Statement of financial position as on 31 December 2018 Rs. in '000


Assets
Non-current assets
Snooker table (960–192–128) 640
Furniture & equipment (720+220–188) 752
1,392

Current assets:
Canteen stock 247
Prepaid rent 25
Subscriptions in arrears 30
Bank (W-3) 1,094
1,396
2,788

General funds
Opening balance (2,024–378)–1,344 (W-2) 302
Excess of income over expenditure 233
535
Non current liabilities:

Life membership fund [1,836(W.2) -228] (W-2) 1,608

Current Liabilities
Canteen creditors 142
Life membership fund 228

Accrued electricity 35
Subscription in advance (W-1) 75
Creditors for equipment (220–66) 154
Canteen wages payable 11
417

2,788

345
Page 28 of 40
W-3: Bank/cash Rs. in '000
Subscriptions 1,055 Opening balance 181
Life membership (W-2) 720 Rent (36 + 36 + 37.5 + 37.5) 147
Sale proceeds from table 212 Salaries 285
Canteen receipts 693 Electricity 263
Canteen creditors 512
Canteen wages (11 x 12) 132
Equipment 66
Closing balance 1,094
2,680 2,680

Advance Rent Rs. in '000


1-1 b/f 24
(J.F 2018) Exp (bal.) 146
1-3 Bank 36 Or (24+36+36+37.5+37.5/3x1)
(M , A , M)
1-6 Bank 36
(J , J , A)
1-9 Bank
(S , A , N ) 37.5 c/d (J , F) 25
1-12 Bank 37.5 (37.5/3 X 2)

Up to Last year:
36 x 4 = 144
From Sept 2018
Annual rent = 144 + 6 = 150
Quarterly Rent = 150 /4 = 37.5

Snooker Table Rs. in '000


Disposal
b/d 960 (960 / 5) 192
Dep 128

c/d (Bal.) 640

346
Page 29 of 40
Furniture and Equipment Rs. in '000
Dep
b/d 720 (720 + 220 ) x 20% 188
Cash (66+66/30x70) 220
c/d (Bal.) 752

Electricity Payable Rs. in '000


Cash 263 b/d 23

Exp (Bal.) 275


c/d 35

Wages Payable Rs. in '000


Cash (11 x 12) 132 b/d 11
Exp (11 x 12) 132
c/d 11
Creditor Rs. in '000
Cash 512 b/d 118

c/d 142 Purchase (bal.) 536

Disposal a/c Rs. in '000


Tables 192 Cash 212

Gain (Bal.) 20

347
Page 30 of 40
Stock Rs. in '000
b/d 215 COS (BAL.) 504
Purchase 536
c/d 247

i.Working of Margin of 2018:


80 + 20 = 100 (up to last year)

Current year:
100 x 10% = 10
So, 100 + 10 = 110
Sales = 504 / 80 x 110 = 693

ii. Equipment:
66 / 30 x 100 = 220
Equipment 220
Bank 66
Payable 154
iii. Cost of snooker tables (for depreciation):

Let’s cost in 2016 is 100


Dep (12.5)
31-12-2016 87.5
Dep 12.5
31-12-2017 75.0

960 / 75 x 100 = 1,280


[1,280 – (1,280 / 5)] = 1,024 x 12.5% = 128

348
Page 31 of 40
Further Practice
1. AB SPORTS AND SOCIAL CLUB
You have agreed to take over the role of bookkeeper for the AB sports and social club.

The summarised statement of financial position on 31 December 2014 as prepared by the previous bookkeeper
contained the following items.
Assets Rs.
Heating oil for clubhouse 1,000
Shop and cafe inventories 7,000
New sportswear, for sale, at cost 3,000
Used sportswear, for hire, at valuation 750
Equipment for groundsman
Cost 5,000
Depreciation 3,500 1,500
Subscriptions due 200
Bank
Current account 1,000
Deposit account 10,000

Capital and liabilities


Accumulated fund 23,150
Payables
Shop and cafe inventories 1,000
Sportswear 300

The bank account summary for the year to 31 December 2015 contained the following items.
Receipts Rs.
Subscriptions 11,000
Bankings
Shop and café 20,000
Sale of sportswear 5,000
Hire of sportswear 3,000
Interest on deposit account 800
39,800

Payments Rs.
Rent and repairs of clubhouse 6,000
Heating oil 4,000
Sportswear 4,500
Grounds person 10,000
Shop and cafe purchases 9,000
Transfer to deposit account 6,000
39,500

349
Page 32 of 40
You discover that the subscriptions due figure as at 31December 2014 was arrived at as follows.
Subscriptions unpaid for 2013 10
Subscriptions unpaid for 2014 230
Subscriptions paid for 2015 40

Corresponding figures at 31 December 2015 are:


Subscriptions unpaid for 2013 10
Subscriptions unpaid for 2014 20
Subscriptions unpaid for 2015 90

Subscriptions paid for 2016 200


Subscriptions due for more than 12 months should be written off with effect from 1 January 2015.
Asset balances at 31 December 2015 include:
Heating oil for club house 700
Shop and cafe inventories 5,000
New sportswear, for sale, at cost 4,000
Used sportswear, for hire, at valuation 1,000

Closing payables at 31 December 2015 are for:


shop and cafe inventories 800

Sportswear 450
heating oil for clubhouse 200

Two thirds of the sportswear purchases made in 2015 had been added to inventory of new sportswear in the figures
given in the list of assets above, and one third had been added directly to the inventory of used sportswear for hire.
Half of the resulting 'new sportswear for sale at cost' at 31 December 2015 is actually over two years old. You decide,
with effect from 31 December 2015, to transfer these older items into the inventory of used sportswear, at a
valuation of 25% of their original cost.

No cash balances are held at 31 December 2014 or 31 December 2015. The equipment for the grounds person is to
be depreciated at 10% per annum, on cost.

Required:
Prepare the income and expenditure account and statement of financial position for the AB sports club for 2015.
(23)

350
Page 33 of 40
2. MONARCH SPORTS CLUB
The Monarch Sports Club has the following summary of its cash book for the year ended 30 June 2015:
Rs. Rs.
Opening bank balance 12,500
Receipts:
Subscriptions 18,000
Life membership fees 3,000
Competition receipts 7,500
Entrance fees 2,500
Equipment sold 1,000 32,000
44,500
Payments:
Transport to matches 3,700
Competition prizes 4,300
Coaching fees 2,100
Repairs to equipment 800
Purchase of new equipment 4,000
Purchase of sports pavilion 35,000
(49,900)
Closing balance (overdrawn) (5,400)

The following information is available regarding the financial position at the beginning and end of the accounting
year:
1 July 2014 30 June 2015
Rs. Rs.
Subscriptions in advance 1,100 900
Subscriptions in arrears 200 300
Coaching fees outstanding 150 450

Of the subscriptions outstanding at the beginning of the year, only half were eventually received.

The equipment sold during the year had a net book value of Rs.1,200 at 1 July 2014. Equipment is to be depreciated
at 20% per annum straight line. Life membership fees are taken to cover 10 years.

The treasurer insists that no depreciation needs to be charged on the sports pavilion, as buildings do not decrease
in value. He says that the last club of which he was treasurer did charge depreciation on its buildings but that when
the club came to replace them, there was still insufficient money in the bank to pay for the new building.

Required: Prepare an income and expenditure account for the Monarch Sports Club for the year ended 30 June
2015. (10)

351
Page 34 of 40
3. LH SPORTS CLUB
The LH Sports Club opened on 1 May 2014 having purchased premises for Rs.80, 000 and furniture for Rs.18,000,
both financed by an interest-free loan from a member. The club secretary has produced the following income and
expenditure account for the year to 30 April 2015.
Income Rs. Rs.
Joining fees (89 members x Rs.200 each) 17,800
Annual subscriptions 12,000
Cafe profits 8,450
Dinner surplus (means profit) 830
Equipment hire receipts 1,750 40,830
Expenditure
Premises costs 10,990
Equipment costs 5,590
Secretary’s expenses 470
Bank charges 125 (17,175)
Surplus for the year 23,655

The income and expenditure account has been prepared after taking into account the following items at 30 April
2015:
□ cafe inventories Rs.1,400
□ payables for cafe supplies Rs.1,320
□ rates and insurances prepaid Rs.2,280

The following items have not been taken into account:


• the equipment costs figure includes Rs.4,000 for the purchase of equipment
• depreciation is to be provided as follows:
✓ at 2% on premises
✓ at 10% on furniture
✓ at 20% on equipment
• joining fees are to be spread over a five-year period
• the annual subscriptions figure includes Rs.960 paid in advance
• subscriptions outstanding at the end of the year, and expected to be collected, amount to Rs.300.

The bank balance at 30 April 2015 was Rs.21,295.

Required:
• Calculate the correct surplus for the year. (6)
• Prepare the statement of financial position at 30 April 2015. (8)

352
Page 35 of 40
Answers:
1. AB SPORTS AND SOCIAL CLUB

AB Sports and social club: Income and expenditure account


Rs. Rs.
Incomes
Subscriptions (W1) 10,720
Shop and cafe profit (W2) 9,200
Sale of sportswear (W3) 1,400
Hire of sportswear (W4) 1,700
Interest on deposit account 800 23,820

Rent of clubhouse 6,000


Expenditure:
Heating oil (1,000 + 4,000 + 200 - 700) 4,500
Grounds person 10,000
Bad debts (unpaid subscriptions = 10 +20) 30
Depreciation (5,000 x 10%) 500 (21,030)
Net surplus 2,790

AB Sports and Social Club statement of financial position as at 31 December 2014


Non-current assets Rs. Rs.
Equipment for grounds person
Cost 5,000
Depreciation (3,500 + 500) 4,000
1,000
Current assets
Heating oil 700
Shop and cafe inventories 5,000
New sportswear (4,000 – 2,000) 2,000
Hire sportswear (1,000 + 500) 1,500
Subscriptions due 90
Bank
Current account (1,000 + 39,800 – 39,500) 1,300
Deposit account (10,000 + 6,000) 16,000 26,590
27,590
Capital and liabilities
Accumulated fund b/f 23,150
Surplus for year 2,790 25,940
Current liabilities
Shop and café 800
Sportswear 450
Heating oil 200
Subscriptions in advance 200 1,650
27,590

353
Page 36 of 40
Workings Inventory of heating oil
Rs.(000) Rs.(000)
b/d 1,000 Cost of sale 4,500

Purchases 4,200 c/d 700

1,100 1,100

Trade payables (heating oil)


Rs.(000) Rs.(000)
Cash 4,000 b/d -
purchase (bal) 4200
c/d 200

77 77

Subscriptions account
Rs.(000) Rs.(000)
b/d (10+230) 240 Bal. b/f 40
Income (bal fig) 10,720 Bank 11,000
Bad debts (10+20) 30
c/d 200 Bal. c/f 90
11,160 11,160

Shop and cafe results Rs. Rs.


Sales 20,000
Opening inventory 7,000
Purchases (9,000 + 800 - 1,000) 8,800
Closing inventory (5,000) (10,800)
Profit (gross) 9,200
Sale of sportswear Rs. Rs.
Sales 5,000
Opening inventory 3,000
Purchases (4,500 + 450 - 300) x 2/3 3,100
Closing inventory (4,000) 2,100
Profit (gross) 2,900
Loss on sportswear transferred (W.1) (1,500)
Profit 1,400
Hire of sportswear Rs. Rs.
Rentals 3,000
Opening balance 750
Additions of cost (4,500 + 450 - 300) x 1/3 1,550
Closing inventory at valuation (1,000) 1,300
Surplus 1,700

354
Page 37 of 40
(W.1)
Old sportswear 500
Loss 1,500
New sports wear 2,000
(4,000 x ½ = 2,000)

2. MONARCH SPORTS CLUB


Monarch Sports Club: Income and expenditure account year ended 30 June 2015
Income Rs. Rs.
Annual subscriptions (W1) 18,400
Life membership (3,000 x 10%) 300
Entrance fees 2,500
Surplus from competitions (W2) (7,500 – 4,350) 3,200
24,400
Expenditure
Transport 3,700
Coaching fees (2,100 - 150 + 450) 2,400
Repairs 800
Bad debts 100
Loss on disposal of equipment (W3) 200
Depreciation (W4) 800 (8,000)
Surplus for the year 16,400

Workings
(W1)
Subscriptions account
Rs. Rs.
Balance b/d (in arrears) 200 Balance b/d (in advance) 1,100
Income (bal) 18,400 Cash
Balance c/d (in advance) 900 Bad debts 100
Balance c/d (in arrears) 300
19,500 19,500

(W2) Competitions
Rs.
Receipts 7,500
Prizes (4,300)
Surplus 3,200

355
Page 38 of 40
(W3) Sale of equipment

Disposal account
Rs. Rs.
Assets 1,200 Cash 1,000
Loss to I & E a/c 200

1,200 1,200

(W4) Depreciation
20% x 4,000 = 800
Equipment account
Rs. Rs.
b/d 1,200 Disposal 1,200
Cash 4,000 Depreciation 800
c/d 3,200

5,200 5,200

3. LH SPORTS CLUB
(a) Surplus for the year
Rs. Rs.
Surplus per draft income and expenditure account 23,655
Add capital expenditure 4,000 Cr.
Deduct depreciation
Premises (80,000 x 20%) 1,600 Dr.
Furniture (18,000 x 10%) 1,800 Dr.
Equipment (4,000 x 20%) 800 Dr.

Less 80% joining fee [17,800 /5 x 4] 14,240 Dr.


Less subscriptions in advance 960 Dr.
Add: subscription outstanding 300 Cr.
New surplus for year 8,555

356
Page 39 of 40
(b) LH Sports Club: Statement of financial position as at 30 April 2015
Rs. Rs.
Assets
Non-current assets
Premises (80,000 – 1,600) 78,400
Furniture (18,000 – 1,800) 16,200
Equipment (4,000 - 800) 3,200 97,800
Current assets
Inventory 1,400
Subscriptions in arrears 300
Prepaid rates and insurance 2,280
Bank 21,295 25,275
Total Assets 123,075

Capital and liabilities


Accumulated fund at 1 May 2014 -
Surplus for year 8,555
Accumulated fund at 30 April 2015 8,555
Joining fees (17,800 – 3,560 -3,560) 10,680
Loan from member(80,000+18,000) 98,000
Current liabilities
Payables 1,320
Joining fees (17,800/5) 10,680
Subscriptions in advance 960
Total Capital and liabilities 123,075
Working
Non-current assets Cost Depreciation Net
Rs. Rs. Rs.
Premises 80,000 (1,600) 78,400
Furniture 18,000 (1,800) 16,200
Equipment 4,000 (800) 3,200
102,000 4,200 97,800

357
Page 40 of 40
Accounting for Not-for-Profit Organisations
Introduction
Not-for-Profit Organisations (NPOs) are organisations organized and operated exclusively for social,
educational, professional, religious, health, charitable or any other not-for-profit purpose. An NPO's
members, contributors and other resource providers do not, in such capacity, receive any financial return
directly from the NPO. (means no dividend)

NPOs may be:


• companies formed under Section 42 of Companies Act, 2017;
• trusts formed under Trust Act, 1882;
• societies formed under the Societies Registration Act, 1860; or
• any other recognisable form of organisation giving value to the groups of people.

The financial objective of a profit-oriented entity is to make profit and maximise shareholders’ wealth
while financial objective of NPO is to provide its services effectively by achieving value for money. NPO
applies or intends to apply its profits, if any, or other income in promoting its objects, and prohibits the
distribution of surplus to its members, sponsors, promoters, etc. (in the form of dividends or drawings).

NPOs have income which they raise and costs which must be paid just like other organisations and although
profit is not their objective but they have to account for their income and costs. NPOs are accountable for
their effectiveness, economy and efficiency in utilising the funds. (to their members or sponsors)

Revenues of NPOs normally arise from donations, government grants and other contributions as well as
from membership fees, the sale of goods, the rendering of services or the use by others of NPO resources
yielding rent, interest, dividends.

Some accounting rules are as relevant to NPOs as to profit-oriented entities, for example, requirements
relating to inventory, non-current assets and recognition of revenue. However, some areas might be
completely irrelevant, for example, earnings per share.

Different terminology
NPOs use different accounting terminology from profit-oriented entities.
Profit-oriented entities Not-for-Profit Organisations
Statement of comprehensive income Statement of income and expenditure
Net profit Excess of income over expenditure or Surplus
Net loss Excess of expenditure over income or Deficit
Equity / Share capital and equity Net assets / Accumulated fund / Accumulated
reserves surplus / Accumulated deficit / Fund balance
Statement of changes in equity Statement of changes in net assets

358
Page 1 of 40
So far we have discussed and applied accounting concepts that are normally used by many small NPOs including
those which do not maintain proper double entry records. However, some NPOs are required to comply with
Accounting standards for non-profit organizations.

Accounting Standard for Not-for-Profit Organisations (ASNPO)


The Institute of Chartered Accountants of Pakistan (ICAP) issued the ‘Accounting Standard for Not-for-Profit
Organisations’ and as per Securities and Exchange Commission of Pakistan’s (SECP) directives, ASNPO is applicable
to associations not-for-profit registered under the Companies Act, 2017.

ASNPO is applicable to NPOs registered under the Companies Act, 2017, however, for other NPOs it is recommended
to prepare financial statements in accordance with ASNPO and stating that its financial statements have been
prepared in accordance with ‘Accounting Standard for Not-for-Profit Organisations’.

An NPO that is registered under the Companies Act, 2017 is also required to comply with the presentation and
disclosure requirements of the Fifth Schedule of the Companies Act, 2017. ASNPO is applicable so far as not in
conflict with the provisions of the Companies Act, 2017.

An NPO applying ASNPO will also apply the primary source of how to account and report transactions and events.

Primary sources: basis of accounting


An NPO applying ASNPO will also apply the primary source of how to account and report transactions and events.
The primary source of how to account and report transactions and events in financial statements will vary according
to the class of the NPO.

Applicable financial reporting framework (primary source)


The primary source for public interest entities (means listed Co. and other non-listed company as defined in third
schedule of Companies Act.) and large sized NPOs is IFRS issued by International Accounting Standards Board (IASB)
and as notified by SECP.

The primary source for medium and small sized NPOs is International Financial Reporting Standards for Small and
Medium-sized Entities (IFRS for SMEs) issued by IASB and as notified by SECP.

Micro NPOs (defined in ASNPO as organisations not registered under Companies Act, 2017 with annual gross
revenue less than Rs. 25 million) may opt to prepare their accounts on cash and disbursement basis (means cash
receipt and payment basis).

However, if a micro NPO opt to prepare and present its financial statements on accrual basis, it will prepare financial
statements in accordance with ASNPO and Accounting and Financial Reporting Standards for Small Sized Entities
(AFRS for SSE) as applicable in Pakistan.
Summary: Primary Source:
Large Sized NPO’s IFRS
Medium and small sized NPO’s IFRS for small and Medium sized entities
Micro NPO’s May opt receipt or payment basis of accounting; if
opts for accrual basis then AFRS

359
Page 2 of 40
Accounting policies
An NPO selects and applies its accounting policies for a period consistently for similar transactions, other
events and circumstances. (as per IAS-8)

When the concepts contained in ASNPO, conflict with a primary source, the requirements of the primary
source shall prevail.

Receipts and payments account


When accounts are prepared on cash or disbursement basis (in case of micro NPOs) rather than accrual
basis of accounting, a receipt and payment account is prepared and presented. This is simply a summary of
cash receipts and payments during the accounting period.

Illustration:

Receipts and payments account


Balance b/d X Donation to Dam Fund X
Subscriptions X Repairs X
Functions – ticket Telephone
revenue X X
Sale of land X Extension of building X
Bank interest X Furniture X
Bequest X Electricity expenses X
Sundry income X Salary and wages X
Sundry expenses X
Balance c/d X
X X
Balance b/d X

Financial statements
Financial statements of NPO shall normally include:
• statement of financial position (or balance sheet)
• statement of income and expenditure (instead of statement of comprehensive income)
• statement of changes in net assets (instead of statement of changes in equity)
• statement of cash flows.
• Notes to financial statements and supporting schedules to which the financial statements are cross-
referenced are an integral part of such statements.

Comparative information
Financial statements shall be prepared on a comparative basis, unless the comparative information is not
meaningful. Comparative information is normally meaningful. However, this may not be the case in some
rare circumstances, such as when the financial structure of the NPO has significantly changed [means e.g.
a micro NPO becomes a large NPO or vice versa.].

360
Page 3 of 40
Endowment Donation: it is cash or any other asset donated for the perpetual (continuous and never ending) benefit
of the non-profit organization. The donation is usually received with the requirement that the principal (original
investment) will remain intact and money earned from investing the principal will be used for the specific purposes
of the non-profit organization.

Most endowments are designed to provide a permanent source of income by keeping the original amount invested
and using the income for the purposes of NPO. An example of endowment is a donation to a university on the
condition that it is invested and that the investment income from the investment are used for scholarship of the
students.

Endowments are given to non-profit organizations with the intention that they be used to advance the mission of
the organization for long term.

RESTRICTIONS are stipulations that specify how resources must be used.

External restrictions are imposed from outside the organization, usually by the contributor of resources.

Internal restrictions are imposed in a formal manner by the organization itself, usually by a resolution of board of
trustees.

However, when we talk about restricted contributions we mean to say externally imposed restrictions only, means
if in a scenario there are internally restricted contributions we will simply consider them as unrestricted
contributions for the purpose of accounting (MCQ number 42).

Example: Types of funds


Question: Consider the following independent circumstances:
(a) A professional body of accountants (the NPO) sets-up a fund for financial support of deserving students. For this
purpose, Rs. 100 million have been allocated that will be invested and 80% of the investment income shall be
used for student support and 20% of investment income shall be added to fund investments. The fund
investments shall not be available for use by the NPO for its operations and the NPO shall preserve the
principal amount of fund.
(b) A healthcare NPO has raised money through special marketing drive in which overseas contributors deposited
$100 each in its ‘Save a life fund’ account. The contributions shall be used for the NPO’s routine operations
which focuses on providing life-saving drugs to patients who cannot afford the cost.
(c) An educational NPO has set-up a fund for development of new school in nearby rural area. The fund-raising
drive has been successful as many people have contributed for the cause. The fund-raising clearly stated that
the funds so raised shall only be used for construction and operations of school at that specific location.

Required:
Identify the type of above funds.

Answer:
(a) Endowment fund
(b) General fund (unrestricted)
(c) Restricted fund

361
Page 4 of 40
CONTRIBUTION REVENUE AND RECEIVABLE
Contribution
Contributions can come from many sources, including individuals, corporations, governments and other
NPOs. Contributions can be in cash or in kind. Contributions include contributions receivable that meet the
criteria for recognition in the financial statements (discussed later).

Definition: Contribution
A contribution is a non-reciprocal transfer to an NPO of cash or other assets (means cash or any other asset
is transferred from a third party with no expectation of payment in exchange) or a non-reciprocal
settlement or cancellation of its liabilities (e.g. a loan is waived off or creditor is not to be repaid
unconditionally). Government funding provided to an NPO is considered to be a contribution (however
usual practice is to refer such fund as grants).

Restrictions
Restrictions (explicit or implicit) on contributions may only be externally imposed.

Types of contribution
Restricted contribution A restricted contribution is a contribution subject to externally imposed stipulations
that specify the purpose for which the contributed asset is to be used.

Endowment contribution An endowment contribution is a type of restricted contribution subject to externally


imposed stipulations specifying that the resources contributed be maintained
permanently, although the constituent assets may change from time to time (means
e.g. cash is received and then building is constructed to earn rental income).

Unrestricted contribution An unrestricted contribution is a contribution that is neither a restricted contribution


nor an endowment contribution.

Example: Types of contributions


Question: Consider the following independent circumstances:
(a) A healthcare NPO received Rs. 10 million from wealthy individuals subject to the condition that this
amount shall only be used for acquisition of land for construction of a hospital in a specific village.
(b) A healthcare NPO received Rs. 25 million contribution from a wealthy individual in the year 2012. The
sole purpose of the amount is to support the NPO’s general operations in the year 2014 and 2015.
(c) An educational NPO received a plot of land from Mr. Salman subject to the condition that this land
shall only be used for construction of a primary education school to be run by that NPO. The fair value
of this plot of land is Rs. 12 million.
(d) An educational NPO received a plot of land from Mr. Jamal and it can be used to achieve general
objectives of that NPO. The fair value of this plot of land is Rs. 15 million.
(e) An educational NPO received Rs. 50 million from alumni (former) donors subject to the condition that
the principal balance shall be invested as per specified investment policy and NPO cannot use the
principal balance to fund operations. However, the NPO can utilise the investment earnings to pay for
things such as academic programs or building new school facilities.

Required:
Identify the type of contributions in above circumstances.

362
Page 5 of 40
Answer:
(a) Restricted contribution
(b) Restricted contribution
(c) Restricted contribution
(d) Unrestricted contribution
(e) Endowment contribution

Revenue recognition
Revenue from contributions is recognised by following either restricted fund method or deferral method (discussed
next). An NPO is required to select one method and apply it consistently over the periods and any change from one
method to the other shall be treated as change in accounting policy (as per IAS 8).

Contribution receivable
Recognition
A contribution receivable should be recognized as an asset when it meets the following criteria:
1. the amount to be received can be reasonably estimated; and
2. ultimate collection is reasonably assured.

Contribution receivable is different from accounts receivable in a business entity because the accounts receivables
are recognized after earning revenue and there is a legally enforceable right. However, there is no such legally
enforceable rights of contribution, as the NPO is not providing any goods or services against contributions.
Therefore, amount receivable should comparatively be more certain.

Pledge
A pledge is a promise to contribute cash or other assets to an NPO (like in function or in program). Similar to any
other contribution receivable, an uncollected pledge would only be recognized:
1. if it meets the above recognition criteria of contribution receivable.
2. there is reasonable assurance that the NPO will comply with conditions, if any, attached to the contribution (e.g
contribution is received if construction of DAM is started); and
3. contribution is not dependant on any contingent event outside NPO’s control (e.g. if Govt will also contribute
100 million).

Bequest/legacy
Bequests are often subject to considerable uncertainty surrounding both the timing of the receipt and the amount
that will actually be received. In many cases, the recognition criteria will not be satisfied and the bequest will not
be recognized until it is received.

Membership fee
Many NPOs receive membership fees. Such fees are considered fees for services when members receive services
having a value commensurate (in proportion to) with fees paid (e.g. to a cricket club or golf club). In other cases,
membership fees may be in substance contributions.

An NPO would decide whether its membership fees are contributions or fees for services and account for them
accordingly on a consistent basis. Some membership fees have characteristics of both fees for services and
contributions. Such fees would be divided into the portion that relates to fees for services and the portion that is in
substance a contribution.

363
Page 6 of 40
Example: Membership fee
Question: ABC Golf Club is members only club providing its members with sports facilities in the grounds
owned and maintained by it against annual subscription fee.
At 30 June 2012, the club had membership subscriptions in arrears amounting to Rs. 48,000,000
and had received Rs. 12,000,000 in advance.
During the year to 30 June 2013, the club received Rs. 650,000,000 from its members. This amount
includes:
▪ Rs. 26,000,000 received as donation from members (no conditions attached).
▪ Rs. 31,200,000 received for membership fee for the year to 30 June 2014.

At 30 June 2013, members owed Rs. 19,200,000 of subscriptions.


Half of the members who were in arrears at the end of the previous period still had not paid by 30
June 2013. It was decided to write these amounts off.

Required:
How the revenue from above should be reported in financial statements of ABC Golf Club for the
year ended on 30 June 2013?

Answer:
The donation of Rs. 26 million received shall be recognised as contribution revenue separately
from fee for services to members.

The subscription income (fee for services) may be calculated as follows:

Subscription Account
Rs. Rs.
b/d 48,000,000 b/d 12,000,000
subscription income (I&E) 600,000,000 Cash (Rs. 650m – 26m) 624,000,000
[fee for services]
Bad debts (Rs. 48m x 50%) 24,000,000
c/d 31,200,000 c/d 19,200,000
679,200,000 679,200,000

Government funding
Certain types of government funding are calculated and paid as if they were fees for services. However,
because the services being funded are provided to the NPO's community of service, and not directly to the
government, government funding is considered to be a contribution.

Example: Government funding


Question: Mujahid Healthcare (MH) is a registered NPO. It has received government funding of Rs. 20
million for which it has to provide vaccine (dosage and administration) for a viral disease to general public
(8,000 dosages x Rs. 2,500 each) without taking any fee from them.

Required:
Discuss the accounting treatment of above from perspective of MH.

364
Page 7 of 40
Answer:
The amount of Rs. 20 million is being calculated on dosage basis (i.e. 8,000 dosages x Rs. 2,500) which might
indicate that Rs. 20 million should be recognised as fee-for-services in statement of income and
expenditure.

However, since the service is not being provided to government but rather to MH’s community of service
(i.e. general public to whom MH provide healthcare services), the government funding of Rs. 20 million
shall be considered as contribution.

Further, since the purpose of government funding is specified, it shall be considered as restricted
contribution.

Factors that may indicate restrictions on Government funding:


Government funding is a significant component of many NPOs’ total contributions. Restrictions on
government funding may be indicated by following factors:
• the fact that the funding is provided based on the NPO's approved operating budget (verified by govt
department).
• the requirement to report to the government as to how the resources were actually used.
• the funding left over at the end of the period must be returned to the government.
• the funding received relates to expenses of the future period being funded.

Contributed materials and services


A contribution of assets other than cash (e.g. food packets, medicines or clothing) would be measured at
fair value estimated using market values. For contributed materials and services (e.g services of doctors by
private hospitals to NPOs as a contribution rather than cash) that are normally purchased, fair value would
be determined in relation to the purchase of similar materials and services.

Contributed materials and services (e.g. cement bags or engineer provided by an engineering firm) that
are part of a constructed or developed capital asset would be recognized at fair value.

When to recognize contributed materials and services


The NPO may choose to recognize contributions of materials and services, but should do so only when:
1. fair value can be reasonably estimated; and
2. when the materials and services are used in the normal course of the NPO's operations and would
otherwise have been purchased (if e.g NPO is not interested in advertisement but a channel has
displayed an advertisement out of its own choice it will not be recognized).

Often these contributions are not recorded because of record-keeping and valuation difficulties. For
example, it may be impractical to record the receipt of contributed services where the NPO depends
heavily on the use of volunteers to provide services (e.g. Hide collection volunteers throughout Pakistan).
Where contributed materials and services meet the criteria of fair value measurement, recording their value
would provide useful information.

365
Page 8 of 40
Inventories
Contribution of materials
When an NPO recognizes contributions of materials and goods, the cost of inventories shall reflect the fair
value at the date of contribution.

Inventories To be distributed at no charge or for a nominal charge (very small amount of money in
comparison with worth of an item e.g medicines)

An NPO shall measure inventories at the lower of cost (fair value if contributed, purchase cost if
purchased) and current replacement cost (current purchase price) when they are held for:
1. distribution at no charge or for a nominal charge (MCQ no. 55); or
2. consumption in the production process of goods e.g somebody has donated chemicals to be used in
production of medicines (to be distributed) at no charge or for a nominal charge.

Example: Inventories

Question: Medicine-for-All is an NPO which provides medicine to communities living in underdeveloped


areas at nominal charge. It has following inventories:
Replacement
Cost NRV* Fair value
Item Type cost
Rupees
Panadol Received in kind Nil 6,000 26,000 28,000
Neubrol 24,000 4,000 24,500 25,000
Imodium 12,000 3,000 12,000 12,500
Purchased for cash
Motilium 15,000 2,500 14,700 15,200
Rijix 18,000 3,500 18,300 17,900

*provided at nominal charge [estimated selling price – estimated selling expenses]


Required:
Calculate the amount of inventory that should be presented in the statement of financial position of
Medicine-for-All from above data.

Answer:
Item Basis Rupees
Panadol Cost equal to fair value but replacement cost is lower[28,000
26,000
and 26,000]
Neubrol Cost (lower)[24,000 and 24,500] 24,000
Imodium Cost / replacement cost (equal) [12,000 & 12,000] 12,000
Motilium Replacement cost (lower) [15,000 & 14,700] 14,700
Rijix Cost (lower) [18,000 & 18,300] 18,000
Total 94,700

366
Page 9 of 40
Collections
Definition: Collections
Collections are works of art(paintings), historical treasures (Kohinoor Diamond) or similar assets that
are:
▪ held for public exhibition, education or research;
▪ protected, cared for and preserved; and
▪ subject to an organisational policy that requires any proceeds from their sale to be used to
acquire other items to be added to the collection or for the direct care of the existing collection.

Cost of collections:
1. Purchase price if acquired
2. Fair value if contributed (if not available then nominal value)

Plus directly attributable costs like PPE.

Although items meeting the definition of a collection exhibit the characteristics of ‘assets’ they are
excluded from the definition of property, plant & equipment, and intangible assets. Collections are
made up of items that are often rare and unique. They have cultural and historical significance.

Although collections are usually held by museums or galleries, other NPOs may also have items that
meet the definition of a collection. For example, an NPO's library may include rare books which might
be considered to be a collection. The regular library materials, however, would not usually meet the
definition of a collection.

NPOs holding collections act as custodians for the public interest. They undertake to protect and
preserve the collection for public exhibition, education or research. The existence of a policy requiring
that any proceeds on the sale of collection items be used to acquire additional items or for the direct
care of the collection provides evidence of the NPO's commitment to act as custodian of the collection.
Costs incurred in protecting and preserving collection is considered as repair and maintenance and
therefore recognise as expense.

Certain works of art and historical treasure not to be depreciated


Certain works of art and historical treasures may have lives that are so long as to be virtually
unlimited. Works of art and historical treasures in this category are those that have cultural, aesthetic
(beauty), or historical value that is worth preserving perpetually. In addition, the NPO must have the
technological and financial ability to continue to protect and preserve them. Works of art and historical
treasures of this type would not be depreciated (if however, carrying amount exceeds fair value then
record a write down as impairment loss).

Property, plant and equipment


Definition: Tangible capital assets
Tangible capital assets are identifiable tangible assets that meet all of the following criteria:
▪ are held for use in the provision of services, for administrative purposes, for production of goods
or for the maintenance (cranes), repair, development or construction of other tangible capital assets;
▪ have been acquired, constructed or developed with the intention of being used on a continuing
basis;

367
Page 10 of 40
▪ are not intended for sale in the ordinary course of operations (not goods for sale); and
▪ are not held as part of a collection.

Recognition
Property, Plant and Equipment (PPE) shall be recognized as an asset, if and only if:
• it is probable that future economic benefits associated with the item will flow to the
NPO; and
• the cost of the item can be measured reliably.

Measurement for contributed PPE:


A contributed asset (e.g. medical equipment) would be recognized at its fair value at the date of
contribution. Fair value of a contributed asset may be estimated using market values.

When an estimate of fair value cannot reasonably be made, both the asset and the related contribution
would be recognized at nominal value (as in IAS 20).

A tangible capital asset purchased by an NPO at a value substantially below fair value would also be
recognized at its fair value with the difference between the consideration paid for the tangible capital asset
and fair value reported as a contribution.

A tangible moveable capital asset purchased from a grant may be recognised at carrying amount
deducting the grant. The grant is recognised in profit or loss over the life of the depreciable asset as a
reduced depreciation expense.

If it is an asset granted for a specified period and the asset has to be returned at the end of the grant
period, asset shall be valued at fair value less present value of the estimated residual amount at the end of
grant period(100,000 – 80,000).

Construction or development over time


The cost of PPE includes direct construction or development costs (such as materials and labour) and
overhead costs directly attributable to the construction or development activity. PPE which is developed
or constructed by an NPO might include contributed materials or labour, which would be recognized at
fair value at the date of contribution.

Land
Land normally has an undeterminable life and would not be depreciated.

Depreciation /Amortisation (under fund accounting means separate calculation of surplus or deficit of
each fund.)

When a fund accounting basis of reporting is used, the choice of the fund or funds to which depreciation
expense would be charged would be based on providing the most meaningful presentation.

Some NPOs may wish to show depreciation as an expense of the operating fund (general fund). This
presentation emphasizes that depreciation is part of the cost of service delivery of NPO [will be discussed
next].

368
Page 11 of 40
Other NPOs may prefer to show depreciation as an expense of the PPE fund. This presentation shows all
revenues and expenses associated with tangible capital assets in a single fund [will be discussed next].
Unamortised deferred contributions related to PPE if it is not used:
When PPE no longer contribute to the NPO's ability to provide services, its carrying amount would be
written down to residual value, if any. A write-down would be necessary, for example, when the NPO no
longer plans to use the asset because it has been damaged or rendered obsolete.

When an asset's carrying amount is written down, a corresponding amount of any unamortized deferred
contributions related to the asset would be recognized as revenue, provided that all restrictions have
been complied with [MCQ 56].

Intangible assets
Definition: Intangible asset
Intangible asset is defined as an identifiable non-monetary asset without physical substance held for use
in the production or supply of goods or services (food panda app), for rental to others [franchises], or for
administrative purposes[ head office Accounting software].

Recognition
The NPO shall recognize an intangible asset as an asset if, and only if:
• it is probable that the expected future economic benefits that are attributable to the asset will flow
to the NPO; and
• the cost or value of the asset can be measured reliably.

Internally generated assets: Expense (IAS 38)


Internally Generated research costs, goodwill, brands, training costs are always expensed out.
Expenditure on intangible item that was initially recognized as an expense shall not subsequently be
capitalized as part of the cost of an intangible asset.

Internally generated assets: Capitalised (IAS 38)


Development costs, which is the next step after research phase, can be capitalized if all of the following
conditions are fulfilled:
(a) the technical feasibility of completing the asset;
(b) the intention to complete the asset exists;
(c) the ability to use or sell the asset;
(d) how the asset will generate the future economic benefit and ability to demonstrate the existence of
market;
(e) the availability of adequate resources to complete; and
(f) the NPO's ability to reliably measure the cost of development of the asset.

Website costs (SIC 32)


Website costs are categorized into five basic stages that are:
• Stage 1: planning stage
• Stage 2: application and infrastructure development
• Stage 3: the graphical design development
• Stage 4: content development
• Stage 5: operations.
369
Page 12 of 40
Costs incurred in stage 1 and stage 5 are always expensed. However, costs incurred from stage 2 to 4 can
be capitalized if it fulfils the criteria of capitalisation of development asset discussed earlier, particularly
criteria (d).

Contributed intangible assets


Same requirements as for PPE.
Presentation of revenues and expenses — gross versus net
Revenues and expenses should be recognized and presented at their gross amounts (separately) and this
information may be presented in income and expenditure account or in the notes to the financial
statements. The determination of whether to report the revenues and expenses on a gross or net basis
depends on the relative facts and circumstances and requires significant judgment.

Example:
An NPO receives funding to undertake a specific research project. The NPO contracts at its own discretion
with a scientist to perform the research. The NPO would not have undertaken the research project had the
funds not been made available.

Required:
Whether the funding revenue and cost of scientist’s services be presented on gross basis or net basis?

Answer:
Although the NPO would not have undertaken the research project without the availability of the funding,
the NPO acts as the principal in contracting with the scientist. It specifies the details of the research to be
carried out by the scientist, and has discretion in selecting the scientist and in establishing the price to be
paid. Thus, the expenses incurred are obligations of the NPO. The funding revenue and cost of scientist
services should be presented on gross basis in statement of income and expenditure.

Example:
A research project is to be undertaken by a textile company, where there is a need for a trained person.
An NPO receives funding (reimbursement of salary) to allocate trained person to a textile company. The
NPO allocates an employee to textile company for the conduct of research. The NPO would be reimbursed
for all the costs related to that employee.

Required:
Whether the reimbursement and employee-related costs be presented on gross basis or net basis?

Answer:
The NPO has an employee who is seconded (send) to a textile company to work under their direction and
the NPO is reimbursed for all of the costs related to that employee. As the NPO is the employer, they would
report their employee-related costs as expenses and would report the reimbursement of their costs as
revenues on gross basis in statement of income and expenditure.

370
Page 13 of 40
Example:
An NPO engages in a number of fundraising activities, including a fundraising telethon, a telephone
campaign, a direct mail campaign, special events and a lottery. The NPO uses an outside fundraising
consultant to conduct the telethon and uses the NPO's own staff and volunteers in the telethon and the
other activities. Funds solicited (gathered) in each of the activities are raised in the name of the NPO.

Required: Whether the funds raised and related costs be presented on gross basis or net basis?

Answer:
Even though the NPO uses an outside fundraising consultant to conduct the telethon, the NPO is the
principal in the relationship with the donors as the funds are raised in its name. The NPO has discretion
in selecting the outside fundraiser, in establishing the fees to be paid and in determining the specifications
of the telethon. The NPO also has the credit risk if donors to the telethon do not pay according to their
pledge. Thus, the NPO should recognize the gross amounts fundraised in each of the activities as revenue
of the NPO, and the total expenses of each activity, including the fees charged by any outside consultant,
as expenses of the NPO, separately.

Example:
An NPO is actively engaged in helping communities in flood affected area. A group of students organised a
sports event, announcing that the net proceeds of the event shall be given to the NPO.

Required:
Whether to report the revenue and costs of the event on gross basis or net basis?

Answer:
The NPO is not the principal in the fundraising event as it was not involved in organizing the event and did
not bear any risks in connection with it. The amount received by the NPO is a donation from the organizers
of the event. Neither the gross revenues nor the gross expenses of the event are recognized in the NPO's
financial statements. The net proceeds received are recognized as a contribution.

Conclusion: it means if the NPO is acting as a principal or the persons involved are of NPO’s employee
then recognise incomes and expenses on gross basis otherwise net.

Fund accounting [maintaining separate set of books for each funding. Purpose is accountability so that
users get inside of activities of each funding]

Definition: Fund accounting


Fund accounting comprises the collective accounting procedures resulting in a self-balancing set of
accounts for each fund established by legal, contractual or voluntary actions of an NPO. Elements of a
fund can include assets, liabilities, net assets, revenues and expenses (and gains and losses, where
appropriate)[means each fund has all the financial statement elements]

Net assets or fund balances may be internally or externally restricted. Internally restricted net assets or
fund balances are often referred to as reserves (or general fund).

371
Page 14 of 40
Definition: Restrictions
Restrictions are stipulations imposed that specify how resources must be used. External restrictions are imposed
from outside the NPO, usually by the contributor of the resources. Internal restrictions are imposed in a formal
manner by the NPO itself, usually by resolution of the board of directors/council/board of trustees.

An NPO that uses fund accounting in its financial statements should provide a brief description of the purpose of
each fund reported.

Financial statements that are reported using fund accounting may follow the multi-column format whereby
resources or similar groups of resources are each assigned a separate column.
An NPO may present its financial statements using different formats for the individual statements. For example,
a statement of income and expenditure and changes in net assets presented in the multi-column format may be
accompanied by a statement of financial position that presents assets, liabilities and net assets in a single column
without presenting each financial statement item by individual fund.
There are two methods of fund accounting:
1. Deferral method and
2. Restricted fund method
1.Revenue recognition: deferral method [separate fund accounting is not mandatory]
When an NPO uses fund accounting in their financial statements without following the restricted fund method,
contributions would be accounted for using the deferral method. The contributions and related income are
recognised as follows:
Contribution Recognition Accounting entries
A. Endowment Recognise as direct increases in Cash xxx
contributions net assets in the current period Endowment fund xxx
and excluded from revenue (In statement of
(MCQ 44). changes in net
(Endowment contributions will assets)
never be available to meet
expenses associated with the
organization operation.
Therefor an organization
following the deferral method
would exclude such contribution
from revenue available for
current expenses by recognising
them as direct increases in net
asset.)
B Restricted Recognise as revenue in current Cash xxx
(a) contributions for period. (MCQ 47) revenue xxx
expenses of (In I&E)
current period
(b) Restricted Defer as a liability and Cash Xxx
contributions for recognise as revenue in the Deferred revenue Xxx
expenses of future same period(s) as the related (liability)
periods expenses are recognised. Deferred revenue xxx

372
Page 15 of 40
Contribution Recognition Accounting entries
When the only restriction on a Revenue xxx
contribution is that it cannot be (when related
used until a particular future expense is
period, the total amount of the recognised)
contribution would be
recognized as revenue in that
future period, whether or not it
has been spent.(e.g a
contribution for third future
period of operations)
(c) Restricted 1.In case of depreciable assets, Cash Xxx
contributions for defer as a liability and recognise Deferred revenue Xxx
the purchase of as revenue on the same basis as (liability)
capital assets the depreciation/amortisation Deferred revenue xxx
expense related to the acquired Revenue xxx
capital assets.

2.In case of non-depreciable Cash xxx


assets, recognise as direct Capital Asset fund xxx
increase in net assets (In statement of
(in this case, it is not possible to changes in net
match the contribution with the assets)
benefits provided since these
benefits are unlimited. Therefor
simply recognise as direct
increase in net assets)
3.A restricted contribution may
be provided for a certain area of
activity e.g research, without
contributor specifying which
portion is to used to acquire
capital asset. In order for a
contribution to be accounted for
as a contribution restricted for
the purchase of a capital asset,
the contributor must specify the
portion of the contribution that
is to be used to purchase capital
assets. If the contributor does
not so specify, then the
contribution would be
recognized as revenue when
spent for the particular purpose
covered by the restriction,(e.g

373
Page 16 of 40
Contribution Recognition Accounting entries
research) regardless of the fact
that some of the expenditures
may relate to the purchase of
capital assets.
(d) Restricted 1.In case debt was incurred to Cash Xxx
contributions for fund expenses of future Deferred revenue Xxx
the repayment of periods, defer and recognise as (liability)
debt revenue in same period(s) as the Deferred revenue xxx
related expenses are recognised Revenue xxx
(MCQ No.49).
2.In case debt was incurred to Cash Xxx
fund the purchase of capital Deferred revenue Xxx
asset (depreciable), defer and (liability)
recognise as revenue on the Deferred revenue xxx
same basis as the Revenue xxx
depreciation/amortisation
expense related to the acquired
capital assets(MCQ No.50).
3.In case debt was incurred to Cash xxx
fund the purchase of capital Capital Asset fund xxx
asset (non-depreciable), (In statement of
recognise as direct increase in changes in net
net assets (MCQ No.51). assets)
4.Otherwise, recognise as Cash xxx
revenue in income and revenue xxx
expenditure account(MCQ (In I&E)
No.52)
(C) Unrestricted Recognise as revenue in the Cash xxx
contributions current period revenue xxx
(In I&E)
(D) Net investment 1.Externally restricted Cash xxx
income (includes investment income that must Endowment fund xxx
revenue e.g rent be added to principal resources (In statement of
income gains or held for endowment are changes in net
losses on recognised as direct increase or assets)
investments. If decrease in net assets. [MCQ 53]
amount is invested
in shares)
2.Other externally restricted Cash xxx
investment income (similarly Deferred revenue xxx
related expenses) are or In capital assets
recognised as per type of fund
restriction discussed above [ in B
and C ] [MCQ 54]

374
Page 17 of 40
Contribution Recognition Accounting entries

3.In case there is no external Cash xxx


restriction, recognise in the revenue xxx
statement of income and (In I&E)
expenditure.
Deferred contributions balances should be presented in the statement of financial position outside net
assets as liability (like grants).
(E) Allocation of This Allocation shall be Unrestricted fund xxx
Unrestricted fund reported as inter fund Restricted fund xxx
transfer in statement of
changes in net assets

2.Revenue recognition: restricted fund method [separate Fund accounting ]


The restricted fund method of accounting for contributions is a specialized type of fund accounting that
involves the reporting of details of financial statement elements (assets, liabilities, incomes, expenses
and equity) by fund in such a way that the NPO reports:
1. total general funds;
2. one or more restricted funds; and
3. an endowment fund, if applicable.
Reporting of financial statement elements segregated on a basis other than that of use restrictions (for
example, by program (e.g. education, medicine or food programs) or geographic location (e.g. World Health
Organization’s Africa, Asia etc programs)) does not constitute the restricted fund method
The contributions and related income are recognised as follows:

Contribution Recognition Accounting entries


1. Endowment contributions Recognise as revenue of the Cash xxx
endowment fund in the current Endowment xxx
period (MCQ 43). fund
(In a separate
column of I&E)
2. Restricted contributions Restricted contributions for Cash xxx
reported in restricted which a corresponding restricted Restricted xxx
fund fund is presented should be fund
recognized as revenue of that (In a separate
fund in the current period(MCQ column of I&E)
45).
3. Restricted contributions Restricted contributions for which Cash xxx
reported in general fund. no corresponding restricted fund General Fund xxx
is presented (explanation below ( in a separate
after the table) should be column of I/E)
recognized in the general fund in
accordance with the deferral
method (MCQ 46).

375
Page 18 of 40
In such case, deferred
contributions balances should be
presented in the statement of
financial position outside net
assets as liability. (MCQ 48)

4. Unrestricted Recognise as revenue of the Cash xxx


contributions general fund in the current General Fund xxx
period. (In a separate
column of I&E)
5. Allocation of unrestricted This allocation shall be reported Unrestricted xxx
resources to restricted as an inter-fund transfer in the Fund
fund statement of changes in net Restricted xxx
assets. fund

6. Net investment income a)Externally restricted investment Cash xxx


(includes revenue, gains income that must be added to Endowment xxx
or losses on investments) principal resources held for fund
endowment, recognise in income (In I&E)
and expenditure in endowment
fund column.
b) Other externally restricted Cash xxx
investment income, in the Restricted xxx
statement of income and fund
expenditure in Appropriate (In I&E)
restricted fund

c) In case there is no external Cash xxx


restriction or if there is no General fund xxx
appropriate fund presented (In I&E)
recognise in the statement of
income and expenditure( in
General fund)

Explanation of point number 3: first of all we might create a separate restrictive fund if it is not already prepared.
However, in some situations it might not be appropriate to create a separate restricted fund separately because
contributions are received for only one-time event. E.g. for Silver Jubilee event.

PREPARATION OF FINANCIAL STATEMENTS


General
The accounting and approach for preparation of financial statements of an NPO is similar to other entities
except for the issues specifically addressed in ASNPO.

376
Page 19 of 40
This section will discuss general presentation requirements and formats of following:
• statement of financial position (or balance sheet)
• statement of income and expenditure
• statement of changes in net assets
• statement of cash flows.

Statement of financial position


The statement of financial position should present the following:
• net assets subject to restrictions to be maintained permanently as endowments;
• designated net assets (means externally restricted);
• unrestricted net assets; and
• total net assets.

Information about the NPO's liquidity is presented by classifying current assets separately from noncurrent
assets and current liabilities separately from non-current liabilities. Cash and other assets subject to
external restrictions limiting their use to beyond one year from the date of the statement of financial
position would be classified as non-current assets.

Illustration:
Statement of financial position (Format)[deferral method]
Not-for-Profit Organisation
Statement of financial position
As at 31 December 2012
2012 2011
Non-current assets Rs. 000 Rs. 000
Property and equipment 1,987 XX
Intangible assets 50 XX
Collections 80 XX
Investments 4,157 XX
6,274 XX
Current assets
Office supplies stock 55 XX
Prepaid expenses 58 XX
Grants/contribution receivable 17 XX
Cash and cash equivalents 183 XX
313 XX
6,587 XXX
Fund balances / Net assets
Net assets: restricted for endowments 208 XX

377
Page 20 of 40
Net assets: internally restricted for special projects 340 XX
General fund / Unrestricted net assets 2,698 XX
3,487 XX
Non-current liabilities
Deferred grants/contributions 1,800 XX
Net assets: Externally restricted for specific projects 241 XX
Loans 300 XX
2,100 XX
Current liabilities
Deferred grants/contributions 600 XX
Accrued expenses 400 XX
1,000 XX
6,587 XX

Illustration:
Statement of financial position (Multi-Columnar Format) [restricted fund method]
Not-for-Profit Organisation
Statement of financial position
As at 31 December 2012
2012 2011
Rs. 000 Rs. 000
General Special
Endowment Total
Non-current assets operations projects
Property and Equipment 1,580 407 1,987 XX
Intangible assets 50 50 XX
Collections 80 80 XX
Investments 3,052 897 208 4,157 XX
4,762 1,304 208 6,274 XXX
Current assets
Office supplies stock 52 3 55 XX
Prepaid expenses 51 7 58 XX
Grants/contribution receivable 17 17 XX
Cash and cash equivalents 166 17 183 XX
286 27 0 313 XX
5,048 1,331 208 6,587 XX

Fund balances / Net assets


Externally restricted 241 208 449 XX
Internally restricted 340 340 XX
Unrestricted 2,698 2,698 XX

378
Page 21 of 40
2,698 581 208 3,487 XX
Non-current liabilities
Deferred grants/contributions 1,300 500 1,800 XX
Loans 300 300 XX
1,600 500 0 2,100 XX
Current liabilities
Deferred grants/contributions 400 200 0 600 XX
Accrued expenses 350 50 400 XX
750 250 0 1,000 XX
5,048 1,331 208 6,587 XX

Statement of income and expenditure


Classification of expenses
NPO may classify expenses in the statement of income and expenditure:
• by object (by name) (for example, salaries, rent, utilities)
• by function (for example, administrative, research); or
• by program [like education, medical, clothing and food].

An NPO would classify its expenses in the manner that results in the most meaningful presentation in the
circumstances. Whether the NPO prepares its budgets by function or object would be a factor to consider
in deciding which method of expense classification would be most appropriate for the NPO's financial
statements[the methods used in budgets should be used in financial statements].

Attribution of expenses
When attributing an expense among various operating functions, an NPO considers an approach such as
the following:
▪ an expense that contributes directly to the output of one function is applied directly to that function,
for example, the cost of a staff member exclusively devoted to that function [e.g. employee of food
program].
▪ an expense that contributes directly to the output of more than one function is attributed on a
reasonable and consistent basis to each function to which it applies (for example, the rent applicable
to the space used for more than one separately reported function, and the remuneration expense of
an executive director of a health care NPO who, in addition to managing the NPO, provides direct health
care services to clients of that NPO).

Statement of income and expenditure — deferral method


Under the deferral method of accounting for contributions, total excess of revenues over expenses for all
funds reports the change in the NPO's unrestricted resources (means transferred to general fund) in the
period.

Illustration:
Statement of income and expenditure (Format)
Not-for-Profit Organisation
Statement of income and expenditure (deferral method)
For the year ended 31.12.2012

379
Page 22 of 40
2012 2011
Income Rs. 000 Rs. 000
Fee-for-services 5,300 XX
Government grants 1,200 XX
Contributions 170 XX
Fundraising events 350 XX
Investment income 31 XX
Other income 2 X
7,053 XXX

ecembe

Expenditures
Salaries 3,070 XX
Rent 1,320 XX
Office supplies used 610 XX
Utilities 880 XX
Marketing and communications 422 XX
Amortisation of capital assets 153 XX
(6,455) (XXX)
Excess of income over expenditure 598 XX

Statement of income and expenditure — restricted fund method


The statement of income and expenditure should present the following for the period:
1. the total for each financial statement item recognized in the general fund;
2. the total for each financial statement item recognized in the restricted funds, other than the
endowment fund;
3. the total for each financial statement item recognized in the endowment fund; and
4. excess or deficiency of revenues and gains over expenses and losses for each of the general fund,
restricted funds other than the endowment fund and the endowment fund.

Under the restricted fund method of accounting for contributions, the general fund presents all revenues
and expenses related to unrestricted resources.

Illustration:
Statement of income and expenditure (Format)
Not-for-Profit Organisation
Statement of income and expenditure (restricted fund method)
For the year ended 31 December 2012

380
Page 23 of 40
2012 2011
Rs. 000 Rs. 000
General Restricted Endowment
Total
Income fund fund fund
Fee-for-services 5,300 5,300 XX
Government grants 1,200 500 1,700 XX
Contributions 170 20 20 210 XX
Fundraising events 350 350 XX
Investment income 31 8 18 57 XX
Other income 2 2 X
7,053 528 38 7,619 XXX
Expenditures
Salaries 3,070 320 3,390 XX
Rent 1,320 1,320 XX
Office supplies used 610 20 630 XX
Utilities 880 57 937 XX
Marketing & communications 422 422 XX
Depreciation of capital assets 153 30 183 XX
(6,455) (427) 0 (6,882) (XX)
Excess of income over expenditure 598 101* 38* 737 XX

*In deferral method, these figures are directly shown in the statement of net assets.
Statement of changes in net assets
The statement of changes in net assets should present changes in the following for the period:
• restricted net assets (to be maintained permanently as endowments);
• internally restricted net assets;
• externally restricted net assets (other than endowment assets);
• unrestricted net assets; and
• total net assets.

The statement of changes in net assets may be referred to as ‘the statement of changes in fund
balances’ when the NPO uses fund accounting in its financial statements.
Inter-fund transfers should be presented in the statement of changes in net assets.

Illustration:
Statement of changes in net assets (Format)
Not-for-Profit Organisation
Statement of changes in net assets (deferral method)
For the year ended 31 December 2012

381
Page 24 of 40
Internally Externally
Externally
Unrestricted restricted restricted
restricted Total
General fund special fund endowment
fund
fund
Rs. 000
Balance 1 Jan 2,145 315 140 150 2,750
Surplus (from I&E) 598 598
Endowment
20 20
Contributions
Restricted grants &
520 520
contributions
Investment income 8 18 26
Fund utilisation(in
(427) (427)
expenses)
Internally imposed
(25) 25 0
restrictions
Transfers (20) 20 0
Balance 31 Dec 2,698 340 241 208 3,487

Illustration:
Statement of changes in net assets (Format)
Not-for-Profit Organisation
Statement of changes in net assets (restricted fund method)
For the year ended 31 December 2012
Internally Externally
Externally
Unrestricted restricted restricted
restricted Total
General fund special fund endowment
fund
fund
Rs. 000
Balance 1 Jan 2,145 315 140 150 2,750
Surplus 598 101 38 737
Internally imposed
(25) 25 0
restrictions
Transfers (20) 20 0
Balance 31 Dec 2,698 340 241 208 3,487

Statement of cash flows [same whether we use deferral method or restricted fund method]
Operating activities
Cash flows from operations include all cash receipts and payments resulting from the main, ongoing service
delivery activities of an NPO and exclude cash flows from financing and investing activities.

Cash receipts from operations include


1. unrestricted contributions,
2. restricted contributions that are to be used for operations and

382
Page 25 of 40
3. other revenues arising from the NPO's ordinary activities, such as fees for services, proceeds on the sale of goods
and unrestricted investment income.

Cash disbursements for operations would comprise expenditures made by the NPO in carrying out its
service delivery activities.(like salaries,rent,utilities)

Investing activities
Components of cash flows from investing activities would include the
1. acquisition of non current capital assets,
2. the purchase of investments, and
3. the proceeds on disposal of major categories of assets, such as capital assets and investments.

Financing activities
Components of cash flows from financing activities would include
1. cash contributed that is restricted for the purpose of acquiring capital assets and
2. cash contributed for endowment.
3. Cash receipts and disbursements related to the obtaining and repayment of debt (loan) would also be
presented as components of cash flows from financing activities.
Illustration:
Statement of cash flows (Format)
Not-for-Profit Organisation
Statement of cash flows
As at 31 December 2012
2012 2011
Cash flows from operating activities Rs. 000 Rs. 000

Surplus (deficit) of income over expenditure 197 (14)


Adjustments:

Depreciation/Amortisation of capital assets 24 30


Amortisation of deferred grant / contributions (84) (80)
Finance cost 16 12

153 (52)
Changes in working capital balances

Office supplies (18) (8)


Prepaid expenses (3) 4

Accrued expenses 8 (11)


Cash generated from operations 140 (67)
Interest paid (20) (15)
Grants and unrestricted contributions received [like unearned 280 250
maintenance income of Q.8 CFS]
Net cash from operating activities 400 168

383
Page 26 of 40
Cash flows from investing activities
Purchase of capital assets (800) (300)
Purchase of investments (1,400) (1,100)

Sale proceeds from investments 81 75


Investment income received 11 18

Net cash used in investing activities (2,108) (1,307)


Cash flows from financing activities
Loan obtained (repaid) 100 (500)
Endowment contributions 1,250 950
Restricted contributions for capital assets 300 800

Cash flows from financing activities 1,650 1,250


Net increase (decrease) in cash and cash equivalent (58) 111

Cash and cash equivalents at beginning of year 211 100


Cash and cash equivalents at end of year 153 211

Example: Professional Sports Club (I)


Question: The following information relates to Professional Sports Club (PSC), a Not-for-Profit Organisation.
Trial balance as at 30 June 2014
Dr. Cr.
Rs. m Rs. m
Total Funds as on 01 July 2013 (note 1) 1,715
Non-current assets (net) (note 5) 428
Investments (long term) 1,204
Short term bank loan 17
Prepaid and accrued expenses 8 11
Cash at bank 43
Fee-for-services 340
Fundraising in various tournaments (net proceeds) [organized by 15
others not the NPO otherwise funding and expenses should be
presented separately]
Contributions(note 2) 94
Government funding(note 3) 150
Investment income(note 4) 144
Salaries(note 8) 403
Rent and utilities (note 8) 354
Other expenses(note 8) 46
2,486 2,486

384
Page 27 of 40
Additional information:
1. The composition of fund balances is as follows:
Rs. m
Fund for Supporting the Young-Talent (Externally imposed stipulation that
resources contributed be maintained permanently)[therefore endowment 50
fund]
Fund for expenses of gymnasium and training centre (Externally imposed
115
stipulation for specific use of resources)
Fund for acquiring a franchise in a popular league (Internally imposed
3
stipulation for specific use of resources)
Fund for general operations: no restrictions 1,547
1,715

2. The details of contributions (same restrictions apply as are applicable to related fund) are as follows:
Rs. m
Contribution for ‘Supporting the Young-Talent’ [in statement of changes in
15 [para A]
net assets]
Contribution for expenses of gymnasium and training centre. 2 [para Bb]
Contribution to acquire freehold land (external restriction). However, the
12 [para B c 2]
land has not been acquired yet.
Contribution to repay loan that was taken to fund current year expenses
8 [para B (d)] 4
(therefore recognise as revenue).
Contributions (unrestricted but PSC itself imposed restriction that Rs. 3
million will be allocated annually for acquiring franchise in a popular
57[ Para [C]]
league) (therefore recognise as revenue, however 3 million will be
transferred through statement of net assets)
94
2. The government funding was received to support PSC general operations for five years starting from
1st January 2014.[para B(b)][treat it like government grant of IAS 20]
3. Investment income in trial balance include investment income of Rs. 6 million which is externally
restricted to be added to principal amount of resources for Young-Talent fund to be maintained
permanently[para D 1 include in endowment fund in statement of net assets]. There is no other
restrictions on investment income [para D 3 include in I & E].
4. Long term assets in the trial balance include freehold land of Rs. 20 million and collections of Rs. 8
million. These collections represent items of such historic value that is worth preserving perpetually
and PSC is committed to protect and preserve them as part of its organisation policy.(collection are to
be presented separately from other non-current assets)
5. Non-current assets are depreciated at 20% reducing balance method. All the depreciation is allocated
to general operations.[in I&E]
6. As part of agreement with contributors of ‘Supporting the Young-Talent’, PSC is required to allocate
Rs. 5 million from unrestricted fund to the endowment fund, annually.[transfer through statement of
changes in net assets]
7. The allocation of expenses is as follows:

385
Page 28 of 40
Gymnasium
General
and training
operations
centre (para
(in I&E)
D 2)
Rs. m Rs. m
Salaries 370 33
Rent and utilities 325 29
Other expenses 40 6
735 68
Required:
Prepare the following (under deferral method) for PSC:
▪ Statement of income and expenditure for the year ended 30 June 2014.
▪ Statement of changes in net assets for the year ended 30 June 2014.
▪ Statement of financial position as at 30 June 2014 (single column).

Answer:
Professional Sports Club
Statement of income and expenditure
For the year ended 30 June 2014
Income Rs. m
Fee-for-services 340
Fundraising proceeds 15
Contributions [8 + 57] 65
Government funding [150 / 5 years x 6/12] 15
Investment income [144 – 6] 138
Total 573
Expenditures
Expenses of gymnasium and training centre (68-68) 0
Salaries 370
Rent and utilities 325
Other expenses 40
Depreciation of long term assets [(428 – 8 – 20) x 20%] 80
Total (815)
Surplus / (Deficit) (242)

386
Page 29 of 40
Professional Sports Club
Statement of changes in net assets (like statement of changes in equity)
For the year ended 30 June 2014
Internally Externally Externally
Unrestricted
restricted Restricted Restricted
Franchise Young-Talent Capital Total
General fund acquisition Endowment asset
fund fund

Balance 1 July 1,547 3 50 - 1,600


Surplus / (deficit) (242) - (242)
Contributions 15 12 27
Investment income 6 - 6
Internally imposed -
(3) 3 0
restrictions
Transfers (5) 5 - 0
Balance 30 June 1,297 6 76 12 1,391

Professional Sports Club


Statement of financial position
As at 30 June 2014
Non-current assets Rs. M
Non-current assets [428 – 8 – 80] 340
Collections 8
Investments 1,204
1,552
Current assets
Prepaid expenses 8
Cash at bank 43
51
1,603
Fund balances / Net assets
Net assets: restricted for endowments 76
Net assets: internally restricted for franchise rights 6
General fund / Unrestricted net assets 1,297
Capital Asset fund / externally restricted 12
1,391

Non-current liabilities
Externally restricted fund for expenses for gymnasium and training centre (115+2- 49
68)
Deferred grants/contributions Government funding [150 – 15 – 30] 105

387
Page 30 of 40
Current liabilities
Deferred grants/contributions [ 150 / 5] (Govt funding current liability) 30
Short term bank loan 17
Accrued expenses 11
58
1,603

Example: Professional Sports Club (II)


Question: The following information relates to Professional Sports Club (PSC), a Not-for-Profit Organisation.
Trial balance as at 30 June 2014
Dr. Cr.
Rs. m Rs. m
Total Funds as on 01 July 2013 (note 1) 1,715
Non-current assets (net) (note 5) 428
Investments (long term) 1,204
Short term bank loan 17
Prepaid and accrued expenses 8 11
Cash at bank 43
Fee-for-services 340
Fundraising in various tournaments (net proceeds) 15
Contributions(note 2) 94
Government funding(note 3) 150
Investment income(note 4) 144
Salaries(note 8) 403
Rent and utilities (note 8) 354
Other expenses(note 8) 46
2,486 2,486

Additional information:
1. The composition of fund balances is as follows:
Rs. m
Fund for Supporting the Young-Talent (Externally imposed stipulation that
resources contributed be maintained permanently) (therefor endowment 50
fund)
Fund for expenses of gymnasium and training centre (Externally imposed
115
stipulation for specific use of resources)
Fund for acquiring a franchise in a popular league (Internally imposed
3
stipulation for specific use of resources)
Fund for general operations: no restrictions 1,547
1,715

388
Page 31 of 40
2. The details of contributions (same restrictions apply as are applicable to related fund) are as follows:
Rs. m
Contribution for ‘Supporting the Young-Talent’ 15 [para 1]
Contribution for expenses of gymnasium and training centre 2 [para 2]
Contribution to acquire freehold land (external restriction). However, the
12
land has not been acquired yet {create a new separate restricted fund}
Contribution to repay loan that was taken to fund current year expenses
8 [para 4]
(in general fund)
Contributions (unrestricted (in general fund) but PSC itself imposed
restriction that Rs. 3 million will be allocated for acquiring franchise in a 57 [para 4]
popular league)[through statement of changes in net assets]
94
3. The government funding was received to support PSC general operations for five years starting from
1st January 2014.[para 3] [treat it like government grant of IAS 20]
4. The investment income of Rs. 6 million is externally restricted to be added to principal amount of
resources for Young-Talent fund to be maintained permanently[para 6 (a)]. There is no other
restrictions on investment income. [para 6 (c)]
5. Long term assets in the trial balance include freehold land of Rs. 20 million and collections of Rs. 8
million. These collections represent items of such historic value that is worth preserving perpetually
and PSC is committed to protect and preserve them as part of its organisation policy. (collection are to
be presented separately from other non-current assets)
6. Non-current assets are depreciated at 20% reducing balance method. All the depreciation is allocated
to general operations.
7. As part of agreement with contributors of ‘Supporting the Young-Talent’, PSC is required to allocate
Rs. 5 million from unrestricted fund to the endowment fund, annually.[through statement of changes
in net assets]
8. The allocation of expenses is as follows:
Gymnasium
General
and training
operations
centre
6(c)
6(d)
Rs. m Rs. m
Salaries 370 33
Rent and utilities 325 29
Other expenses 40 6
735 68

Required:
Prepare the following (under restricted fund method) for PSC:
▪ Statement of income and expenditure for the year ended 30 June 2014.
▪ Statement of changes in net assets for the year ended 30 June 2014.
▪ Statement of financial position as at 30 June 2014 (single column).

389
Page 32 of 40
Note: in this method, present the income and expenditure of each fund in column form in I&E and then add their
respective surplus or deficits in their columns in statement of net assets. However, interfund transfers are recorded
through statement of net assets.

Answer:
Professional Sports Club
Statement of income and expenditure
For the year ended 30 June 2014
Income Unrestricte Restricte Restricted Endowmen Total
d d Capital t
Gymnasi fund Young
um and talent
training
fund
Rs. in “million”
Fee-for-services 340 340
Fundraising proceeds 15 15
Contributions [8 + 57] 65 2 12 15 82
Government funding 15 15
[150 / 5 years x 6/12]
Investment income[144-6] 138 6 144
Total 573 2 12 21 608
Expenditures
Salaries 370 33 403
Rent and utilities 325 29 354
Other expenses 40 6 46
Depreciation [(428 – 8 – 20) x 20%] 80 80
Total (815) (68) 0 (883)
Surplus / (Deficit) (242) (66) 12 21 (275)

Professional Sports Club


Statement of changes in net assets
For the year ended 30 June 2014
Internally
Unrestricted Externally restricted
restricted
Franchise Gymnasium Young-Talent Capital Total
General fund acquisition & Training Endowment asset fund
Centre Fund fund
Rs. m
Balance 1 July 1,547 3 115 50 - 1,715
Surplus / (deficit) (242) (66) 21 12 (275)
Internally imposed
(3) 3 0
restrictions
Transfers (5) 5 0
Balance 30 June 1,297 6 49 76 12 1,440

390
Page 33 of 40
Professional Sports Club
Statement of financial position
As at 30 June 2014
Non-current assets Rs. M
Non-current assets [428 – 8 – 80] 340
Collections 8
Investments 1,204
1,552
Current assets
Prepaid expenses 8
Cash at bank 43
51
1,603

Fund balances / Net assets


Net assets: restricted for endowments 76
Net assets: Externally restricted gymnasium and training centre 49
Net assets: internally restricted for franchise rights 6
Net assets: capital asset fund / restricted 12
General fund / Unrestricted net assets 1,297
1,440
Non-current liabilities
Deferred grants/contributions [150 – 15 – 30] 105

Current liabilities
Deferred grants/contributions [ 150 / 5] 30
Short term bank loan 17
Accrued expenses 11
70
1,603

391
Page 34 of 40
Example:
Following is the trial balance of Chongtar International Hospital as on 31 December 2019:

Debit Credit
---- Rs. in million ----
Burns ward - capital work in progress 55.3
Cafeteria sales 24.4
Cash and bank balances 8.4
Donations for burns ward during the period [externally restricted][para B 75.1
c 1]
Expenses and gifts for ‘walk on diabetes day’ 2.6
Fees from patients 125.0
General donations 82.6
General fund 195.6
Inventory - cafeteria 4.7
Inventory - medicines 19.4
Inventory - hospital supplies 8.5
Medical equipment 185.4 64.2
Miscellaneous expenses 8.5
Other fixed assets 110.7 54.7
Payables 38.9
Purchases - cafeteria 16.4
Purchases - medicines 60.5
Purchases - hospital supplies 18.7
Receivables - panel corporates 31.4
Rent 19.6
Sponsorship for ‘walk on diabetes day’ 2.2
Salaries - administrative staff 24.0
Salaries - doctors and nursing staff 38.2
Short term investments 38.0
Utilities 12.4
662.7 662.7

Additional information:
(i) Cost of closing physical inventory of medicines and hospital supplies was Rs. 25.8 million
and Rs. 13.8 million respectively. Medicines costing Rs. 3.1 million were found expired. Medicines
are only used to treat the admitted patients and are not sold separately.
(ii) Year-end physical count of cafeteria inventory could not take place. Goods are sold in cafeteria at a
gross margin of 25% on sales.
(iii) Rent outstanding at year-end was Rs. 1.4 million.
(iv) 15% of salaries of administrative staff and 10% of rent are related to cafeteria.
(v) Walk on diabetes day’ was organized in December 2019 [As it is the event conducted by the NPO,
which means income and expenditures should be presented separately]. Expenses relating to the
event amounting to Rs. 1.2 million were outstanding and unrecorded at year end.
(vi) Medical equipment having fair value of Rs. 36.8 million were received as donation on 01.01.2019.

392
Page 35 of 40
These have been brought into use but have not been recorded in the books.(contributed
depreciable asset should be recognized at fair value and treat the contribution as income over the
useful life of asset.)
(vii) Depreciation is charged on all assets on reducing balance method at 15% per annum.

Required: Prepare the following (using deferral method):


(a) Income and expenditure account for the year ended 31 December 2019. (12)
(b) Statement of financial position as on 31 December 2019. (06)

Answer
Chongtar International Hospital
Income and Expenditure Account
For the year ended 31-12-2019
“Rs. in Millions”
Incomes:
Fees from Patients 125.0
General donation 82.6
Recognition of revenue of contribution of Medical equipment [36.8 x 15%] 5.52
Sponsorship for walk on diabetes day 2.2
Profit from Cafeteria (W- 1) 0.4
Total 215.7
Expenditures:
Loss of Medicines 3.1
Salaries of Administrative staff (24 - 3.6) 20.4
Salaries - Doctors and nursing staff 38.2
Medicines used (19.4 + 60.5 - 3.1 - 25.8) 51.0
Hospital supplies used (8.5 + 18.7 - 13.8) 13.4
Rent (19.6 + 1.4 - 2.1) 18.9
Walk on diabetes day (2.6 + 1.2) 3.8
Depreciation – Medical equipment 23.7
Depreciation – Other fixed assets 8.4
Utilities 12.4
Misc. expenses 8.5
Total 201.80
Surplus 13.92

393
Page 36 of 40
Workings:
1. Profit of Cafeteria:
Sales 24.4
Cost of Sales:
Opening stock 4.7
Purchases 16.4
Closing stock (Bal) 2.8 (18.3)
Gross Profit 6.1
Expenses:
Salaries (24 x 15%) (3.6)
Rent (19.6 + 1.4) x 10% (2.1)
Net Profit 0.4

Chongtar International Hospital


Statement of Financial Position
As on 31-12-2019
Non – Current Assets: “Rs. in Millions”
Medical equipment (185.4 + 36.8 - 64.2 - 23.7) 134.3
Other fixed assets (110.7 - 54.7 - 8.4) 47.6
Burn ward – CWIP 55.3
Current Assets:
Cafeteria stock 2.8
Medicine stock 25.8
Hospital supplies 13.8
Receivables 31.4
Short term investments 38.0
Cash and bank 8.4
Total 357.4
Fund and Liabilities:
General Fund[unrestricted] 195.6
Add: Surplus 13.92 209.52
Total
Non-current liabilities
Burn Ward Fund [restricted] 75.1
Deferred contributions [36.8 -5.52-(31.3 x 15%) Current portion 26.58
based on reducing balance method]
Current Liabilities:
Payable 38.9
Deferred contributions [31.3× 15%) 4.7
Rent payable 1.4
Expenses payable on walk on diabetes day 1.2
Total 357.4

394
Page 37 of 40
Workings:
i) Loss of Medicines 3.1
Purchase of medicines 3.1

ii) Cost of Sales of Cafeteria sales to get closing stock of Cafeteria:


Sales = 24.4
Margin = 25% on sales
24.4/100 x 75% = 18.3
Inventory
b/d 4.7 Cost of Sales 18.3
Purchases 16.4 c/d (bal) 2.8

iii) Rent expense 1.4


Rent payable 1.4
iv) Salary expense related to cafeteria
24 x 15% = 3.6
Rent expense related to cafeteria:
Given = 19.6
+ Recorded = 1.4
Total 21
x10% 2.1

v) Expenses related to walk on diabetes day:


Expenses on Diabetes day 1.2
Exp payable 1.2
Medical Equipment 36.8
Deferred contribution 36.8

Medical Equipment:
(185.4 – 64.2) x 15% = 18.18
+ 36.8 x 15% = 5.52
Total 23.7

vii) Other fixed assets (110.7 - 54.7) x 15% = 8.4

Recognition and measurement


Recognition criteria
The recognition criteria under ASNPO are as follows:
• the item has an appropriate basis of measurement and a reasonable estimate can be made of the
amount involved; and
• for items involving obtaining or giving up economic benefits, it is probable that such benefits will be
obtained or given up.

395
Page 38 of 40
The recognition criteria provide general guidance on when an item is recognized in the financial statements.
Professional judgment is required to consider the specific circumstances to identify whether any particular item is
recognized or not.
Revenue Revenues are generally recognized when performance is achieved and reasonable assurance
regarding measurement and collectability of the consideration exists.

Unrestricted Unrestricted contributions to NPO do not normally arise from the sale of goods or the
contributions rendering of services and, consequently, performance achievement is generally not relevant
to the recognition of unrestricted contributions; such revenues are generally recognized
when received or receivable.

Restricted Restricted contributions are recognized based on the nature of the related restriction.
contributions
Gains Gains are generally recognized when realized.
Expenses and Expenses and losses are generally recognized when an expenditure or previously recognized
losses asset does not have future economic benefit. Expenses are related to a period on the basis
of transactions or events occurring in that period or by allocation applying the matching
concept(like depreciation).

Measurement
Financial statements of NPOs are prepared primarily using the historical cost basis of measurement whereby
transactions and events are recognized in financial statements at the amount of cash or cash equivalents paid or
received or the fair value ascribed to transactions and events when they took place.
Financial statements are prepared with capital maintenance measured in financial terms [which means financial
capital maintenance (money terms) as in MCQ number 39] and with no adjustment being made for the effect on
capital of a change in the general purchasing power of the currency during the period.

Types of funds (Sources of Contributions):


The funds can be classified into following three categories:

1.Endowment fund
An endowment fund is a self-balancing set of accounts which reports the accumulation of endowment
contributions. Only endowment contributions and investment income subject to restrictions stipulating
that it be added to the principal amount of the endowment fund would be reported as revenue of the
endowment fund.

Allocations of resources to the endowment fund that result from the imposition of internal restrictions are
recorded as inter-fund transfers (in statement of net assets)

2.Restricted fund
A restricted fund is a self-balancing set of accounts the elements of which are restricted or relate to the
use of restricted resources.
Allocations of resources that result from the imposition of internal restrictions are recorded as inter-fund
transfers to the restricted fund (in statement of net assets)

396
Page 39 of 40
3.General fund / unrestricted fund
A general fund is a self-balancing set of accounts which reports all unrestricted revenue and restricted
contributions for which no corresponding restricted fund is presented. The fund balance represents net assets that
are not subject to externally imposed restrictions.

397
Page 40 of 40
Question 1
Following amounts have been extracted from the financial statements of Lithops Limited:

2020 2019
----- Rs. in million -----
Sales 500 450
Cost of sales 378 300
Trade receivables 95 80
Trade payables 72 60
Inventory 93 75
Cash at bank 12 16

All sales and purchases are made on credit.

Required:
a) Calculate working capital cycle days for 2020. (Assume a 360-day year)
(04)
b) Suggest four possible measures that can be taken to reduce working capital cycle days.
(03)
Answer 1
(a) Working capital cycle days:

Trade A 𝐴𝑣𝑒𝑟𝑎𝑔𝑒 𝐷𝑒𝑏𝑡𝑜𝑟𝑠 87.5[(95 + 80) ÷ 2] ⇒ 63 days


𝑥 360 𝑥 360
receivables 𝐶𝑟𝑒𝑑𝑖𝑡 𝑆𝑎𝑙𝑒 500
collection period
Inventory holding B 𝐴𝑣𝑒𝑟𝑎𝑔𝑒 𝐼𝑛𝑣𝑒𝑛𝑡𝑜𝑟𝑦 84[(93 + 75) ÷ 2] ⇒ 80 days
𝑥 360 𝑥 360
period 𝑐𝑜𝑠𝑡 of sale 378
Trade payables C 𝐴𝑣𝑒𝑟𝑎𝑔𝑒 𝐶𝑟𝑒𝑑𝑖𝑡𝑜𝑟𝑠 66[(72 + 60) ÷ 2] ⇒ 60 days
𝑥 360 𝑥 360
payment period 𝐶𝑟𝑒𝑑𝑖𝑡 𝑃𝑢𝑟𝑐ℎ𝑎𝑠𝑒 (𝑤 − 1) 396
Working capital cycle =A+B−C = 63 + 80 ‒ 60 ⇒ 83 days

W-1: Calculation of purchases Rs. in million


Cost of goods sold 378
Add: Closing inventory 93
Less: Opening inventory (75)
Purchases (on credit) 396

(b) Measures to improve working capital cycle days:


▪ Give incentives to customers to pay on time
▪ Do not transact with customers who have a history of defaulting/late payments
▪ Automate the monitoring of accounts receivables
▪ Resolve disputes with customer as early as possible

398
Page 1 of 16
Q.2 The draft financial statements of Barbary Cement Limited (BCL) for the year ended 31 December 2020
include a plant having a carrying value of Rs. 400 million. Due to technological change, the remaining useful life of the
plant has been reduced to 4 years.

Following information has been gathered for impairment testing of the plant:
(i) Inflows from sale of product to be manufactured by the plant for the year 2021 are estimated at Rs. 200
million. These inflows are subject to 10% decrease in each subsequent year due to declining demand.
(ii) Outflows from operational cost for 2021 are estimated at Rs. 80 million. These outflows would increase by 5% in
each subsequent year despite decline in demand due to inflation and increase in plant’s wear and tear.
(iii) BCL’s net profit is subject to income tax of 20%.
(iv) Depreciation on plant is calculated using straight line method.
(v) The plant’s net disposal proceeds at the end of the useful life is estimated at Rs. 100 million.
(vi) Pre-tax and post-tax discount rates are 12% and 9.6% per annum respectively.
(vii) A technologically advanced plant with similar capacity can be purchased at Rs. 350 million. BCL has
received an offer to buy the existing plant for Rs. 250 million. BCL will have to incur shipping cost of Rs. 7 million,
to dispatch the existing plant to the purchaser.

Required:
Compute the impairment loss to be recognized as at 31 December 2020. (07)

A.2
------ Rs. in million ------
Impairment loss:
Carrying value 400.0
Recoverable amount
Value in use (W-1) 333.6
Fair value less cost of sell (250‒7) 243.0
Higher of above (333.6)
66.4

W-1: Value in use 2021 2022 2023 2024 Total


--------------- Rs. in million ---------------
Inflows from sale 200 180 162 145.80
Operational cost (80) (84) (88.2) (92.61)
Disposal value - - - 100
Cash flows undiscounted 120 96 73.8 153.19
Discount factor @ 12% 0.8929 0.7972 0.7118 0.6355
PV of cash flows /value in use 107.2 76.5 52.5 97.4 333.6

399
Page 2 of 16
Q.4

An asset was purchased on 1 January 2017 for Rs. 100 million with useful life of 6 years and residual value
(i) of Rs. 10 million. On 1 January 2020, it is revalued to Rs. 120 millionwith remaining useful life of 3 years
and expected residual value of Rs. 15 million. How much depreciation will be charged for the year ended
31 December 2020?
(a) Rs. 15 million (b) Rs. 35 million
(c) Rs. 20 million (d) Rs. 25 million (01)
When items of property, plant and equipment are stated at revalued amounts, which ofthe following
(iii) disclosures shall be made?

(a) Any restrictions on the distribution of the revaluation surplus to shareholders


(b) The carrying amount of temporarily idle property, plant and equipment
(c) The gross carrying amount of any fully depreciated property, plant and equipmentthat is still in use

(d) All of the above (01)


(iv) Which of the following concepts measures profit in terms of an increase in theproductive
capacity of an entity?
(a) Physical capital maintenance
(b) Historical cost accounting
(c) Financial capital maintenance (money terms)
(d) Financial capital maintenance (real terms) (01)

(v) Which of the following should be included in the initial cost of investment property?
(a) Cost incurred on opening ceremony to celebrate completion of property
(b) Operating losses incurred before the property achieves the planned level of
occupancy
(c) Abnormal waste of materials incurred in construction of property
(d) Property transfer taxes
(01)
(vi) An entity purchased an investment property on 1 January 2018 for Rs. 35 million. The property had an
estimated useful life of 35 years with no residual value. At 31 December 2020, the property had a fair
value of Rs. 42 million. On 1 January 2021, the property was sold for net proceeds of Rs. 40 million. Calculate the
profit or loss on disposal under both the cost and fair value models.

Cost model Fair value model


(a) Gain of Rs. 2 million Gain of Rs. 2 million
(b) Gain of Rs. 8 million Loss of Rs. 2 million
(c) Gain of Rs. 7 million Loss of Rs. 2 million
(d) Gain of Rs. 8 million Gain of Rs. 5 million

Which of the following is not considered as transaction with owners with reference tostatement of changes in
equity?
(2)

400
Page 3 of 16
(a) Issuance of shares at par (b) Issuance of shares at premium
(c) Profit for the year (d) Bonus issue of shares
(01)
(viii) Which two of the following factors could cause a company’s gross profit percentage onsales
to be above the expected level?
(b) Over-statement of closing inventories
(c) Sales were higher than expected
(d) Inclusion of disposal proceeds of non-current assets in sales
(e) Decrease in carriage charges borne by the company on goods sent to customers
(01)

A.4 (i) (b) Rs. 35 million


(iii) (a) Any restrictions on the distribution of the revaluation surplus to shareholders
(iv) (a) Physical capital maintenance
(v) (d) Property transfer taxes
(vi) (b) Gain of Rs. 8 million Loss of Rs. 2 million
(vii) (c) Profit for the year
(viii) (a) Over-statement of closing inventories
(c) Inclusion of disposal proceeds of non-current assets in sales

Q.6 Following are the extracts from the financial statements of Saguaro Limited (SL) for theyear ended 30
June 2021:

Statement of financial position as on 30 June 2021


Assets 2021 2020 Equity & liabilities 2021 2020
Rs. in million Rs. in million
Operating fixed assets 820 848 Share capital (Rs. 10 each) 700 500
Accumulated depreciation (300) (262) Share discount (40) -
Capital work in progress 84 - Retained earnings 220 315
Inventories 274 245 Long-term loans 175 210
Trade receivables 177 204 Trade payables 180 130
Insurance claim - 31 Accrued expenses 48 43
Advance to supplier 78 60 Current portion of long-
Cash and bank balances 193 112 term loans 43 40
1,326 1,238 1,326 1,238
Statement of profit or loss for the year ended 30June
2021
Rs. in million
Sales 757
Cost of sales (485)
Gross profit 272
Operating expenses (310)
Gain on disposal of equipment 17
Loss before interest (21)

401
Page 4 of 16
Other information:
(i) SL declared a final dividend of 10% on 30 September 2020 which was paid
in December 2020.
(ii) 20 million shares were issued in May 2021.
(iii) Insurance claim was related to plant and machinery destroyed in April 2020. The
plant had cost and book value of Rs. 63 million and Rs. 42 million respectively.
(iv) During the year, SL disposed of equipment having cost and net bookvalue of Rs.
75 million and Rs. 35 million respectively.
(v) Current portion of long-term loans include accrued interest of Rs. 5 million.
(2020: Rs. 1 million)
(vi) Trade payables include an amount of Rs. 14 million payables against capitalwork
in progress.

Required:
(a) Prepare SL’s statement of cash flows for the year ended 30 June 2021. (16)
(b) Calculate SL’s cash flow from operations for the year ended 30-06-2021 by using direct method
(07)

A.6 Saguaro Limited


Statement of cash flows for the year ended 30 June 2021
Indirect method
Rs. in million
Cash flows from operating activities
Loss for the year (45)
Adjustments for:
Depreciation on property, plant and equipment 78
Interest expense 24
Gain on disposal of property, plant and equipment (17)
Profit before working capital changes 40
Changes in working capital:
Increase in inventory (29)
Decrease in trade receivables 27
Increase in advance to supplier (18)
Increase in accrued expenses 5
Increase in trade payables 36
21
Cash generated from operations 61

Interest paid (20)


Net cash flows from operating activities 41

Cash flows from investing activities


Purchase of property, plant and equipment (47)

402
Page 5 of 16
Proceeds from claim of machinery disposed last year 31
Payment for CWIP (70)
Proceeds from disposal of equipment 52
Net cash flows used in investing activities (34)

Cash flows from financing activities


Proceeds from issue of shares 160
Repayment of long term loan (36)
Dividend paid (50)
Net cash flows from financing activities 74
Net increase in cash and cash equivalents 81
Cash and cash equivalents at beginning of the year 112
Cash and cash equivalents at the end of the year 193

(b) Saguaro Limited Statement of Cash


Flow For the year ended June
30,2021(Direct Method)
Receipt from customers (w-1)
784Payment to Suppliers (w-2)
(496)Payment for exp (w-3) (227)Cash generated from Operation

Workings:
Cash and Cash Equivalents: 2021 2020
Cash and bank balances 193 112

Fixed Assets A/c Acc.dep A/c


b/d 848 Disposal 75 Disposal 40 b/d 262
Cash 47 c/d 820 Dep 78
c/d 300
895 895 340 340

CWIP A/c Inventory A/c


b/d - b/d 245
Payable 14 Increase 29

Cash 70 c/d 84 c/d 274


84 84 274 274
Trade Insurance claim receivable
Receivable A/c b/ a/c
b/d 204 Decrease 27 d
31 31
c/d 177
c/d -
204
204 31 31

403
Page 6 of 16
Advance to Share Capital a/c
supplier A/c 500
b/d
b/d 60
Cash 160
Increase
Discount
18 c/ 700 40
c/d 78 d 700
700
78 78

Share Discount Retained Earnings a/c


A/c 315
Dividend b/d
b/d -
(500 x 10%)
Share 40 Loss 45
cap. c/d
220 315
c/d 40
40 40

loan A/c Interest Payable a/c


249 b/d 1
b/d Cash(bal.) 20 Interest 24
Cash (210+40-1)
expense
25
c/d(175+43- 213 c/d 5

5) Trade Payable A/c 249 CWIP Payable


25 A/c
130
b/d b/d
Increase 36 CWIP 14

c/d(180-14) 166 c/ 14
d 14
166 166 14
Calculation of interest expense
Accrued exp A/c Gain on disposal A/c
43 -
b/d Fixed 75 Acc.dep 40
Increase
Cash(bal) 52
c/ 48 Gain 17
d 92
48 48 92

Loss after Interest 45


Interest (bal) (24)
Loss before Interest (Given) 21

404
Page 7 of 16
Working for direct Method:

Debtor A/c(w-1) Stock a/c


b/d 204

Sales 757 Cash(bal.) 784

c/d 177
961 961

b/d 245
COS 485
Purchases(bal. 514
)
c/d 274

759 759
Advance + Creditors A/c(w-2) Accrued exp a/c(w-2)
b/d 60 b/d 130 b/d 43
Cash 227
Cash 496 Purchases 514 Exp 232
(310-78)
c/d 166 c/d 78 c/d 48
(180-14)
722 722 759 759

Q.7 Following information pertains to non-current assets of Bunny Ear Limited (BEL):

Land:
In January 2019, the government allotted a piece of land to BEL subject to the condition that BEL will establish a factory
building on it. The land was recorded at its fair value of Rs. 100 million.

Factory building:
On 1 March 2019, BEL started construction of the factory building. The construction work was completed on 30 June
2020. Payments related to the construction of the factory were as follows:

Description Date of payment Rs. in million


1st bill of contractor 1-Mar-2019 130
2nd bill of contractor 1-Aug-2019 190
3rd bill of contractor 1-Jan-2020 180
Last bill of contractor 1-Jul-2020 100

405
Page 8 of 16
The project was financed through:
i. Government grant of Rs. 200 million received on 1 February 2019. Unused funds from government grant were
invested in a saving account @ 8% per annum.
ii. Withdrawals from the following existing running finance facilities obtained from Bank A and Bank B.

The relevant details are:


Bank A Bank B
Obtained on 1 January 2019 1 January 2020
Markup rate 12% 14%
-------- Rs. in million --------
Balance on 31 December 2019 250 -
Markup for 2019 22 -
Balance on 31 December 2020 350 200
Average balance during 2020 300 150
Markup for 2020 36 21

Manufacturing plant:
The manufacturing plant was purchased on 1 August 2020 at cost of Rs. 420 million.Rs. 240 million was
financed through an interest free loan from government. The loan will be forgiven if the plant is operated for atleast 4
years by BEL. Upon acquisition, there is a reasonable assurance that BEL will comply with this condition.

Other information:
• BEL uses cost model for subsequent measurement of property, plant and equipment.
• All government grants are recorded as deferred income and a part of it is transferred to income each year.
• Useful life of the factory building and manufacturing plant has been estimated at 25 years and 10 years
respectively.

Required:
Prepare relevant extracts (including comparative figures) from BEL’s statement of profit or loss for the year ended
31 December 2020 and statement of financial position as on that date. (Notes to the financial statements are not required.
Borrowing costs are to be calculated on the basis of number of months) (16)

406
Page 9 of 16
A.7 Bunny Ear Limited
Extract from statement of profit and loss for the year ended 31 December 2020

2020 2019
------------Rs.in million--------------
Depreciation
• Factory building 12.5 -
• Manufacturing plant 17.5 -
Income from saving account (w-3) 3.66
Grant income
• Land 2.0 -
• Factory building 4.0
• Manufacturing plant 10.0
Interest expense (36+21-19),(22-6) 38.0 16.0

Extract from statement of financial position as on 31 December 2020

2020 2019
------------Rs.in million--------------
Non-current assets
Property plant and equipment
• Land 100.0 100.0
• Factory building [ (625(w1)-12.5(625/25×6/12)] 612.0
• Manufacturing plant (420-17.5(420/10×5/12)) 402.5
• Capital work in process-factory building (w-1) - 326.0

Non-current liabilities
Deferred government grant
• Land (100-2 (100/25×6/12)-4(100/25) 94 100.0
• Factory building (200-4 (200/25×6/12)-8(200/25) 188 200.0
• Manufacturing plant (240-10) (240/10×65/12)-24(240/10) 206

Current liabilities
Running finance 350+200 550.0 250.0
Deferred Govt grant (next year income)
Land 4 -
Factory building 8 -
Manufacturing plant 24 -

407
Page 10 of 16
W-1: cost of factory building Rs. In million
Payments in 2019 (130+190) 320.0
Borrowing cost capitalized in 2019 (w-2) 6.0
Balance as at 31 December 2019 326.0
Payments in 2020 (180+100) 280.0
Borrowing cost capitalized in 2019 (w-2) 19.0
625.0

W-2: Details of payment:


Funds from
Date Description Amounts Govt grant General
1-3-2019 1st bill 130 130 -
1-8-2019 2nd bill 180 70 120
1-1-2020 3rd bill 180 - 180

W-3: Borrowing cost to be capitalized:


2019:
120×12%×5*/12 =6 * August to December
Only loan from the bank A was available in 2019,so its rate is capitalization rate

2020:
120×12.67%×6*/12 =7.60 *January to June
180×12.67%×6*/12 =11.40 *January to June
To be capitalized in 2020 =19.00

Capitalization rate:
36 + 21
× 100 = 12.67%
300 + 150

W-4 Investment income from investment of govt grant :


200×8%×1*/12 =1.33 *February
70×8%×5*/12 =2.33 *March to July
= 3.66

408
Page 11 of 16
Q.8 The accountant of Cereus Golf Club (CGC) was terminated on charges of fraud and you have been assigned the
task of preparing the accounts for the year ended 31 December 2020. You have found thatthe proper books had not been
maintained. The management of CGC has given you the following information:
(i) Cash and bank balances at 1 January 2020 amounted to Rs. 0.5 million and Rs. 2 million respectively.
However, as on 31 December 2020, there was no cash balance and Rs. 4.2 million in the bank.
(ii) The members are required to pay 3 years’ subscription in advance upon admission/renewal.Full year
subscription is charged from members joining during the year. Number of
subscriptions received are as under:

Year No. of 3 years’


memberships subscriptionper
member
2018 100 Rs. 60,000
2019 140 Rs. 75,000
2020 160 Rs. 90,000

During 2020, 10 members were awarded membership on special permission but they had not paid the
subscription till year-end.

After year-end, 5 more members informed that they had paid the 3 years’ subscription amount in 2020. It was
found out that the amount was misappropriated by the accountant.
(iii) CGC had received a donation of Rs. 8 million in 2019 to meet the repair and maintenance expenditure of its
golf course. Out of total donation, the club has spent Rs. 2.2 million and Rs. 2.8 million in 2019 and 2020
respectively.
(iv) CGC started purchasing golf kits in 2020 for sales as well as for rent purposes. 20% of the purchases were
unpaid at year-end. Two third of the golf kit purchases made in 2020 had been added to inventory of golf kits for
sale and remaining had been added directly to golf kits forrent.
(v) Golf kits are sold for cash at cost plus 40%. Cost of closing inventory of golf kits for sale amounted to Rs. 1 million. It
was decided to transfer half of these kits into golf kits for rent at 30% of their original cost.
(vi) Some of the receipts and payments during the year were as follows:
Rupees
Rent of golf kits 650,000
Golf kits purchases 4,800,000
Annual insurances (paid till April 2021) 660,000
Salaries (including Rs. 350,000 for 2019) 2,800,000
Other expenses 2,320,000
(vii) CGC has a fidelity insurance policy and any cash deficiency upto a maximum of Rs. 2
million is recoverable under the policy.
(viii) Fixed assets at 1 January 2020 had a book value of Rs. 25 million. All fixed assets are to be
depreciated at 15% per annum.
Required:
a. Prepare income and expenditure account for the year ended 31 December 2020. (11)
b. Prepare statement of financial position as on 31 December 2020. (09)

409
Page 12 of 16
A.8 Cereus Golf Club
(a) Income and expenditure account for the year ended 31 December 2020

Rs. in '000
Income:
Subscription income (W-1) 10,750
Rent on golf kits 650
Profit on sale of golf kit (4200-3000) 1200
12,600
Expenditures:
Insurance expense 660–220(660÷12×4) (440)
Salaries expense 2,800–350 (2,450)
Loss on golf kits transferred 500×0.7 (350)
Depreciation 3,750(25,000×15%) (3,750)
Other expenses (2,320)
Loss on misappropriation (450+4170-2000) (W-2) (2,620)
(11,930)
Excess of income over expenditure(surplus) 670

(b) Statement of financial position as on 31 December 2020


Rs. in '000
Non-current assets:
Fixed assets 21,250
21,250
Current assets:
Golf kits 500
Insurance claim receiveable 2,000
Golf kits on rent 2,150
Prepaid Insurance 220
Subscription in arrears (W-1) 300
Bank balance 4,200
9,370
30,620
General funds:
Opening balance (W-3) 12,350
Excess of income over expenditure(surplus) 670
13,020

410
Page 13 of 16
Liabilities:
Repair and maintenance Donation in advance 3,000
Creditors - golf kits 1,200
Subscription in advance 13,400
30,620

411
Page 14 of 16
412
Page 15 of 16
*Accounting entry Golf kit for rent 150
Loss (I&E) 350
Golf kits for sale 500
*(140×75,000×1/3+160×90,000×2/3+5×90,000×2/3) =13,400

Calculation of Sales: 3,000/100 x 140 = 4,200

Net profit on sales=4200-3000=120

(W-3): Opening general fund: Rs. in '000


Cash and bank balance 2,500
Fixed assets 25,000
Subscription in advance (9,000)
Accrued salaries (350)
Repair and maintenance donation fund (5,800)
12,350

413
Page 16 of 16

You might also like